Csat Pyq Solved _(2011-2022)-Final Updated

You might also like

Download as pdf or txt
Download as pdf or txt
You are on page 1of 258

Previous Years Solved Papers

(2011-2022)

by
Manjul Kumar Tiwari Sir

DELHI CENTRE:
Old Rajinder Nagar: 27-B, Pusa Road, Metro Pillar no. 118, Near Karol Bagh Metro, New Delhi-110060 | Phone: 8081300200
Mukherjee Nagar: 1422, Main Mukherjee Nagar Road, Near Batra Cinema, New Delhi-110009 | Phone: 8081300200
BHOPAL CENTRE: Plot No. 46 Zone - 2, M.P Nagar, Bhopal - 462011 | Phone: 8827664612, 8081300200

JAIPUR CENTRE: A-1, Lion's Colony, Sitabari, Near The Theme Hotel, Tonk Road, Jaipur-302018 | Phone: 9358200511
Previous Years
Solved Papers
(2011-2022)

Contents
S.No. Year Pg.No.

1. CSAT Solved Paper : 2011 -------------------------------------------------------- 1 - 19

2. CSAT Solved Paper : 2012 ------------------------------------------------------ 20 - 40

3. CSAT Solved Paper : 2013 ------------------------------------------------------ 43 - 59

4. CSAT Solved Paper : 2014 ------------------------------------------------------ 60 - 77

5. CSAT Solved Paper : 2015 ------------------------------------------------------ 78 - 96

6. CSAT Solved Paper : 2016 ----------------------------------------------------- 97 - 116

7. CSAT Solved Paper : 2017 --------------------------------------------------- 117 - 137

8. CSAT Solved Paper : 2018 --------------------------------------------------- 138 - 159

9. CSAT Solved Paper : 2019 --------------------------------------------------- 160 - 180

10. CSAT Solved Paper : 2020 --------------------------------------------------- 181 - 202

11. CSAT Solved Paper : 2021 --------------------------------------------------- 203 - 230

12 CSAT Solved Paper : 2022 --------------------------------------------------- 231 - 258


Previous Year Solved Papers 3

Previous Year
CSAT : 2011 Solved Papers

Directions for the following 8 (eight) items: 1. According to passage :


Read each of the following two passages and answer 1. The objective of inclusive growth was laid
the items that follow. Your answers to these items should down by the founding fathers of the nation.
be based on the passages only. 2. Need of the hour is to have an enabling
Government.
Passage-1
3. The Government should engage in maximum
For achieving inclusive growth there is-a critical need to interference in market processes.
rethink the-role of the State. The early debate among 4. There is a need to change the size of the
economists about the size of the Government can be Government.
misleading. The need of the hour is to have an enabling Which of the statements given above are correct?
Government. India is too large and complex a nation for (a) 1 and 2 only (b) 2 and 3 only
the State to be able to deliver all that is needed. Asking
(c) 1 and 4 only (d) 1, 2, 3 and 4
the Government to produce all the essential goods, create
Sol. (a)
all the necessary jobs, and keep a curb on the prices of all
goods is to lead to a large cumbersome bureaucracy and 2. According to the passage, the strategy of inclusive
widespread corruption. growth can be effected by focussing on

The aim must be to stay with the objective of inclusive (a) meeting all the needs of every citizen in the
growth that was laid down by the founding fathers of country.
the nation and also to take a more modern view of what (b) Increasing the regulations over the
the State can realistically deliver. manufacturing sector.
(c) Controlling the distribution of manufacturing
This is what leads to the idea of an enabling State, that is,
goods.
a Government that does not try to directly deliver to the
(d) Delivery of the basic services to the deprived
citizens everything that they need. Instead, it (1) creates
sections of the society.
an enabling ethos for the market so that individual
Sol. (d)
enterprise can flourish and citizens can, for the most part,
provide for the needs of one another, and (2) steps in to The last paragraph clearly opines that the
help those who do not manage to do well for themselves, government's focus on delivery of basic services to
the deprived sections of the society promotes
for there will always be individuals, no matter what the
inclusive growth.
system, who need support and help. Hence we need a
Government that, when it comes to the market, sets 3. What constitutes an enabling Government?
effective, incentive-compatible rules and remains on the 1. A large bureaucracy.
sidelines with minimal interference, and, at the same 2. Implementation of welfare programmes
time, plays an important role in directly helping the poor through representatives.
by ensuring that they get basic education and health 3. Creating an ethos that helps individual
services and receive adequate nutrition and food. enterprise

CSAT by Manjul Kumar Tiwari Sir


4 CSAT : 2011

4. Providing resources to those who are Passage-2


underprivileged. The concept of ‘creative society’ refers to a phase of
5. Offering direct help to the poor regarding development of a society in which a large number of
basic services. Select the correct answer from potential contradictions become articulate and active.
the codes given below : This is most evident when oppressed social groups get
(a) 1, 2 and 3 only (b) 4 and 5 only politically mobilised and demand their rights. The
upsurge of the peasants and tribals, the movements for
(c) 3, 4 and 5 only (d) 1, 2, 3, 4 and 5
regional autonomy and self-determination, the
Sol. (c) environmental movements, and the women’s movements
Statements 3, 4 and 5 have been stated in the in the developing countries are signs of emergence of
passage. Statement 2 is not mentioned in the passage creative society in contemporary times. The forms of
and statement 1 is not supported by the passage. In social movements and their intensity may vary from
fact, it is against the idea of asking the government country to country and place to place within a country.
to produce all the necessary goods and jobs as that But the very presence of movements for social
would lead to a large bureaucracy. transformation in various spheres of a society indicates
the emergence of a creative society in a country.
4. Why is the State unable to deliver “all that is
needed”? 6. What does the author imply by “creative society”?
1. It does not have sufficient bureaucracy. 1. A society where diverse art forms and literary
2. It does not promote inclusive growth. writings seek incentive.
Select the correct answer from the codes given 2. A society where social inequalities are
below : accepted as the norm.
(a) 1 only (b) 2 only 3. A society where a large number of
(c) Both 1 and 2 (d) Neither 1 nor 2 contradictions are recognised.
Sol. (d) 4. A society where’ the exploited and the
The passage clearly mentions that large oppressed groups grow conscious of their
bureaucracy for delivering inclusive growth is a human rights and upliftment.
problem (it is not state can’t have large bureaucracy). Select the correct answer using the codes given
The second statement, is an effect of an Ineffective below :
state. It is not the cause for state non delivery. (a) 1, 2 and 3 (b) 4 only

5. What is essential message being conveyed by the (c) 3 and 4 (d) 2 and 4
author of the passage? Sol. (c)
(a) The objectives of inclusive growth laid down The statements – “The upsurge of the peasants and
by the founding fathers of the nation should tribals, the movements for regional autonomy and
be remembered. self-determination” and “a society in which a large
(b) The Government needs to make available number of potential contradictions become
more schools and health services. articulate and active” are reflected in option (c).
(c) The Government needs to establish markets
and industries to meet the needs of the poor 7. What according to the passage are the
strata of the society. manifestations of social movements?
(d) There is a need to rethink the role of the State 1. Aggressiveness and being incendiary.
in achieving inclusive growth. 2. Instigation by external forces.
Sol. (d) 3. Quest for social equality and individual
The very first few lines contain the whole essence freedom.
of the passage that role of the state needs a rethink 4. Urge for granting privileges and self-respect
Rest all can be easily eliminated. to disparaged sections of the society.

CSAT by Manjul Kumar Tiwari Sir


Previous Year Solved Papers 5
Select the correct answer using the codes given Directions for the following 2 (two) items:
below:
Each of the following two items consists of four
(a) 1 and 3 only (b) 2 and 4 only
statements. Of these four statements, two cannot both be
(c) 3 and 4 only (d) 1, 2, 3 and 4
true, but both can be false. Study the statements carefully
Sol. (c) and identify the two that satisfy the above condition.
Statements 1 and 2 are not correct as they don’t find Select the correct answer using the codes given below
a mention in the passage each set of statements :
Hence, the answer has to be (c). 10. Examine the following statements:

8. With reference to the passage consider the 1. All animals are carnivorous.
following statements: 2. Some animals are not carnivorous.
1. To be a creative society, it is essential to have 3. Animals are not carnivorous.
a variety of social movements. 4. Some animals are carnivorous.
2. To be a creative society, it is imperative to have Codes:
potential contradictions and conflicts. Which
(a) 1 and 3 only (b) 1 and 2 only
of the statements given above is/are correct?
(c) 2 and 3 only (d) 3 and 4 only
(a) 1 only (b) 2 only
Sol. (a)
(c) Both 1 and 2 (d) Neither 1 nor 2
Sol. (c) Check the validity of each option against the
remaining three.
9. Consider the following three statements: Consider S1: If all animals are carnivorous, then
1. Only students can participate in the race. the statement 3 is wrong. And if 3 is right, 1 is wrong.
2. Some participants in the race are girls. So, both are not true together. But if some animals
3. All girl participants in the race are invited for are carnivorous (as per S4), the both of these are
coaching. false. Hence correct option is (a).
Which one of the following conclusions can be 11. Examine the following statements:
drawn from the above statements? 1. All trains are run by diesel engine.
(a) All participants in the race are invited for
2. Some trains are run by diesel engine.
coaching.
3. No train is run by diesel engine.
(b) All students are invited for coaching.
4. Some trains are not run by diesel engine.
(c) All participants in the race are students.
Codes:
(d) None of the statements (a), (b) and (c) given
(a) 1 and 2 only (b) 2 and 3 only
above is correct.
(c) 1 and 3 only (d) 1 and 4 only
Sol. (c)
Option C can be easily seen from statement 1. Sol. (c)

The venn diagram is as follows. 12. Consider the four age pyramids given below
namely A, B, C and D representing four different
countries.

CSAT by Manjul Kumar Tiwari Sir


6 CSAT : 2011

Which one of them indicates the declining 14. Which of the two towns has a higher number of
population? persons with Diabetes?
(a) A (b) B
Distribution of diseases in Town-A
(c) C (d) D
Sol. (c)
The base in the pyramid denotes the number of
children and the top denotes number of senior
citizens. Now clearly if the top is wider than the
bottom, it means fewer births are taking place and
more deaths are taking place. So population will
reduce.

13. The followings figures has four curves namely A,


B, C and D, Study the figure and answer the item
that follows. Distribution of diseases in Town-B

Which curve indicates the exponential growth? (a) Town A


(a) A (b) B (b) Town B
(c) C (d) D (c) Same in Town A and Town B
Sol. (c) (d) No inference can be drawn
An Exponential curve is the one where the slope of Sol. (d)
the curve increases exponentially. The equation is Since the total population of Town A and Town B
y = ex is not given we cannot determine the number of
Curve A is not exponential as it shows a slight persons with diabetes.
decrease. Curve B is also not exponential (it is linear)
15. What can we say about persons with more than
Curve D is not exponential as it remains constant
one disease from these graphs?
for a period of almost upto 1000 years and then
(a) There are likely to be persons with more than
shows a sudden increase.
one disease in Town A.
Curve C is exponential as it increases continuously
(b) There are likely to be persons with more than
and also slowly in the early stage and at a fast pace
one disease in Town B.
in later stages.
(c) There are likely to be persons with more than
Directions for the following 2 (two) items:
one disease in both Towns A and B.
The following pie charts show the break-up of disease
(d) No inference can be drawn.
categories recorded in the patients from two towns, Town
A and Town B. Pie charts plot the disease Categories as Sol. (b)
percentage of the total number of patients. Based on On addition of all the percentages of the pie chart
these, answer the two items that follow the charts. for both the towns, town A = 100%, but town

CSAT by Manjul Kumar Tiwari Sir


Previous Year Solved Papers 7
B = 121%. This means that there is an overlap of The rapidly expanding human population has greatly
diseases in town B. modified the Earth’s ecosystems to meet their increased
16. Consider the following Velocity-Time graph. It requirements of some of the goods and services,
shows two trains starting simultaneously on particularly food, fresh water, timber, fibre and fuel.
parallel tracks. These modifications have contributed substantially to
human well being and economic development. The
benefits have not been equally distributed. Some people
have actually been harmed by these changes. Moreover,
short-term increases in some ecosystem goods and
services have come at the cost of the long-term
degradation of others. For example, efforts to increase
the production of food and fibre have decreased the
ability of some ecosystems to provide clean water,
regulate flooding and support biodiversity.
With reference to the above graph, which one of
the following statements is not correct? 17. With reference to the passage, consider the
(a) Train B has an initial acceleration greater than following statements. Expanding human’
that of Train A. population has an adverse effect on:
(b) Train B is faster than Train A at all times. 1. Spiritual fulfilment

(c) Both trains have the same velocity at time to’ 2. Aesthetic enjoyment
(d) Both trains travel the same distance in time to 3. Potable fresh water
units. 4. Production of food and fibre
Sol. (d) 5. Biodiversity
Acceleration in a velocity time graph can be seen
Which of the statements given above are correct?
from the graph’s slope (tilt with respect to
horizontal). B’s slope is more than A, which means (a) 1, 2 and 3 only (b) 2, 4 and 5 only
for the same time period, it is changing velocity (c) 3 and 5 only (d) 1, 2, 3, 4 and 5
faster than that of A. Hence more acceleration. Sol. (c)
Option (b) and (c) can be verified from the graph The paragraph – “For example, efforts to increase
since distance = Speed × time area under the curve the production of food and fibre have decreased the
in a velocity time graph gives distance covered.
ability of some ecosystems to provide clean water,
Directions for the following 6 (six) items: regulate flooding and support biodiversity”
Read each of the following two passages and answer mentions the adverse effects. Remaining options are
the items that follow. Your answers to these items should services provided by ecosystem to humans.
be based on the passages only.
18. The passage mentions that “some people have
Passage-1 actually been harmed by these changes.” What
Ecosystems provide people with a variety of goods does it imply?
and services; food, clean water, clean air, flood control, 1. The rapid expansion of population has
soil stabilization, pollination, climate regulation, adversely affected some people.
spiritual fulfilment and aesthetic enjoyment, to name
2. Sufficient efforts have not been made to
just a few. Most of these benefits either are irreplaceable
increase the production of food and fibre.
or the technology necessary to replace them is
prohibitively expensive. For example, potable fresh water 3. In the short term some people may be harmed,
can be provided by desalinating sea-water, but only at but in the long term everyone will benefit from
great cost. modifications in the Earth’s ecosystems.

CSAT by Manjul Kumar Tiwari Sir


8 CSAT : 2011

Which of the statements given above is/are moral one. How can a man understand morality who
correct? does not use his own intelligence and power of thought,
(a) 1 only but lets himself be swept along like a log of wood by a
(b) 2 current? Sometimes a man defies convention and acts
on his own with a view to absolute good.
(c) 1 and 3
(d) None of the statements given above 20. Which of the following statements best describe/
describes the thought of the writer?
Sol. (a)
1. A moral act calls for using our discretion.
The paragraph “The benefits have not been equally
distributed. Some people have actually been harmed 2. Man should react to a situation immediately.
by these changes.” – answers the question. 3. Man must do his duty.
Other statements run contrary to the passage. 4. Man should be able to defy convention in
order to be moral.
19. With reference to the passage, consider the
following statements: Select the correct answer from the codes given
1. It is imperative to modify the Earth’s below :
ecosystems for the well being of mankind. (a) 1 only (b) 1 and 3

2. Technology can never replace all the goods (c) 2 and 3 (d) 1 and 4
and services provided by ecosystems. Which Sol. (d)
of the statements given above is/are correct? First sentence supports statement 1. Statement 2
(a) 1 only (b) 2 only does not specify the situation and is out of context.
(c) Both 1 and 2 (d) Neither 1 nor 2 Statement 3 can‘t be true with the example given in
the passage, of the messenger and the king.
Sol. (d)
“The rapidly expanding human population has 21. Which of the following statements is the nearest
greatly modified the Earth’s ecosystems to meet their definition of moral action, according to the writer?
increased requirements of some of the goods and (a) it is a mechanical action based on official
services, particularly food, fresh water, timber, fibre orders from superiors.
and fuel. These modifications have contributed (b) It is an action based on our sense of discretion.
substantially to human well being and economic (c) It is a clever action based on the clarity of
development”. So statement 1 is the opposite of the purpose.
problem stated in the passage.
(d) It is a religious action based on understanding.
Statement 2 is also incorrect.
Sol. (b)
Paasage-2 The passage says that moral act must be based on
A moral act must be our own act; must spring from our our own discretion. Other statements are quoted
own will. If we act mechanically, these is no moral out of context.
content in our act. Such action would be moral. If we
22. The passage contains a statement “lets himself be
think it proper to act like a machine and do so. For, in
swept along like a log of wood by a current.”
doing so, we use our discrimination. We should bear in
Among the following statements, which is/are
mind the distinction between acting mechanically and
nearest in meaning to this?
acting intentionally. It may be moral of a king to pardon
1. A person does not use his own reason.
a culprit. But the messenger carrying out the order of
pardon plays only a mechanical part in the king’s moral 2. He is susceptible to influence/pressure.
act. But if the messenger were to carry out the king’s 3. He cannot withstand difficulties/challenges.
order considering it to be his duty, his action would be a 4. He is like a log of wood.

CSAT by Manjul Kumar Tiwari Sir


Previous Year Solved Papers 9
Select the correct answer using the codes given 24. Consider the following figures:
below:
(a) 1 only (b) 1 and 2 only
(c) 2 and 3 only (d) 1 and 4 only
Sol. (b)
Last few sentences indicate that man does not use
What is the missing number?
his own reason and he allows himself to be
(a) 7 (b) 8
influenced by external factors. Hence statement 1
and 2 are apt. Statement 3 is not presented in the (c) 9 (d) 10
passage and statement 4 is irrelevant. Sol. (c)
6, second column = 2 multiplied by 3;
23. Consider the following distance - time graph. The
graph shows three athletes A, B and C running 24 = 6 × 4 and 80 = 2 × 40
side by side for a 30 km race. Use the same pattern. So, 9 should be multiplied by
4 to get 36. 3 × 3 = 9

25. Study the following figure:

A person goes from A to B always moving to the


right or downwards along the lines. How many
different routes can he adopt?

Select the correct answer from the codes given


With reference to the above graph consider the below:
following statements : (a) 4 (b) 5
1. the race was won by A. (c) 6 (d) 7
2. B was ahead of A up to 25 km. Sol. (c)
3. C ran very slowly from the beginning. Consider a p × q rectangular grid with top left corner
Which of the statements given above is/are A and bottom right corner B. The number of distinct
correct? paths available to travel from A to B moving
downward and rightward is p + qCp or p + qCq.
(a) 1 only (b) 1 and 2 only
We have a 2 × 2 rectangular grid in this case. So, the
(c) 2 and 3 only (d) 1, 2 and 3
number of paths to travel from A to B = 4C2 = 6.
Sol. (b)
According to the graph, A has taken just below 30 26. Consider the following figure and answer the item
min to complete the race whereas B has taken more that follow:
than 35 min and C did not finish the race. So
statement 1 is correct.
B has taken less time than A upto 25 km. Hence B
was ahead of A upto 25 km. So statement 2 is also
correct.
It can be observed that C has taken less time than A
and B upto 25 km. So, statement 3 is wrong.

CSAT by Manjul Kumar Tiwari Sir


10 CSAT : 2011

What is total number of triangles in the above grid? 27. The passage mentions that “this world is evidently
(a) 27 (b) 26 not meant for them”. It refers to people who
(c) 23 (d) 22 1. seek freedom from foreign domination.
Sol. (c) 2. live in starvation and misery.
The triangles given are equilateral triangles of 3. become revolutionaries.
different lengths 1,2,3 and 4 units. The number of
Which of the statements given above is/are
triangles of 1 unit length = 1 + 3 + 5 + 3 = 12
correct?
The number of triangles of length 2 units = 1 + 2 +
(a) 1 and 2 only (b) 2 only
3 = 6. Plus there is one inverted triangle of length 2
units. So the number of triangles of length 2 (c) 2 and 3 only (d) 3 only
units = 7 Sol. (b)
The number of triangles of length 3 units = 1 + 2 = 3 Statement 1 is for countries but not in the context
The number of triangles of length 4 units = 1 question asks. Statement 2 is correct. If they cannot
So, the total number of triangles = 12 + 7 + 3 + 1 = 23 become revolutionaries, this world is not for them
as per the passage.
Directions for the following 4 (four) items:
28. Consider the following assumptions :
Read the following passage and answer the items that
follow. Your answers to these items should be based on 1. A country under foreign domination cannot
the passage only. indulge in spiritual pursuit.
2. Poverty is an impediment in the spiritual
Passage
pursuit.
A country under foreign domination seeks escape from
3. Subject peoples may become other-worldly.
the present in dreams of a vanished age, and finds
consolation in visions of past greatness. That is a foolish With reference to the passage, which of the above
and dangerous pastime in which many of us indulge. assumptions is/are valid?
An equally questionable practice for us in India is to (a) 1 and 2 only (b) 2 only
imagine that we are still spiritually great though we have (c) 2 and 3 only (d) 3 only
come down, in the world in other respects. Spiritual or
Sol. (c)
any other greatness cannot be founded on lack of
freedom and opportunity, or on starvation and misery. 29. The passage thematically centres on
Many western writers have encouraged that notion that (a) the state of mind of oppressed people
Indians are other-worldly. I suppose the poor and
(b) starvation and misery
unfortunate in every country become to some extent
other-worldly, unless they become revolutionaries, for (c) the growth of civilization
this world is evidently not meant for them. So also subject (d) body, mind and spirit of people in general
peoples. Sol. (a)

As a man grows to maturity he is not entirely engrossed The essence of the passage is that when a person is
in, or satisfied with, the external objective world. He seeks dissatisfied with external world he seeks inner
also some inner meaning, some psychological and meaning. This refers directly to the state of mind of
physical satisfactions. So also with peoples and the oppressed people as in the passage. Hence, (a).
civilizations as they mature and grow adult. Every Option (b) is a reason why a person is dissatisfied
civilization and every people exhibit these parallel with the external world, so it cannot be the central
streams of an external life and an internal life. Where idea. Option (c) is too general. In option (d), ‘people
they meet or keep close to each other, there is an in general’ makes it incorrect. The passage does not
equilibrium and stability. When they diverge conflict talk about people in general; it talks about the
arises and the crises that torture the mind and spirit. oppressed people in the society.

CSAT by Manjul Kumar Tiwari Sir


Previous Year Solved Papers 11
30. According to the passage, the torture of the mind (a) A (b) B
and spirit is caused (c) C (d) D
(a) by the impact of foreign domination. Sol. (b)
(b) by the desire to escape from foreign
33. Which region of the curve indicates that the
domination and find consolation in visions
treatment yielded effective relief?
of past greatness.
(a) C
(c) due to lack of equilibrium between an external
life and an internal life. (b) D
(d) due to one’s inability to be either (c) E
revolutionary or other-worldly. (d) The curve does not indicate the treatment
Sol. (c) Sol. (c)

Directions for the following 3 (Three) items: Relief is when the number of bacteria starts going
Read the passage given below, study the graph that down.
follows and answer the three items given below the figure. 34. There are four routes to travel from city A to city B
During a party, a person was exposed to contaminated and six routes from city B to city C. How many
water. A few days later, he developed fever and loose routes are possible to travel from the city A to
motions. He suffered for some days before going to a
city C?
doctor for treatment. On starting the treatment, he soon
became better and recovered completely a few days later. (a) 24 (b) 12
The following graph shows different phases of the (c) 10 (d) 8
person’s disease condition as regions A, B, C, D and E of Sol. (a)
the curve.
For every one route, you will have six routes to take.
So, total number of routes is 6 × 4 = 24

35. A contract on construction job specifies a penalty


for delay in completion of the work beyond a
certain date is as follows: Rs. 200 for the first day,
Rs. 250 for the second day, Rs. 300 for the third
day etc., the penalty for each succeeding day being
50 more than that of the preceding day. How much
penalty should the contractor pay if he delays the
31. Which region/regions of the curve correspond/ work by 10 days?
corresponds to incubation phase of the infection? (a) Rs. 4950 (b) Rs. 4250
(a) A only (c) Rs. 3600 (d) Rs. 650
(b) B only Sol. (b)
(c) Band C The penalty for delay on first day is 200, for second
(d) No part of the curve indicates the incubation day is 250, for third day is 300 and so on.
phase The penalties form an arithmetic progression (AP)
Sol. (a) with a common difference of 50 and first element
(a) as 200.
Incubation means where the bacteria just started
multiplying after insertion in the body. Thus A. The sum of penalties for 10 days is thus same as the
sum of numbers in the A.P. with n as 10. Sum of AP
32. Which region of the curve indicates that the person (formula) = n/2 [2a + (n – 1) d] = 10/2 [ 2 × 200 +
began showing the symptoms of infection? (10 – 1) × 50 ] = 4250

CSAT by Manjul Kumar Tiwari Sir


12 CSAT : 2011

36. Consider the figure given below and answer the Sol. (a)
items that follows: You can paint the figure using the colours A, B and
P2 C like this.

O P1
In the figure shown above, OP 1 and OP 2 are two
plane mirrors kept perpendicular to each other. S is
the direction of a beam of light falling on the mirror
OP 1. The direction of the reflected beam of light
from the mirror OP 2 will be
(a) Perpendicular to the direction S.
(b) At 45° to the direction S. 38. Consider the following figure and answer the items
that follows:
(c) Opposite and parallel to the direction S.
(d) At 60° to the direction S.
Sol. (c)
The rays will reflect at equal angles from OP1 and
OP2 : as per the diagram.
P1 A square is divided into four rectangles as shown
above. The lengths of the sides of rectangles are
natural numbers. The areas of two rectangles are
indicated in the figure. What is the length of each
S
side of the square?
(a) 10
O P2
(b) 11
Hence, its opposite and parallel to the direction of S.
(c) 15
37. Consider the following figure and answer the item (d) Cannot be determined as the given data are
that follows: Insufficient.
Sol. (b)
Area 15 units can be 15 × 1 or 5 × 3.
Area 48 units can be 48 × 1 or 24 × 2 or 16 × 3 or
12 × 4 or 8 × 6
Since the bigger figure is a square choose a
combination with the length and breadth that forms
a square: So if you take 5 units (from 15 units area)
+ 6 units (from 48 units area) or 8 units (from 48
units area) + 3 units (from 15 units area), it is equal
What is the minimum number of different colours to a side 11 square.
required to paint the figure given above such that 39. A person has only Rs. 1 and Rs. 2 coins with her. If
no two adjacent regions have the same colour? the total number of coins that she has is 50 and the
(a) 3 (b) 4 amount of money with her is Rs. 75, then the
(c) 5 (d) 6 number of Rs. 1 and Rs. 2 coins are, respectively

CSAT by Manjul Kumar Tiwari Sir


Previous Year Solved Papers 13
(a) 15 and 35 (b) 35 and 15 43. A village having a population of 4000 requires 150
(e) 30 and 20 (d) 25 and 25 litres of water per head per day. It has a tank
measuring 20 m x 15 m x 6 m. The water of this
Sol. (d)
tank will last for
Let the Rs. 1 and Rs. 2 coins be x and y in number
(a) 2 days (b) 3 days
respectively. Given x + y = 50 and x + 2y = 75.
(c) 4 days (d) 5 days
Solving the above two equations, you get x = 25 and
y = 25. Sol. (b)
Total requirement = 4000 × 150 = 6,00,000 litres
40. Three persons start walking together and their
Volume (capacity) of tank = L × B × H (for cuboids)
steps measure 40 cm, 42 cm and 45 cm respectively.
= 20 × 15 × 6 = 1800 m3 1m = 1000 litres
What is the minimum distance each should walk
so that each can cover the same distance in So, capacity = 18,00, 000
complete steps? So water will last for 18,00,000/6,00,000 = 3 days.
(a) 25 m 20 cm (b) 50 m 40 cm
Directions for the following 4 (four) items:
(c) 75 m 60 cm (d) 100 m 80 cm
Read the following passage and answer the items that
Sol. (a)
follow. Your answers to these items should be based on
Take the LCM of 40, 42 and 45 to know at what the passage only.
point will they find a common multiple. It is 2520 cm
i.e. 25 m 20 cm. Passage
A species that exerts. an influence out of proportion to
41. If a bus travels 160 km in 4 hours and a train travels its abundance in an ecosystem is called a keystone
320 km in 5 hours at uniform speeds, then what is species. The keystone species may influence both the
the ratio of the distances travelled by them in one species richness of communities and the flow of energy
hour? and materials through ecosystems. The sea star Pisaster
(a) 8 : 5 (b) 5 : 8 the flow of energy and materials through ecosystems.
(c) 4 : 5 (d) 1 : 2 The sea star Pisaster ochraceus, which lives in rocky
intertidal ecosystems on the Pacific coast of North
Sol. (b)
America, is also an example of a keystone species. Its
Speeds of bus = 40 kmph, so 40 kms in one hour.
preferred prey is the mussel Mytilus californianus . In
Speed of train = 64 kmph, so 64 kms in one hour.
the absence of sea- stars, these mussels crowd out other
Ratio = 40 : 64 = 5:8 competitors in a broad belt of the intertidal zone. By
consuming mussels, sea star creates bare spaces that
42. There are 100 students in a particular class. 60%
are taken over by a variety of other species.
students play cricket, 30% student play football
and 10% student play both the games. What is the A study at the University of washington demonstrated
number of students who play neither cricket nor the influence of Pisaster on species richness by removing
football? sea stars from selected parts of the intertidal zone
(a) 25 (b) 20 repeatedly over a period of five years. Two major changes
occurred in the areas from which sea stars were removed.
(c) 18 (d) 15
First, the lower edge of the mussel bed extended farther
Sol. (b) down into the intertidal zone, showing that sea stars
If 60% play cricket, and 30% play football then total are able to eliminate mussels completely where they are
percentage that plays both = 60 + 30 = 90% covered with water most of the time. Second, and more
But out of these, 10% that play both need to be dramatically, 28 species of animals and algae
subtracted as they are counted twice. So, 80% play disappeared from the sea star removal zone. Eventually
games. Those who play neither will be only Mytilus, the dominant competitor, occupied the
100 – 80 = 20%. entire substratum. Through its effect on competitive

CSAT by Manjul Kumar Tiwari Sir


14 CSAT : 2011

relationships, predation by Pisaster largely determines 47. Consider the following assumptions:
which species live in these rocky intertidal ecosystems. 1. The food chains/food web in an ecosystem
are influenced by keystone species.
44. What is the crux of the passage?
2. The presence of keystone species is a specific
(a) Sea star has a preferred prey.
characteristic of aquatic ecosystems.
(b) A preferred prey determines the survival of a
keystone species. 3. If the keystone species is completely removed
from an ecosystem, it will lead to the collapse
(c) Keystone species ensures species diversity.
of the ecosystem.
(d) Sea star is the only keystone species on the
Pacific coast of North America. With reference to the passage, which of the above
Sol. (c) assumptions is/are valid?
This has been explained with the example of the (a) 1 only (b) 2 and 3 only
sea star and the passage concludes on the same (c) 1 and 3 only (d) 1, 2 and 3
lines.
Sol. (c)
45. With reference to the passage, consider the Statements 1 has been mentioned in the passage.
following statements : Statements 2 is an out of context generalization.
1. Mussels-are generally the dominant species Although S is an extreme statement the paragraph
in intertidal ecosystems. provider evidence for this s.
2. The survival of sea stars is generally 48. Consider the following argument:
determined by the abundance of mussels.
“In, order to be a teacher one must graduate from
Which of the statements given above is /are correct?
college. All poets are poor. Some Mathematicians
(a) 1 only (b) 2 only are poets. No college graduate is poor.”
(c) Both 1 and 2 (d) Neither 1 nor 2
Which one of the following is not a valid
Sol. (d)
conclusion regarding the above argument?
Statement 1 is incorrect as it is not mentioned in the
(a) Some Mathematicians are not teachers.
passage. Statement 2 is also incorrect and conveys
opposite to what the passage says. (b) Some teachers are not Mathematicians.
(c) Teachers are not poor.
46. Which of the following is/are implied by the
passage? (d) Poets are not teachers.
1. Mussels are always hard competitors for sea Sol. (b)
stars. Since teachers can only be college graduates and
2. Sea stars of the Pacific coast have reached the since no college graduates are poor, so C is correct.
climax of their evolution. Moreover, since poets are poor, no poet can be a
3. Sea stars constitute an important component teacher. Also, since some mathematicians are also
in the energy flow in intertidal ecosystem. poets (and poor) they can’t be teachers. So, A is also
correct.
Which of the statements given above is/are
correct? B cannot be said with absolute certainty. Hence B is
incorrect and the answer.
(a) 1 and 2 (b) 2 only
(c) 1 and 3 (d) 3 only 49. A student on her first 3 tests receives on an average
Sol. (d) score of N points. If she exceeds her previous
Keystone species influence the flow of energy average score by 20 points on her fourth test, then
through ecosystems, and sea stars are a keystone what is the average score for the first 4 tests?
species. Hence only 3 is true. Other statements are (a) N + 20 (b) N + 10
out of context generalizations. (c) N + 4 (d) N + 5

CSAT by Manjul Kumar Tiwari Sir


Previous Year Solved Papers 15
Sol. (d) So, distance between C-D will be 2 × 7.07 = 14.12 km
The average score of student in 3 tests is N points. approx.
Hence the total score = 3N points. Directions for the following 5 (five) items:
Given the score in fourth test = N + 20, the average Read the following passage and answer the items that
score of student in four tests will be = (3N +N+20)/ follow. Your answers to these items should-be based on
4=N+5 the passage only.

50. In a group of persons, 70% of the persons are male Passage


and 30% of the persons are married. If two sevenths Now India’s children have a right to receive at least eight
of males are married, what fraction of the females years of education, the gnawing question is whether’ it
is single? will remain ‘on paper’ or ‘become a reality. One hardly
(a) 2/7 (b) 1/3 needs a reminder that this right is different from the
others enshrined in the Constitution, that the beneficiary
(c) 3/7 (d) 2/3 - a six year old child cannot demand it, nor can she or he
Sol. (d) fight a legal battle when the right is denied or violated.
70% males, so 30% females. In all cases, it is the adult society which must act on
behalf of the child. In another peculiarity, where a child’s
If 2/7th of males are married, this means 2/7 (70 %)
right to education is denied, no compensation offered
= 20% males are married. So, 10% females are
later can be adequate or relevant. This is so because
married (out of 30% total married). So, single females
childhood does not last if a legal battle fought on behalf
will be 20% out of total 30% females. of a child is eventually won, it may be of little use to the
The fraction is 2/3. boy or girl because the opportunity missed at school
during childhood cannot serve the same purpose later
51. The houses of A and B face each other on a road in life. This may be painfully true for girls because our
going north-south, A’s being on the western side. society permits them only a short childhood, if at all.
A comes out of his house, turns left, travels 5 km, The Right to Education (RTE) has become law at a point
turns right, travels 5 km to the front of D’s house. B in India’s history when the ghastly practice of female
does exactly the same and reaches the front of C’s infanticide has resurfaced in the form of foeticide. This
house. In this context, which one of the following is “symptomatic of a deeper turmoil” in society which
statements is correct? compounding the traditional obstacles to the education
(a) C and D live on the same street. of girls. “Tenacious prejudice against the intellectual
potential of girls runs across our cultural diversity and
(b) C’s house faces south.
the system of education has not been able to address it.
(c) The houses of C and D are less than 20 km
apart. 52. With reference to the passage, consider the
(d) None of the above following statements:
1. When children are denied education, adult
Sol. (c)
society does not act on behalf of them.
From the arrangement, it is clearly visible that
2. Right to Education as a law cannot be
enforced in the country.

Which of the statements given above is/are


correct?
(a) 1 only (b) 2 only
(c) Both 1 and 2 (d) Neither 1 nor 2
The distance between C and D, is twice the distance
Sol. (d)
between C and A.
When children are denied rights, the adult should
Between C and A, we use Pythagoras theorem i.e.
act on behalf of them. Hence 1 is wrong. And, 2 is

square root of 5  5 
2 2
 50  7.07 an out of context statement.

CSAT by Manjul Kumar Tiwari Sir


16 CSAT : 2011

53. According to the passage, what could be the 55. Which one of the following statements conveys
traditional obstacles to the education of girls? the key message of the passage?
1. Inability of parents to fight a legal battle when (a) India has declared that education is
the Right to Education is denied to their compulsory for its children.
children. (b) Adult society is not keen on implementing the
2. The traditional way of thinking about girl’s Right to Education.
role in society. (c) The Right to Education, particularly of a girl
3. The prejudice against the intellectual child, needs to be safeguarded.
potential of girls.
(d) The system of education should be address
4. Improper system of education. the issue of right to education.
Select the correct answer from the codes given Sol. (c)
below:
56. Which one of the following statements conveys
(a) 1 and 2 only
the inference of the passage?
(b) 2, 3 and 4 only
(a) The society has a tenacious prejudice against
(c) 1, 3 and 4 only
the intellectual potential of girls.
(d) 1, 2, 3 and 4
(b) Adults cannot be relied upon to fight on behalf
Sol. (b)
of children for their Right to Education.
Statement 1 is wrong as the passage clearly tells
(c) The legal fight to get education for children is
that parents fight a legal battle and even though
often protracted and prohibitive.
late, there is a chance that they might win it. The
last 2 sentences of the passage support statements (d) There is no sufficient substitute for education
2, 3 and 4. received in childhood.
Sol. (d)
54. On the basis of the passage, consider the following
statements: Since (a) is mentioned in the passage, it is not an
inference.
1. Right to Education is a legal right and not a
fundamental right. (b) and (c) are generalizations. Hence (d) can be
2. For realising the goal of universal education, inferred from the discussion about child education
the education system in the country must be rights from the passage.
made identical to that of developed countries.
Read the following passage and answer (three) items
Which of the statements given above is/are that follow:
correct?
A, B, C, D and E are members of the same family. There
(a) 1 only (b) 2 only
are two fathers, two sons, two wives, three males and
(c) Both 1 and 2 (d) Neither 1 nor 2
two females. The teacher was the, wife of a lawyer who
Sol. (d) was the son of a doctor. E is not male, neither also a wife
The passage mentions that it is a constitutional right of a professional. C is the youngest person in the family
in these lines – “One hardly needs a reminder that and D is the eldest. B is a male.
this right is different from the others enshrined in
57. How is D related to E?
the Constitution”. But we cannot be sure whether it
is a legal right as per the passage or fundamental (a) Husband (b) Son
right. At the same time to totally deny that is a legal (c) Father (d) Wife
right and not a fundamental right would be wrong. Sol. (a)
So since we cannot draw any inference, this Clearly the family has three generations. We are
statement should be marked wrong. putting the male first and then the female in the
2 is out of context generalization. couple. Now, there can be 2 possibilities.

CSAT by Manjul Kumar Tiwari Sir


Previous Year Solved Papers 17
Sol. (a)
1425 people voted for the proposal. Hence the
number of males who voted for the proposal
= 1425 – 600 = 825. Therefore the number of males
who were undecided = 1652 – 825 – 796 = 31

62. How many females were not in favour the


From the first arrangement, D is E’s husband. proposal?
58. Who are the females in the group? (a) 430 (b) 496
(a) C and E (b) C and D (c) 586 (d) 1226
Sol. (a)
(c) E and A (d) D and E
Refer above equation .
Sol. (c)
Refer to arrangement 63. In a queue, Mr. X is fourteenth from the front and
Mr. Y is seventeenth from the end, while Mr. Z is
59. Whose wife is the teacher? exactly in between Mr. X and Mr. Y. If Mr. X is
(a) C (b) D ahead Mr. Y and there are 48 persons in the queue,
(c) A (d) B how many persons are then between Mr. X and
Mr. Z?
Sol. (d)
(a) 6 (b) 7
Refer to arrangement
(c) 8 (d) 9
Read the following passage and answer the 3 (three) Sol. (c)
items that follow: Since X is 14th from the front, Y is 17th from the end,
and also X is ahead of Y, there must be 48 – (14 + 17)
In a survey regarding a proposal measure to be
= 17 persons in between X and Y.
introduced, 2878 person took part of which 1652 were
Now, as there are equal number of people between
males. 12 persons voted against the proposal which 796
X and Z; and Z and Y. So, the number of persons
were males. 1425 persons vote for the proposal. 196
between X and Z has to be 8.
females yet undecided. It will be like – 13 —X ——(8)——Y——(8)——Z
60. How many females voted for the proposal? Directions for the following 9 (nine) items:
(a) 430 (b) 600 The following nine items (Questions 64 to 72) are based
(c) 624 (d) 640 on three passages in English to test the comprehension
Sol. (b) of English language and therefore these items do not have
Hindi version. Read each passage and answer the items
Total number of males = 1652. Hence the number of
that follow.
females = 1226.
Passage-1
Given 796 males voted against the proposal out of He walked several miles that day but could not get
1226. Therefore 430 females voted against the anything to eat or drink except some dry bread and some
proposal. We also know 196 females were water, which he got from cottagers and farmers. As night
undecided. fell, he slept under a haystack lying in a meadow. He
Therefore, the number of females who voted for the felt frightened at first, for the wind blew awfully over
proposal = 1226 – 430 – 196 = 600. the empty fields. He felt cold and hungry, and was feeling
more lonely than he had ever felt before. He however,
61. How many males were undecided? soon fell asleep, being much tired with his long walk.
(a) 31 (b) 227 When he got up next day, he was feeling terribly hungry
so he purchased a loaf of bread with a few coins that
(c) 426 (d) 581
he had.

CSAT by Manjul Kumar Tiwari Sir


18 CSAT : 2011

64. When the night fell, he slept (a) unpleasant (b) sad
(a) in the open field (c) fantastic (d) amusing
(b) under a pile of dry grass Sol. (a)
(c) in a farmer’s cottage “just as I was going to close it, a horrible idea
(d) under a tree occurred to me - Had I packed my toothbrush” –
Sol. (b) this is neither amusing or fantastic. This is also not
sad as its more in the tone of unpleasantness.
65. He soon fell asleep because
(a) he was exhausted 68. What makes his life miserable whenever he
(b) he was all alone undertakes travelling?
(c) he had not slept for days (a) Going to railway station
(d) he was very frightened (b) Forgetting the toothbrush
Sol. (a) (c) Packing his bag
The line “he was tired with his long walk” is
(d) Bad dreams
reflected in 'a'.
Sol. (b)
66. With reference to the passage, consider the
following statements : 69. His toothbrush is finally
1. He was walking through the countryside, (a) in his bag (b) in his bed
2. The cottagers and farmers gave his enough (c) in his handkerchief (d) lost
food so that he could sleep at night without Sol. (c)
feeling hungry. “and have to rug upstairs for it at the last moment
Which of the statements given above is/are and carry it to the railway station, wrapped up in
correct? my pocket-handkerchief.” – this means it is in the
(a) 1 only (b) 2 only handkerchief.

(c) Both 1 and 2 (d) Neither 1 nor 2 Passage-3


Sol. (a) In spring, polar bear mothers emerge from dens with three
The description of the scenery and the farmers, month old cubs. The mother bear has fasted for as long
cottages etc. indicate clearly that it is countryside. as eight months but that does not stop the young from
“He felt cold and hungry” means 2 is wrong. demanding full access to her remaining reserves. If there
are triplets, the most persistent stands to gain an extra
Passage - 2
meal and it may have the meal at the expense of others.
I opened the bag and packed the boots in; and then, just
The smallest of the litter forfeits many meals to stronger
as I was going to close it, a horrible idea occurred to me -
Had I packed my toothbrush? I don’t know how it is, but siblings. Females are protective of their cubs but tend to
I never do know whether I’ve packed my toothbrush. My ignore family rivalry over food. In 21 years of
toothbrush is a thing that haunts me when I’m travelling, photographing polar bears, I’ve only once seen the
and makes my life a misery, I dream that haven’t packed smallest of triplets survive till autumn.
it, and wake up in a cold perspiration, and get out of bed
70. Female polar bears give birth during
and hunt for it. And, in the morning, I pack it before I
(a) spring (b) summer
have used it, and it is always the last thing I turn out of
the bag; and then repack and forget it, and have to rug (c) autumn (d) winter
upstairs for it at the last moment and carry it to the railway Sol. (d)
station, wrapped up in my pocket-handkerchief. “In spring, polar bear mothers emerge from dens
67. When he was going to close the bag, the idea that with three month old cubs”. So, three months before
occurred to him was spring is winter.

CSAT by Manjul Kumar Tiwari Sir


Previous Year Solved Papers 19
71. Mother bear Sol. More preferred (a); less preferred (b).
(a) takes sides over cubs Admitting your fault would be succumbing to
(b) lets the cubs fend for themselves unjust pressure. It is not expected of a civil servant.
(c) feeds only their favourites Putting responsibility on someone else would be
(d) see that all cubs get an equal share unfair, wrong and irresponsible.
Sol. (b) The other two are in line with upholding the truth
“Females are protective of their cubs but tend to and being sincere with your official responsibilities.
ignore family rivalry over food.” Suggest option (b)
74. A local thug (bad element) has started illegal
72. With reference to the passage, the following construction on your vacant plot. He has refused
assumptions have been made: your request to vacate and threatened you of dire
1. Polar bears fast as long as eight months due consequences in case you do not sell the property
to non-availability of prey. at a cheap price to him. You would
2. Polar bears always give birth to triplets. (a) sell the property at a cheap price to him.

Which of the assumptions given above is/are (b) go to the police for necessary action.
valid? (c) ask for help from your neighbours.
(a) 1 only (d) negotiate with the goon to get a higher price.
(b) 2 only
Sol. More preferred (b); less preferred (d)
(c) Both 1 and 2
Selling the property at cheap price would be
(d) Neither 1 nor 2
succumbing to pressure. It shows lack of capacity
Sol. (d) to fight for justice.
1 is partly true. Non-availability of prey is nowhere Asking for help from neighbours would not be a
mentioned in the passage. Statement 2 is also
good idea as he is a local thug. So the neighbours
incorrect. The passage says ‘if there are triplets'.
may not support you.
Directions for the following 8 (eight) items: Negotiating for a higher price (though less
preferable) will ensure that you don’t get a bad deal.
Given below are eight items. Each item describes a
situation and is followed by four possible responses. Willing to go to police shows that you can fight
Indicate the response you find most appropriate. Choose against injustice. This is a trait civil servants should
only one response for each item. The responses will be have. UPSC asks such questions to test the same.
evaluated based on the level of appropriateness for the
75. You have to accomplish a very important task for
given situation.
your headquarters within the next two days.
Please attempt all the items. There is no penalty for
Suddenly you meet with an accident. Your office
wrong answers for these eight items.
insists that you complete the task. You would
73. You have been asked to give an explanation for (a) ask for an extension of deadline.
not attending an important official meeting. Your
(b) inform Headquarters of your inability to finish
immediate boss who has not informed you about
on time.
the meeting is now putting pressure on you not to
place an allegation against him / her. You would (c) Suggest alternate person to headquarters who
may do the needful.
(a) send a written reply explaining the fact.
(b) seek an appointment with the top boss to (d) stay away till you recover.
explain the situation. Sol. More preferred (c); less preferred (a).
(c) admit your fault to save the situation. Best is (c) as it shows your professionalism towards
(d) put the responsibility on the coordinator of work keeping in mind your own personal
the meeting for not informing. limitations.

CSAT by Manjul Kumar Tiwari Sir


20 CSAT : 2011

76. You are an officer-in-charge for providing basic Threatening the boat owners might take some time,
medical facilities to the survivors of an earthquake and given the urgency may not be that practical.
affected area. Despite your best possible effort,
78. You are the officer-in-charge of a village
people put allegations against you for making
administering distribution of vaccine in an
money out of the funds given for relief. You would
isolated epidemic hit village, and you are left with
(a) let an enquiry be set up to look into the matter.
only one vaccine. There is a requirement of that
(b) ask your senior to appoint some other person
vaccine from the Gram Pradhan and also a poor
in your place.
villager. You are being pressurised by the Gram
(c) not pay attention to allegations. Pradhan to issue the vaccine to him. You would
(d) stop undertaking any initiative till the matter (a) initiate the procedure to expedite the next
is resolved. supply without issuing the vaccine to either.
Sol. More preferred (a); less preferred (c) (b) arrange vaccine for the poor villager from the
Asking for someone else to be appointed would be distributor of another area.
succumbing to public pressure, which is like a daily (c) ask both to approach a doctor and get an input
affair in the bureaucracy. about the urgency.
Stopping any further initiative will affect the relief
(d) arrange vaccine for the Gram Pradhan from
operations and is not advisable. Public interest
the distributor of another area.
should come first.
Sol. More preferred (d); less preferred (b)
Letting an inquiry in the matter is a sign of highest
integrity and is expected of civil servants. Integrity 79. You have taken up a project to create night-shelters
simple means subjecting oneself to the greatest for homeless people during the winter season.
scrutiny possible. Within a week of establishing the shelters, you
have received complaints from the residents of the
77. You have been made responsible to hire boats at a
area about the increase in theft cases with a
short notice to be used for an area under flood. On
demand to remove the shelters. You would
seeing the price mentioned by the boat owners you
found that the lowest price was approximately (a) ask them to lodge a written complaint in the
three times more than the approved rate of the police station.
Government. You would (b) assure residents of an enquiry into the matter.
(a) reject the proposal and call for a fresh price. (c) ask residents to consider the humanitarian
(b) accept the lowest price. effort made.
(d) continue with the project and ignore their
(c) refer the matter to the Government and wait.
complaint.
(d) threaten the boat owners about a possible
Solution: More preferred (b); less preferred (c)
cancellation of the licence.
A complaint in the police station may lead to the shelters
Sol. More preferred (b); less preferred (d).
being removed by police. Its the winter season. It will
Rejecting the proposal and calling for fresh prices
result in undue harassment for the poor shelterless
is a long drawn process. You do not have time as its
people.
urgent for floods.
Continuing with the project ignoring the complaint
Same problem is there with referring to the
would be abdicating duty and is not a sign of integrity,
government and waiting.
fairness and impartiality.
Since its very urgent best would be to accept the
lowers price for now since its required very urgently Asking them to consider the humanitarian efforts made
for relief operations. You may be dragged into would at least save the homeless people’s lives in the
controversies later, but public interest should be put winter. Then you can monitor the situation and take
first. Service should be selfless. appropriate action.

CSAT by Manjul Kumar Tiwari Sir


Previous Year Solved Papers 21
80. You, as an administrative authority, have been Sol. More preferred (a); less preferred (d)
approached, by the daughter-in-law of an (d) is less preferred because the girl might not go to
influential person regarding harassment by her police because of social pressures as mentioned in
in-laws on account of insufficient dowry. Her the question.
parents are not able to approach you because of
Since you are an administrative authority, you
social pressures. You would
cannot handle dowry cases. Best would be to assist
(a) call the in-laws for an explanation. the lady by calling the in-laws for explanation.
(b) counsel the lady to adjust, given such a This would improve the faith of people in the
circumstance. administration; it shows capacity for justice; and
(c) take action after her parents approach you. sensitivity towards important social issues.
(d) ask her to lodge complaint with the police.
  

CSAT by Manjul Kumar Tiwari Sir


22 CSAT : 2012

Previous Year
CSAT : 2012 Solved Papers

Directions for the following 6 (six) items: Select the correct Solution: using the codes given
below:
Read the following two passages and Solution: the items
(a) 1, 2, 3 and 4 (b) 1, 2 and 3 only
that follow each passage. Your Solutions to these items (c) 2 and 4 only (d) 1, 3 and 4 only
should be based on the passages only.
Sol. (b)
Passage 1 “protection against free market individualism” tells
The poor especially in market economics need the that statement 3 is correct. Rest others are very
strength that collectivities offer for creating more explicitly mentioned in the passage.
economic, social and political space for themselves, for
2. What does the author imply by “gender impact”?
enhancing their socio- economic well-being and voice,
(a) Women are doubtful participants in
and as a protection against free market individualism.
cooperatives.
It has been argued that a group approach to farming,
(b) Family cooperatives may not include women
especially in the form of bottom up agricultural (c) Women benefitting from group farming.
production collectivities, offers substantial scope for (d) Women’s role in transition economies is
poverty alleviation and empowering the poor as well as highly restrictive.
enhancing agricultural productivity. To realize this Sol. (c)
potential, however, the groups would need to be
Clear from the passage.
voluntary in nature, small in size, participative in
decision making and equitable in work sharing and 3. Consider the following assumptions:
benefit distribution. There are many notable examples 1. It is imperative for transition economies to
of such collectivities to be found in varied contexts, such have agricultural collectivities.
as in transitions economies. All of them bear witness to 2. Agricultural productivity can be increased by
the possibility of successful cooperation under given group approach to farming.
conditions. And although the gender impact of the With reference to the above passage which of these
family cooperatives in the transition economies are assumptions is/are valid?
uncertain, the Indian examples of women-only groups (a) 1 only (b) 2 only
farming offer considerable potential for benefiting (c) Both 1 and 2 (d) Neither 1 nor 2
women. Sol. (b)

1. Agricultural collectivities such as group based The word “imperative” is an extreme word. It means
farming can provide the rural poor something which is absolutely necessary. The
1. Empowerment passage only says that they are helpful.
2. Increased agricultural productivity. Passage 2
3. Safeguard against exploitative markets. In a typical Western liberal context, deepening of
4. Surplus production of agricultural democracy invariably leads to consolidation of ‘liberal
commodities. values’. In the Indian context, democratization is

CSAT by Manjul Kumar Tiwari Sir


Previous Year Solved Papers 23
translated into greater involvement of people not as (d) Relative unimportance of hereditary identities
‘individuals’ which is a staple to liberal discourse, but over class identities.
as communities or groups. Individuals are getting Sol. (b)
involved in the public sphere not as ‘atomized’ Option (a) could have been true; but its very general
individuals but as members of primordial communities – it talks about public sphere; and not Indian politics
drawn on religious or caste identity. Community-identity in specific.
seems to be the governing force. It is not therefore
surprising that the so-called peripheral groups continue 6. What is the “silent revolution” that has occurred
to maintain their identities with reference to the social in the Indian democratic process?
groups {caste, religion or sect} to which they belong while (a) Irrelevance of caste and class hierarchies in
political processes
getting involved in the political processes despite the
(b) Loosening of social strictures in voting
fact that their political goals remain more or less
behavior and patterns.
identical. By helping to articulate the political voice of
(c) Social change through transfer of power from
the marginalized, democracy in India has led to ‘a
upper caste elites to subaltern groups.
loosening of social strictures’ and empowered the
(d) All the statements a), b) and c) given above are
peripherals to be confident of their ability to improve
correct in this context.
the socio economic conditions in which they are placed.
Sol. (c)
This is a significant political process that had led to a
silent revolution through a meaningful transfer of power Option (a) is incorrect, which also makes option (d)
from the upper caste cities to various subaltern groups incorrect.
within the democratic framework of public governance. Option (b) is incorrect, because the loosening has
not been in the voting behaviour; it has been
4. According to the passage, what does “deepening towards the social structures and participation in
of democracy” mean in the Western context? the political process.
(a) Consolidation of group and class identities.
(b) Democratization translated as greater Directions for the following 5 (five) items:
involvement of people. Examine the information given in the following
(c) Democratization as greater involvement of paragraph and Solution: the items that follow:
‘atomized’ individuals in the public sphere. Guest lectures on five subjects viz., Economics, History,
(d) None of the statements a, b and c given above Statistics, English and Mathematics have to be arranged
is correct in this context. in a week from Monday to Friday. Only one lecture can
Sol. (c) be arranged on each day. Economics cannot be scheduled
Option (a) talks about the Indian context. Option on Tuesday. Guest faculty for History is available only
on Tuesday. Mathematics lecture has to be schedules
(b) talks about democracy in general.
immediately after the day of Economics lecture. English
Option (c) talks about the consequences of
lecture has to be scheduled immediately before the day
democratization in a normal society. In the Western of Economics lecture.
context it simply means people embrace liberal
values. 7. Which lecture is scheduled on Monday?
(a) History (b) Economics
A careful reading of passage will give you the
(c) Mathematics (d) Statistics
solutions.
Sol. (d)
5. Greater democratization in India has not Since Tuesday's schedule is fixed - for English to
necessarily led to come immediately before economics, and economic
(a) The dilution of caste and communal identities to come immediately before maths, the following
in the public sphere. will be the arrangement.
(b) Irrelevance of community identity as Monday - Statistics, Tuesday – H (History)
governing force in Indian politics. Wednesday – English, Thursday – Economics,
(c) Marginalization of elite groups in society. Friday –Mathematics

CSAT by Manjul Kumar Tiwari Sir


24 CSAT : 2012

8. Which lecture is scheduled between Statistics and (c) Most machines are operated on electric
English? energy.
(a) Economics (b) History (d) Electrically operated machines are preferable
(c) Mathematics (d) No lecture to use.
Sol. (b) Sol. (d)

9. Which lecture is the last one in the week? Option 1: Just because all machines consume
(a) History (b) English energy; and electricity provides energy, does not
(c) Mathematics (d) Economics mean that electricity is the only source of energy.
Sol. (c) Machines can run on other sources of energy too.
So it is wrong.
10. Which lecture is located scheduled on Option 2: Electricity provides energy does not mean
Wednesday? it is only source providing energy. Hence, Option 2
(a) Statistics (b) Economics
is also wrong.
(c) English (d) History
Option 3: In the light of the above discussion,
Sol. (c)
Option3 is also wrong.
11. Which lecture is scheduled before the Mathematics Option 4: From statements 3 and 4, this seems the
lecture? most reasonable option. Hence, (d)
(a) Economics (b) History
(c) Statistics (d) English 14. Examine the following statements:
Sol. (a) 1. None but the rich ran afford air-travel.
2. Some of those who travel by air become sick
12. Two glasses of equal volume are respectively half 3. Some of those who become sick require
and three-fourths filled with milk. They are then treatment
filled to the brim by adding water. Their contents
Which one of the following conclusions can be
are then poured into another vessel. What will be
drawn from the above statements?
the ratio of milk to water in this vessel?
(a) All the rich persons travel by air.
(a) 1 : 3 (b) 2 : 3
(b) Those who travel by air become sick
(c) 3 : 2 (d) 5 : 3
(c) All the rich persons become sick.
Sol. (d)
(d) All those who travel by air are rich
Lets say both glasses have 100 ml capacity. Glass 1:
Sol. (d)
50 ml water and 50 ml milk
Option (a) is incorrect because even if the rich can
Glass 2: 25 ml water and 75 ml milk
afford air travel, its not necessary that every rich
Total when poured together = 200 ml Milk = 125 ml person will travel by air.
Water = 75 ml
Since (b) is also incorrect, (c) has to be incorrect.
Ration = 125 : 75 = 5 : 3 Option (d) is the correct answer.
13. Consider the following statements: 15. In five flats, one above the other, live five
1. All machines consume energy professionals. The professor has to go up to meet
2. Electricity provides energy his IAS officer friend. The doctor is equally friendly
3. Electrically operated machines are cheap to to all, and has to go up as frequently as go down.
maintain The engineer has to go up to meet his MLA friend
4. Electrically operated machines do not cause
above whose flat lives the professor’s friend.
pollution.
Which one of the following inferences can be From the ground floor to the top floor, in what order
drawn from the above statements? do the five professionals live?
(a) All machines are run by electric energy. (a) Engineer, Professor, Doctor, IAS officer, MLA
(b) There is no form of energy other than electricity (b) Professor, Engineer, Doctor, IAS officer, MLA

CSAT by Manjul Kumar Tiwari Sir


Previous Year Solved Papers 25
(c) IAS officer, Engineer, Doctor, Professor, MLA (b) Education is meant to fulfill human needs
(d) Professor, Engineer, Doctor, MLA, IAS officer (c) The purpose of education is to train the
Sol. (d) human intellect
It is clear that the doctor is at the centre and the (d) Education is meant to achieve moral
MLA stays above the Engineer. development
It is also given that IAS officer is professor’s friend. Sol. (a)
So, IAS officer must stay above Engineer and MLA. Observe the following lines: “But in its deepest sense
Only the last option is satisfying the above education is not instrumentalist. That is to say, it is
conditions. not to be justified outside of itself…”
You can also check it by drawing a diagram. Options 2, 3 and 4 are mentioned in the passage as
S1: IAS  Prof the outcomes of education. Hence, the correct
S2: Doc is in the middle S3: IAS > .. MLA >..> answer is option 1.
Engineer 17. According to the passage, education must be
Now only way in which these can be satisfied is: respected in itself because
IAS > MLA > Doctor > Engineer > Professor a) It helps to acquire qualifications for
Directions for the following 15 (fifteen) items: employment.
Read the following three passages and solve the items (b) It helps in upward mobility and acquiring
that follow each passage. Your Solutions to these items social status.
should be based on the passages only. (c) It is an inner process of moral and intellectual
development
Passage 1
(d) All the a), b) and c) given above are correct in
Education, without a doubt, has an important functional,
this context.
instrumental and utilitarian dimension. This is revealed
Sol. (c)
when one asks questions such as ‘what is the purpose
of education?’. The Solutions, too often, are ‘to acquire 18. Education is a process in which
qualifications for employment/upward mobility’, (a) Students are converted into trained
‘wider/higher (in terms of income) opportunities’, and professionals.
‘to meet the needs for trained human power in diverse (b) Opportunities for higher income are generated
field for national development’. But in its deepest sense (c) individuals develop self-critical awareness
education is not instrumentalist. That is to say, it is not and independence of thought
to be justified outside of itself because it leads to the (d) qualifications for upward mobility are
acquisition of formal skills or of certain desired acquired
psychological – social attributes. It must be respected in
Sol. (c)
itself. Education is thus not a commodity to be acquired
or possessed and then used, but a process of inestimable This has been clearly mentioned in the last few lines.
importance to individuals and society, although it can Passage 2
and does have enormous use value. Education then, is a
Chemical pesticides lose their role in sustainable
process of expansion and conversion, not in the sense
agriculture if the pests evolve resistance. The evolution
of conversion turning students into doctors or engineers,
of the pesticide resistance is simply natural selection in
but the widening and turning out of the mind – the
action. It is almost certain to occur when vast numbers of
creation, sustenance and development of self-critical
a genetically variable population are killed. One or a few
awareness and independence of thought. It is an inner
individuals may be unusually resistant (perhaps because
process of moral-intellectual development.
they possess an enzyme that can detoxify the pesticide).
16. What do you understand by the ‘instrumentalist’ If the pesticide is applied repeatedly, each successive
view of education? generation of the pest will contain a larger proportion of
(a) Education is functional and utilitarian in its resistant individuals. Pests typically have a high intrinsic
purposes. rate of reproduction, and so a few individuals in one

CSAT by Manjul Kumar Tiwari Sir


26 CSAT : 2012

generation may give rise to hundreds or thousands in (d) None of the statements a), b) and c) given above
the next, and resistance spreads very rapidly in a is correct.
population. Sol. (c)
This problem was often ignored in the past, even though Option (a) and (b) are incorrect as they are nowhere
the first case of DDT (dichlorodiphenyltrichloroethane) mentioned in the passage. Option (c) is correct
resistance was reported as early as 1946. There is an because it follows the logic of evolution and natural
exponential increase in the numbers of invertebrates that selection.
have evolved resistance and in the number pesticides 20. With reference to the passage, consider the
against which resistance has evolved. Resistance has following statements:
been recorded in every family of arthropod pests 1. Use of chemical pesticides has become
(including dipterans such as mosquitoes and house flies, imperative in all the poor countries of the
as well as beetles, moths, wasps, fleas, lice and mites) as world.
well as in weeds and plant pathogens. Take the Alabama 2. Chemical pesticides should not have any role
leaf worm, a moth pest of cotton, as an example. It has in sustainable agriculture
developed resistance in one or more regions of the world 3. One pest can develop resistance to many
to aldrin, DDT, dieldrin, endrin, lindane and toxaphene. pesticides
If chemical pesticides brought nothing but problems, - if Which of the statements given above is/are
their use was intrinsically and acutely unsustainable – correct?
then they would already have fallen out of widespread (a) 1 and 2 only (b) 3 only
use. This has not happened. Instead, their rate of (c) 1 and 3 only (d) 1, 2 and 3
production has increased rapidly. The ratio of cost to Sol. (b)
benefit for the individual agricultural producer has
Statements 1 and 2 are extreme statements and out
remained in favour of pesticide use. In the USA,
of context generalizations. So the answer has to be
insecticides have been estimated to benefit the agricultural
(b).
products to the tune of around $5 for every $1 spent.
21. Though the problems associated with the use of
Moreover, in many poorer countries, the prospect of
chemical pesticides is known for a long time, their
imminent mass starvation, or of an epidemic disease, are
widespread use has not waned. Why?
so frightening that the social and health costs of using
(a) Alternatives to chemical pesticides do not
pesticides have to be ignored. In general the use of
exist at all.
pesticides is justified by objective measures such as ‘lives
(b) New pesticides are not invented at all.
saved’, ‘economic efficiency of food production’ and ‘total
(c) Pesticides are biodegradable.
food produced’. In these very fundamental senses, their
(d) None of the statements a), b) and c) given above
use may be described as sustainable. In practice,
is correct.
sustainability depends on continually developing new
Sol. (d)
pesticides that keep at least one step ahead of the pests –
pesticides that are less persistent, biodegradable and more Consider the last paragraph “Moreover, in many
accurately targeted at the pests. poorer countries, the prospect of imminent mass
starvation, or of an epidemic disease, are so
19. “The evolution of pesticide resistance is natural frightening that the social and health costs of using
selection in action.” What does it actually imply? pesticides have to be ignored. In general the use of
(a) It is very natural for many organisms to have pesticides is justified by objective measures such as
pesticide resistance. ‘lives saved’, ‘economic efficiency of food
(b) Pesticide resistance among organisms is a production’ and ‘total food produced’.”
universal phenomenon. So, none of the options are correct.
(c) Some individuals in any given population
show resistance after the application of 22. How do pesticides act as agents for the selection
pesticides of resistant individuals in any pest population?

CSAT by Manjul Kumar Tiwari Sir


Previous Year Solved Papers 27
1. It is possible that in a pest population the (c) There is no scope for the improvement of
individuals will behave differently due to their pesticides and making their use sustainable
genetic makeup. (d) Both the statements a) and b) above are correct.
2. Pests do possess the ability to detoxify the Sol. (d)
pesticides.
Passage 3
3. Evolution of pesticide resistance is equally
distributed in pest population. Today’s developing economies use much less energy per
capita than developed countries such as the United State
Which of the statements given above is/are did at similar incomes, showing the potential for lower-
correct? carbon growth. Adaptation and mitigation need to be
(a) 1 only (b) 1 and 2 only integrated into a climate-smart development strategy that
(c) 3 only (d) 1, 2 and 3 increases resilience, reduces the threat of further global
Sol. (b) warming, and improves development outcomes.
Adaptation and mitigation measures can advance
Statement 1 has been mentioned clearly in the
development, and prosperity can raise incomes and
passage and is the very base on which the whole
foster better institutions. A healthier population living
passage stands.
in better – built houses and with access to bank loans
23. Why is the use of chemical pesticides generally and social security is better equipped to deal with a
justified by giving the examples of poor and changing climate and its consequences. Advancing
developing countries? robust, resilient development policies that promote
1. Developed countries can afford to do away adaptation is needed today because changes is the
with use of pesticides by adapting to organic climate, already begun, will increase even in the short
term.
farming, but it is imperative for poor and
developing countries to use chemical The spread of economic prosperity has always been
pesticides. intertwined with adaptation to changing ecological
2. In poor and developing countries, the conditions. But as growth has altered the environment
pesticide addresses the problem of epidemic and as environmental change has accelerated, sustaining
diseases of crops and eases the food problem. growth and adaptability demands greater capacity to
3. The social and health costs of pesticide use understand our environment, generate new adaptive
are generally ignored in poor and developing technologies and practices, and diffuse them widely. As
countries. economic historians have explained, much of
humankind’s creative potential has directed at adapting
Which of the statements given above is/are correct? to the changing world. But adaptation cannot cope with
(a) 1 only (b) 1 and 2 only all the impacts related to climate change, especially as
(c) 2 only (d) 1, 2 and 3 larger changes unfold in the long term.
Sol. (c)
Countries cannot grow out of harm’s way fast enough to
Statement 3 wrong mentions both poor and match the changing climate. And some growth strategies,
developing countries have to ignore the costs. In whether driven by the government or the market, can
the passage, its only poor countries that ignore the also add to vulnerability – particularly if they over exploit
cost. natural resources. Under the Soviet development plan,
Statement 1’s first part is out of context irrigated cotton cultivation expanded in water-stressed
generalizations. Central Asia and led to the near disappearance of the
Aral Sea, threatening the livelihoods of fishermen, herders
24. What does the passage imply? and farmers. And clearing mangroves- the natural
(a) Alternative options to chemical pesticides coastal buffers against storm surges – to make way for
should be promoted. intensive farming or housing development, increases the
(b) Too much use of chemicals is not good for the physical vulnerability of coastal settlements, whether in
ecosystem. Guinea or in Louisiana.

CSAT by Manjul Kumar Tiwari Sir


28 CSAT : 2012

25. Which of the following conditions of growth can Select the correct Solution: using the codes given
add to vulnerability? below:
1. When the growth occurs due to excessive (a) 1 only (b) 2 and 3 only
exploitation of mineral resources and forests (c) 1 and 3 only (d) 1, 2, and 3
2. When the growth brings about a change in Sol. (b)
humankind’s creative potential. Second paragraph mentions “But as growth has
3. When the growth is envisaged only for altered the environment....technologies and
providing houses and social security to the practices and diffuse them widely.” Hence,
people. statements 2 and 3 are correct. Answer is (b).
4. When the growth occurs due to emphasis on
farming only. Select the correct Solution: using 28. Which of the following inferences can be made
the codes given below: from the passage?
(a) 1 only (b) 2, 3 and 4 only 1. Rainfed crops should not be cultivated in
(c) 1 and 4 only (d) 1, 2, 3 and 4 irrigated areas
Sol. (a) 2. Farming under water-deficient areas should
not be a part of development strategy.
In the last paragraph, “overexploitating natural
resources” is mentioned as one of the factors adding Select the correct Solution: using the codes given
to vulnerability. Statement 2 and 4 mentioned in below:
the passage but not in this context. Statement 3 with (a) 1 only (b) 2 only
“social security” is incorrect. So, only statement 1 (c) Both 1 and 2 (d) Neither 1 nor 2
is correct. Sol. (d)
26. What does low-carbon growth imply in the present See again here both the statements are extreme
context? generalizations. The passage mentions only a
1. More emphasis on the use of renewable specific case related to both the statements. Hence,
sources of energy. both are wrong.
2. Less emphasis on manufacturing sector and 29. Consider the following assumptions:
more emphasis on agricultural sector. 1. Sustainable economic growth demands the
3. Switching over from monoculture practices use of creative potential of man.
to mixed farming 2. Intensive agriculture can lead to ecological
4. Less demand for goods and services. backlash.
Select the correct Solution: using the codes given 3. Spread of the economic prosperity can
below: adversely affect the ecology and environment
(a) 1 only
With reference to the passage, which of the above
(b) 2, 3 and 4 only
assumptions is/are valid?
(c) 1 and 4 only
(a) 1 only (b) 2 and 3 only
(d) None of the above implies low-carbon growth
(c) 1 and 3 only (d) 1, 2 and 3
Sol. (a)
Sol. (d)
Renewable energy is generally understood as a
“....much of humankind’s creative potential has
strategy for low carbon growth.
been directed at adapting to the changing world”.
27. Which of the following conditions is/are For e.g. the passage talks about Soviet development
necessary for the sustainable economic growth? plan – cotton cultivation and mangroves of Guinea
1. Spreading of economic prosperity more. etc. This shows statement 1 is true.
2. Popularizing/spreading of adaptive Then “for intensive farming or housing
technologies widely development, increases the physical vulnerability
3. Investing on research in adaptation and of coastal settlements, whether in Guinea or in
mitigation technologies. Louisiana”...shows that statement 2 is true.

CSAT by Manjul Kumar Tiwari Sir


Previous Year Solved Papers 29
Further, paragraph 2 talks about economic One of the major reasons for the world’s great biodiversity
prosperity being intertwined with ecological is the occurrence of centres of endemism so that similar
conditions. Hence, 3 is also true. habitats in different parts of the world are occupied by
different groups of species that happen it have evolved
30. Which one of the following statements constitutes
there. If the species naturally had access to everywhere
the central theme of this passage? on the globe, we might expect a relatively small number
(a) Countries with greater economic prosperity of successful species to become dominant in each biome.
are better equipped to deal with the The extent to which this homogenisation can happen
consequences of climate change. naturally is restricted by the limited powers of dispersal
(b) Adaptation and mitigation should be of most species in the face of the physical barriers that
integrated with development strategies exist to dispersal. By virtue of the transport opportunities
(c) Rapid economic growth should not be offered by humans, these barriers have been breached by
pursued by both developed and developing an ever-increasing number of exotic species.
economies.
The effects of introductions have been to convert a hugely
(d) Some countries resort to over exploitation of
diverse range of local community compositions into
natural resources for the sake of rapid
something much more homogenous.
development.
Sol. (b) It would be wrong, however, to conclude that introducing
species to a region will inevitably cause a decline in
Option (a) is an observation. Option (c) and (d) are
species richness there. For example, there are numerous
extreme generalizations of specific cases mentioned
species of plants, invertebrates and vertebrates found in
in the passage.
continental Europe but absent from the British Isles
Option (b) is the theme.
(many because they have so far failed to recolonize after
Directions for the following 11 questions: the last glaciations). Their introduction would be likely
to augment British biodiversity. The significant
Read the following three passages and solve the items detrimental effect noted above arises where aggressive
that follow each passage. Your Solutions to these passages species provide a novel challenge to endemic biotas ill-
should be based on these passages only. equipped to deal with them.
Passage - 1 31. With reference to the passage, which of the
Invasions of exotic species into new geographical areas following statements is correct?
sometimes occur naturally without human agency. (a) Introduction of exotic species into new
However, human actions have increased this trickle to a geographical areas always leads to reduced
flood. Human caused introductions may occur either biodiversity.
accidently or as a consequence of human transport, or (b) Exotic species introduced by man into new
intentionally but illegally to serve some private purpose areas have always greatly altered the native
or legitimately to procure some hoped-for public benefit ecosystems.
by bringing a pest under control, producing new (c) Man is the only reason to convert a hugely
agricultural products or providing novel recreational diverse range of local community
opportunities. Many introduced species are assimilated compositions into more homogenous ones.
into communities without much obvious effect. However (d) None of the statements (a), (b), and (c) is correct
some have been responsible for dramatic changes to in this context.
native species and native communities. For example, the Sol. (d)
accidental introduction of the brown tree snake, Boiga All three statements are extreme generalizations
irregularis into Guam, an island in the pacific, has through based on specific references from the passage. For
nest predation reduced 10 endemic forest bird species to e.g. “However, SOME (introductions) have been
the point of extinction. responsible for dramatic changes in native species.”

CSAT by Manjul Kumar Tiwari Sir


30 CSAT : 2012

32. Why does man introduce exotic species into new 35. What can be the impact of invasion of exotic
geographical areas? species on an ecosystem?
1. To bread exotic species with local varieties. 1. Erosion of endemic species.
2. To increase agricultural productivity. 2. Change in the species composition of the
community of the ecosystem
3. for beautification and landscaping
Which of the above statements is/are correct? Select the correct Solution: using the codes given
(a) 1 only (b) 2 and 3 only below:
(a) 1 only (b) 2 only
(c) 1 and 3 only (d) 1, 2 and 3
(c) Both 1 and 2 (d) Neither 1 nor 2
Sol. (d)
Sol. (c)
The paragraph: “serve some private purpose or
As per the first paragraph invasion of exotic species
legitimately to procure some hoped-for public
has led to dramatic changes to native species and
benefit by bringing a pest under control, producing
natural communities. Hence, statement 1 is correct.
new agricultural products or providing novel
recreational opportunities”. is reflected in The last few lines of the passage state that the
option (d). introduction of exotic species has a detrimental
effect and poses a challenge to endemic biotas ill-
33. How is homogenization prevented under natural equipped to deal with them. Hence, S2 is also
conditions? correct.
(a) Evolution of groups of species specific to local Passage - 2
habitats.
(b) Presence of oceans and mountain ranges Most champions of democracy have been rather reticent
in suggesting that democracy would itself promote
(c) Strong adaptation of groups of specific to local
development and enhancement of social welfare – they
physical and climatic conditions
have tended to see them as good but distinctly separate
(d) All the statements (a), (b) and (c) given above
and largely independent goals. The detractors of
are correct in this context.
democracy, on the other hand, seemed to have been quite
Sol. (d)
willing to express their diagnosis of what they see as
34. How have the human beings influenced the serious tensions between democracy and development.
biodiversity? The theorists of the practical spirit - “Make up your mind:
1. By smuggling live organism do you want democracy, or instead, do you want
development?”- often came ,at least to start with, from
2. By building highways
East Asian countries, and their voice grew in influence
3. By making ecosystems sensitive so that new
as several of these countries were immensely successful
species are not allowed
– through the 1970s and 1980s and even later – in
4. By ensuring that new species do not have
promoting economic growth without pursuing
major impact on local species.
democracy.
Which of the statements given above are correct?
To deal with these issues we have to pay particular
(a) 1 and 2 (b) 2 and 3
attention to both the content of what can be called
(c) 1 and 3 (d) 2 and 4 development and to the interpretation of democracy (in
Sol. (a) particular to the respective roles of voting and of public
As per the first paragraph, “Human-caused reasoning). The assessment of development cannot be
introductions may occur either accidentally as a divorced from the lives that people can lead and the real
consequence of human transport, or intentionally freedom that they enjoy. Development can scarcely be
but illegally to serve some private purpose.” Hence, seen merely in terms of enhancement of inanimate objects
point 1- smuggling (illegal) and point 2 – building of convenience, such as a rise in the GNP (or in personal
highways (transport) are correct. incomes), or industrialisation – important as they may

CSAT by Manjul Kumar Tiwari Sir


Previous Year Solved Papers 31
be as means to the real ends. Their value must depend on (c) Rise in the savings and consumption trends.
what they do to the lives and freedom of the people (d) Extent of real freedom that citizens enjoy.
involved, which must be central to the idea of Sol. (d)
development. As per the second paragraph, “The assessment of
If development is understood in a broader way, with a development cannot be divorced from the lives that
focus on human lives, then it becomes immediately clear people can lead and the real freedom that they
that the relation between development and democracy enjoy.” Hence, (d).
has to be seen partly in terms of their constitutive
connection, rather than only through their external links. 38. What does a “constitutive” connection between
Even though the question has often been asked whether democracy and development imply?
political freedom is “conductive to development”, we (a) The relation between them has to be seen
must not miss the crucial that political liberties and through external links.
democratic rights are among the “constitutive (b) Political and civil rights only can lead to
components” of development does not have to be economic development
established indirectly through their contribution to the (c) Political liberties and democratic rights are
growth of GNP. essential elements of development.
(d) None of the statements (a), (b) and (c) given
36. According to the passage, why is a serious tension above is correct in this context.
perceived between democracy and development
Sol. (c)
by the detractors of democracy?
As per the last paragraph, “….we must not miss the
(a) Democracy and development are distinct and
crucial recognition that political liberties and
separate goals
democratic rights are among the ‘constituent
(b) Economic growth can be promoted
components’ of development.” So, answer is (c).
successfully without pursuing a democratic
system of governance Passage - 3
(c) Non-democratic regimes deliver economic
The need for Competition Law becomes more evident
growth faster and far more successfully than
when foreign direct investment (FDI) is liberalised. The
democratic ones.
impact of FDI is not always pro-competitive. Very often
(d) All the statements (a), (b) and (c) given above
FDI takes the form of a foreign corporation acquiring a
are correct in this context.
domestic enterprise or establishing a joint venture with
Sol. (b) one. By making such an acquisition the foreign investor
Option (a): this is mentioned by champions of may substantially lessen competition and gain a
democracy, not detractors. So, it is wrong which dominant position in the relevant market, thus charging
automatically makes (d) also wrong. higher prices. Another scenario is where the affiliates of
While passage denotes that several non-democratic two separate multinational companies (MNCs) have
regimes in East Asia have delivered growth been established in competition with one another in a
successfully; it does not say generally that non- particular developing economy, following the
democratic regimes deliver faster growth. Hence, liberalisation of FDI. Subsequently, the parent companies
its an extreme generalization. So (c) is wrong. overseas merge. With the affiliates no longer remaining
independent, competition in the host country may be
(b) can be observed from the last lines of the first
artificially inflated. Most of these adverse consequences
paragraph.
of mergers and acquisitions by MNCs can be avoided if
37. According to the passage, what should be the an effective competition law is in place. Also, an economy
ultimate assessment/aim/view of development? that has implemented an effective competition law is in
(a) Rise in the per capita income and industrial a better position to attract FDI than one that has not. This
growth rates. is not just because most MNCs are expected to be
(b) Improvement in the Human Development accustomed to the operation of such a law in their home
Index and GNP. countries and know how to deal with such concerns but

CSAT by Manjul Kumar Tiwari Sir


32 CSAT : 2012

also that MNCs expect competition authorities to ensure 41. What is the inference from this passage?
a level playing field between domestic and foreign firms. (a) Foreign investors and multinational
companies always dominate domestic
39. With reference to the passage, consider the
market.
following statements:
(b) It is not in the best interest of domestic
1. It is desirable that the impact of Foreign Direct
economy to allow mergers company.
investment should be pro-competitive.
(c) With competition law, it is easy to ensure a
2. The entry of foreign investors invariably leads
level playing field between domestic and
to the inflated prices in domestic markets.
foreign firms.
Which of the statements given above is/are (d) For countries with open economy Foreign
correct? Direct investment is essential for growth.
(a) 1 only (b) 2 only Sol. (c)
(c) Both 1 and 2 (d) Neither 1 nor 2 Options (a) and (b) are extreme generalizations.
Sol. (a) Option (d) has been quoted out of context. Hence,
the answer is (c) which is the essence of passage.
The sentence “The need for Competition Law
becomes more evident when foreign direct 42. Examine the following statements:
investment (FDI) is liberalised. The impact of FDI is 1. I watch TV only if I am bored
not always pro-competitive.” clearly points to 2. I am never bored when I have my brother’s
state.1 company.
Statement 2 is valid only when the parent 3. Whenever I go to the theatre I take my brother
companies merge overseas. along.

40. According to the passage, how does a foreign Which one of the following conclusions is valid in
investor dominate the relevant domestic market? the context of the above statements?
1. Multinational companies get accustomed to (a) If I am bored I watch TV
domestic laws. (b) If I am bored, I seek my brother’s company.
2. Foreign companies establish joint ventures (c) If I am not with my brother, than i’ll watch
with domestic companies. TV.
(d) If I am not bored I do not watch TV.
3. Affiliates in a particular market/sector lose
Sol. (d)
their independence as their parent companies
overseas merge. The logical flow diagram is as follows
4. Foreign companies lower the cost of their
products as compared to that of products of
domestic companies.

Which of the statements given above are correct?


(a) 1 and 2 only
(b) 2 and 3 only
(c) 1, 2 and 3 only Here being bored or X or Y is leading to the action
“Watch TV”. But if the person is watching TV, it
(d) 1, 2, 3 and 4
does not mean that he was necessarily bored. The
Sol. (b)
cause could have been X or Y or anything. So, option
Statement 1 is favourable to competition. (a) is incorrect. But, at the same time, he will not
Statement 4 as been quoted out of the passage. watch TV, if he is note bored.
Hence, only 2 and 3 are correct as they have been Same diagram can be visualized for the relation
directly stated in the passage. between “not being bored” and “being in brother’s

CSAT by Manjul Kumar Tiwari Sir


Previous Year Solved Papers 33
company “. His brother’s company is not the only In Venn diagrams, sometimes there is more than
way in which he can escape boredom. This tells one possibility. For e.g. here possibility 1 is:
that (c) is wrong because he may not be bored when (that people who dance may not be club members
he doesn’t have his brother’s company. at all)

43. Only six roads A, B, C, P, Q and R connect a


Members
military camp to the rest of the country. Only
of Club
one out of A, P, and R are open at any one time.
Students
If B is closed so is Q. Only one of A & B is open
during storms. P is closed during floods. In the
context, which one of the following statements is
correct?
(a) Under normal conditions only three roads are
open.
Dance Married
(b) During storms at least one road is open.
(c) During floods only three roads are open Possibility 2: Those who dance can be club members
(d) During calamities all roads are closed or even married students.
Sol. (b)
In this of first option case, if R is open, then B, C and Club members
Q can also be open. 4 roads can be open. So its

d
rie
wrong.
ar
M
In case of second option - In storm either A or B is Students
open. Hence, it has to be correct.
Invited
In case of third option - P is closed during flooding.
for the dance
Therefore, if A is open then neither of B and Q will
Now, check options individually.
be open. So, at most two roads can be open. So, this
may or may not be true. Option (a) is wrong, because not all students are
In case of 4th option- From option 2, we know that married, so, cannot be invited for dance.
at least one road is open. Hence, this is incorrect. Option (b) is also wrong, because all married
In all three given situations - Normal, Storms and students may not be the member of the club. Its true
Floods, there is no closed condition on road C. that only students are the members; but it does not
Hence, (b). mean every student is a club member (this is clearly
shown in the Venn diagrams).
44. Examine the following statements:
1. None but students are the members of the club. Option (c) is wrong because clearly only some are
2. Some members of the club are married. married in the club. So, correct option is (d).
3. All married persons are invited for dance. 45. Four political parties W, X, Y and Z decided to set
up a joint candidate for the coming parliamentary
Which one of the conclusions can be drawn from
elections. The formula agreed by them was the
the above statements?
acceptance of a candidate of the most of the parties.
(a) All students are invited for dance
For aspiring candidates A, B, C and D approached
(b) All married students are invited for dance
the parties for their ticket.
(c) All members of the club are married person
(d) None of the above conclusions can be drawn A was acceptable to W but not Z
Sol. (b) B was acceptable to Y but not X
This is a typical syllogism question. Draw Venn C was acceptable to W and Y
diagrams to solve easily or use logic, whichever
D was acceptable to W and X
way you feel comfortable.

CSAT by Manjul Kumar Tiwari Sir


34 CSAT : 2012

When candidate B was preferred by W and Z, 47. Consider the following statements:
candidate C was preferred by X and Z and The Third World War, if it ever starts will end very
candidate A was acceptable to X but not Y: Who quickly with possible end of civilization. It is only
got the ticket? the misuse of nuclear power which will trigger it.
(a) A (b) B Based on the above statement which one of the
following inferences is correct?
(c) C (d) D
(a) Nuclear power will be used in Third World
Sol. (c) War.
As it is given, A is accepted by W, Z and X. (b) There will be no civilization left after the third
world war.
B accepted by Y, W and Z.
(c) The growth of nuclear power will destroy
C accepted by W, Y, X and Z.
civilization in the long run.
D accepted by W and X. (d) The third world war will not take place.
So, C can be accepted by all the four parties. Solution: a)

46. Consider the following statements: The passage very clearly mentions that only the
misuse of nuclear power will trigger world war. So
1. All X-brand cars parked here are white. nuclear power has to be used in the third world
2. Some of them have radial tyres war.

3. All X-brand cars manufactured after 1986 have 48. Figures given below are changing with certain
radial tyres are parked here. rules as we observe them from left to right:

4. All cars are not X-brand.

Which one of the following conclusions can be


drawn from the above statements?
According to this rule which of the following
(a) Only white cars are parked here. would be the next figure if the changes were made
in the same rule?
(b) Some white X-brand cars with radial tyres are
parked here.

(c) Cars other than X-brand cannot have radial


tyres.

(d) Most of the X-brand cars are manufactured


before 1986.

Sol. (b)

Consider option (a): Its wrong because only X-brand


cars parked here are white.

Consider option (b): All X-brand cars are white and


some have radial tyres which means its correct. Sol. (d)

Consider option (c): Its an out of the league Observe the triangle movement. See the triangle is
statement. moving corners anti-clockwise and turning upside
down successively. So in the answer figure it has to
Consider option (d): An illogical and out of the be at the left top and vertical up. Only one option
context statement. (d) satisfies this condition.

CSAT by Manjul Kumar Tiwari Sir


Previous Year Solved Papers 35
49. Consider the following information regarding the In the above figure, circle P represents
performance of class of 1000 students in four hardworking people, circle Q represents intelligent
different tests: people, Circle R represents truthful people and
circle S represents honest people. Which region
represents the people who are intelligent, honest
and truthful but not hardworking?
(a) 6 (b) 7
(c) 8 (d) 11
Sol. (a)
Choose a sector that excludes P, but includes all
others Q, R and S. It is 6.
If a student scores 74 marks in each of the four
52. Three views of a cube following a particular motion
tests, in which one of the following tests is her
are given below:
performance the best comparatively?
(a) Test I (b) Test II
(c) Test III (d) Test IV
Sol. (b)
If she scores 74 marks, we can assume that this will
be her best performance. S if he scored 74 marks in
a subject which has least highest marks and the What is letter opposite to A?
least higher range of marks. Test II satisfies this best. (a) H (b) P
(c) B (d) M
50. Six squares are coloured, front and back, red(R),
Sol. (a)
blue(B), yellow (Y), green (G), white (W), orange
(O) and are hinged together as shown in the figure After observing first two figures, A, B, H and M are
given below. If they were folded to form a cube adjacent to K. So, letter opposite to A has to be either
what would be the face opposite to white face? H or M.

Now, there are two conditions. One, if M is opposite


to A, then H will be opposite to B. But, by third
figure, we see it is not possible.

Hence, H has to be opposite to A.

(a) R (b) G 53.


(c) B (d) O
Sol. (c)
If you take G as the base, these will be opposite
faces: GO, YR, BW. Even if you take Y as the base,
you will find the same opposite faces.

51.

Which one of the figures shown below occupies


the blank space(?) in the matrix given below?

CSAT by Manjul Kumar Tiwari Sir


36 CSAT : 2012

S3: If frustrated people MAY become drug addicts


(not all), then Frustrated (F) can be a smaller circle
(a) (b)
cutting the drug circle.

(c) (d)

Sol. (d)
In first and third set, the black portion of middle Now, check the options.
image is diagonally opposite to that of first image. Option (a) can be dismissed easily.
This is the pattern. So, second image in second set
Option (b) has to be correct, because no matter what
must be diagonally opposite to the first one.
the drug circle has to cut the whimsical circle.
Hence, (d).
Option (c) too can be dismissed based on the
54. Consider the following statements: diagram.
1. All artists are whimsical. Option (d) is also wrong, because the diagram does
2. Some artists are drug addicts. not support it.
3. Frustrated people are prone to become drug
addicts. 55. Examine the following statements:
1. Either A & B are of same age or A is older
From the above three statements it may be
than B
concluded that:
2. Either C & D are of same age or D is older
(a) Artists are frustrated
than C
(b) Some drug addicts are whimsical
3. B is older than C
(c) All frustrated people are drug addicts.
(d) Whimsical people are generally frustrated Which of the following conclusions can be drawn
Sol. (b) from the above statements?
S1: If all artists are whimsical, it means that (a) A is older than B
whimsical is a big category (or circle) and all artists (b) B and D are of the same age
will be one small circle in that. (c) D is older than C
(d) A is older than C
Sol. (d)
From S1: A  B from S2: D  C from S3: B > C
Now if B (which is either equal or older than A) is
greater than C, then A has to be older than C.

56. Examine the following statements:


1. Only those who have a pair of binoculars can
S2: If some artists are drug addicts, draw a circle become the member of the birdwatcher’s club.
that cuts the circle of Drug addicts. Now, that circle 2. Some members of the birdwatcher’s club have
can either be completely inside whimsical OR cameras.
outside it cutting it. 3. Those members who have cameras can take
part in photo-contests.

Which of the following conclusions can be drawn


from the above statements?
(a) All those who have a pair of binoculars are
members of the birdwatcher’s club.

CSAT by Manjul Kumar Tiwari Sir


Previous Year Solved Papers 37
(b) All members of the birdwatcher’s club have a Sol. (b)
pair of binoculars. Shirt Pant
(c) All those who take part in photo-contests are A - Black Green
members of the birdwatcher’s club. B - Blue Orange
(d) No conclusion can be drawn.
C - Orange
Sol. (b)
Now, from the above information, this can be the
Option 1 and 3 are general and not necessarily true.
deduced table.
But option 2 is a correct conclusion as all the club
Shirt Pant
members definitely have a pair of binoculars.
A - Black Blue/Orange Green, Yellow
57. During the summer vacation Ankit went to a B - Blue Black/Orange Orange
summer camp where he took part in hiking,
C - Orange Blue Green
swimming and boating. This summer, he is looking
forward to a music camp where he hopes to sing, Answer is green and blue.
dance and learn to play the guitar. 59. Ten new TV shows started in January- 5 sitcoms, 3
Based on the above information four conclusions drama and 2 news magazines. By April, only seven
as given below, have been made. Which one of of the new shows were still on, five of them being
these logically follows from the information given sitcoms.
above? Based on the above information, for conclusions,
(a) Ankit’s parents want him to play guitar. as given below, have been made. Which of these
(b) Ankit prefers music to outdoor activities. logically follows from the information given above?
(c) Ankit goes to some type of camp every summer. (a) Only one news magazine show is still going
on.
(d) Ankit likes to sing and dance
(b) Only one of the drama show is still going on.
Sol. (d)
(c) At least one discontinued show was a drama.
Parents are not even in the picture. So, no to (a).
(d) Viewers prefer sitcoms over drama.
(b) conclusion cannot be made.
Sol. (c)
(c) would be too general to say, because the
paragraph talks only about two summer camps that Three shows were discontinued as given. Since no
he has been to. sitcoms shows were discontinued, the discontinued
ones must be either drama or news magazines. Now,
(d) is the choice left which is evident from his joining
there were only two news magazines. So, atleast
the camps.
one discontinued show was a drama.
58. Three persons A, B & C wear shirts of Black, Blue
and Orange colours (not necessarily in the order) 60. Read the passage given below and the two
and pants of green, yellow and orange (not statements that follow (given on the basis of the
necessarily in that order). No person wore shirt and passage):
pant of the same colour. Further, it is given that,
Four men are waiting at Delhi airport for Mumbai
1. A did not wear shirt of black colour.
flight. Two are doctors and other two are
2. B did not wear shirt of blue colour. businessman. Two speak Gujarati and two speak
3. C did not wear shirt of orange colour. Tamil. No two of the same profession speak the
4. A did not wear the pants of green colour same language. Two are Muslims and two are
5. B wore pants of orange colour. Christians. No two of the same religion are of the
What were the colours of the pants and shirts worn same profession, nor do they speak same language.
The Tamil speaking doctor is Christian.
by C respectively?
(a) Orange and black (b) Green and blue 1. The Christian-Businessman speaks Gujarati.
(c) Yellow and blue (d) Yellow and black 2. The Gujarati-speaking doctor is a Muslim.

CSAT by Manjul Kumar Tiwari Sir


38 CSAT : 2012

Which of the above statements is/are correct Which means 0.2 x N is covered in 10 minutes i.e.,
conclusion/conclusions? 1/6 hr Hence, Average speed = distance/time =
(a) 1 only (b) 2 only 0.2 × N /(1/6) = 48
(c) Both 1 & 2 (d) Neither 1 nor 2  x = 48/6 x 0.2 =40 km
Sol. (c)
63. Gita is prettier than Sita but not as pretty as Rita.
Assume these are the 4 people - Doc 1, Doc 2,
Then,
Business 1, Business 2
(a) Sita is not as pretty as Gita
Now, if Doc 2(Tamil, Christian), then Doc 1 (has to
(b) Sita is prettier than Rita
be Muslim, Gujarati) and Business 1 has to be
(c) Rita is not as pretty as Gita
(Gujarati, Christian); so Business 2 will be (Muslim,
(d) Gita is prettier than Rita
Tamil)
Sol. (a)
61. Consider the following statement: R>G>S
“Though quite expensive, television is not a luxury Options (b), (c) and (d) are false.
item, as one can learn many things through
64. Given that,
television”.
1. A is the brother of B
Which of the following is the valid inference from 2. C is the father of A.
the above given statement? 3. D is brother of E.
(a) All expensive things are regarded as luxury. 4. E is the daughter of B then, the uncle of D is?
(a) A (b) B
(b) All essential things for learning are not
(c) C (d) E
luxury.
Sol. (a)
(c) Television is essential for learning.
(d) Television is not a luxury item.
Sol. (d)
Check option (a): Too general.
Option (b): Again a general statement. Although
TV is a learning item and not a luxury, even then
TV is not essential for learning. We cannot infer
this option.
Uncle of D is A.
Option (c): Clear from (b) that its wrong. Choice has
to be (d). 65. Examine the following statements:

62. Mr. Kumar drives to work at an average speed of 1. Rama scored more than Rani
48km/hr. The time taken to cover the first 60% of 2. Rani scored less than Ratna
the distance is 10 minutes more than the time taken 3. Ratna scored more than Rama
to cover the remaining distance. How far is his 4. Padma scored more than Rama but less than
office? Ratna. Who scored the highest?
(a) 30 km (b) 40 km (a) Rama (b) Padma
(c) 45 km (d) 48km (c) Rani (d) Ratna
Sol. (b) Sol. (d)
Let the distance be N. S1: Rama > Rani
First 60% distance is covered in 10 more minutes S2: Ratna > Rani
more time than that of the rest 40% distance S3: Ratna>...>Rama> Rani
So, 0.6 x N distance is covered in 10 more minutes S4: Ratna>Padma>Rama>Rani Ratna scored the
more than that of the rest 0.4 x N distance highest.

CSAT by Manjul Kumar Tiwari Sir


Previous Year Solved Papers 39
Directions for the following eight items: 68. The author of the passage seems to suggest that
The following eight items (questions 66 to 73) are based (a) It is possible to adjust one-self to uncongenial
on three passages in English to test the comprehension surroundings.
of the English language and therefore these items do not (b) The conditions in Indian prisons are not bad
have Hindi version. Read each item and Solution: the
(c) It is not difficult to spend one’s time in prison
items that follow.
(d) There is a need to improve the conditions in
Passage-1 our jails.
For fourteen and half months I lived in my little cell or Sol. (a)
room in Dehradun jail, and I began to feel as if I was
The prisoner familiarized himself with his
almost a part of it. I was familiar with every bit of it, I
surroundings. He befriended his natural
knew every mark and dent on the whitewashed walls
environment as a means of survival and to
and on the uneven floors and on the ceiling with the
overcome his aloofness. So most appropriate
moth eaten rafters. In the little yard outside I greeted little
option is (a).
tufts of grass and odd bits of stone as old friends. I was
not alone in my cell, for several colonies of wasp and Passage-2
hornets lived there, and many lizards found the home
behind the rafters, emerging in the evening in the search We started pitching the highest camp that has been ever
of prey. made. Everything took five times as long as it would have
taken in the place where there was enough air to breathe;
66. Which of the following explains best the sentence but at last we got tent up, and when we crawled in, it
in the passage “I was almost a part of it”?
was not too bad. There was only a light wind and inside
(a) I was not alone in the cell
it was not too cold for us to take off our gloves. At night
(b) I was familiar with every bit of the cell
most climbers take off their boots; but I prefer to keep
(c) I greeted little tufts of grass like old friends.
them on. Hilary, on the other hand took his off and lain
(d) I felt quite at home in the cell
them next to his sleeping bag.
Sol. (b)
(a) and (c) are not convincing arguments to explain 69. What does the expression “pitching the highest
the question statement. camp” imply?
Option (d) is an extrapolation of what is given in (a) They reached the summit of the highest
the passage. but (b) option is more objective, specific mountain in the world.
and mentioned in the passage. (b) Those who climbed that far earlier did not
pitch any camp.
67. The passage attempts to describe:
(a) The general conditions of the country’s jail (c) So far nobody climbed that high.
(b) The prisoner’s capacity to notice the minute (d) They were too many climbers and needed to
details of his surroundings pitch a big camp
(c) The prisoner’s conscious efforts to overcome Sol. (c)
the loneliness
It can’t be the summit because they are talking about
(d) The prisoner’s ability to live happily with other
creatures the highest camp that has ever been made (by
humans). (b) is also inappropriate. And (d) is an
Sol. (c)
absurd statement. (c) is the right choice.
(a) is not correct
(b) is one of the elements of the passage, not what 70. They took a long time to finish the work because:
the passage attempts to describe. (d) is a subjective
(a) They were very tired.
view of what has been mentioned in the passage.
(b) There was not enough air to breathe
Moreover, there were other things too that the
prisoner was doing for e.g. noticing the walls. (c) It was very cold
So, best choice would be (c). (d) It was very dark

CSAT by Manjul Kumar Tiwari Sir


40 CSAT : 2012

Sol. (b) Choose only one response for each item. The responses
“Everything took five times as long as it would have will be evaluated based on the level of appropriateness
taken in a place where there was enough air to for the given situation.
breathe.” Means that the place didn’t have enough Please attempt all the items. There is no penalty for wrong
air to breathe. Solutions for these seven items.

71. When they crawled into the tent 74. You have differences of the opinion regarding the
(a) They took off their gloves because it was not final report prepared by your subordinate that is
very cold to be submitted urgently. The subordinate is
(b) They could not take off their gloves because it justifying the information given in the report. You
was very cold. would
(c) They took of their gloves though it was very (a) Convince the subordinate that he is wrong
cold. (b) Tell him to reconsider the results
(d) They did not take off their gloves though it (c) Revise the report on your own
was not very cold.
(d) Tell him not to justify the mistake
Sol. (a)
Solution: More preferred (c); less preferred (a)
Passage-3 The question tests your organizational, managerial
A local man, staying on the top of the floor of an old and administrative skills alongwith emotional
wooden house, was awakened at midnight by fire. Losing intelligence.
his way in the smoke-filled passage, he missed the Since the report is to be submitted urgently, you
stairway and went into another room. He picked a bundle don’t have time for the results to be reconsidered or
to protect his face from fire and immediately fell through to keep arguing with the subordinate. So, best way
the floor below where he managed to escape through a out is (c), if not then (c).
clear doorway. The “bundle” proved to be the baby of
B or D will not lead to any results.
the Mayor’s wife. The “hero” was congratulated.
75. You are competing with your batch-mate for the
72. The man went to another room because
prestigious award to be decided based on an oral
(a) He did not know where the stairway was
presentation. You have been asked by the
(b) The passage was full of smoke
committee to finish on time. Your friend however,
(c) He was extremely nervous
is allowed more than the stipulated time period.
(d) He stumbled on bundle
(a) Lodge a complaint to chairperson against the
Sol. (b)
discrimination.
73. The man was called hero because (b) Not listen to any justification from the
(a) Expressed his willingness to risk his life to committee
save others (c) Ask for withdrawal of your name
(b) Managed to escape from fire (d) Protest and leave the place
(c) Showed great courage in fighting the fire. Solution: More preferred (a); less preferred (d).
(d) Saved a life This question checks capacity and inclination for
Sol. (d) justice, and withstanding odd situations. Option
(b) is a knee-jerk reaction and shows your emotional
The bundle that he had used to save face, was the
intelligence in poor light.
Mayor’s baby, who also got saved in his escaping
from fire. Option (c) shows a lack of interest in fighting for
justice.
Directions for the following 7(seven) items: Going through option (a), you are going by the
Given below are the seven items. Each item describes a official machinery of grievance redressal; fighting
situation and is followed by four possible responses. for justice as well withstanding the odds that will
Indicate the response that you find most appropriate. come in the way.

CSAT by Manjul Kumar Tiwari Sir


Previous Year Solved Papers 41
Option (d) shows the inclination towards fighting Or you can even ask the committee for views. This
for justice, but it is not result-oriented. Leaving the is a more democratic approach since you are
place will not help in any way. heading the committee.

76. You are handling a time bound project. During Other options are a little bit on the extreme side and
the project review meeting, you find that project is not preferable in this case. The reputation of the
likely to get delayed due to lack of cooperation of athlete is also at stake.
the team members. You would, Take a balanced, pragmatic approach.
(a) Warn the team members for their non-
cooperation 78. You are handling a priority project and have been
(b) Look into the reasons for non-cooperation meeting all the deadlines and planning your leave
(c) Ask for the replacement of the team members during the project. Your immediate boss does not
(d) Ask for the extension of time citing reasons grant leave citing the urgency of the project. You
Solution: More preferred (b); less preferred (a) would...
(a) Proceed on leave without waiting for the
The question too tests your organizational,
sanction
managerial and administrative skills along with
(b) Pretend to be sick and take leave
emotional intelligence.
(c) Approach higher authority to reconsider the
Asking for replacement or extension without
leave application.
looking into reasons or talking to team members
(d) Tell the boss that it is not justified.
will be a wrong decision. Moreover, it is a time
bound project. So, options (c) and (d) are out. Solution: More preferable (d); less preferable (c)
(a) is preferable because you will at least get the This question checks for balance in decision-making and
work done. (b) is even more preferable because you handling organizational pulls and pressures on your
will get to the core of the issue, and then ensure that personal life.
the work is completed on time. (a) and (b) are not right for obvious reasons like
dishonesty; disobeying official orders and protocols; and
77. You are a chairperson of a state sports committee.
irresponsible behaviour.
You have received a complaint and later it was
found that an athlete in junior age category who (c) can get you the leave, within the official channels, but
has won a medal has crossed the age criteria by 5 your relationship with immediate boss will be spoiled.
days. You would... He will not like a higher authority overruling him on a
(a) Ask the screening committee for a matter of leave for you, especially when its an urgent
clarification project.
(b) Ask the athlete to return the medal (d) is a more nuanced approach; because its an urgent
(c) Ask the athlete to get an affidavit from the project (and your leave is not all that urgent and
court declaring his/her age. important).
(d) Ask the members of the committee for their
views 79. You are involved in setting up a water supply
project in remote area. Full recovery of cost is
Solution: More preferable (a); less preferable (d).
impossible in any case. The income levels in the
This question checks your administrative acumen
area are low and 25% of the population is below
and sense of balance in decision-making.
poverty line (BPL). When a decision has to be taken
Since the investigation in the case is over, asking on pricing you would....
for clarification from the committee that screened (a) Recommended that the supply of water be free
the athlete is the best way out. You will get to know of charge in all respects
if it was a procedural error or operational error or (b) Recommended that the users pay a onetime
an unintended mistake. Then you can take action. fixed sum for installation of taps and the
Best is (a). usage of water be free

CSAT by Manjul Kumar Tiwari Sir


42 CSAT : 2012

(c) Recommended that a fixed monthly charges (a) Give a bribe


be levied on the non-BPL families and for BPL (b) Behave as if you have not understood the
families water should be free feelers and persist with your application
(d) Recommended that the users pay a charge (c) Go to the higher officer for help verbally
based on the consumption of water with complaining about feelers.
differentiated charges for Non-BPL and BPL (d) Send in a formal complaint
families.
Solution: More preferable (c); less preferable (b)
Solution: (d) more preferable; (c) less preferable.
This checks tolerance for corruption; uprightness;
This question checks your administrative acumen; and integrity.
sense of balance in decision-making and empathy
Sending in a formal complaint will be a wrong step
for the marginalised.
as there is no evidence that the official has asked
Option (a) is implausible/impractical.
you for a bribe.
Option (b) - fixed installation charges will be difficult
Giving a bribe will of course be wrong.
for BPL to pay. And later making it free for everyone
is impractical. Behaving as if you have not understood will be a
less preferable option because the official has not
C or D is a more humane and sympathetic approach
done it just once. He has been doing it again and
because BPL are getting either water free or at a
again.
differentiated (lower cost) while others are paying
to ensure economic viability. So going to the higher officer for help will be the
D is preferable because otherwise there will be best approach. Because this way not only your case
misuse of BPL water. will be quickly resolved, that other official may stop
asking future bribes from others in fear of action
80. As a citizen you have some work with a from higher-ups.
government department. The official calls you
again and again; and without directly asking you,

sends out feelers for a bribe. You want to get your
work done. You would...

CSAT by Manjul Kumar Tiwari Sir


Previous Year Solved Papers 43

Previous Year
CSAT : 2013 Solved Papers

Directions for the following 2 (two) items: Select the correct answer using the codes given
below:
Read the following passage and answer the two items
(a) 1 only (b) 1 and 2
that follow. Your answers to these items should be based
(c) 2 and 3 (d) 1, 2 and 3
on the passage only.
Sol. (a)
Passage
3. In a rare coin collection, there is one gold coin for
Ecological -research over the last quarter of the century every three non-gold coins. 10 more gold coins are
has established the deleterious effects of habitat added to the collection and the ratio of gold coins
fragmentation due to mining, highways and such other to non-gold coins would be 1: 2. Based on the
intrusions on forests. When a large block of forests gets information; the total number of coins in the
fragmented into smaller bits, the edges of all these bits collection now becomes
come into contact with human activities resulting in the (a) 90 (b) 80
degradation of the entire forests. Continuity of forested (c) 60 (d) 50
landscapes and corridors gets disrupted ‘affecting several Sol. (a)
extinction-prone species of wildlife. Habitat
Initial ratio = Gold (G) : Non-Gold(N)
fragmentation, is therefore considered as the most serious
threat to biodiversity conservation. Ad hoc grants of =1:3 ... (i)
forest lands to mining companies coupled with rampant, After adding 10 gold coins, the ratio becomes
illegal mining is aggravating this threat. (10+G):N = 1:2 ... (ii)
On solving (i) & (ii), you get, G = 20 and N = 60.
1. What is the central focus of this passage?
(a) Illegal mining in forests Therefore, now the total number of coins in the
(b) Extinction of wildlife collection = (10 + G) + N = 10 + 20 + 60 = 90
(c) Conservation of nature
4. A gardener has 1000 plants: He wants to plant
(d) Disruption of habitat
them in such a way that the number of rows and
Sol. (d) the number of columns remains the same. What is
The essence of the passage, is reflected in: the minimum number of plants that he needs more
“Due to some activities like mining, highways etc, for this purpose?
there is disruption in landscapes and corridors. This (a) 14 (b) 24
is affecting several species of wild life.” (c) 32 (d) 34
Sol. (b)
2. What is the purpose of maintaining the continuity
If the number of rows and number of columns are to
of forested landscapes and corridors?
be equal, then the total number of trees should be a
1. Preservation of biodiversity.
perfect square. As 1000 is not a perfect square, find
2. Management of mineral resources.
3. Grant of forest lands for human activities. a perfect square nearest to 1000.

CSAT by Manjul Kumar Tiwari Sir


44 CSAT : 2013

It’s 1024, which is square of 32. So he needs to add 7. In a garrison, there was food for 1000 soldiers for
24 more trees to get 1024. one month. After 10 days, 1000 more soldiers joined
the garrison. How long would the soldiers be able
5. A sum of RS. 700 has to be used to give seven cash
to carry on with the remaining food?
prizes to the students of a school for their overall
(a) 25 days (b) 20 days
academic performance. If each prize is Rs. 20 less (c) 15 days (d) 10 days
than its preceding prize, then what is the least
Sol. (d)
value of the prize?
After 10 days, the remaining food would be
(a) Rs. 30 (b) Rs. 40
sufficient for the 1000 soldiers for 20 more days
(c) Rs. 60 (d) Rs. 80
Sol. (b) So, If 1000 more soldiers are added, it shall be
sufficient for only 10 days as the no. of soldiers are
Let the least value price be X. Then the next value is
doubled, the days are halved
X + 20, and the next value is X + 40 and on and on
up to 7 values. 8. The tank-full petrol in Arun’s motor-cycle lasts for
It’s nothing but an arithmetic Progression with sum 10 days. If he starts using 25% more everyday, how
700. many days will the tank-full petrol last?
(a) 5 (b) 6
Technique 1: (Manually)
(c) 7 (d) 8
X + X+20 + X + 40 +… X + 120 = 700
Sol. (d)
 7X + (20 + 40 +…+ 120) = 700
Total consumption is say N. So daily consumption
 7X + 20(1 + 2 + 3+…+ 6) = 700  7X + 20(21) is N/10. Now if daily increases by 25%, then totally
=700 its 1.25N. So, when you divide 10 by 1.25N, it will
 7X = 280 X = 40 be N/8. It will go for 8 days.
Technique 2: (Using AP Formula) 9. A person can walk a certain distance and drive
Number of prizes = 7 (N). back in six hours. He can also walk both ways in
So sum till nth term will be: n/2 (2X+ (n – 1) × 20) 10 hours. How much time will he take to drive
= 700. Put the value of N as 7. 7/2 (2X + 120) = 700 both ways?
(a) Two hours
 X = 40
(b) Two and a half hours
6. Out of 120 applications for a post, 70 are male and (c) Five and a half hours
80 have a driver’s license. What is the ratio between (d) Four hours
the minimum to maximum number of males Sol. (a)
having driver’s license? 2-way walk = 10 hrs, therefore 1-way walk = 5 hrs
(a) 1 to 2 (b) 2 to 3
1-way walk + 1-way Drive = 6 hrs; therefore 1-way
(c) 3 to 7 (d) 5 to 7
drive = 6-5= 1 hr Hence, 2-way drive = 2 hrs
Sol. (c)
Directions for the following 7 (seven) items:
Number of females = 120 -70 = 50
So, a case where minimum number of males have Read the following two passages and answer the items
driving license would mean all females have driving that follow each passage. Your answers to these items
license. All 50 females have driver’s license. So, only should be based on the passages only.
30 males with license (minimum number). Passage-1
Now, where maximum number of males will have
The law in many parts of the world increasingly restricts
license, all 70 should have license (only 10 females
the discharge of agricultural slurry into watercourses.
would have license then).
The simplest and often the most economically sound
Ratio: 30/70 = 3 to 7
practice returns the material to the land as semisolid

CSAT by Manjul Kumar Tiwari Sir


Previous Year Solved Papers 45
manure or as sprayed slurry. This dilutes its 3. The discharge may lead to the eutrophication
concentration in the environment to what might have of water bodies.
occurred III a more primitive and sustainable type of Select the correct answer using the codes given
agriculture and converts pollutant into fertilizer. Soil below
microorganisms decompose the organic components of (a) 1 only (b) 2 and 3 only
sewage and slurry and most of the mineral nutrients (c) 1 and 3 only (d) 1, 2 and 3
become available to be absorbed again by the vegetation. Sol. (c)
The excess input of nutrients, both nitrogen and Consider this: “The law in many parts of the world
phosphorus - based, from agricultural runoff (and increasingly restricts the discharge of agricultural
human sewage) has caused many ‘healthy’ oligotrophic slurry in to water courses. The simplest and most
lakes (low nutrient concentrations, low plant economically sound practice returns the material
productivity with abundant water weeds, and clear to the land as semisolid manure or as sprayed
water) to change to eutrophic condition where high slurry”. So, these sentences from 1st paragraph hint
nutrient inputs lead to high phytoplankton productivity that loosing the nutrients in this way is not a good
(sometimes dominated by bloom- forming toxic species). economical practice. Hence, option 1 is correct.
This makes the water turbid, eliminates large plants and, Options 2 and 3 is self-evident from 2nd paragraph.
in the worst situations, leads to anoxia and fish kills; so
called cultural eutrophication. Thus, important 11. The passage refers to the conversion of “pollutant
ecosystem services are lost, including the provisioning to fertilizer”. What is pollutant and what is
service of wild-caught fish and the cultural services fertilizer in this context?
associated with recreation. (a) Decomposed organic component of slurry is
pollutant and microorganisms in soil
The process of cultural eutrophication of lakes has been
constitute fertilizer.
understood for some time. But only recently did scientists
(b) Discharged agricultural slurry is pollutant
notice huge ‘dead zones’ in the oceans near river outlets,
and decomposed slurry in soil is fertilizer.
particularly those draining large catchment areas such
(c) Sprayed slurry is pollutant and watercourse
as the Mississippi in North America and the Yangtze in
is fertilizer.
China. The nutrient-enriched water flows through
(d) None of the above expressions is correct in
streams, rivers and lakes, and eventually to the estuary
this context.
and ocean where the ecological impact may be huge,
Sol. (b)
killing virtually all invertebrates and fish in areas up to
70,000 km 2 in extent. More than 150 sea areas 12. According to the passage, what are the effects of
worldwide are now regularly starved of oxygen as a indiscriminate use of fertilizers?
result of decomposition of algal blooms, fuelled 1. Addition of pollutants to the soil and water.
particularly by nitrogen from agricultural runoff of 2. Destruction decomposer of microorganisms
fertilizers and sewage from large cities. Oceanic dead in soil.
zones are typically associated with industrialized 3. Nutrient enrichment of water bodies.
nations and usually lie off’- ‘countries that subsidize 4. Creation of algal blooms.
their agriculture, encouraging farmers to increase Select the correct answer from the codes given
productivity and use more fertilizer. below:
(a) 1, 2 and 3 only (b) 1, 3 and 4 only
10. According to the passage, why should the (c) 2 arid 4 only (d) 1, 2, 3 and 4
discharge of agricultural slurry into watercourses Sol. (b)
be restricted?
1. Losing nutrients in this way is not a good 13. What is/are the characteristics of a water body
practice economically. with cultural eutrophication?
2. Watercourses do not contain the 1. Loss of ecosystem services
microorganisms that can decompose organic 2. Loss of flora and fauna
components of agricultural slurry. 3. Loss of mineral nutrients

CSAT by Manjul Kumar Tiwari Sir


46 CSAT : 2013

Select the correct answer using the code given 16. With reference to the passage, the following
below assumptions have been made:
(a) 1 only (b) 1 and 2 only 1. The author gives primary importance to
(c) 2 and 3 only (d) 1, 2 and 3 physical and material help in eradicating
Sol. (b) human misery.
2. Charitable homes, hospitals, etc. can remove
14. What is the central theme of this passage? human misery to a great extent. Which of the
(a) Appropriate legislation is essential to protect assumptions is/are valid?
the environment. (a) 1 only (b) 2 only
(b) Modern agriculture is responsible for the (c) Both 1 and 2 (d) Neither 1 nor 2
destruction of environment.
Sol. d)
(c) Improper waste disposal from agriculture can
17. Consider the following figures 1, 2, 3 and 4:
destroy the aquatic ecosystems.
(d) Use of chemical fertilizers is undesirable in
agriculture.
Sol. (c)
Option C is evident from para:
In the figure from 1 to 4 above, two symbols are
Even though the passage discussed the negative shown to change their position in a regular
impact of the fertilizers, it never says that they be direction. Following the same sequence, which one
done away with or they are undesirable. If used of the following will appear at the fifth stage?
and disposed properly, its useful.

Passage - 2

The miseries of the world cannot be cured by physical


help only. Until man’s nature changes, his physical needs
will always arise, and miseries will always be felt, and
no amount of physical help will remove them completely.
The only solution of the problem is to make mankind
pure. Ignorance is the mother of evil and of all the misery
we see. Let men have light, let them be pure and
spiritually strong and educated; then alone will misery Sol. (b)
cease in the world. We may convert every house in the The 'sum' is following an anti-clockwise rotation
country into a charitable asylum, we may fill the land in square figure. The triangle is following a zigzag
with hospitals, but human misery will continue until pattern.
man’s character changes.
Directions for the following 2 (two) items:
15. According to the passage, which of the following
In each item, there are two sets of figures; first four figures
statements is most likely to be true as the reason
named Problem figures and next four figures named
for man’s miseries?
Answer figures indicated as (a), (b), (c) and (d). The
(a) The poor economic and social conditions
problem figures follow a particular sequence. In
prevailing in society.
accordance with the same, which one of the four answer
(b) The refusal on the part of man to change his
figures should appear as the fifth figure?
character.
(c) The absence of physical and material help 18. Problem figures:
from his society.
(d) Ever increasing physical’ needs due to
changing social structure.
Sol. (b)

CSAT by Manjul Kumar Tiwari Sir


Previous Year Solved Papers 47
Answer figures: (a) diagram I (b) diagram II
(c) diagram III (d) diagram IV
Sol. (d)
Diagram I: Even for zero x, there exists some y (i.e. it
assumes some value). This means, even if there are
Sol. (c) no men, there exist some working days, which is
The triangle is rotating clockwise by an angle of wrong.
120 degrees. And, the arrow is following a patten of Diagram II: If x is increased y is also increased. This
up-down in subsequent figures. means if no. of men are increased then the no. of
19. Problem figures: working days increased, which is wrong.
Diagram III: This has two parts in it, a vertical line
and a horizontal line. Vertical line suggests infinite
working days for some number of workers, which
Answer figures: is wrong. Horizontal line suggests, same working
days even if the no of men is increased/decreased
which is also wrong.
Diagram IV: This shows the correct relation i.e. no.
of working days reduces as no. Of men are increased.
Sol. b) You can see this in terms of a mathematical equation
The three segments of each figure ( two Ls and one too (Technique 2):
arrow); are Here x and y are inversely related. If x, the number
(i) Moving left of men increases, then, y, the number of days for
(ii) When they reach the extreme left (first position), completing the work decreases. So, mathematically
they come back again from the other left (but they are inversely proportional.
with an inverted image) => x multiplied(1/y) => xy = constant
So, when the arrow in the first figure moves to
This is an equation of a rectangular hyperbola
second figure, it becomes inverted. And until it goes
which is shown in diagram IV.
again to the extreme left and moves, it would not
invert. 21. Consider the following matrix:
20. Consider the following diagrams: 3 370 7
x men, working at constant speed, do a certain job 2 224 6
in y days. Which one of these diagrams shows the 1 730 X
relation between x and y?
What is the number at ‘X’ in the above matrix?
(a) 5 (b) 8
(c) 9 (d) 11
Sol. (c)
Add all the digits in the central column, to get the
addition of the numbers in the first and second
column.
For e.g. 3 + 7 + 0 = 10; which is equal to 3 + 7 = 10
Same in Second row: 2 + 2 + 4 = 8 which is equal to
2+6=8
Same should be in third row: 7 + 3 + 0 = 10 which
should be equal to 1 + x = 10  x = 9

CSAT by Manjul Kumar Tiwari Sir


48 CSAT : 2013

22. Four cars are hired at the rate of Rs. 6 per km plus 24. A cube has six numbers marked 1, 2, 3, 4, 5 and 6
the cost of diesel at Rs. 40 a litre. In this context, on its faces. Three views of the cube are shown
consider the details given in the following table: below:
Car Mileage Hours Total Payment
(km/l) (Rs.)
A 8 20 2120
B 10 25 1950
C 9 24 2064
D 11 22 1812
Which car maintained the maximum average
speed?
(a) Car A (b) Car B
(c) Car C (d) Car D
Sol. (a)
(a) 2 and 3 (b) 6 and 1
Find the total distance travelled by and divide it by (c) 1 and 4 (d) 3 and 1
total time. To find the distance, first form a correct Sol. (a)
equation that sums up the relation of all variables
of each Car – A , B, C , D. Direction for the following 5 (five) items:
Let the distance be N. So diesel consumed for car A Study the two figures given below and answer the five
will be N/8 litres. Cost will be (N/8) × 40 = 5N items that follow:
Moreover, 6 Rs. needs to paid per km. So additional
cost will be 6N Total cost = 11N = Rs. 2120
 N = 192.7 Km
Average speed of Car A = 192.7/20 = Around 9.6
km/hr
Now, instead of repeating this process for every car,
form a general equation: Total cost = [(Distance/
mileage) × Fuel Cost ] + Distance x per km. Cost ]
Find distance from here and then average speed.
The answer is A.

23. Examine the following three figures in which the


numbers follow a specific pattern:

The missing number (?) in the third figure above is


(a) 7 (b) 16
(c) 21 (d) 28
Sol. (b)
Observe the first two figures carefully. The pattern
is as follows. 2 × (84/12)=14 25. How many Physics professors belong to the age
2 × (81/9)=18 group 35 - 44?
Hence for the third figure, 2 × (88/11)= X  X = 2 × (a) 18 (b) 16
8  X = 16 (c) 14 (d) 12

CSAT by Manjul Kumar Tiwari Sir


Previous Year Solved Papers 49
Sol. (b) 29. If the Psychology professors in the University
From figure 2, 40% of Physics professors belong to constitute 2% of all the professors in the University,
the age group 35-44. From figure 1, that there are then what is the number of professors in the
32 male and 8 female Physics professors. University?
(a) 400 (b) 500
Totally there are 40 Physics professors - out of which
(c) 600 (d) 700
40% belong to 35-44 age group. 40% of 40 will be 16
Sol. (b)
– answer.
30. Consider the following figures:
26. Which one of the following disciplines has the
highest ratio of males to females?
(a) Physics (b) Mathematics
(c) Chemistry (d) Economics
Sol. (a)

27. What percentage of all Psychology professors are


females?
(a) 40% (b) 50%
(c) 60% (d) 70%
Which one of the following figures would logically
Sol. (c) come in the 7th position indicated above by a
There are 4 male and 6 female Psychology question mark?
professors - totally 10 Psychology professors. There
are 6 female professors out of all Psychology
professors. So, (6/10) X 100 = 60%

28. If the number of female Physics professors in the


age group 25 - 34 equals 25% of all the Physics
professors in that age group, then what is the
number of male Physics professors in the age group
25 - 34?
(a) 9 (b) 6
(c) 3 (d) 2
Sol. (a)

Look at figure 2 - 30% of Physics professors belong Sol. (d)


to the age group 25-34. Then from figure1, we know The pattern is: Outer arrows - first clockwise, then
there are 32 male and 8 female Physics professors. anti-clock wise, then two facing each other, with
the below one in the anti-clockwise direction. Inner
So, totally 40 Physics professors, out of that 30%
circle and square change alternatively.
belong to 25-34 age group.
So there are 12 (30% of 40) Physics professors in the Directions for the following 8 (eight) items:
age group 25-34 Read the following four passages and answer the items
that follow each passage. Your answers to these items
Further its given that the number of female Physics
should be based on the passages only.
professors in the age group 25 - 34 equals 25% of all
the Physics professors in that age group, so, 25% of Passage -1
12 i.e. 3 are female professors in that age group The subject of democracy has become severely muddled
From S1 and S2: 12 – 3 = 9 male professors belong because of the way the rhetoric surrounding it has been
to 25-34 age group. used in recent years, There is, increasingly, an oddly

CSAT by Manjul Kumar Tiwari Sir


50 CSAT : 2013

confused dichotomy between those who want to ‘impose’ 32. With reference to the passage, the following
democracy on countries In the non-Western world (in assumptions have been made:
these countries’ ‘own interest’, of course) and those who 1. Many of the non-Western countries are unable
are opposed to such ‘imposition’ (because of the respect to have democracy because they take
for the countries’ ‘own ways’). But the entire language of democracy to be a specialized cultural
‘imposition’, used by both sides, is extraordinarily product of the West.
inappropriate since it makes the implicit assumption that 2. Western countries are always trying to impose
democracy belongs exclusively to the West, taking it to democracy on non-Western countries. Which
be a quintessentially ‘Western’ idea which has originated
of the above is/are valid assumption/
and flourished only in the West.
assumptions?
But the thesis and the pessimism it generates about the (a) 1 only (b) 2 only
possibility of democratic practice in the world would be (c) Both 1 and 2 (d) Neither 1 nor 2
extremely hard to justify. There were several experiments Sol. (d)
in local democracy in ancient India. Indeed, in
Statement 1 is false. But Statement 2: If read
understanding the roots of democracy in the world, we
carefully, in the lines so state 2 is also false “those
have to take an interest in the history of people
who want to ‘impose’ democracy on countries in the non-
participation and public reasoning in different parts of
Western world (in these countries’ ‘own interest’, of
the world. We have to look beyond thinking of democracy
course” who are ‘those’ has not been mentioned?
only in terms of European and American evolution. We
So, it may or may not be the West.
would fail to understand the pervasive demands for
participatory living, on which Aristotle spoke with far- Passage - 2
reaching insight, if we take democracy to be a kind of a Corporate governance is based on principles such as
specialized cultural product of the West. conducting the business with all integrity and fairness,
It cannot, of course, be doubted that the institutional being transparent with regard to all transactions, making
structure of the contemporary practice of democracy is all the necessary disclosures and decisions, complying
largely the product of European and American experience with all the laws of the land, accountability and
over the last few centuries. This is extremely important responsibility towards the stakeholders and commitment
to recognize since these developments in institutional to conducting business in an ethical manner. Another
formats were immensely innovative and ultimately point which is highlighted on corporate governance is
effective. There can be little doubt that there is a major the need for those in control to be able to distinguish
‘Western’ achievement here. between what are personal and corporate funds while
managing a company.
31. Which of the following is closest to the view of
democracy as mentioned in the above passage? Fundamentally, there is a level of confidence that is
(a) The subject of democracy is a muddle due to a associated with a company that is known to have good
desire to portray it as a Western concept, corporate governance. The presence of an active group
‘alien’ to non-Western countries. of independent directors on the board contributes a great
(b) The language or imposition of democracy is deal towards ensuring confidence in the market.
inappropriate. There is, however, a need to Corporate governance is known to be one of the criteria
consider this concept in the backdrop of that foreign institutional investors are increasingly
culture of ‘own ways’ of non-Western society. depending on when deciding on which companies to
(c) While democracy is not essentially a Western
invest in. It is also known to have a positive influence on
idea belonging exclusively to the West, the
the share price of the company. Having a clean image on
institutional structure of current democratic
the corporate governance front could also make it easier
practices has been their contribution.
for companies to source capital at more reasonable costs.
(d) None of the statements (a), (b) and (c) given
Unfortunately, corporate governance often becomes the
above is correct.
centre of discussion only after the exposure of a large
Sol. (c)
scam.

CSAT by Manjul Kumar Tiwari Sir


Previous Year Solved Papers 51
33. According to the passage, which of the following to increase awareness regarding the child’s food needs
should be the practice/practices in good corporate and how to satisfy them.
governance?
35. According to the passage, malnutrition in children
1. Companies should always comply with
can be reduced
labour and tax laws of the land.
(a) if the children have regular intake of food.
2. Every company in the country should have a
(b) after they cross the age of five.
government ‘representative as one of the
(c) if the food needs of younger children are
independent directors on the board to ensure
known.
transparency.
(d) if the responsibility of feeding younger
3. The manager of a company should never children is given to adults.
invest his personal, funds in the company.
Sol. (c)
Select the correct answer using the codes given
There is a catch in option (a) – regular does not
below:
mean ‘enough’ and ‘adequate’. So it is not the most
(a) 1 only (b) 2 and 3 only
appropriate answer.
(c) 1 and 3 only (d) 1, 2 and 3
Option (d) could have been the answer if option (c)
Sol. (a)
was not there, because (c) is more appropriate. The
34. According to the passage, which of the following passage talks primarily about raising awareness
is/are the major benefit/benefits of good corporate about child’s actual food needs; and then any one
governance? can satisfy them.
1. Good corporate governance leads to increase
36. According to the author, poverty is not the main
in share price of the company.
cause of malnutrition, but the fact that
2. A company with good corporate governance
1. taking care of younger ones is not a priority
always increases its business turnover
for working mothers.
rapidly.
2. awareness of nutritional needs is not
3. Good corporate governance is the main
propagated by the Public Health authorities.
criterion for foreign institutional investors
when they decide to buy a company. Select the correct answer using the code given
Select the correct answer using the codes given below:
below: (a) 1 only (b) 2 only
(a) 1 only (b) 2 and 3 only (c) Both 1 and 2 (d) Neither 1 nor 2
(c) 1 and 3 only (d) 1, 2 and 3 Sol. (b)
Sol. (a)
Passage - 4
In statement 3, the word “main” is an extreme word
A number of empirical studies find that farmers are risk-
Passage - 3 averse, though only moderately in many cases. There is
also evidence to show that farmers’ risk aversion results
Malnutrition most commonly occurs between the ages of
in cropping patterns and input use designed to reduce
six months and two years. This happens despite the
risk rather than to maximize income. Farmers adopt a
child’s food requirements being less than that of an older
number of strategies to manage and cope with
child. Malnutrition is often attributed to poverty, but it
agricultural risks. These include practices like crop and
has been found that even in households where adults
field diversification, non-farm employment, storage of
eat adequate quantities of food, more than 50 per cent of
stocks and strategic migration of family members. There
children-under-five do not consume enough food. The are also institutions ranging from share tenancy to
child’s dependence on someone else to feed him/her is kinship, extended family and informal credit agencies.
primarily responsible for the malnutrition. Very often the One major obstacle to risk sharing by farmers is that the
mother is working and the responsibility of feeding the same type of risks can affect a large number of farmers in
young child is left to an older sibling. It is therefore crucial the region. Empirical studies show that the traditional

CSAT by Manjul Kumar Tiwari Sir


52 CSAT : 2013

methods are not adequate. Hence there is a need for policy In the light of the above statements, which one of
interventions, especially measures that cut across the following is true?
geographical regions. (a) All families are primary groups.
(b) All primary groups are families.
Policies may aim at tackling agricultural risks directly or (c) A group of smaller size is always a primary
indirectly. Examples of risk-specific policies arc crop group.
insurance, price stabilization and the development of (d) Members of a primary group know each other
varieties resistant to pests and diseases. Policies which intimately.
affect risk indirectly are irrigation, subsidized credit and Sol. (d)
access to information. No single risk-specific policy is
Statement 3 says family may be an example (not
sufficient to reduce risk and is without side-effects,
necessarily) a primary group. So, (a) is wrong. By
whereas policies not specific to risk influence the general reasoning options (b) and (c) can eliminate d.
situation and affect risks only indirectly. Crop insurance,
as a policy measure to tackle agricultural risk directly, 40. Four friends, A, B, C and D distribute some money
deserves careful consideration in the Indian context and among themselves in such a manner that A gets
in many other developing countries because the majority one less than B, C gets 5 more than D, D gets 3
of farmers depend on rain-fed agriculture and in many more than B. Who gets the smallest amount?
areas yield variability is the predominant cause of their (a) A (b) B
(c) C (d) D
income instability.
Sol. (a)
37. The need for policy intervention to mitigate risks S1 – A= B – 1
in agriculture is because
S2 – C=D + 5
(a) farmers are extremely risk-averse.
S3 – D = B + 3
(b) farmers do not know how to mitigate risks.
(c) the methods adopted by farmers and existing So, C > D > B > A
risk sharing institutions are not adequate. Directions for the following 4 (four) items:
(d) majority of farmers depend on rain-fed Read the following statements and answer the four items
agriculture. that follow:
Sol. (c) Five cities P, Q, R, S and T are connected by different
Refer to the last lines of the 1st paragraph. modes of transport as follows: P and Q are connected by
boat as well as rail.
38. Which of the following observations emerges from Sand R are connected by bus and boat. Q and T are
the above passage? connected by air only.
(a) One can identify a single policy that can
P and R are connected by boat only.
reduce risk without any side-effect.
T and R are connected by rail and bus.
(b) No single risk-specific policy is sufficient to
reduce agricultural risk. 41. Which mode of transport would help one to reach
(c) Policies which affect risk indirectly can R starting from Q, but without changing the mode
eliminate it. of transport?
(d) Government’s policy intervention can (a) Boat (b) Rail
mitigate agricultural risk completely. (c) Bus (d) Air
Sol. (b) Sol. (a)
39. Consider the following statements:
I. A primary group is relatively smaller in size.
II. Intimacy is an essential characteristic of a
primary group.
III. A family may be an example of a primary
group.

CSAT by Manjul Kumar Tiwari Sir


Previous Year Solved Papers 53
42. If a person visits each of the places starting from P Sol. (c)
and gets back to P, which of the following places If B is selected, W will not be selected. So options (a)
must he visit twice? and (d) are wrong. As C cannot play with Z, option
(a) Q (b) R (b) is also wrong.
(c) S (d) T
Sol. (b) 47. If all the three males’ are selected, then how many
combinations of four member teams are possible?
Because the node at R leads to S from where there is
(a) 1 (b) 2
no other way. So, one has to return to R in any
possible way. (c) 3 (d) 4
Sol. (b)
43. Which one of the following pairs of cities is
If male A, B, C are selected, then one more person is
connected by any of the routes directly without
needed to make up a four member team.
going to any other city?
Since B is selected, W cannot be selected and as C is
(a) P and T (b) T and S
(c) Q and R (d) None of these selected, Z cannot be selected. So the either X or Y
has to be chosen.
Sol. (d)
(A,B,C,X) or (A,B,C,Y) . So these two only can be
44. Between which two cities among the pairs of cities formed.
given below are there maximum travel options
available? 48. The music director of a film wants to select four
(a) Q and S (b) P and R persons to work on “different aspects of the
(c) P and T (d) Q and R composition of a piece of music. Seven persons are
Sol. (a) available for this work; they are Rohit, Tanya,
Shobha, Kaushal, Kunal, Mukesh and Jaswant.
Directions for the following 3 (three) items: Rohit and Tanya will not work together. Kunal
Read the following passage and answer the three items and Shobha will not work together. Mukesh
that follow: and Kunal want to work together.

A tennis coach is trying to put together a team of four Which of the following is the most acceptable
players for the forthcoming tournament. For this 7 players group .of people that can be selected by the music
are available: males A, Band C; and females W, X, Y and director?
Z. All players have equal capability and at least 2 males (a) Rohit, Shobha, Kunal and Kaushal
will be there in the team. For a team of four, all players (b) Tanya, Kaushal, Shobha and Rohit
must be able to play with each other. But, B cannot play (c) Tanya, Mukesh, Kunal and Jaswant
with W, C cannot play with Z and W cannot play with Y. (d) Shobha, Tanya, Rohit and Mukesh
45. If Y is selected and B is rejected, the team will Sol. (c)
consist of which one of the following groups? Option (a) is not possible as Shobha and Kunal will
(a) A, C, Wand Y (b) A, C, X and Y not work together Option (b) is not possible as Rohit
(c) A, C, Y and Z (d) A, W, Y and Z and Tanya will not work together Option (d) is not
Sol. (b) possible as Rohit and Tanya will not work together
If Y is selected, W will not be selected. So, options Hence, option (c)
(a) and (d) are wrong. As C cannot play with Z,
49. Five people A, B, C, D and E are, seated about a
option (c) is also wrong.
round table, Every chair is spaced equidistant from
46. If B is selected and Y is rejected, the team will adjacent chairs,
consist of which one of the following groups? I. C is seated next to A.
(a) A, B, C and W (b) A, B, C and Z II. A is seated two seats from D.
(c) A, B, C and X (d) A, W, Y and Z III. B is not seated next to A.

CSAT by Manjul Kumar Tiwari Sir


54 CSAT : 2013

Which of the following must be true? (a) While playing chess with Geeta and Meena,
I. D is seated next to B. Bipin often loses.
II. E is seated next to A. (b) Geeta is the oldest among the three.
(c) Geeta hates to 10 the game.
Select the correct answer from the codes given
(d) Meena is the youngest of the three.
below:
Sol. (d)
(a) I only (b) II only
(c) Both I and II (d) Neither I nor II Geeta is older than her cousin Meena  G > M
Sol. (c) Meena’s brother Bipin is older than Geeta  B > G
Therefore, B > G > M
These are the two possibilities:
Directions for the following 4 (four) items:

Read the following passage and answer the four items


that follow. YOUR’ answers to these items should be
based on the passage only.

Passage
In both the cases, D is placed next to B. And this is Financial markets in India have acquired greater depth
true for A and E. and liquidity over the years. Steady reforms since 1991
have led to growing linkages and integration of the Indian
Directions for the following 3 (three) items:
economy and its financial system with the global
Examine carefully the following statements and answer
economy. Weak global economic prospects and
the three items that follow:
continuing uncertainties the in international financial
Out of four friends A, B, C and D, A and B play football
markets therefore, have had their impact on the emerging
and cricket, Band C play cricket and hockey,
market economies. Sovereign risk concerns, particularly
A and D play basketball and football, C and D play in the Euro area, affected financial markets for the greater
hockey and basketball. part of the year, with the contagion of Greece’s sovereign
50. Who does not play hockey? debt problem spreading to India and other economies by
(a) D (b) C way of higher-than-normal levels of volatility.
(c) B (d) A The funding constraints in international financial
Sol. (d) markets could impact both the availability and cost of
foreign funding for banks and corporates. Since the
51. Who plays football, basketball and hockey? Indian financial system is bank dominated, banks’
(a) D (b) C ability to withstand stress is critical to overall financial
(c) B (d) A stability. Indian banks, however, remain robust,
Sol. (a) notwithstanding a decline in capital to risk-weighted
assets ratio and a rise in non- performing asset levels in
52. Which game do B, C and D play? the recent past. Capital adequacy levels remain above
(a) Basketball (b) Hockey the regulatory requirements. The financial market
(c) Cricket (d) Football infrastructure continues to function without any major
Sol. (b) disruption. With further globalization, consolidation,
deregulation, and diversification of the financial system,
53. Geeta is older than her cousin Meena, Meena’s
the banking business may become more complex and
brother Bipin is older than Geeta. When Meena
riskier. Issues like risk and liquidity management and
and Bipin visit Geeta, they like to play chess. Meena
enhancing skill therefore assume greater significance.
wins the game more often than Geeta. Based on
the above information, four conclusions, as given 54. According to the passage, the financial markets in
below, have been made. Which one of these the emerging market economies including India
logically follows from the information given above? had the adverse impact in recent years due to

CSAT by Manjul Kumar Tiwari Sir


Previous Year Solved Papers 55
1. weak global economic prospects. gardening. The gardening group meets every
2. uncertainties in the international financial second day, the electronics group meets every third
markets. day, the chess group meets every fourth day, the
3. sovereign risk concerns in the Euro area. yachting group meets every fifth day and the
4. bad monsoons and the resultant crop loss. photography group meets every sixth day. How
many times do all the five groups meet on the same
Select the correct answer using the code given
day within 180 days?
below:
(a) 3 (b) 5
(a) 1 and 2 only (b) 1, 2 and 3
(c) 10 (d) 18
(c) 2 and 3 only (d) 2, 3 and 4
Sol. (a)
Sol. (b)
LCM of 2,3,4,5 and 6. gives 60 i.e. all the groups
55. The Indian financial markets are affected by global meet on the same day on every 60th day. Hence,
changes mainly due to the only three times.
(a) increased inflow of remittances from abroad.
(b) enormous increase in the foreign exchange 59. A, B, C, D and E belong to five different cities P, Q,
reserves. R, Sand T (not necessarily in that order). Each one
(c) growing global linkages and integration of the of them comes from a different city. Further it is
Indian financial markets. given that:
(d) contagion of Greece’s sovereign debt problem. 1. B and C do not belong to Q.
Sol. (c) 2. B and E do not belong to P and R.
3. A and C do not belong to R, Sand T.
This is the very theme of the passage.
4. D and E do not belong to Q and T.
56. According to the passage, in the Indian financial
Which one of the following statements is not
system, banks’ ability to with stand stress is critical
correct?
to ensure overall financial stability because Indian
(a) C belongs to P (b) D belongs to R
financial system is
(c) A belongs to Q (d) B belongs to S
(a) controlled by the Government of India.
(b) less integrated with banks. Sol. (d)
(c) controlled by Reserve Bank of India. 60. Seven men, A, B, C, D, E, F and G are standing in a
(d) dominated by banks. queue in that order. Each one is wearing a cap of a
Sol. (d) different colour like violet, indigo, blue, green,
yellow, orange and red. D is able to see in front of
57. Risk and liquidity management assumes more him green and blue, but not violet. E can see violet
importance in the Indian banking system in future and yellow, but not red. G can see caps of all colours
due to
other than orange. If E is wearing an indigo coloured
1. further globalization.
cap, then the colour of the cap worn by F is
2. more consolidation and deregulation of the
(a) Blue (b) Violet
financial system.
(c) Red (d) Orange
3. further diversification of the financial system.
4. more financial inclusion in the economy. Sol. (c)
You can construct the following table from the
Select the correct answer using the code given
statements.
below:
Person Cap
(a) 1, 2 and 3 (b) 2, 3 and 4
A/B/C green/blue/yellow
(c) 1 and 2 only (d) 3 and 4 only
D violet
Sol. (a)
E indigo
58. There are five hobby clubs in a college viz, G orange
photography, yachting, chess, electronics and So F is wearing red cap.

CSAT by Manjul Kumar Tiwari Sir


56 CSAT : 2013

61. There are some balls of red, green and yellow colour 63. Sedimentary rock leads to the formation of oil
lying on a table. There are as many red balls as deposits because
there are yellow balls. There are twice as many (a) there are no saline conditions below it.
yellow balls as there are green ones. The number (b) it allows some dissolved oxygen to enter the
of red balls dead organic matter below it.
(a) is equal to the sum of yellow and green balls. (c) weight of overlying sediment layers causes
(b) is double the number of green balls. the production of heat.
(c) is equal to yellow balls minus green balls. (d) it contains the substances that catalyze the
(d) cannot be ascertained. chemical reactions required to change dead
Sol. (b) organisms into oil.

R=Y= 2G Sol. (c)


64. In a class of 45 students, a boy is ranked 20th. When
Directions for the following 2 (two) items: two boys joined, his rank was dropped by one. What
Read the following passage and answer the two items is his new rank from the end?
that follow. Your answers to these items should be based (a) 25th (b) 26th
on the passage only. (c) 27th (d) 28th
Sol. (c)
Passage
Initial Condition = 19 guys – The Boy (20th Rank) –
Crude mineral oil comes out of the earth as a thick brown 25 other guys
or black liquid with a strong smell. It is a complex mixture Now = 20 guys – The Boy – 26 other guys (Since his
of many different substances, each with its own rank has shifted by one, one guy should be ahead
individual qualities. Most of them are combinations of and one behind)
hydrogen and carbon in varying proportions. Such
So, his rank from end is 27th.
hydrocarbons are also found in other forms such as
bitumen, asphalt and natural gas. Mineral oil originates 65. A thief running at 8 km/hr is chased by a
from the carcasses of tiny animals and from plants that policeman whose speed is 10 km/hr. If the thief is
live in the sea. Over millions of years, these dead creatures 100 m ahead of the policeman, then the time
form large deposits under the sea-bed; and ocean currents required for the policeman to catch the thief will
cover them with a blanket of sand and silt. As this mineral be
hardens, it becomes sedimentary rock and effectively (a) 2 min (b) 3 min
shuts out the oxygen, so preventing the complete (c) 4 min (d) 6 min
decomposition of the marine deposits underneath. The Sol. (b)
layers of sedimentary rock become thicker and heavier. Use relative speed concept. Relative speed of
Their pressure produces heat, which transforms the tiny policeman w.r.t the thief is 2 Km/hr (10-8).
carcasses into crude oil in a process that is still going on His Speed in m/min will be (2 X1000)/60 =
today. 100/3 m/min

62. Mineral oil deposits under the sea do not get He has to cover 100 m. So time needed is 3 mins.
completely decomposed because they 66. A train travels at a certain average speed for a
(a) are constantly washed by the ocean currents. distance of 63 km and then travels a distance of 72
(b) become rock and prevent oxygen from entering km at an average speed of 6 km/hr more than its
them. original speed. If it takes 3 hours to complete the
(c) contain a mixture ‘of hydrogen and carbon. total journey, what is the original speed of the train
(d) are carcasses of organisms lying in saline in km/hr?
conditions. (a) 24 (b) 33
Sol. (b) (c) 42 (d) 66

CSAT by Manjul Kumar Tiwari Sir


Previous Year Solved Papers 57
Sol. (c) (c) he was satisfied with the condition of his
Let the original speed be ‘S’. bicycle.
Total time taken = 3 hr  63/S + 72/S+6 = 3 (d) they were late only by 30 minutes.
Sol. (c)
You can solve the question easily if you just do an
option check. Otherwise the long way is to solve The statement “All right” after examining the
the equation completely. Save time. bicycle points to option C.
As we have 63 in the numerator, we can assume Passage - 2
that S may be a multiple of 7. The only option
It was already late when we set out for the next town,
available is 4. So, let us try with that. which according to the map was about fifteen kilometres
63/42 + 72/48 = 3/2 + 3/2 = 3 away on the other side of the hills. There we felt that we
would find a bed for the night. Darkness fell soon after
Directions for the following 8 (eight), items:
we left the village, but luckily we met no one as we drove
The following eight items (items 67 to 74) are based on swiftly along the narrow winding road that led to the
three passages in English to test the comprehension of hills. As we climbed higher, it became colder and rain
English language and therefore these items do not have began to fall, making it difficult at times to see the road. I
Hindi version. Read each passage and answer the items asked John, my companion, to drive more slowly. After
that follow. we had travelled for about twenty kilometres, there was
still no sign of the town which was marked on the map.
Passage - 1 We were beginning to get worried. Then without warning,
the car stopped and we found we had run out of petrol.
Seven-year-old Jim came home from the park without his
new bicycle. “An old man and a little boy borrowed it,” 69. The author asked John to drive more slowly
he explained. “They are going to bring it back at four because
o’clock.” His parents were upset that he had given his (a) the road led to the hills.
expensive new bicycle, but were secretly proud of his (b) John was an inexperienced driver.
kindness and faith. Came four o’clock, no bicycle. The (c) the road was not clearly visible.
parents were anxious. But at 4:30, the door bell rang, and (d) they were in wilderness.
there stood a happy man and a boy, with the bicycle and Sol. (c)
a box of chocolates. Jim suddenly disappeared into his
70. The travellers set out for the town although it was
bedroom, and then came running out. “All right,” he getting dark because
said, after examining the bicycle. ‘’You can have your (a) they were in a hurry.
watch back!” (b) the next town was a short distance away and
67. When Jim came home without his bicycle, his was a hill-resort.
(c) they were in wilderness.
parents
(d) the next town was a short distance away and
(a) were angry with him.
promised a good rest for the night.
(b) were worried.
Sol. (d)
(c) did not feel concerned.
(d) were eager to meet the old man and the little 71. The travellers were worried after twenty kilo metres
boy. because
Sol. (b) (a) it was a lonely countryside.
Anger would be an extreme word. So B, not A. (b) they probably feared of having lost their way.
(c) the rain began to fall
68. Jim returned the watch to the old man and the little (d) it was getting colder as they drove.
boy because Sol. (b)
(a) they gave him chocolates. Since the place mentioned in the map was not
(b) his father was proud of him. found, they thought they had lost the way.

CSAT by Manjul Kumar Tiwari Sir


58 CSAT : 2013

Passage - 3 Please attempt all the items. There is no penalty for, wrong
answers for these six items.
A stout old lady was walking with her basket down the
middle of a street in Petrograd to the great confusion of 75. You are the head of your office. There are certain
the traffic and no small peril to herself. It was pointed houses reserved for the allotment to the office staff
out to her that the pavement was the place for foot- and you have been given the discretion to do so. A
passengers, but she replied, “I m going to walk where I set of rules for the allotment of the houses has been
like. We’ve got liberty now.” It did not occur to the dear laid down by you and has been made public. Your
lady that if liberty entitled the foot-passenger to walk personal secretary, who is very close to you, comes
down the middle of the road it also entitled the taxi-driver to you and pleads that as his father is seriously ill,
to drive on the pavement, and that the end of such liberty he should be given priority in allotment of a house.
would be universal chaos. Everything would be getting The office secretariat that examined the request as
in everybody else’s way and nobody would get per the rules turns down the request and
anywhere. Individual liberty would have become social recommends the procedure to be followed
anarchy. according to the rules. You do not want to annoy
your personal secretary. In such circumstances,
72. It was pointed out to the lady that she should walk
what would you do ?
on the pavement because she was
(a) Call him over to your room and personally
(a) a pedestrian (b) carrying a basket
explain why the allotment cannot be done.
(c) stout (d) an old lady
(b) Allot the house to him to win his loyalty.
Sol. (a)
(c) Agree with the office note to show that you
73. The lady refused to move from the middle of the are not biased and that you do not indulge in
street because favouritism.
(a) she was not afraid of being killed. (d) Keep the file with you and not pass any orders.
(b) she felt that she is entitled to do whatever she Solution: More preferred (a), less preferred (c)
liked.
76. While travelling in a Delhi-registered commercial
(c) she did not like walking on the pavement.
taxi from Delhi to an adjacent city (another State),
(d) she was confused.
your taxi driver informs you that as he has no
Sol. (b)
permit for running the taxi in that city, he will stop
74. The old lady failed to realise that at its Transport Office and pay the prescribed fee
(a) she was not really free. of Rs. forty for a day. While paying the fee at the
(b) her liberty was not unlimited. counter you find that the transport clerk is taking
(c) she was an old person. an extra fifty rupees for which no receipt is being
(d) roads are made for motor vehicles only given. You are in a hurry for your meeting. In such
Sol. (b) circumstances, what would you do?
(a) Go up to the counter and ask the clerk to give
D could have been the answer, but she knew and
back the money which he has illegally taken.
realized that the roads were for motor vehicles only,
(b) Do not interfere at all as this is a matter
yet choose to walk there. So B is the more appropriate
between the taxi driver and the tax authorities.
response.
(c) Take note of the incident and subsequently
Directions for the following 6 (six) items: report the matter to the concerned authorities.
(d) Treat it as a normal affair and simply forget
Given below are six items. Each item describes a situation about it.
and is followed by four possible responses. Indicate the
Solution: More preferable (a); less preferable (c)
response you find most appropriate. Choose only one
response for each item. The responses will be evaluated 77. A person lives in a far off village which is almost
based on the level of appropriateness for the given two hours by bus. The villager’s neighbour is a
situation. very powerful landlord who is trying to occupy

CSAT by Manjul Kumar Tiwari Sir


Previous Year Solved Papers 59
the poor villager’s land by force. You are the Solution: More preferable (c); less preferable (b) Option
District Magistrate and busy in a meeting called (a) is nepotism and corruption straightaway.
by a local Minister. The villager has come all the
79. You are in-charge of implementing the Family
way, by bus and on foot, to see you and give an
Planning programme in an area where there is a
application seeking protection from the powerful
strong opposition to the present policy. You want
landlord. The villager keeps on waiting outside
to convince the residents of the need for keeping
the meeting hall for an hour. You come out of the
small families. What would be the best way of
meeting and are rushing to another meeting. The
communicating this message?
villager follows you to submit his application.
(a) By logically explaining to the residents the
What would you do?
need for family planning to improve the health
(a) Tell him to wait for another two hours till you
and living standards.
come back from your next meeting.
(b) By encouraging late marriages and proper
(b) Tell him that the matter is actually to be dealt
spacing of children.
by a junior officer and that he should give the
(c) By offering incentives for adopting family
application to him.
planning devices.
(c) Call one of your senior subordinate officers (d) By asking people who have been sterilized or
and ask him to solve the villager’s problem. are using contraceptives to directly talk to the
(d) Quickly take the application from him, ask him residents.
a few relevant questions regarding his
problem and then proceed to the meeting. Solution: More preferable (a); less preferable (c)

Solution: More preferable (c); less preferable (d) 80. You are a teacher in a University and are setting a
question paper on a particular subject. One of your
78. There is a shortage of sugar in your District where colleagues, whose son is preparing for the
you are the District Magistrate. The Government examination on that subject, comes to you and
has ordered that only a maximum amount of 30 kg informs you that it is his son’s last chance to pass
sugar is to be released for wedding celebrations. A that examination and whether you could help him
son of your close friend is getting married and your by indicating what questions are going to be in the
friend requests you to release at least 50 kg sugar examination. In the past, your colleague had helped
for his son’s wedding. He expresses annoyance you in another matter. Your colleague informs you
when you tell him about the Government’s that his son will suffer from depression if he fails
restrictions on this matter. He feels that since you in this examination. In such circumstances, what
are the District Magistrate you can release any would you do?
amount. You do not want to spoil your friendship (a) In view of the help he had given you, extend
with him. In such circumstances, how would you your help to him.
deal with the situation? (b) Regret that you cannot be of any help to him.
(a) Release the extra amount of sugar which your (c) Explain to your colleague that this would be
friend has requested for violating the trust of the University
(b) Refuse your friend the extra amount and authorities and you are not in a position to
strictly follow the rules. help him.
(c) Show your friend the copy of the Government (d) Report the conduct of your colleague to the
instructions and then persuade him to accept higher authorities
the lower amount as prescribed in the rules.
(d) Advise him to directly apply to the allotting Solution: More preferable (c); less preferable (b)
authority and inform him that you do not
interfere in this matter.   

CSAT by Manjul Kumar Tiwari Sir


60 CSAT : 2014

Previous Year
CSAT : 2014 Solved Papers

Passage - 1 1. Consider the following statements:


The Himalayan ecosystem is highly vulnerable to According to the passage, the adverse impact of
damage, both due to geological reasons and on account climate change on an ecosystem can be a
of the stress caused by increased pressure of population, 1. Permanent disappearance of some of its flora
exploitation of natural resources and other related and fauna.
challenges. These aspects may be exacerbated due to the 2. Permanent disappearance of ecosystem itself.
impact of climate change. It is possible that climate Which of the statements given above is/are
change may adversely impact the Himalayan ecosystem correct?
through increased temperature altered precipitation (a) 1 only (b) 2 only
patterns, episodes of drought and biotic influences. (c) Both 1 and 2 (d) Neither 1 nor 2
This would not only impact the very sustenance of the Sol. (a)
indigenous communities in uplands but also the life of Statement 2: says “disappearance of the ecosystem
downstream dwellers across the country and beyond. itself”. First statement too is extreme, but it can be
Therefore, there is an urgent need for giving special found in passage.
attention to sustain the Himalayan ecosystem. This
2. Which one of the following statements best implies
would require conscious efforts for conserving all the
the need to shift toward contemporary conservation
representative systems.
approach?
Further, it needs to be emphasized that the endemics with
(a) Exploitation of natural resources causes a
restricted distribution, and most often with specialized
stress on the Himalayan ecosystem.
habitat requirements, are among the most vulnerable
(b) Climate change alters precipitation patterns,
elements. In this respect the Himalayan biodiversity
causes episodes of drought and biotic
hotspot, with rich endemic diversity, is vulnerable to
interference.
climate change.
(c) The rich biodiversity, including endemic
The Threats include possible loss of genetic resources
diversity, makes the Himalayan region a
and species, habitats and concomitantly a decrease in
biodiversity hotspot.
ecosystem services. Therefore, conservation of endemic
(d) The Himalayan biogeographic region should
elements in representative ecosystems/habitats assumes
be enabled to adapt to climate change
a great significance while drawing conservation plans
smoothly.
for the region.
Sol. (d)
Towards achieving the above, we will have to shift
towards contemporary conservation approaches, which 3. What is the most important message conveyed by
include a paradigm of landscape level interconnectivity the passage?
between protected area systems. The concept advocates (a) Endemism is a characteristic feature of
a shift from the species-habitat focus to an inclusive focus Himalayan region.
on expanding the bio- geographic range so that natural (b) Conservation efforts should emphasize on
adjustments to climate change can proceed without being biogeographic ranges rather than on some
restrictive. species or habitats.

CSAT by Manjul Kumar Tiwari Sir


Previous Year Solved Papers 61
(c) Climate change has adverse impact on the to have generally low levels of taxation and government
Himalayan ecosystem. expenditure should be kept to the minimum to abide by
(d) Without Himalayan ecosystem, the life of the the principle of fiscal prudence. All these are policy
communities of uplands and downstream actions on the domestic front and are not directly related
will have no sustenance. to the core items of the globalization agenda, namely free
Sol. (b) international flow of goods and finance.
5. According to the passage, under the globalization,
4. With reference to the passage, the following
government interventions are viewed as processes
assumptions have been made:
leading to
1. To maintain natural ecosystems, exploitation
of natural resources should be completely (a) distortions and inefficiency in the economy.
avoided. (b) optimum use of resources.
2. Not only anthropogenic but also natural (c) more profitability to industries.
reasons can adversely affect ecosystems.
(d) free play of market forces with regard to the
3. Loss of endemic diversity leads to the
industries.
extinction of ecosystems.
Sol. (a)
Which of the above assumptions is/are correct?
Direct mention has been made in the passage.
(a) 1 and 2 (b) 2 only
(c) 2 and 3 (d) 3 only 6. According to the passage, the basic philosophy of
Sol. (b) globalization is to
(a) give absolute freedom to producers to
Passage - 2 determine prices and production.
It is often forgotten that globalization, is not only about (b) give freedom to producers to evolve
policies on international economic relationships and distribution patterns.
transactions, but has equally to do with domestic policies
(c) give absolute freedom to markets to determine
of a nation. Policy changes necessitated by meeting the
prices, production and employment.
internationally set conditions (by WTO etc.) of free trade
and investment flows obviously affect domestic (d) give freedom to producers to import and
producers and investors. But the basic philosophy export.
underlying globalization emphasizes absolute freedom Sol. (c)
to markets to determine prices and production and 7. According to the passage, which of the following
distribution patterns, and view government interventions is/are necessary for ensuring globalization?
as processes that create distortions and bring in
1. Privatization of public enterprises
inefficiency. Thus, public enterprises have to be
privatized through disinvestments and sales; sectors and 2. Expansionary policy of public expenditure
activities hitherto reserved for the public sector have to 3. Free play of market forces to determine wages
be opened to the private sector. This logic extends to the and employment
social services like education and health. Any restrictions 4. Privatization of social services like education
on the adjustments in workforce by way of retrenchment and health.
of workers should also be removed and exit should be
Select the correct answer using the code given
made easier by removing any restrictions on closures.
below:
Employment and wages should be governed by free play
of market forces, as any measure to regulate them can (a) 1 only (b) 2 and 3 only
discourage investment and also create inefficiency in (c) 1, 3 and 4 (d) 2, 3 and 4
production. Sol. (c)
Above all, in line with the overall philosophy of reduction Statement 2 is incorrect. So (b) and (d) are out.
in the role of the state, fiscal reforms should be undertaken Statement 3 is ofcourse correct.

CSAT by Manjul Kumar Tiwari Sir


62 CSAT : 2014

8. According to the passage, in the process (a) Non-increasing (b) Non -decreasing
globalization the State should have (c) Steady (d) Fluctuating
(a) expanding role Sol. (b)
(b) reducing role
(c) statutory role 13. The following table shows the marks obtained by
(d) none of the above roles. two students in different subjects:

Sol. (b)

Directions for the following 4 (four) items:

The following graph shows the average profit of two fruit-


sellers A and B in thousands (Rs.) per year from the year
1995 to 2000.

Consider the graph and answer the 4 (four) items that


follow:
The difference in the mean aggregate percentage
marks of the students is
(a) 2.5% (b) 13.75%
(c) 1.25% (d) Zero
Sol. (d)

14. Examine the following figure:

Which one of the following figures has the above


figure embedded in it?
9. In which year is the average profit of A and B same?
(a) 1995 (b) 1996
(c) 1997 (d) 1998
Sol. (b) (a) (b) (c) (d)
Sol. (c)
10. What is the difference between the average profit
of B and A in the year 1998? 15. Consider the following matrix:
(a) - Rs. 100 (b) - Rs. 1,000
(c) + Rs. 600 (d) - Rs. 300
Sol. (c)

11. How much more average profit did A make in the


year 2000 than in the year 1999?
(a) Rs. 200 (b) Rs. 1,000
(c) Rs. 1,500 (d) Rs. 2,000
Sol. (d)

12. What is the trend of the average profit of B from the Which one of the following figures fits into blank
year 1997 to the year 2000? part of the above matrix?

CSAT by Manjul Kumar Tiwari Sir


Previous Year Solved Papers 63
18. Consider the following matrix with one empty
block in the lower extreme corner:
(a) (b)

(c) (d)

Sol. (b)
The pattern is – of the two arrows in the first column,
the second arrow flips by 180 degrees in the next
column (second). In the third column, both arrows
Which of the following figures could fit in the
flip by 180 degrees.
empty block and thus complete the matrix?
16. The following table gives population and total
income of a city for four years:

Sol. (a)
The squares are increasing and circles decreasing
in a particular fashion across the next right columns.

19. With reference to the figure given below number of


Which one of the following statements correct in
different routes from S to T without retracing from
respect of the above data ?
U and/or V (T is the next node after V), is
(a) Population increased by 5% or more every
year.
(b) Income increased by 10% or more every year.
(c) Per capita income was always above 5,000.
(d) Per capita income was highest in 1994. T
Sol. (c)

17. Consider the table given below in which the (a) 3 (b) 6
numbers bear certain relationship among (c) 9 (d) 18
themselves along the rows: Sol. (d)
There are two ways of doing it.
Technique 1: The Long one and obvious
You keep counting the number of routes starting
from S to U to V to T.
Technique 2: Short cut
Multiply the number of nodes from S, U and V
Which one of the following numbers is the missing leading towards T. It is 3 X 2 X 3 = 18.
number indicated above by X ? Number of ways you can reach U from S is 3. The
(a) 19 (b) 15 number of ways you can reach V from U is 2. So the
(c) 14 (d) 8 number of ways you can reach from S to V will
Sol. (b & d) logically be 3 X 2 = 6. Similarily logically extend it
Both B and D can be the answer to T, it becomes 6X3 =18.

CSAT by Manjul Kumar Tiwari Sir


64 CSAT : 2014

20. Consider the following figures: (a) the values of all things would remain constant.
(b) the values of the things sold would be
doubled.
(c) the values of the things bought would be
halved.
Change in positions of beads in the figures above (d) the value of money only would be halved.
follows a sequence. Follows the same sequence, Sol. (d)
which of the figures look should appear as the
fifth figure above? 23. A and B decide to travel from place X to place Y by
bus. A has Rs. 10 with him and he finds that it is
(a) (b) 80% of the bus fare for two persons. B finds that he
has Rs. 3 with him and hands it over to A. In this
context, which one of the following statements is
(c) (d) correct?
(a) Now the money A has just enough to buy two
Sol. (b) tickets.
The sequence followed is like Left (L), Right(R), (b) A still needs Rs. 2 for buying the tickets
Top(T) and bottom(B) in cycles for all beads. (c) After buying the two tickets A will be left with
Sequence for Top left bead across four figures – LR 50 paise.
- BT (d) The money A now has is still not sufficient to
For Bottom left bead – RL – TB (total mirror image buy two tickets.
of top bead) For Top Right bead – TB – RL (exact Sol. (c)
reverse of top left bead) Rs. 10 is 80% of fare. So actual fare is Rs. 12.50
For bottom right bead – BT – LR (total mirror image
B gives Rs. 3 to A. So he has Rs. 9.50, i.e. 50 paisa
of the above) Repeat the same pattern. And you get
short of bus fare.
the answer.
24. As per agreement with a bank, a businessman had
21. A bell rings every 18 minutes. A second bell rings
to refund a loan in some equal installments
every 24 minutes. A third bell rings every 32
without interest. After paying 18 installments he
minutes. If all the three bells ring at the same time
found that 60 percent of his loan was refunded.
at 8 o’clock in the morning, at what other time will
How many installments were there in the
they all ring together?
agreement?
(a) 12 : 40 hrs (b) 12 : 48 hrs
(a) 22 (b) 24
(c) 12 : 56 hrs (d) 13 : 04 hrs
Sol. (b) (c) 30 (d) 33

Take the LCM of 18, 24 and 32 which comes out to Sol. (c)
be 288 minutes i.e. 4 hours and 48 minutes. Add to 60% is equal to 18 installments.
8’o clock. The time is 12:48 hours. 1% is equal to 18/60 installments
22. “Price is not the same thing as value. Suppose that So 100% is equal to (18/60)X100 = 30 installments.
on a day the price of everything viz., coal, bread,
25. A worker reaches his factory 3 minutes late if his
postage stamps, a day’s labour, the rent of houses,
speed from his house to the factory is 5 km/hr. If
etc. were to double. Prices then would certainly
he walks at a speed of 6 km/hr then he reaches the
rise, but- values of all things except one would
factory 7 minutes early the distance of the factory
not.”
from his house is
The writer wants to say that if prices of all things (a) 3 km (b) 4 km
were doubled (c) 5 km (d) 6 km

CSAT by Manjul Kumar Tiwari Sir


Previous Year Solved Papers 65
Sol. (c) must not be treated as an end in itself but as an instrument
Use time, speed and distance equation for both for spreading prosperity to all. India’s own past
situations. experience and the experience of other nations suggests
that growth is necessary for eradicating poverty but it is
Let distance of factory from house be X in Kms. Let
not a sufficient condition. In other words, policies for
the ideal time be T in hours. (X/5) = T + (3/60)
promoting growth need to be complemented with policies
eqn (i)
to ensure that more and more people join’ in the growth
(X/6) = T – (7/60)eqn (ii) process and, further,· that there are mechanisms in place
Solving both together, you get (X/5)-(X/6) = 10/60 to redistribute some of the gains to those who are unable
X/30 = 1/6 X = 5 Km. to partake in the market process and, hence, get left behind.

26. “Liberty, therefore, is never real unless the A simple way of giving this idea of inclusive growth a
Government can be called to account when it sharper form is to measure a nation’s progress in terms
invades rights.” of the progress of its poorest segment, for instance the
bottom 20 per cent of the’ population. One could measure
Which one of the following is the best justification the per capita income of the bottom quintile of the
of the above statement? population and also calculate the growth rate of income;
(a) In the realisation that the government can be and evaluate our economic success in terms of these
brought to book in a court of law measures that pertain to the poorest segment. This
(b) In identifying a man as a political unit in a approach is attractive because it does not ignore growth
way which distinguishes him from other like some of the older heterodox criteria did. It simply
citizens looks at the growth of income of the poorest sections of
(c) In a decentralized society wherein the basic the population. It also ensures that those who are outside
needs of men can find satisfaction of the bottom quintile do not get ignored. If that were
(d) In the understanding that liberty and, done, then those people would in all likelihood drop
restraints are complementary down into the bottom quintile and so would
Sol. (d) automatically become a direct target of our policies.
Hence the criterion being suggested here is a statistical
Direction 8 for the following 5 (five) items:
summing up of the idea of inclusive growth, which, in
Read the following two passages and answer the items turn, leads to two corollaries: to wish that India must
that follow- each passage. Your answers to these items strive to achieve high growth and that we must work to
should be based on the passages only. ensure that the weakest segments benefit from the growth.

Passage -1 27. The author’s central focus is on


In recent times, India has grown fast not only compared (a) applauding India’s economic growth not only
to its own past but also in comparison with other nations. against its own past performance, but against
But there cannot be any room for complacency because it other nations.
is possible for the Indian economy to develop even faster (b) emphasizing the-need for economic growth
and also to spread the benefits of this growth more widely which is the sole determinant of a country’s
than has been done thus far. Before going into details of prosperity.
the kinds of micro-structural changes that we need to (c) emphasizing inclusive growth where gains
conceptualize and then proceed to implement, it is of growth are shared widely by the
worthwhile elaborating on the idea of inclusive growth population.
that constitutes the defining concept behind this (d) emphasizing high growth.
Government’s various economic policies and decisions. Sol. (c)
A nation interested in inclusive growth views the same
growth differently depending on whether the gains of 28. The author supports policies which will help
the growth are heaped primarily on a small segment or (a) develop economic growth.
shared widely by the population. The latter is cause for (b) better distribution of incomes irrespective of
celebration but not the former. In other words, growth rate of growth.

CSAT by Manjul Kumar Tiwari Sir


66 CSAT : 2014

(c) develop economic growth and redistribute 3. cutting costs.


economic gains to those getting left behind. 4. selling its equity shares at higher prices.
(d) put an emphasis on the development of the Which of the statements given above is/are
poorest segments of society. correct?
Sol. ( c) (a) 1 only (b) 2 and 3
(c) 3 and 4 (d) 1, 2 and 4
29. Consider the following statements:
Sol. (b)
According to the author, India’s economy his 31. Consider the following statements:
grown but there is no room for complacency as According to the passage, private oil companies
1. growth eradicates poverty. re-enter the oil producing market if
2. growth has resulted in prosperity all. Which
1. a transparent rule-based petrol pricing exists.
of the statements given above is/a correct?
2. There is no government interference in the oil
(a) 1 only (b) 2 only
producing market.
(c) Both 1 and 2 (d) Neither 1 nor 2
3. subsidies are given by the government.
Sol. (d)
4. regulations of anti-trust are removed . Which
The author does not mention any of these. He of the statements given above are correct?
mentions the opposite that growth has not been able
(a) 1 and 2 (b) 2 and 3
to life large number of people from poverty; and
(c) 3 and 4 (d) 2 and 4
distribution needs to be looked at.
Sol. (a)
Passage -2
32. Five persons fire bullets at a target at an interval of
It is easy for the government to control State-owned 6, 7, 8, 9 and 12 seconds respectively. The number
companies through nods and winks. So what really of times they would fire the bullets together at the
needs to be done as first step is to put petrol pricing on a target in an hour is
transparent formula - if the price of crude is x and the (a) 6 (b) 7
exchange rate y, then every month or fortnight, the (c) 8 (d) 9
government announces a maximum price of petrol, which Sol. (b)
anybody can work out from the x and Y. The rule has to Take the LCM of 6,7,8,9 and 12 which comes out to
be worked out to make sure that the oil-marketing can, in be 504 seconds.
general, cover their costs. This will mean that if one Divide 3600 seconds (1 hour) by 504 = It will be
company can innovate and cut costs, it will make greater between 7 and 8. So the answer is (b).
profits. Hence, firms will be more prone to innovate and
33. A group of 630 children is seated in rows for a group
be efficient under this system. Once the rule is
photo session. Each row contains three less
announced, there should be no interference by the
children than the row in front of it. Which one of
government. If this is done for a while, private companies
the following number of rows is not possible?
will re-enter this market. And once a sufficient number
(a) 3 (b) 4
of them are in the fray, we can remove the rule-based
(c) 5 (d) 6
pricing and leave it truly to the market (subject to, of
Sol. (d)
course, the usual relations of anti-trust and other
competition laws). Let the number of children in the front row be X. So
children in the back rows respectively will be
30. Consider the following statements: According to X-3
the passage, an oil company can make greater
X-6
profits, if a transparent formula for petrol pricing
X-9
is announced every fortnight or month, by
1. promoting its sales. X – 12
2. undertaking innovation. X – 15

CSAT by Manjul Kumar Tiwari Sir


Previous Year Solved Papers 67
And so on..... Which three villages are in a line?
Check if 3 rows are possible i.e. if they sum upto (a) A, C, B (b) A, D, E
630 (X)+ (X-3)+(X-6) = 630 (c) C, B, F (d) E, B, D
=> 3X = 639 => X = 213 Sol. (b)

Since X is an integer, 3 rows are possible.

34. There are seven persons up on a ladder, A, B, C, D,


E, F and G (not in that order). A is further up than
E but is lower than C. B is in the middle. G is
between A and B. E is between B and F. If F is
between E and D, the person on the bottom step of
the ladder will be
Each arrow represents 1 Km of distance. So, A, D
(a) B (b) F
and E are in a line.
(c) D (d) E
Sol. (c) 37. Four children are sitting in a row. A is occupying
Statement 1 (S1): C> .......A > E the seat next to B but not next to C. If C is not sitting
S2: B in the middle. next to D, who is/are occupying seat/seats
adjacent to D?
S3: G is between A and B. So the first four on top of
(a) B (b) A
ladder have to be C>A>G>B E
(c) B and A (d) Impossible to tell
S4: F between E and D. So, sequence becomes Sol. (b)
something like C>A>G>B>E>F>D D is at the bottom
First sentence, this can be the possibility – AB or
ladder.
BA, but not CAB or BAC
35. Consider that: Second sentence, CD or DC is not possible. So C has
1. A is taller than B. to be next to B, and D next to A. It can be either
2. C is taller than A. DABC, or CBAD.
3. D is taller than C.
38. Assume that
4. E is the tallest of all.
1. the hour and minute hands of a clock move
If they are made to sit in the above order of their without jerking.
height, who will occupy the mid position? 2. the clock shows a time between 8 o’clock and
(a) A (b) B 9 o’clock.
(c) C (d) D 3. the two hands of the clock are one above the
Sol. (c) other.
S1: A>. B.. After how many minutes (nearest integer) with the
two hands will be again lying one above the other?
Adding S2: C>....A>....B..
(a) 60 (b) 62
Adding S3: D>...>C>...A>...B... Adding S4: (c) 65 (d) 67
E>D>C>A>B Sol. (c)
C will occupy the middle position. When the minute hand travels 60 minutes, the hour
36. Consider the following statements: There are six hand only travels 5 minutes.
villages A, B, C, D, E and F. F is 1 km to the west The relative speed between them (both hands in
of D. same direction) = 60 – 5 = 55 minutes
B is 1 km to the east of E. A is 2 km to the north of E. They would meet again after = 60  55 = 1.09 hours
C is 1 km to the east of A. 1.09 hours = 1.09 × 60 = 65.4 minutes
D is 1 km to the south of A. Hence, they would meet again after 65 minutes.

CSAT by Manjul Kumar Tiwari Sir


68 CSAT : 2014

Directions for the following 6 (six items): For this, something has to be observed very
The following six items are based on two passages in carefully, with seriousness.
English to test the comprehension of English language Therefore, the expression "a stray dog watched the
and therefore these items do not have Hindi version. Read procession philosophically" implies careful, serious
each passage and answer the items that follow. watching by the dog.
Passage- 1 Passage- 2
Cynthia was a shy girl. She believed that she was plain
In front of us was walking a bare-headed old man in
and untalented. One day her teacher ordered the entire
tattered clothes. He was driving his beasts. They were all class to show up for audition for the school 900 play.
laden with heavy loads of clay from the hill and looked Cynthia nearly died of fright when she was told that
tired. The man carried a bug whip which perhaps he she would have to sfort stand on stage in front of the
himself made. As he walked down the road he stopped entire class and deliver dialogues. The mere we thought
now and then to eat the wild berries that grew on bushes of it made her feel sick. But a remarkable transformation
along the uneven road. When he threw away the the occurred during the audition. A thin, shy girl, her knees
bold birds and then begametimes a straydon seedched quaking, her stomach churning mughts in terror, began
the procession philosophically and then began to bark. to stun everyone with her excellent performance. Her
When this happened, my two little sons would stand bored classmates suddenly stopped their noisy chat to
still holding my hands firmly. A dog can sometimes be stare at her slender figure on the stage. At the end of her
dangerous indeed. audition, the entire room erupted in thunderous
applause
39. The author's children held his hands firmly
because 41. Cynthia was afraid to stand on stage because
(a) they were scared of the barking dogs. (a) she felt her classmates may laugh at her.
(b) they wanted him to pluck berries. (b) her stomach was churning.
(c) they saw the whip in the old man's hand. (c) she lacked self-confidence.
(d) the road was uneven. (d) she did not like school plays.
Sol. (a) Sol. (c)
The answer can be found in the following lines: In the passage, it has been provided that Cynthia
"Sometimes a stray dog watched the procession believed that she was plain and untalented.
philosophically and then began to bark. When this She did not have faith in her capabilities.
happened, my two little sons would stand still That is, she lacked self confidence.
holding my hands firmly."
These lines imply that the children were scared of 42. Cynthia's classmates were chatting because
barking dogs. (a) it was their turn to act next.
(b) they were bored of the performances.
Therefore, they held the author's hands firmly.
(c) Cynthia did not act well.
40. The expression "a stray dog watched the (d) the teacher had no control over them.
procession philosophically" means that Sol. (b)
(a) the dog was restless and ferocious. It is because Cynthia's classmates were not
(b) the dog stood aloof, looking at the procession interested in the previous performances.
with seriousness. 43. Cynthia's knees were quaking because
(c) the dog looked at the procession with big, (a) she felt nervous and shy.
wondering eyes. (b) the teacher scolded her.
(d) the dog stood there with his eyes closed. (c) she was very thin and weak.
Sol. (b) (d) she was afraid of her classmates.
Philosophy refers to some serious, deep thoughts Sol. (a)
about an object, social event, etc. She felt nervous and shy.

CSAT by Manjul Kumar Tiwari Sir


Previous Year Solved Papers 69
44. The transformation that occurred during the Sol. (a)
audition refers to Person (Gender – Profession), X, Y and Z are used
(a) the nervousness of Cynthia. until the person is unknown.
(b) the eruption of the entire room in thunderous
Statement1 (S1): A (M - Engineer) – “X”
applause.
(F- Stenographer) S2: “Y” (M/F – Judge) – “Z” (M/
(c) the surprise on the faces of her classmates.
F- Lawyer)
(d) the stunning performance of Cynthia.
S3: F (M – Draughtsman) – Son of B and brother of E
Sol. (a)
This means that: E is Doctor and son of B
45. If the 3rd day of a month is Monday, which one of
By S4 the following are clear: C lawyer will the wife
the following will be the fifth day from 21st of this
of B judge. Moreover, D is the stenographer. By S5:
month?
D is the grandmother of F.
(a) Monday (b) Tuesday
(c) Wednesday (d) Friday So, this is the final relationship:

Sol. (c) First Generation : A (M - Engineer) – D (F-


Stenographer)
46. For a charity show, the total tickets sold were 420. |
Half of these tickets were sold at the rate of Rs. 5
Second Generation: B (M, Judge) – C (F, Lawyer)
each, one-third at the rate of Rs. 3 each and the rest
|
for Rs. 2 each. What was the total amount received?
(a) Rs. 900 (b) Rs. 1,540 Third Generation: F (M – Draughtsman) & E (M/F
(c) Rs. 1,610 (d) Rs. 2,000 – Doctor)

Sol. (c) 48. Which of the following is/are couple/couples?


Half of 420 tickets = 210: Price per ticket – Rs. 5; (a) AD only (b) BC only
total = Rs. 1050 One-third at the rate of 3 Rs. Each : (c) Both AD and BC (d) Both AC and BD
total= 140 X 3 = Rs. 420 Sol. (c)
Rest for Rs. 2: Total = 70X 2 = 140 Gross sales = Rs.
49. What is the profession of D?
1,610
(a) Judge
Directions for the following 3 (three) items: (b) Stenographer
(c) Doctor
Read the passage given below and answer the items that
(d) Cannot be determined
follow.
Sol. (b)
A, B, C, D, E, F are members of a family. They are engineer,
Directions for the following 7 (seven) items: Read the
stenographer, doctor, draughtsman, lawyer and judge
following two passages and answer the items that follow
(not in order). A, the engineer is married to the lady
each passage. Your answers to these items should be
stenographer.
based on the passages only.
The judge is married to the lawyer. F, the draughtsman is
Passage -1
the son of B and brother of E. C, the lawyer is the daughter-
in-law of D. E is the unmarried doctor. D is the Many nations now place their faith in capitalism and
grandmother of F. There are two married couples in the governments choose it as the strategy to create wealth for
family. their people. The spectacular economic growth seen in
47. What is the profession of B? Brazil, China and India after the liberalisation of their
(a) Judge economies is proof of its enormous potential and success.
(b) Lawyer However, the global banking crisis and the economic
(c) Draughtsman recession have left many bewildered. The debates tend
(d) Cannot be determined to focus on free market operations and forces, their

CSAT by Manjul Kumar Tiwari Sir


70 CSAT : 2014

efficiency and their ability for self correction. Issues of 1. refers to the false ideology of capitalism.
justice, Integrity and honesty are rarely elaborated to 2. underlies the righteous claims of the free
highlight the failure of the global banking system. The market.
apologists of the system continue to justify the success of 3. shows the benevolent face of capitalism.
capitalism and argue that the recent crisis was a blip. 4. ignores resultant gross inequity.

Their arguments betray an Ideological bias with the Which of the statements given above is/are
assumptions that an unregulated market is fair and correct?
(a) 1 only (b) 2 and 3
competent, and that the exercise of private greed will be
(c) 1 and 4 (d) 4 only
in the larger public interest.
Sol. (c)
Few recognize the bidirectional relationship between
Passage - 2
capitalism and greed; that each reinforces the other.
Net profits are only 2.2% of their total assets for central
Surely, a more honest conceptualisation of the conflicts
public sector undertakings, lower than for the private
of interest among the rich and powerful players who
corporate sector. While the public sector or the State-led
have benefited from the system, their biases and ideology
entrepreneurship played an important role in triggering
is needed; the focus on the wealth. creation should also
India’s industrialization, our evolving development
highlight the resultant gross inequity.
needs, comparatively less-than-satisfactory performance
50. The apologists of the “Free Market System”, of the public sector enterprises, the maturing of our
according to the passage, believe in private sector, a much larger social base now available
(a) market without control by government for expanding entrepreneurship and the growing
authorities. institutional capabilities to enforce competition policies
(b) market without protection by the government. would suggest that the time has come to review the role
(c) ability of market to self correct. of public sector.
(d) market for free goods and services. What should the portfolio composition of the government
Sol. (c) be? It should not remain static all times. The airline
industry works well as a purely private affair. At the
Now, you need to be a little careful and be objective in opposite end, rural roads, whose sparse traffic makes
selecting the options. You need to select the answer tolling unviable, have to be on the balance-sheet of the
“according to the passage”. State. If the government did not own rural roads, they
would not exist.
Option (c) is resulted in lines – “The debates tend to focus
on free market operations and forces, their efficiency and their Similarly, public health capital in our towns and cities
will need to come from the public sector. Equally,
ability for self correction”.
preservation and improvement of forest cover will have
51. With reference to “ideological bias”, the passage to be a new priority for the public sector assets.
implies that Take the example of steel. With near-zero tariffs, India is
(a) free market is fair but not competent. a globally competitive market for the metal. Indian firms
(b) free market is not fair but competent. export steel into the global market which demonstrates
(c) free market is fair and competent. there is no gap in technology. Indian companies are
(d) free market is neither fair nor biased. buying up global steel companies, which shows there is
Sol. (c) no gap in capital availability. Under these conditions,
The ideological bias of the apologists is that free private ownership works best.
markets are fair and competent. But the passage Private ownership is clearly desirable in regulated
criticizes this bias. This implies that free markets industries, ranging from, finance to infrastructure, where
are neither fair, nor competent. a government agency performs the function of regulation
and multiple competing firms are located in the private
52. “The exercise of private greed will be in the larger sector. Here, the simple and clean solution - government
public interest” from the passage as the umpire and the private sector as the players is

CSAT by Manjul Kumar Tiwari Sir


Previous Year Solved Papers 71
what works best. In many of these industries, we have a (d) Private sector need not have any social
legacy of government ownership, where productivity responsibility.
tends to be lower, fear of bankruptcy is absent, and the Sol. (b)
risk of asking for money from the tax payer is ever present. The correct answer is 'Private sector cannot have
There is also the conflict of interest between government monetary gains in this.'
as an owner and as the regulator.
In the passage, it has been argued that there is sparse
The formulation and implementation of competition traffic on the rural roads which makes toll tax
policy will be more vigorous and fair if government collection unviable for the private sector.
companies are out of action. This is because the private sector is more interested
53. According to the passage, what is/are the reason/ in profits and monetary gains.
reasons for saying that the time has come to review Therefore, the private sector does not find it
the role of public sector? advantageous enough to enter into the domain of
1. Now public sector has lost its relevance in rural road development.
the industrialization process. 55. The portfolio composition of the government
2. Public sector does not perform satisfactorily. refers to
3. Entrepreneurship in private sector is (a) Public sector assets quality.
expanding. (b) Investment in liquid assets.
4. Effective competition policies are available (c) Mix of government investment in different
now. industrial sectors.
Which of the statements given above is/are correct (d) Buying Return on Investment yielding capital
in the given context? assets.
(a) 1 and 3 only (b) 2 only Sol. (c)
(c) 2, 3 and 4 only (d) 1, 2, 3 and 4 The correct answer is 'Mix of government
Sol. (c) investment in different industrial sectors.'
According to the passage, the time has come to Economics portfolio refers to a collection of
review the role of public sector because of following financial investments like stocks, bonds,
factors: commodities, etc.
our evolving development needs, comparatively In terms of government presence in the economy, it
less-than-satisfactory performance of the public would refer to government investment in different
sector enterprises, the maturing of our private sector, sectors of the economy.
larger social base now available for expanding The portfolio composition would mean how
entrepreneurship the growing institutional government makes investments in the different
capabilities to enforce competition policies. sectors.
However, the relevance of the public sector is not 56. The author prefers government as the umpire and
completely lost. For e.g., the health industry requires private sector as players because
substantial government presence.“Hence, the (a) Government prescribes norms for a fair play
correct answer is '2, 3 and 4 only'. by the private sector.
(b) Government is the ultimate in policy
54. According to the passage, rural roads should be
formulation.
in the domain of public sector only. Why?
(c) Government has no control over private sector
(a) Rural development work is the domain of
players.
government only.
(d) None of the above statements is correct in this
(b) Private sector cannot have monetary gains in
context.
this.
(c) Government takes money from tax payers and Sol. (a)
hence it is the responsibility of government Passage talks about competition laws and
only. government being out of action.

CSAT by Manjul Kumar Tiwari Sir


72 CSAT : 2014

57. A question paper must have a question on one of 59. The letters L, M, N, 0, P, Q, R, S and T in their order
the eight poets: A, B, C, D, E, F, G or H The first four are substituted by nine integers 1 to 9 but not in
belong to the medieval period while the rest are that order. 4 is assigned to P. The difference
considered modern poets Generally, modern poets between P and T is 5. The difference between N
figure in the question paper in alternate years. and T is 3. What is the integer assigned to N?
Generally those who like H like G also; and those (a) 7 (b) 5
who like F like E also. The paper-setter does not like (c) 4 (d) 6
to ask about F as he has written a book on F, but he Sol. (d)
likes F. Last year, the paper contained a question P = 4,
on A. On the basis of the information given, this T = 4 + 5 = 9,
year’s paper is most likely to contain a question on So, N = 9 – 3 = 6
(a) C (b) E
(c) F (d) H 60. The number of deaths among the army personnel
Sol. (b) is 8 in 1000, but among the civilian population it
is 20 per 1000. Which one of the following
The following is clear from the question:
inferences can be drawn from this statement?
• A,B,C,D – Medieval (a) It is better to join the army.
(b) relationship is fortuitous.
• E,F,G,H – Modern
(c) lity of Life Index is very high within the armed
• Modern poets come alternate years forces.
• H-G, and F-E are liked together. (d) groups cannot be compared due to their
heterogeneity.
• Last year A came (medieval poet) which means Sol. (d)
this year, a modern poet will come
Option (c) cannot be the answer, because just a death
Now, F is out of the scene, but since the examiner rate data can not give quality of life index (that too
likes to ask about F, he also likes E. So, E is the most “very high”).
probable poet for the year. 61. Given the statement: “Buses are the cause of more
58. In a group of six women there are four dancers, accidents than cars, and trucks causes fewer
accidents than buses”, which of the following
four vocal musicians, one actress and three
conclusions can we draw?
violinists. Girija and Vanaja are among the
violinists while Jalaja and Shailaja do not know (a) There are more buses on the road than trucks.
(b) Car drivers are more careful than bus drivers.
how to play on the violin. Shailaja and Tanuja are
(c) Truck drivers are more skilled than either car
among the dancers. Jalaja, Vanaja, Shailaja and
or bus drivers.
Tanuja are all vocal musicians and two of them
(d) None of the above
are also violinists. If Pooja is an actress, who
Sol. (d)
among the following is certainly a dancer and a
violinist? None of the conclusions can be deduced logically
(a) Jalaja (b) Poosa from the statement.
(c) shailaja (d) Tanuja 62. “If political leadership fails to emerge, there is
Sol. (d) likelihood of military taking over power in
4 dancers, 4 vocal, 1 actress, 3 violin. Girija = violin developing countries. Radical student groups or
Vanaja = violin, vocal Jalaja = no violin, vocal labour may try to raise revolution but they are not
likely to compete with the military. Military
Shailja = no violin, dancer, vocal
intervention, rule, and withdrawal from politics
Tanuja = dancer, vocal, violin (2 of them are also is closely related to a society’s level of political
violin) Pooja = actress development.”

CSAT by Manjul Kumar Tiwari Sir


Previous Year Solved Papers 73
In the context of political development, the Tues = Sports
assumption in the above passage is that Wed = Maths
(a) political leadership is not an effective Thus = History
instrument. Friday = not Lit,
(b) military fills in political vacuum.
(c) military intervention is inevitable for 65. In a row ‘A’ is in the 11th position from the left
development. and ‘B’ is in the 10th position from the right. If ‘A’
(d) None of the above and ‘B’ interchange, then ‘A’ becomes 18th from
Sol. (b) the left. How many persons are there in the row
The key word is assumption, the line in passage. other than ‘A’ and ‘B’?
(a) 27 (b) 26
If the political leadership fails then emerge, there is
(c) 25 (d) 24
military taking rest over. Its shows vaccum is filled
by the military leadership. Sol. (c)
Here is the arrangement as per the question:
63. Four persons, Alok, Bhupesh, Chander and Dinesh
10 people - A – ‘x’ people in between – B - 9 people.
have a total of Rs. 100 among themselves. Alok
and Bhupesh between them have as much money OR
as Chander and Dinesh between them, but Alok 10 people – B – ‘x’ people - A - 9 people.
has more money than Bhupesh; and Chander has Now,
only half the money that Dinesh has. Alok has in 10 + 1 + x = 17. X = 6. Total people other than A and
fact Rs. 5 more than Dinesh has. Who has the B = 10 + 6 + 9 = 25.
maximum amount of money?
(a) Alok (b) Bhupesh 66. Location of B is north of A and location of C is east
(c) Chander (d) Dinesh of A. The distances AB and AC are 5 km and 12 km
Sol. (a) respectively. The shortest distance (in km) between
the locations B and C is
Alok + Bhupesh = Chander + Dinesh
(a) 60 (b) 13
Alok > Bhupesh
(c) 17 (d) 7
Dinesh = 2 Chander Sol. (b)
Alok = Dinesh + 5
Here is the arrangement.
Clearly, Alok has highest.
B
64. Examine the following statements: |
1. George attends Music classes on Monday. A ____ C
2. He attends Mathematics classes on
Applying the Pythagoras theorem:
Wednesday.
3. His Literature classes are not on Friday. BC2 BC2 = AB2 +AC2
4. He attends History classes on the day = AB2
following the day of his Mathematics classes. + AC2
5. On Tuesday, he attends his Sports classes.
= 52
If he attends just one subject in a day and his + 122
Sunday is free, then he is also free on
= 25 + 144 = 169.
(a) Monday (b) Thursday
(c) Saturday (d) Friday So BC = 13.
Sol. (d) 67. Two cars start towards each other, from two places
The statements point out that A and B which are at a distance of 160 km. They
Monday = Music start at the same time 08 : 10 AM. If the speeds of

CSAT by Manjul Kumar Tiwari Sir


74 CSAT : 2014

the cars are 50 km and 30 km per hour respectively, 1. melting of glaciers.


they will meet each other at 2. water availability and temperature suitability
(a) 10 : 10 AM (b) 10 : 30 AM at other locations.
(c) 11 : 10 AM (d) 11 : 20 AM 3. poor productivity of crops.
Sol. (a) 4. wider adaptability of crop plants.
Total distance = 160 km. Which of the statements given above are correct?
Using the concept of relative speed, the speed at (a) 1, 2 and 3 (b) 2 and 3 only
which one car will approach the other (if other is (c) 1 and 4 only (d) 1, 2, 3 and 4
kept stationary) is = 50 + 30 = 80 km/h. So they will Sol. (b)
meet after 160 / 80 = 2 hours. So time of meet is
The correct answer is "b".
10:10 AM.
The second para clearly mentioned that to ensure
Directions for the following 6 (six) items: continued yields.
Read the following two passages and answer the items
It is necessary to change the location because of
that follow each passage. Your answers to these items
water availability, draught proofing, poor produvity
should be based on the passages only.
of crops.
Passage -1 Passage doesn't mention about melting glaciers
Climate change poses potentially devastating effects on impact and water adaptability.
India’s agriculture. While the overall parameters of
climate change are increasingly accepted a 1°C average 69. According to the passage, why is it important to
temperature increase over the next 30 years, sea level rise promote agricultural research in India?
of less than 10 cm in the same period, and regional (a) To predict variations in monsoon patterns and
monsoon variations and corresponding droughts - the to manage water resources
impacts in India are likely to be quite site and crop specific. (b) To make long term investment decisions for
Some crops may respond favourably to the changing economic growth
conditions, others may not. This emphasizes the need to (c) To facilitate wider adaptability of crops
promote agricultural research and create maximum (d) To predict drought conditions and to recharge
flexibility in the system to permit adaptations. aquifers
The key ingredient for “drought proofing” is the managed Sol. (c)
recharge of aquifers. To ensure continued yields of The central idea of the passage is to facilitate wider
important staple crops (e.g. wheat), it may also be adaptability of crops for which it advocates
necessary to shift the locations where these crops are promoting agricultural research.
grown, in response to temperature changes as well as to
Passage – 2
water availability. The latter will be a key factor in making
It is essential that we mitigate the emissions of
long term investment decisions.
greenhouse gases .and thus avoid some of the worst
For example, water runoff from the Himalayas is
impacts of climate change that would take place in coming
predicted to increase over the next 30 years as glaciers
years and decades. Mitigation would require a major
melt, but then decline substantially thereafter. It will be
shift in the way we produce and consume energy. A shift
critical to provide incentives to plan for these large-scale
shifts in agro-ecological conditions. away from overwhelming dependence on fossil fuels is
now long overdue, but unfortunately, technological
India needs to make long term investment in research
development has been slow arid inadequate largely
and development in agriculture. India is likely to
because government policies have not promoted
experience changed weather patterns in future.
investments in research and development, myopically
68. Consider the following statements: as a result of relatively low prices of oil. It is now, therefore,
Climate change may force the shifting of locations
imperative for a country like India treating the
of the existing crops due to
opportunity of harnessing renewable energy on a large

CSAT by Manjul Kumar Tiwari Sir


Previous Year Solved Papers 75
scale as a national imperative. This country is extremely Select the correct answer using the code given
well endowed with solar, wind and biomass sources of below:
energy. Where we have lagged, unfortunately, is in our (a) 1, 2 and 3 (b) 2, 3 and 4
ability to develop and to create technological solutions (c) 1 and 3 only (d) 2 and 4 only
for harnessing these resources. Sol. (b)
One particular trajectory for carrying out stringent In the passage, it has been argued that in order to
mitigation of greenhouse gas emissions assessed by the reduce greenhouse gas emissions and mitigate the
Inter-governmental Panel on Climate Change (IPCC) worst impacts of climate change, it is required that
clearly shows the need for ensuring that global emissions along with conscious technological efforts to reduce
global emissions of greenhouse gases, changes in
of greenhouse gases peak no later than 2015 and reduce
lifestyle are a must.
rapidly thereafter. The cost associated with such a
trajectory is truly modest” and would amount, in the The lifestyle in today's world is dominated by high
levels of consumerism, that is, purchasing more and
estimation of IPCC, to not more than 3 percent of the
more commodities based on what is trending and
global GDP in 2030. In other words, the level of prosperity
socially desirable rather than what is required.
that the world would have reached without mitigation
Also, high levels of meat consumption contribute
would at worst be postponed by a few months or a year
as much as 18% to greenhouse gas emissions.
at the most. This is clearly not a very high price to pay
for protecting hundreds of millions of people from the Therefore, reducing the consumption of meat and
reducing consumerism would help in the mitigation
worst risks associated with climate change. Any such
of greenhouse gases.
effort, however, would require lifestyles to change
appropriately also. Mitigation of greenhouse gas 71. Why do we continue to depend on the fossil fuels
emissions is not a mere technological fix, and clearly heavily?
requires changes in lifestyles and transformation of a 1. Inadequate technological development
2. Inadequate funds for research and
country’s economic structure, whereby effective
development
reduction in emissions is brought about, such as through
3. Inadequate availability of alternative sources
the consumption of much lower quantities of animal
of energy
protein. The Food and Agriculture Organization (FAO)
has determined that the emissions from the livestock Select the correct answer using the code given
below:
sector amount to 18 percent of the total. The reduction of
(a) 1 only (b) 2 and 3 only
emissions from this source is entirely in the hands of
(c) 1 and 3 only (d) 1, 2 and 3
human beings, who have never questioned the impacts
Sol. (d)
that their dietary habits of consuming more and more
The passage mentioned that a major shift from fossil
animal protein are bringing about. Mitigation overall
fuels is long over due because of :
has huge co-benefits, such as lower air pollution and
1. Slow technological development
health benefits, higher energy security and greater
2. Inadequate fund
employment.
3. Lack of alternative sources
70. According to the passage, which of the following
would help in the mitigation of greenhouse gases? 72. According to the passage, how does the mitigation
1. Reducing the consumption of meat of greenhouse gases help us?
1. Reduces expenditure on public health
2. Rapid economic liberalization
2. Reduces dependence on livestock
3. Reducing the consumerism 3. Reduces energy requirements
4. Modern management practices of livestock 4. Reduces rate of global climate change

CSAT by Manjul Kumar Tiwari Sir


76 CSAT : 2014

Select the correct answer using the code given 75. A gardener increased the area of his rectangular
below: garden by increasing its length by 40% and
(a) 1, 2 and 3 (b) 1, 3 and 4 decreasing its width by 20%. The area of the new
(c) 2, 3 and 4 (d) 1 and 4 only garden
Sol. (b) (a) has increased by 20%.
In the passage, it has been highlighted that (b) has increased by 12%.
mitigation of climate change has the following co- (c) has increased by 8%.
benefits: (d) is exactly the same as the old area.

Improved public health, leading to reduced Sol. (b)


expenditure on health“Reduced rate and impact New length = 1.4 times the old length (say L) = 1.4L
of climate change. New width = 0.8 W
Energy security, a dimension of which is demand- Area (new) = 1.12 LW (12% increase)
side management, i.e, reducing energy
requirements. 76. Six books are labelled A, B, C, D, E and F and are
However, reduced dependence on livestock- placed side by side. Books B, C, E and F have green
related more to a strategy of climate change covers while others have yellow covers. Books A,
mitigation rather than a direct benefit. B and D are new while the rest are old volumes.
Books A, Band Care law reports while the rest are
73. What is the essential message of the passage?
medical extracts. Which two books are old medical
(a) We continue to depend on fossil fuel heavily
extracts and have green covers?
(b) Mitigation of the greenhouse gases is (a) B and C (b) E and F
imperative (c) C and E (d) C and F
(c) We must invest in research and development Sol. (b)
(d) People must change their lifestyle BCEF – Green; AD – Yellow
Sol. (b) ABD – New; CEF – Old
The clear answer is mitigation of GHG is imperative, ABC – Law; DEF – Medical
As passage argued this in complete passage. E and F are medical books with green covers.

74. There are 50 students admitted to a nursery class. 77. A straight line segment is 36 cm long. Points are to
Some students can speak only English and some be marked on the line from both the end points.
can speak only Hindi. 10 student can speak both From each end, the first point is at a distance of 1
English and Hindi. If the number of students who cm from the end, the second point is at a distance
can speak English 21, then how many students of 2 cm from the first point and the third point is at
can speak Hindi how many can speak only Hindi a distance of 3 cm from the second point and so
and how many can speak only English ? on. If the points on the ends are not counted and
(a) 21, 11 and 29 respectively the common points are counted as one, what is the
(b) 28, 18 and 22 respectively number of points?
(a) 10 (b) 12
(c) 37, 27 and 13 respectively
(c) 14 (d) 16
(d) 39, 29 and 11 respectively
Sol. (b)
Sol. (d)
First point is at a distance of 1 cm, 2nd at 2 cm and
Diagram Since 21 people can speak English (and
so on – therefore, the distance of each point from
10 can speak both), so only English speaking crowd
the left edge of the scale will be
is 11. Total is 50, so only Hindi people will be
1st point – 1 cm
50 – 11 = 39.
Eng (11) 10 Hindi (39) 2nd– 3 cm

CSAT by Manjul Kumar Tiwari Sir


Previous Year Solved Papers 77
3rd– 6 cm (a) 45 (b) 44
4th– 10 cm (c) 38 (d) 30

5th– 15 cm Sol. (b)

6th– 21 cm Using Venn diagrams

7th– 28 cm You can also see this by drawing a Set diagram.


G V
8th– 36 cm
Similarily from the right edge too, the same story 40 Y

repeats. So we have totally 8+8 = 16 points. 6


Z
But, common points are to be counted as one and X
end points not to be counted.
End points are the 8th point drawn from both the
sides. So reduce 2 points from 16 which becomes F

14. A= 6
Now common points – so 5th point from the left; X+Y+Z (who play only two) = 30
and 6th point from the right are common and will G+V+F+(X+Y+Z+A) = 120 [G,V,F are people who
repeat twice. So they have to be counted only once. can play only guitar, violin or flute]
Reduce 2 more points from 14 which becomes 12 as So, put all the data, and you have
the answer. 40 + V+ F + 30 + 6 = 120

78. If Sohan, while selling two goats at the same price, So, V+F (those who can play flute alone or violin
makes a profit of 10% on one goat and suffers a alone) = 44
loss of 10% on the other 80. Six identical cards are placed on a table. Each card
(a) he makes no profit and no loss. has number ‘1’ marked on one side and number
(b) he makes a profit of 1%. ‘2’ marked on its other side. All the six cards are
(c) he suffers a loss of 1%. placed in such a manner that the number ‘1’ is on
(d) he suffers a loss of 2%. the upper side. In one try, exactly four (neither
Sol. (c) more nor less) cards are turned upside down.
Let cost of 1st goat be Rs. 100. Then its selling price In how many Ieast number of tries can the cards be
= 110 = selling price of goat 2. But he suffers loss of turned upside down such that all the six cards
10% on goat 2 which means cost of goat 2 = 110/.9 show number ‘2’ on the upper side?
= 122.22 (a) 3
Total cost = 222.22, total selling price = 220. Means (b) 5
loss of ~1%. (c) 7
(d) This cannot be achieved
79. Out of a total of 120 musicians in a club, 5% can Sol. (a)
play all the three instruments, guitar, violin and
Before trying, the Cards show: 111111
flute. It so happens that the number of musicians
First try (make any 4 cards show 2): 222211
who can play any and only two of the above
instruments is 30. The number of musicians who 2nd try (take one of those showing 1 and three of
those showing two and flip): 211121
can play guitar alone is 40. What is the total
number those who can play violin alone or flute 3rd try (Flip all the ones): 222222...
alone? 

CSAT by Manjul Kumar Tiwari Sir


78 CSAT : 2015

Previous Year
CSAT : 2015 Solved Papers

Directions for the following 8 (eight) items: preserve farmland. Poor countries, they say, can improve
Read the following six passages and answer the items crop storage and packaging and rich nations could cut
that follow. Your answers to these items should be based back on resource-intensive foods like meat.
on the passages only.
2. Which one of the following statements best sums
Passage -1 up the above passage?
Human history abounds in claims and theories confining (a) The population of the world is growing very
the right of governing to a few select citizens. Exclusion fast.
of the many is justified on the ground that human beings (b) Food security is a perennial problem only in
may be rightfully segregated for the good of society and developing countries.
viability of the political process. (c) The world does. not have enough resources
to meet the impending food scarcity.
1. Which one of the following statements is least (d) Food security is increasingly a collective
essential as a part of the argument in the above challenge.
passage?
Sol. (d)
(a) Man seeks control over external things
affecting him. The passage talks about the increasing population
(b) In society, there are 'super' and 'sub' human of the world and the effort that needs to be made in
beings. order to ensure that the growth of population does
(c) Exceptions to participation are efficacy. not outstrip the food production. Option (d) sums it
universal citizen conducive to systemic up most accurately.
(d) Governing implies recognition of disparities
Passage - 3
in individual capacities.
Many people in India feel that if we cut our defence
Sol. (a)
expenditure on weapon-building, we can create a climate
The passage is about the difference in disparities in of peace with our neighbours, subsequently reducing the
individual capacities and how this impacts the conflict or creating a no-war situation. People who
polity. Least essential to the argument is option (a). proclaim such ideas are either the victims of war or the
Passage -2 propagators of false argument.
By 2050, the Earth's population will likely have swelled 3. With reference to the above passage, which of the
from seven to nine billion people. To fill all thm,e following is the most valid assumption?
stomachs - while accounting for shifting consumption (a) Building of weapons systems by us has
patterns, climate change, and a finite amount of arable instigated our neighbours to wage wars
land and potable water - some experts say food against us.
production will have to double. How can we make the (b) The greater spending on weapon-building by
numbers add up? Experts say higher yielding crop us would lessen the possibility of armed
varieties and more efficient farming methods, vill be conflict with our neighbours.
crucial. So will waste reduction. Experts urge cities to (c) It is necessary to have state of the art weapons
reclaim nutrients and water from waste streams and systems for national security.

CSAT by Manjul Kumar Tiwari Sir


Previous Year Solved Papers 79
(d) Many people in India believe that we are during processing and transportation, destruction of rain
wasting our resources on weapon-building. forests, reduced nutritional content, increased demand
Sol. (b) for preservation and packaging. Food insecurity also
The passage talks about the argument that increase increases as the produce comes from regions that are not
in defence expenditure creates a climate of peace. feeding their own population properly.
The most valid assumption in favour of this 5. With reference to the above passage, which of the
argument is that of the option (b). following statements is/are true?
Passage - 4 1. Consuming regionally grown food and not
India accounts for nearly a fifth of the world's child depending on long travelled food is a part of
eco-friendly behaviour.
deaths. In terms of numbers, it is the highest in the world
2. Food processing industry puts a burden on
- nearly 16 lakhs every year. Of these, more than half die
our natural resources.
in the first month of life. Officials believe that the reason
for this is the absence of steps to propagate basic health Select the correct answer using the code given
practices relating to breast feeding and immunisation. below:
Also the large reproductive population of 2.6 crore (a) 1 only (b) 2 only
remains bereft of care during the critical phases of (c) Both 1 and 2 (d) Neither 1 nor 2
pregnancy and post-delivery. Added to this is the Sol. (c)
prevalence of child marriages, anaemia among young The passage talks about the harmful impact of
women and lack of focus on adolescent sanitation, all of processed food on environment. Statement 1 is
which impact child death rates. implied. Also food processing puts a lot of strain
on our natural resources. Statement 2 is explicitly
4. Which is the critical inference that can be made
mentioned. Hence the correct answer is option (c).
from the above passage?
(a} A lot of Indians are illiterate and hence do not Passage-6
recognize the value of basic health practices. I must say that, beyond occasionally exposing me to
(b) India has a very huge population and the laughter, my constitutional shyness has been of no
government alone cannot manage public disadvantage whatever. In fact I can see that, on the
health services. contrary, it has been all to my advantage. My hesitancy
(c) Universalization and integration of maternal in speech, which was once an annoyance, is now a
health and child health services can pleasure. Its greatest benefit has been that it has taught
effectively address the problem. me the economy of words. I have naturally formed the
(d) The nutrition of women in child bearing age habit of restraining my thoughts. And I can now give
does not affect child mortality rate. myself the certificate that a thoughtless word hardly ever
Sol. (c) escapes my tongue or pen. I do not recollect ever having
had to regret anything in my speech or writing. I have
The passage talks about lack of healthcare facilities
thus been spared many a mishap and waste of time.
as one of the major reasons for high mortality. Hence
Experience has taught me that silence is part of the
the inference that can be drawn is universalization
spiritual discipline of a votary of truth. Proneness to
of maternal and child health services can effectively
exaggerate, to suppress or modify the truth, wittingly or
address the problem. Hence the correct answer is
unwittingly, is a natural weakness of man, and silence
option (c).
is necessary in order to surmount it. A man of few words
Passage - 5 will rarely be thoughtless in his speech; he will measure
Foods travel more than the people who eat them. Grocery every word. We find so many people impatient to talk.
stores and supermarkets are loaded with preserved and There is no chairman of a meeting who is not pestered
processed foods. This, however, often leads to with notes for permission to speak. And whenever the
environmental threats, such as pollution generated by permission is given the speaker generally exceeds the
long distance food transportation and wastage of food time-limit, asks for more time, and keeps on talking

CSAT by Manjul Kumar Tiwari Sir


80 CSAT : 2015

without permission. All this talking can hardly be said 10. An automobile owner reduced his monthly petrol
to be of any benefit to the world. It is so much waste of consumption when the prices went up. The price-
time. My shyness has been in reality my shield and consumption relationship is as follows:
buckler. It has allowed me to grow. It has helped me in
Price (in Rs. per litre) 40 50 60 75
my discernment of truth.
Monthly consumption (in litres) 60 48 40 32
6. The author says that a thoughtless word hardly
ever escapes his tongue or pen. Which one of the If the price goes up to Rs. 80 per litre, his expected
following is not a valid reason for this? consumption (in litres) will be
(a) He has no intention to waste his time. (a) 30 (b) 28
(b) He believes in the economy of words. (c) 26 (d) 24
(c) He believes in restraining his thoughts. Sol. (a)
(d) He has hesitancy in his speech.
If we look at the given data, the total expenditure is
Sol. (a) always Rs. 2,400 If the price goes up to Rs 80 per
litre, the expected consumption would be 2,400/80
7. The most appropriate reason for the author to be
30 litres. The correct answer is option (a).
spared many a mishap is that
(a) he hardly utters or writes a thoughtless word. 11. Consider the figures given below :
(b) he is a man of immense patience.
(c) he believes that he is a spiritual person.
(d) he is a votary of truth.
To fit the question mark, the correct answer is:
Sol. (a)
The most appropriate reason why the author has
been spared a mishap is that he had never written a
thoughtless word. Option (a) is the best answer.

8. For the author, silence is necessary in order to


surmount
(a) constitutional shyness.
(b) hesitancy in speech.
(c) suppression of thoughts.
(d) tendency to overstate.
Sol. (d)
According to the author tendency to overstate is the
natural for a human. Silence helps him to surmount
and overcome this tendency. The best answer is
option (d)
Sol. (a)
9. Twelve people form a club. By picking lots, one of The correct answer is option (a)
them will host a dinner for all once in a month.
The number of dinners a particular member has to 12. Consider the following matrix:
host in one year is
(a) One (b) Zero
(c) Three (d) Cannot be predicted
Sol. (d)
What is the missing number at ‘?’ in the matrix?
In the question no additional conditions are given
(a) 5 (b) 0
like a member can host a party only once. Hence the
correct answer is option (d). (c) 7 (d) 3

CSAT by Manjul Kumar Tiwari Sir


Previous Year Solved Papers 81
Sol. (a) 15. In a test, a candidate attempted only 8 questions
The logic is simple_3X -3 = 6; 8 × 8 - 8. 56 and so on. and secured 50% marks in each of the questions. If
The last calculation should be 5 X 5-5 = 20. Hence he obtained a total of 40%, in the test and all
the correct answer is option (a). questions in the test carried equal marks, how
many questions were there in the test?
13. What is the missing number ‘X’ of the series (a) 8 (b) 10
7, X, 21, 31, 43? {c) 15 (d) 16
(a) 11 (b) 12 Sol. (b)
(c) 13 (d) 14
Given:
Sol. (c)
Candidate attempt only 8 questions
The sequence of the series 7, X, 21, 31, 43 is as
Explanation:
follows:
Let the marks of each question is 10.
Total marks got by the candidate = 8 × (50% of 10)
= 8 × 5 = 40 marks
According to question, he obtained total of 40%
Hence the missing number is 13. marks

14. Four cardboard pieces of specific shapes are shown So, 40% = 40 marks
in the following figure :  100% = 100 marks
Total marks of the test = 100
 Total number of questions = 100/10 = 10
 The require answer is 10.
Hence, option (b) is correct.

16. A father is nine times as old as his son and the


Which one of the following figures given can be mother is eight times as old as the son. The sum of
formed by joining these pieces together? the father’s and the mother’s age is 51 years. What
is the age of the son?
(a) 7 years (b) 5 years
(c) 4 years (d) 3 years
Sol. (d)
Suppose the son’s age is x years.
Father’s age = 9x
Mother’s age = 8x
Sum of the ages is 9x + 8x = 17x = 51
Hence x = 3 which is the age of the son.
Hence the correct answer is option (d)

17. Four persons A, B, C and D consisting of two


married couples are in a group. Both the women
are shorter than their respective husbands. A is
the tallest among the four. C is taller than B. D is
Sol. (d) B’s brother. In this context, which one of the
The correct answer is option (d). following statements is not correct?

CSAT by Manjul Kumar Tiwari Sir


82 CSAT : 2015

(a) All four have family ties. 20. All good athletes want to win and all athletes who
(b) B is the shortest-among the four. want to win eat a well-balanced diet; therefore all
(c) C is taller than D. athletes who do not eat a well-balanced diet are
(d) A is B’s husband. bad athletes.
The best conclusion from this statement is that
Sol. (c)
(a) no bad athlete wants to win.
A is the tallest, hence has to be a male. (b) no athlete who does not eat a well-balanced
D is B’s brother, hence D is a male diet is a good athlete.
Therefore the women are B and C. (c) every athlete who eats a well-balanced diet is
a good athlete.
A cannot be D’S wife. Hence B is A’s wife. Therefore,
(d) all athletes who want to win are good
C is D’S wife.
athletes.
Hence D is taller than C. Correct answer is option (c). Sol. (b)
18. Consider the following statements :
1. A man had a wife, two sons and two Directions for the following 8 (eight) items:
daughters in his family. Read the following seven passages and answer the items
2. The daughters were invited to a fest and the that follow. Your answers to these items should be based
male members of the family went out to take on the passages only.
part in a picnic.
Passage -1
3. The man’s father did not return from his
work. The richer States have a responsibility to cut down carbon
Which of the following statements is true? emissions and promote clean energy investments. These
(a) Only the man’s wife was left at home. are the States that got electricity, grew faster and now
(b) It is likely that the man’s wife was left at home. have high per capita income, making them capable of
(c) None was left at home. sharing India’s burden of becoming eco-friendly. Delhi,
(d) More than one person was left at home. for example, can help by generating its own clean
Sol. (b) electricity using solar rooftop panels or even help poor
States finance their clean energy projects. It is no secret
19. Geeta : Naresh has become a better boxer since he
that State Electricity Boards, which control 95% of the
started meditation.
distribution network, are neck-deep in losses. These
Radha : Impossible. A boxer’s most important asset losses further discourage State utilities from adopting
is his aggressiveness. renewable energy as it is more expensive than fossil
Radha’s statement reflects her belief that fuels.
(a) meditation tends to make a person less
21. Which among the following is the most logical
aggressive.
and ratwnal assumption that can be made from
(b) meditation has little or no effect on the person
the above passage?
who practises it.
(c) Naresh was a poor boxer earlier because he (a) The richer States must lead in the production
was not aggressive enough. and adoption of renewable energy.
(d) Naresh would not have taken to meditation (b) The poor States always have to depend on
as he was a boxer. rich States for electricity.
Sol. (a)
(c) The State Electricity Boards can improve their
Radha says that a boxer’s most important asset is
finances by undertaking clean energy
his aggressiveness and it is impossible that Naresh
projects.
becomes a better boxer because he has started
meditation. Hence, she believes that meditation (d) The high economic disparity between the rich
would tend to make a person less aggressive. The and poor States is the major cause of high
correct answer is option (a). carbon emissions in India.

CSAT by Manjul Kumar Tiwari Sir


Previous Year Solved Papers 83
Sol. (a) (a) The true aim of government is to secure the
The passage states that “the richer states have a citizens their social and political freedom.
responsibility to cut down carbon emissions and (b) The primary concern of government is to
promote clean energy investments. Hence the provide absolute social security to all its
correct answer is option (a). citizens.
(c) The best government 1s the one that allows the
Passage - 2 citizens to enjoy absolute liberty in all matters
Set against a rural backdrop, ‘Stench of kerosene’ is the of life.
stoq of a couple, Guleri and Manak, who have been (d) The best government is the one that provides
happily married for several years but do not have a child. absolute physical security to the people of the
Mnnak’s mother is desperate to have a grandchild to country.
carry on the family name. Hence, she gets Manak Sol. (a)
remarried in Guleri’s absence. Manak, who acts as a
According to the passage, the objective of
reluctant but passive spectator, is meanwhile, informed
Government is free every man from fear and give
by a friend that. Guleri, on hearing about her husband’s
him all freedom to develop his mindset. Option (a)
second marriage, poured kerosene on her clothes and
is the best possible answer.
set fire to them. Manak is heartbroken and be1-,rins to
livP as if hP ·were a dead man. \iVhen his second wife Passage- 4
delivers a son, Manak stares at the child for a long time Our municipal corporations are understaffed. The issue
and blurts out, “Take him away! He stinks of kerosene.” of skills and competencies of the staff poses an even
greater challenge. Urban services delivery and
22. This is a sensitive issue-based story which tries to
infrastructure are complex to plan and execute. They
sensitise the readers about
require a high degree of specialization and
(a) Male chauvinism and infidelity
professionalism. The current framework within which
(b) Love and betrayal
municipal employees, including senior management, are
(c) Lack of legal safeguards for women
recruited does not adequately factor m the technical and
(d) Influence of patriarchal mindset
managerial competencies required. Cadre and
Sol. (d)
recruitment rules only specify the bare minimum in
Option (a) is eliminated because there is no infidelity academic qualifications. There is no mention of
involved. Manak was forcibly remarried. Also, managerial or technical competencies, or of relevant
betrayal is not the theme of the passage. More than work experience. This is the case with most municipal
lack of legal safeguards the passage brings out the corporations. They also suffer from weak organisation
influence of a patriarchal mindset. Hence the correct design and structure. logical and rational assumption
answer is option (d). that can be made from the above passage?
Passage -3 24. Which among the following is the most logical
The ultimate aim of government rn not to rule or control and rational assumption that can be made from
by fear, nor to demand obedience, but conversely, to free the above passage?
every man from fear, that he may live in all possible (a) The task of providing urban services is a
security. In other words, to strengthen his natural right complex issue which requires the
to exist and work without injury to himself or others. organisational expansion of municipal bodies
The object of government is not to change men from all over the country.
rational beings into beasts or puppets. It should enable (b) Our cities can pro\·ide better quality of life if
them to develop their minds and bodies in security, and our local government bodies have adequate
to employ their reason unshackled. staff with required skills and competencies.
23. Which among the following is the most logical (c) Lack of skilled staff is due to the absence of
and rational inference that can be made from the institutions which offer the requisite skills in
above passage? city management.

CSAT by Manjul Kumar Tiwari Sir


84 CSAT : 2015

(d) Our country is not taking advantage of the 26. What is the most logical, rational and crucial
demographic dividend to manage the message that is implied in the above passage?
problems associated with rapid urbanization. (a) MGNREGS should be extended only to those
Sol. (b) who have a bank account.
The theme of the passage is lack of a specialization (b) The paper-based system of payments is more
and professionalism within the municipal efficient than electronic payment in the
employees and understaffing in municipal present scenario.
corporations. Hence the correct answer is option (b). (c) The goal of electronic wage payments was not
to eliminate mediation by village leaders.
Passage - 5 (d) It is essential to provide financial literacy to
Flamingos in large flocks in the wild are social and the rural poor.
extremely loyal. They perform group mating dances.
Sol. (d)
Parents are very fond of their chicks, gathering them
into creches for protection while both males and females Though most of the passage discusses the result of
fly off to search for food. a study of MGNREGS, it is used as an example to
prove the disadvantages of financial illiteracy.
25. Which among the following is the most logical Hence the correct answer is option (d)
corollary to the above passage?
(a) Mass nesting in all species of birds is essential Passage- 7
to ensure complete survival of their offspring. Individuals, groups and leaders who promote human
(b) Only birds have the capacity to develop social development operate under strong institutional,
behaviour and thus can do mass nesting to structural and political constraints that affect policy
raise their chicks in safety. options. But experience suggests broad principles for
(c) Social behaviour in some species of birds
shaping an appropriate agenda for human development.
increases the odds of survival in an unsafe
One important finding from several decades of human
world.
(d) All species of birds set up creches for their development experience is that focusing exclusively on
chicks to teach them social behaviour and economic growth is problematic. While we have good
loyalty. knowledge about how to advance health and education,
Sol. (c) the causes of growth are much less certain and growth is
The passage mentions that flamingos are social and often elusive. Further, an unbalanced emphasis on
how they protect the chicks. Hence option (c) is the growth is often associated with negative environmental
best answer. consequences and adverse distributional effects. The
experience of China, with its impressive growth record,
Passage-6
reflects these broader concerns and underlines the
Vast numbers of Indian citizens without bank accounts importance of balanced approaches that emphasize
live in rural areas, are financially and functionally
investments in the non-income aspects of human
illiterate, and have little experience with technology. A
development.
research study was conducted in a particular area in
wJ::i.ich electronic wage payments in Mahatma Gandhi 27. With reference to the above passage, consider the
National Rural Employment Guarantee Scheme following statements :
(MGNREGS) are meant to go directly to the poor. It was 1. In developing countries, a strong institutional
observed that recipients often assume that the village framework is the only requirement for human
leader needs to mediate the process, as was the case development and policy options.
under the previous paper-based system. Among
2. Human development and economic growth
households under this research study area who claimed
are not always positively inter-related.
to have at least one bank account, over a third reported
still receiving MGNREGS wages in cash directly from a 3. Focusing only on human development
village leader. should be the goal of economic growth.

CSAT by Manjul Kumar Tiwari Sir


Previous Year Solved Papers 85
Which of the above statements is/are correct? (a) C is the richest.
(a) 1 only (b) 2 and 3 only (b) Dis the poorest.
(c) 2 only (d) 1, 2 and 3 (c) C has more than what A and D have together.
Sol. (c) (d) B is richer than D.
The passage specifically mentions that focusing Sol. (c)
only on economic growth has been problematic for C has exactly as much as A and D put together.
human development. However it does not mention Hence statement (c) is incorrect answer.
that focusing on human development should be the
31. In a town, 45% population read magazine A, 55%
only objective. Hence, statements 1 and 3 are wrong
read magazine B, 40% read magazine C, 30% read
and statement 2 is correct. The correct answer is
magazines A and B, 15% read magazines B and C,
option (c).
25% read magazines A &nd C; and 10% read all
28. With reference to the above passage, the following the three magazines.
assumptions have been made: What percentage do not read any magazine?
1. Higher economic growth is essential to ensure (a) 10% (b) 15%
reduction in economic disparity. (c) 20% (d) 25%
2. Environmental degradation is sometimes a Sol. (c)
consequence of economic growth.
Which of the above assumption/assumptions?
(a) 1 only (b) 2 only
(c) Both 1 and 2 (d) Neither 1 nor 2
Sol. (b)
Statement 2 is mentioned in the passage. Higher
economic growth is essential for human
development. However, benefits of this economic
growth should accrue to all sections of the society.
Total percent of people who read at least
29. If A runs less fast than B, and B runs as fast but not 1 magazine 80%. 20% do not read any magazine
faster than C; then, as compared to A, C runs The correct answer is option (c)
(a) slower than A
(b) faster than A 32. Examine the following statements:
(c) with same speed as A 1. Lady’s finger is tastier than cabbage.
(d) Given data is not sufficient to determine 2. Cauliflower is tastier than lady’s finger.
Sol. (b) 3. Cabbage is not tastier than peas.
B and C would be faster than A. Hence the correct The conclusion that can be drawn from these
answer is option (b). statements is that
A B C D
(a) peas are as tasty as lady’s finger.
Initial Amounts 100 100 100 100 (b) peas are as tasty as cauliflower and lady’s
+20 - - - finger.
A pays Rs. 20 to B
80 120 100 100 (c) cabbage is the least tasty of the four vegetables.
-10 +10 - -
B pays Rs. 0 to C (d) cauliflower is tastier than cabbage.
80 110 120 100
C gets Rs. 30 from D - - +30 -30 Sol. (d)
Final Amounts 80 110 140 70
According to statements 1 and 2, Lady’s finger is
tastier than cabbage and Cauliflower is tastier than
30. Each of A, B, C and D has ~ 100. A pays ~ 20 to B,
lady’s finger. Hence it can be concluded that
who pays ~ 10 to C, who gets ~ 30 from D. In this
cauliflower is tastier than cabbage. The correct
context, which one of the following statements is
answer is option (d).
not correct?

CSAT by Manjul Kumar Tiwari Sir


86 CSAT : 2015

33. Shahid and Rohit start from the same point in Water in second glass = 1 – 1/4 = 3/4
opposite directions. After each 1 km, Shahid always So, the total water = 2/3 + 3/4 = 17/12
turns left and Rohit always turns right. Which of
Then ratio of milk and water = (7/12) : (17/12)
the following statements is correct?
Ratio of milk and water = 7 : 17
(a) After both have travelled 2 km, the distance
between them is 4 km.  The required answer is 7 : 17.
(b) They meet after each has travelled 3km.
36. Out of 130 students appearing in an examination,
(c) They meet for the first time after each has
62 failed in English, 52 failed in Mathematics,
travelled 4 km.
whereas 24 failed in both English and Mathematics.
(d) They go on without ever meeting again.
The number of students who passed finally is
Sol. (b) (a) 40 (b) 50
(c) 55 (d) 60
Sol. (a)

If Shahid and Rohit start from point A, they meet at


point B. Till then both have travelled 3 km each.
Hence the correct answer is option (b)

34. In a 500 metres race, B starts 45 metres ahead of A, Correct answer is option (a).
but A wins the race while B is still 35 metres behind.
37. In a group of persons travelling in a bus, 6 persons
What is the ratio of the speeds of A to B assuming
can speak Tamil, 15 can speak Hindi and 6 can
that both start at the same time?
speak Gujarati. In that group none can speak any
(a) 25: 21 (b) 25: 20
other language. If 2 persons in the group can speak
(c) 5:3 (d) 5:7
two languages only and one person can speak all
Sol. (a)
he three languages, then how many persons are
According to the question in the time in which A there in the group?
covers 500 metres, B covers ; 500 – 45 – 35 = 420 (a) 21 (b) 22
metres. (c) 23 (d) 24
Ratio of distances; Sol. (c)
500 : 420 T=6 H = 15
25 : 21
3 2 12
Therefore ratio of speeds
1
The correct answer is option (a) 0 0

35. Two equal glasses of same type are respectively 5 G=6


1/3 and 1/4 full of milk. They are then filled up
with water and the contents are mixed in a pot. According to the diagram, number of persons in
What is the ratio of milk and water in the pot? the group are 23.
(a) 7 : 17 (b) 1 : 3 Hence option (c) is the correct answer.
(c) 9 : 21 (d) 11 : 23
38. In a parking area, the total number of wheels of all
Sol. (a)
the cars (four-wheelers) and scooters/ motorbikes
Total milk in both glass = 1/3 + 1/4 = 7/1
(two-wheelers) is 100 more than twice the number
Now, water in first glass = 1 – 1/3 = 2/3 of parked vehicles. The number of cars parked is

CSAT by Manjul Kumar Tiwari Sir


Previous Year Solved Papers 87
(a) 35 (b) 45 Directions for the following 7 ( seven) items :
(c) 50 (d) 55 Read the following six passages and answer the items
Sol. (c) that follow. Your answers to these items should be based
Let the number of two-wheelers = x on the passages only.
Let the number of four-wheelers = y Passage -1
So, the total vehicles = x + y Climate change is already making many people hungry
Total number of wheels = 2x + 2y all over the world, by disrupting crop yields and pushing
up prices. And it is not just food but nutrients that are
According to question,
becoming scarcer as the climate changes. It is the poorest
 2x + 4y = 2(x + y) + 100
communities that will suffer the worst effects of climate
 2x + 4y = 2x + 2y + 100 change, including increased hunger and malnutrition
 2y = 100 as crop production and livelihoods are threatened. On
 y = 50 the other hand, poverty is a driver of climate change, as
desperate communities resort to unsustainable use of
The correct answer is option (c).
resources to meet current needs.
39. The mangroves can shed tons of leaves per acre
41. Which among the following is the logical corollary
every year; fungi and bacteria break down this leaf
to the above passage?
litter and consume it, they then are consumed by
(a) Government should allocate more funds to
tiny worms and crustaceans, which in turn feed
poverty alleviation programmes and increase
small fish, which feed larger fish and birds and
food subsidies to the poor communities.
crocodiles.
(b) Poverty and climate impacts reinforce each
Which among the following is the most logical
other and therefore we have to re-imagine our
inference of the above statement?
food systems.
(a) Coastal areas cannot have food chains
(c) All the countries of the world must unite in
without mangroves.
fighting poverty and malnutrition and treat
(b) Mangroves are an essential component of all
poverty as a global problem.
marine ecosystems.
(d) We must stop unsustainable agricultural
(c) Mangroves have a crucial role in some of the
practices immediately and control food prices.
coastal food chains.
Sol. (b)
(d) The composition of marine flora and fauna is
largely determined by mangroves. The passage talks about the cause-effect
Sol. (c) relationship between poverty and climate change.
Only option (b) links both these issues and hence is
The correct answer is option (c)
the most correct answer.
40. "By liberty I mean the eager maintenance of that
Passage-2
atmosphere in which men have the opportunity to
The Global Financial Stability Report finds that the share
be their best selves."
of portfolio investments from advanced economies in the
Which one of the following expresses the view total debt and equity investments in emerging economies
implied in the above statement? has doubled in the past decade to 12 percent. The
(a) Liberty is the absence of restraint on human phenomenon has implications for Indian policy makers
action. as foreign portfolio investments in the debt and equity
(b) Liberty is what law permits people to perform. markets have been on the rise.
(c) Liberty is the ability to do what one desires. The phenomenon is also flagged as a threat that could
(d) Liberty is the maintenance of conditions for compromise global financial stability in a chain reaction,
the growth of human personality. in the event of United States Federal Reserve's imminent
Sol. (d) reversal of its "Quantitative Easing" policy.

CSAT by Manjul Kumar Tiwari Sir


88 CSAT : 2015

42. Which among the following is the most rational (d) Open defecation is a public health problem
and critical inference that can be made from the in all developing countries.
above passage? Sol. (c)
(a) Foregin portfolio investments are not good for The passage talks about the consequences of open
emerging economies. defecation such as malnourishment, mortality,
(b) Advanced economies undermine the global stunted growth of babies. This has an impact on
financial stability. the efficiency of labour_ The passage does not talk
(c) India should desist from accepting foreign about lack of resources at the Centre and the other
portfolio investments in the future. developing countries. Hence option (c) is the most
(d) Emerging economies are at a risk of shock from correct answer.
advanced economies.
Sol. (d) Passage - 4
We generally talk about democracy but when it comes to
The passage talks about increasing participation
any particular thing, we prefer a belonging to our caste or
of Portfolio investments in emerging economies that
community or religion. So long as we have this kind of
propel growth. Hence an event adversely impacting
the global financial stability will have major temptation, our democracy will remain a phoney kind of
implications for these emerging economies as well. democracy. We must be in a position to respect a man as
However the passage does not recommend that this a man and to extend opportunities for development to those
situation should be reversed. Hence the most correct who deserve them and not to those who happen to belong
answer is option (d). to our community or race. This fact of favouritism has
been responsible for much discontent and ill-will i:r1 our
Passage-3 country.
Open defecation 1s disastrous when practised in very
densely populated areas, where it is impossible to keep 44. Which one of the following statements bestsums
away human faeces from crops, wells, food and up the above passage?
children's hands. Groundwater is also contaminated by (a) Our country has a lot of diversity with its
open defecation. Many ingested genns and worms many castes, communities and religions.
spread diseases. They prevent the body from absorbing (b) True democracy could be established by
calories and nutrients. Nearly one~half of India's providing equal opportunities to all.
children remain malnourished. Lakhs of them die from (c) So far none of us have actually understood
preventable conditions. Diarrhoea leaves Indians’ the meaning of democracy.
bodies smaller on average than those of people m some (d) It will never be possible for us to establish
poorer countries ‘!Nhere people eat fewer calories. truly democratic governance in our country.
Underweight mothers produce stunted babies prone to Sol. (b)
sickness who may fail to develop their full cognitive The theme of this passage is that providing equal
potential. The germs released into environment harm opportunities for all should be the basic function of
rich and poor alike, even those who use latrines. a democracy. Hence, option (b) is the correct answer.
43. Which among the following is the most critical Passage - 5
inference that can be made from the above The existence/establishment of formal financial
passage? institutions that offer safe. reliable, and alternative
(a) The Central and State governments in India financial instruments is fundamental in mobilising
do not have enough resources to afford a savings. To save, individuals need access to safe and
latrine for each household. reliable financial institutions, such as banks, and to
(b) Open defecation is the most important public appropriate financial instruments and reasonable
health problem of India. financial incentives. Such access is not always available
{ c) Open defecation reduces the human capital to all people in developing countries like India and more
of India's workforce. so, in rural areas. Savings help poor households manage

CSAT by Manjul Kumar Tiwari Sir


Previous Year Solved Papers 89
volatility in cash flow, smoothen consumption, and build many number of tasks to perform. Where discretion has
working capital. Poor households without access to a to be used, there must be rules and safeguards to prevent
formal savings mechanism encourage immediate misuse of that power. Systems have to be devised which
spending· temptations. minimise, if not prevent, the abuse of discretionary
power. Government work must be conducted within a
45. With reference to the above passage, consider the
framework of recognised rules and principles, and
following statements :
decisions should be similar and predictable.
1. Indian financial institutions do not offer any
financial instruments to rural households to 47. Which among the following is the most logical
mobilise their savings. assumption that can be made from the above
2. Poor households tend to spend their passage?
earnings/savings due to lack of access to (a) Government should always be given wide
appropriate financial instruments. discretionary power in all matters of
Which of the statements given above is/are administration.
correct? (b) The supremacy of rules and safeguards
(a) 1 only (b) 2 only should prevail as opposed to the influence of
(c) Both 1 and 2 (d) Neither 1 nor 2 exclusive discretion of authority.
Sol. (b) (c) Parliamentary democracy is possible only if
the Government has wider discretionary
The passage does not talk about India specifically
but only mentions India as an example. Hence power.
statement 1 can be considered as incorrect The last (d) None of the above statements is a logical
sentence of the passage mentions that in the absence assumption that can be made from this
of formal saving mechanisms, household spend the passage.
money. Hence statement 2 is correct and the correct Sol. (b)
answer is option (b). The question asks the most logical assumption that
46. What is the crucial message conveyed in the can be made from the passage. Amongst options
passage? (a), (d) and (c), none can be considered as a logical
assumption. Hence the correct answer is option (b).
(a) Establish more banks
(b) Increase the Gross Domestic Product (GDP) 48. A selection is to be made for one post of Principal
growth rate and two posts of Vice-Principal. Amongst the six
(c) Increase the interest rate of bank deposits candidates called for the interview, only two are
(d) Promote financial inclusion eligible for the post of Principal while they all are
Sol. (d) eligible for the post of Vice-Principal. The number
Restricting ourselves to the passage only option (a) of possible combinations of selectees is
is mentioned which is the correct answer. A student (a) 4 (b) 12
might be tempted to mark option (d) but the passage (c) 18 (d) None of'the above
does not talk about financial inclusion. Sol. (d)
2C × 5C2 = 2 × 10 = 20 ways
Passage - 6 1
Governments may have to take, steps which would The correct answer is option (d).
otherwise be an infringement on the Fundamental Rights
of individuals, such as acquiring a person's land against 49. A student has to opt for 2 subjects out of 5 subjects
his will, or refusing permission for putting up a for a course, namely, Commerce, Economics,
building, but the larger public interest for which these Statistics, Mathematics I and Mathematics II.
are done must be authorized by the people (Parliament). Mathematics II can be offered only if Mathematics
Discretionary powers to the administration can be done I is also opted. The number of different
away with. It is becoming more and more difficult to combinations of two subjects which can be
keep this power within limits as the government has opted is

CSAT by Manjul Kumar Tiwari Sir


90 CSAT : 2015

(a) 5 (b) 6 Sol. (d)


(c) 7 (d) 8 The situation is impossible. The correct answer is
Sol. (c) option (d).
The subjects are C, E, S, Ml and Mll The different
52. The graph below depicts the earnings of A and B
combinations are. over the period 2000 to 2010 :
CE, CS, CMI, ES, EMI, SMI, Ml and Mll
The total numbers of possible combinations are 7.

50. A person ordered 5 pairs of black socks and some


pairs of brown socks. The price of a black pair was
thrice that of a brown pair. While preparing the
bill, the bill clerk interchanged the number of black
and brown pairs by mistake which increased the
From the graph, which one of the following can be
bill by 100%. What was the number of pairs of concluded?
brown socks in the original order?
(a) On the average A earned more than B during
(a) 10 (b) 15
this period.
(c) 20 (d) 25
(b) On the average B earned more than A during
Sol. (d) this period.
Let the original number of brown socks be k. (c) The earnings of A and B were equal during
Let the prices of the black and brown socks be x and this period.
y respectively. (d) The earnings of A were less as compared to B
during this period.
We have, x, 3y
Sol. (a)
Actual bill is 5x + ky
From 2000 to 2007, A earned more than B. This
The bill prepared is kx + 5y situation was reversed from 2007 - 2010. However,
According to the condition given the question according to the graph, during these three years,
2(5x + ky) = kx + 5y a’s earning wasmarginally more than A. Hence It
can be concluded that overall A earned more than
Substituting and simplifying we get k = 25
B during this period.
Hence the correct answer is option (d).
53. Two pipes A and B can independently fill a tank
51. The number of persons who read magazine X only completely in 20 and 30 minutes respectively. If
is thrice the number of persons who read magazine both the pipes are opened simultaneously, how
Y. The number of persons who read magazine Y much time will they take to fill the tank completely?
only is thrice the number of persons who read (a) 10 minutes (b) 12 minutes
magazine X. Then, which of the following (c) 15 minutes (d) 25 minutes
conclusions can be drawn? Sol. (b)
1. The number of persons who read both the Using the formula directly, time taken
magazines is twice the number of persons
20 x 30/20 + 30 = 600/50 = 12
who read only magazine X.
The correct answer is option (b).
2. The total number of persons who read either
one magazine or both the magazines is twice 54. Each of the six different faces of a cube has been
the number of persons who read both the coated with a different colour i.e., V, I, B, G, Y and
magazines. O. Following information is given :
Select the correct answer using the code given 1. Colours Y, O and B are on adjacent faces.
below : 2. Colours f, G and Y are on adjacent faces.
(a) 1 only (b) 2 only 3. Colours B, G and Y are on adjacent faces.
(c) Both 1 and 2 (d) Neither 1 nor 2 4. Colours O, V and B are on adjacent faces.

CSAT by Manjul Kumar Tiwari Sir


Previous Year Solved Papers 91
Which is the colour of the face opposite to the face (c) D = l, E = 0
coloured with O? (d) D = 1, E = 2
(a) B (b) V Sol. (c)
(c) G (d) I The calculation given above is a simple rule for
Sol. (c) multiplication of a number by 1001
The correct answer is option (c). Hence D = 1 and E = 0.
55. Consider the following statements followed by two 58. Year-wise variation of the price of a certain
conclusions: commodity is shown in the following graph :
Statements : Some men are great.
Some men are wise. Conclusion 1: Men are either
great or wise.
Conclusion II: Some men are neither great nor wise.
Which one of the following is correct?
(a) Only conclusion I is valid
(b) Only conclusion II is valid
(c) Both the conclusions are valid The price of the commodity in the year 1990
(d) Neither of the conclusions is valid (a) must have been Rs. 10/-
Sol. (d) (b) must have been 12/-
(c) must have been anywhere between Rs. 10/-
Conclusion I is ruled out because both the
and Rs. 20/-
statements use ‘some’ and not ‘remaining’
(d) is higher than that in the year 1991
Conclusion Il is also invalid because some and some
may mean all. Hence the correct answer is option Sol. (c)
(d). From the graph it can be concluded that option (c)
56. Consider the following statements : is the correct answer:
1. Some claim to have seen UFOs (Unidentified
59. The proportion of expenditure on various items
Flying Objects).
by two families A and B are represented in the
2. Life on other heavenly bodies is considered
following Bar Charts :
to be a possibility.
3. Voyage to space is now an established fact.
50% Food 10% Food
From the above statements, it may be concluded
that 30% 60%
(a) UFOs are heavenly bodies 30% Education Other Other
items items
(b) UFOs are sent from other heavenly bodies
(c) Some living species in other heavenly bodies 20% 30%
are more intelligent than man Education Education
(d) Nothing definite can be said about the UFOs
Sol. (d) Family A Family B
Statements 1, 2 and 3 do not seem to be inter- Total expenditure : Total expenditure:
connected at all. Hence option (d) is the best possible Rs. 20,000 per month Rs. 1,00,000 per month
answer. From these charts, we can conclude that
(a) Family A spent more money on food than
57. If ABC x DEED = ABCABC; where A, B, C, D and Family B.
E are different digits, what are the values of D (b) Family B spent more money on food than
and E? Family A.
(a) D = 2, E = 0 (c) Family A and Family B spent the same
(b) D = 0, E = 1 amount on food.

CSAT by Manjul Kumar Tiwari Sir


92 CSAT : 2015

(d) The expenditure on food by Family A and (c) There is no solution in sight for the problems
Family B cannot be compared. of continuing poverty and inflation in India
Sol. (c) in the near future.
The expenditure is as follows (d) Economic reforms can often create a high
inflation economy.
Sol. (d)
According to the passage economic reforms have
resulted in an increase in prices and a high inflation.
The effort to reduce subsidies is just one of the many
causes for high inflation. Hence the most crucial
60. Usha runs faster than Kamala, Priti runs slower message of the passage is given in option (d) which
than Swati, Swati runs slower than Kamala. Who brings out the relation between economic reforms
is the slowest runner? and price rise.
(a) Kamala (b) Priti
Passage - 2
(c) Swati (d) Usha
No Eight is absolute, exclusive or inviolable. The Right
Sol. (b)
of personal property, similarly, has to be perceived in
The runners is descending speeds are Usha,
the larger context of its assumed legitimacy. The Right
Kamala, Swati and Priti. The slowest runner is Priti
of personal property should unite the principle of liberty
and the correct answer is option (b)
with that of equality, and both with the principle of
Directions for the following 7 (seven) items : cooperation.
Read the following four passages and answer the items
62. In the light of the argument in the above passage,
that follow. Your answers to these items should be based
on the passages only. which one of the following statements is the most
convincing explanation?
Passage - 1 (a) The Right of personal property is a Natural
India has suffered from persistent high inflation. Right duly supported by statutes and
Increase in administered prices, demand and supply scriptures.
imbalances, imported inflation aggravated by rupee (b) Personal property is a theft and an instrument
depreciation, and speculation - have combined to keep of exploitation. The Right of personal
high inflation going. If there is an element common to property is therefore violative of economic
all of them, it is that many of them are the outcomes of justice.
economic reforms. India's vulnerability to the effects of (c) The Right of personal property is violative of
changes in international prices has increased with trade distributive justice and negates the principle
liberalisation. The effort to reduce subsidies has resulted of cooperation,
in a continuous increase in the prices of commodities (d) The comprehensive idea of economic justice
that are administered. demands that the Right of each person to
61. What is the most logical, rational and crucial acquisition of property has to be reconciled
message that is implied in the above passage? with that of others.
(a) Under the present circumstances, India Sol. (d)
should completely avoid all trade Option (a) goes directly against what is mentioned
liberalisation policies and all subsidies. in the passage. Option (b) is an exaggeration. The
(b) Due to its peculiar socio-economic situation, passage talks about individual rights uniting with
India is not yet ready for trade liberalisation the broader idea of equality. Option (d) is the most
process, suitable answer.

CSAT by Manjul Kumar Tiwari Sir


Previous Year Solved Papers 93
Passage - 3 funding through the multilateral mechanism of the
The conflict between man and State is as old as State Convention will enhance their domestic capacity to
history. Although attempts have been made for centuries finance the mitigation efforts.
to bring about a proper adjustment between the
64. According to the passage, which of the following
competing claims of State and the individual, the
is/are a matter of intense debate in the multilateral
solution seems to be still far off. This is primarily because
negotiations under UNFCCC regarding the role of
of the dynamic nature of human society where old values
developing countries in climate change?
and ideas constantly yield place to new ones. It is
1. The scale and size of required financial
obvious that if individuals are allowed to have absolute
support.
freedom of speech and action, the result would be chaos,
2. The crop loss due to climate change in the
ruin and anarchy.
developing countries.
63. The author's viewpoint can be best summed up in 3. To enhance the mitigation and adaptation
which of the following statements? actions in the developing countries.
(a) The conflict between the claims of State and Select the correct answer using the code given
individual remains unresolved. below :
(b) Anarchy and chaos are the obvious results of (a) 1 only (b) 2 and 3 only
democratic traditions. (c) 1, and 3 only (d) 1, 2 and 3
(c) Old values, ideas and traditions persist Sol. (c)
despite the dynamic nature of human society. According to the passage the scale and size of funds
(d) Constitutional guarantee of freedom of speech involved to enhance the mitigation and adaptation
is not in the interest of society. strategies in developing countnes are a matter of
Sol. (a) intense speculation and debate in the IJNFCC.
The passage is narrative in nature and just states s Hence statements 1 and 3 are correct and the correct
viewpoint. It says that the conflict between Man answer is option (c).
and State is as old as the society and does not
65. In this passage, the Convention puts the
mention anything about the problem being resolved
responsibility for the provision of financial support
Hence option (a) is the correct answer.
on the developed countries because of
Passage 1. their higher level of per capita incomes.
Climate change is a complex policy issue with major 2. their large quantum of GDP.
implications in terms of finance. All actions to address 3. their large contribution to the stock of GHGs
climate change ultimately involve costs. Funding is vital in the atmosphere.
for countries like India to design and implement Select the correct answer using the code given
adaptation and mitigation plans and projects. Lack of below :
funding is a large impediment to implementing (a) 1 only (b) 1 and 2 only
adaptation plans. The scale and magnitude of the (c) 3 only (d) 1, 2 and 3
financial support required by developing countries to Sol. (c)
enhance their domestic mitigation and adaptation Statement 3 is directly mentioned in the passage.
actions are a matter of intense debate in the multilateral No mention is made about the higher level of per
negotiations under the United Nations Framework capita income and the large quantum of GDP of the
Convention on Climate Change (UNFCCC). The developed countries. Hence the correct answer is
Convention squarely puts the responsibility for
option (c).
provision of financial support on the developed
countries, taking into account their contribution to the 66. With regards to developing countries, it can be
stock of greenhouse gases (GHGs) in the atmosphere. inferred from the passage that climate change is
Given the magnitude of the task and the funds required, likely to have implications on their
domestic finances are likely to fall short of the current 1. domestic finances.
and projected o needs of the developing countries. Global 2. capacity for multilateral trade.

CSAT by Manjul Kumar Tiwari Sir


94 CSAT : 2015

Select the correct answer using the code given Sol. (c)
below : The correct answer is option (c).
(a) 1 only (b) 2 only
(c) Both 1 and 2 (d) Neither 1 nor 2 70. The monthly incomes of Peter and Paul are in the
Sol. (a) ratio of 4 : 3. Their expenses are in the ratio of 3 : 2.
If each saves 6,000 at the end of the month, their
The passage clearly mentions that “domestic
monthly incomes respectively are (in W)
finances are expected to fall short of the current and
projected needs of the developing countries.” (a) 24,000 and 18,000
Hence, statement 1 is correct. No mention is made (b) 28,000 and 21,000
about the capacity of the developing countries in (c) 32,000 and 24,000
multilateral trade.
(d) 34,000 and 26,000
Statement 2 is wrong and the correct answer is
Sol. (a)
option (a)
Check from the options.
67. Which one of the following is essentially
In option (a) the expenses ar 8,000 and 12,000 which
discussed in the passage?
is in the ration 3:2_ Hence option (a) is the correct
(a) Conflict between developed and developing
answer.
countries regarding support for mitigation
(b) Occurrence of climate change due to excessive 71. Two cities A and B are 360 km apart. A car goes
exploitation of natural resources by the from A to B with a speed of 40 km/hr and returns
developed countries to A with a speed of 60 km/hr. What is the average
(c) Lack of political will on the part of all the speed of the car?
countries to implement adaptation plans
(.a) 45 km/hr (b) 48 km/hr
(d) Governance problems of developing countries
(c) 50 km/hr (d) 55 km/hr
as a result of climate change
Sol. (c) Sol. (b)

The passage s out the contrast in developed and Using the formula the average speed would be
developing countries with regard to climate change. 2 x 40 x 60/40 + 60 km/hr
Hence the best answer is option (c). Hence the correct answer is option (b).
68. Between 6 PM and 7 PM the minute hand of a Directions for the following 2 (two) items :
clock will be ahead of the hour hand by 3 minutes at
Read the following passage and answer the 2 (two) items
(a) 6 : 15 PM (b) 6 : 18 PM
that follow:
(c) 6 : 36 PM (d) 6 ; 48 PM
Sol. (c) A, B, C, D, E and F are cousins. No two cousins are of the
same age, but all have birthdays on the same day of the
At 6-15 and 6:18 the minutes hand will be behind
same month. The youngest is 17 years old and the oldest
the hours hand. Options (a) and (b) are eliminated.
E is 22 years old. F is somewhere between B and D in age.
At 6: 48 the minutes hand would be at least 9
A is older than B. C is older than D. A is one year older
minutes ahead of the hours hand. The only option
than C.
possible is option (c).
72. Which one of the following is possible?
69. There are 5 tasks and 5 persons. Task-1 cannot be (a) D is 20 years old (b) F is 18 years old
assigned to either person-1, or person-2. Task-2 (c) F is 19 years old (d) F is 20 years old
must be assigned to either person-3 or person-4.
Every person is to be assigned one task. In how 73. What is the number of logically possible orders of
many ways can the assignment be done? all six cousins in terms of increasing age?
(a) 6 (b) 12 (a) 1 (b) 2
(c) 24 (d) 144 (c) 3 (d) 4

CSAT by Manjul Kumar Tiwari Sir


Previous Year Solved Papers 95
For Q.72 to Q.73 Which of the following statements is true?
According to the data the brother is descending order of (a) Both the Tamil speakers can drive a car.
age are
(b) Both the Marathi speakers can drive a car.
E (22 years) A (21 years) C (20 years) B (19 years) - F (18
(c) Both of those who can drive a car speak
years) D (17 years)
Marathi.
Sol. (b)
(d) One of those who can drive a car speaks
The correct answer is option (b) Tamil.
Sol. (b)
Sol. (d)
The correct answer is option (b)
Only option (d) can be concluded and is the correct
74. In a society it is customary for friends of the same answer.
sex to hug and for friends of opposite sex to shake
76. In a plane, line X is perpendicular to line Y and
hands when they meet. A group of friends met in a
parallel to line Z; line U is perpendicular to both
party and there were 24 handshakes.
lines V and W; line X is perpendicular to line V.
Which one among the following numbers
Which one of the following statements is correct?
(a) 39 (b) 30
(a) Z, Uand W are parallel.
(c) 21 (d) 20
(b) X, Vand Y are parallel.
Sol. (c)
(c) Z, Vand U are all perpendicular to W.
Let the numbero f males be M and the number of
females be F. (d) Y, Vand W are parallel.

Each male or each female will shake hands with Sol. (d)
each of the member of the opposite sex. Option (d) is the correct answer.
The total number of handshakes = M × F = 24
77. A cow costs more than 4 goats but less than
Possibilities for the number of males and females in 5 goats. If a goat costs between Rs. 600 and
the society (not necessarily in the same order) = Rs. 800, which of the following is a most valid
(2, 12), (3, 8), (4, 6), (1, 24) conclusion?
In case of (2, 12) the number of hugs would be = (a) A cow costs more than Rs. 2,500.
2C + 12C = 2!(2!x1) + 12!/(2!x10! = 1 + 66 = 67
2 2
(b) A cow costs less than Rs. 3,600.
Clearly, it is not in the option.
(c) A cow costs between Rs. 2,600 and Rs 3,800.
In case of (3, 8), the number of hugs would be = 3C2
+ 8C2 = 3!/2!x1) + 8!/(2!x6! = 3 + 28 = 31 (d) A cow costs between Rs. 2,400 and Rs. 4,000.

Clearly, it is not in the option either. Sol. (d)

In case of (4, 6), the numbero f hugs would be = 4C2 Minimum cost of 4 goats = Rs_ 2,400
+ 6C2 = 4!/(2!x2!) + 6!(2!x4!) = 6 + 15 = 21 Maximum cost of 5 goats = Rs_ 4,000
Clearly, it is one of the options. Hence option (d) is the correct answer.
The correct answer is option (c). 78. A society consists of only two types of people -
75. Two men, Anil and David, and two women, fighters and cowards. Two cowards are always
Shabnam and Rekha are in a sales group. Only friends. A fighter and a coward are always enemies.
two speak Tamil. The other two speak Marathi. Fighters are indifferent to one another. If A and B
Only one man and one woman can drive a car. are enemies, C and D are friends, E and F are
Shabnam speaks Marathi. Anil speaks Tamil. Both indifferent to each other, A and E are not enemies,
Rekha and David can drive. while B and F are enemies.

CSAT by Manjul Kumar Tiwari Sir


96 CSAT : 2015

Which of the following statements is correct? 80. Candidates in a competitive examination consisted
(a) B, C and F are cowards. of 60% men and 40% women. 70% men and 75%
women cleared the qualifying test and entered the
(b) A, E and F are fighters. final test where 80% men and 70% women were
(c) B and E are in the same category. successful.
(d) A and F are in different categories. Which of the following statements is correct?

Sol. (b) (a) Success rate is higher for women.

According to the information given in the question. (b) Overall success rate is below 50%.
(c) More men cleared the examination than
Fighters -A, E and F
women.
Towards - C, D and B
(d) Both (a) and (b) above are correct.
Hence the correct answer is option (b).
Sol. (c)
79. In a box of marbles, there are three less white Let the total number of students be x
marbles than the red ones and five more white
Number of men = 0.6x; number of women = 0.4x
marbles than the green ones. If there are a total of
10 white marbles, how many marbles are there in Men clearing the qualifying test = 0.7 × 0.6 × 0.42x
the box? Women clearing the qualifying test 0.75 × 0.4x ×
(a) 26 (b) 28 0.30x

(c) 32 (d) 36 Men clearing the final test 0.8 × 0.42x × 0.336x

Sol. (b) Women clearing the final test = 0.7 × 0.3 × 0.2x

Total number of white marbles 10 The success rate for men is marginally higher than
women. Option (a) is wrong.
Number of red marbles 10+3= 13
Overall success rate is above 50% Option (b) is
Number of green marbles 10-5=5 wrong.
Hence total number of marbles 28 More men cleared the examination than women.
The correct answer is option (b). Option (c) is correct.


CSAT by Manjul Kumar Tiwari Sir


Previous Year Solved Papers 97

Previous Year
CSAT : 2016 Solved Papers

Directions for the following 8 (eight) items: Passage - 2


Read the following eight passages and answer the items Climate adaptation may be rendered ineffective if
that follow the passages. Your answer to these items policies are not designed in the context of other
should be based on the passages only. development concerns. For instance, a comprehensive
strategy that seeks to improve food security in the context
Passage - 1
of climate change may include a set of coordinated
By killing transparency and competition, crony
measures related to agricultural extension, crop
capitalism is harmful to free enterprise, opportunity and
diversification, integrated water and pest management
economic growth. Crony capitalism, where rich and the
and agricultural information series. Some of these
influential are alleged to have received land and natural
measures may have to do with climate changes and
resources and various licences in return for payoffs to
others with economic development.
venal politicians, is now a major issue to be tackled. One
of the greatest dangers to growth of developing 2. What is the most logical and rational inference
economies like India is the middle-income where crony that can be made from the above passage?
capitalism creates oligarchies that slow down the growth. (a) It is difficult to pursue climate adaptation in
the developing countries
1. Which among the following is the most logical (b) Improving food security is a far more complex
corollary to the above passage? issue than climate adaptation
(a) Launching more welfare schemes and (c) Every developmental activity is directly or
allocating more finances for the current indirectly linked to climate adaptation
schemes r are urgently needed (d) Climate adaptation should be examined in
(b) Efforts should be made to push up economic tandem with other economic development
growth by other meand provide licences to ptions
the poor Sol. (d)
(c) Greater transparency in the functioning of the
Only option (d) sounds sensible, complete and not
government and promoting the financial
extreme. Option (a) is wrong. Option (b) is not
inclusion are needed at present
mentioned anywhere. Option (c) is wrong. Hence
(d) We should concentrate more on developing
option (d) is correct.
manufacturing sector than service sector
Sol. (c) Passage - 3
In the passage, the first line “By killing transparency Understanding of the role of biodiversity in the
.......” clearly indicates that greater transparency in hydrological cycle enables better policy-making. The term
the functioning of the government is needed. India biodiversity refers to the variety of plants, animals,
is a growing economy, and crony capitalism can microorganisms, and the ecosystems in which they occur.
harm rapid economic growth. A “corollary” is a Water and biodiversity are interdependent. In reality, the
proposition that follows from a proven one. Hence hydrological cycle decides how biodiversity functions.
option (c) is correct. In turn, vegetation and soil drive the movement of water.

CSAT by Manjul Kumar Tiwari Sir


98 CSAT : 2016

Every glass of water we drink has, at least in part, passed Passage - 5


through fish, trees, bacteria, soil and other organisms. Safe and sustainable sanitation in slums has
Passing through these ecosystems, it is cleansed and immeasurable benefits to women and girls in terms of
made fit for consumption. The supply of water is a critical their health, safety, privacy and dignity. However,
service that the environment provides. women do not feature in most of the schemes and policies
on urban sanitation. The fact that even now the manual
3. Which among the following is the most critical
scavenging exists, ones to show that not enough has been
inference that can be made from the above passage? done to promote pour-flush toilets and discontinue the
(a) Biodiversity sustains the ability of nature to use of dry latrines. A more sustained and rigorous
recycle water campaign needs to be launched towards the right to
(b) We cannot get potable water without the sanitation on a very large scale. This should primarily
existence of living organisms focus on the abolition of manual scavenging.
(c) Plants, animals and microorganisms
continuously interact among themselves 5. With reference to the above passage, consider the
(d) Living organisms could not have come into following statements:
1. Urban sanitation problems can be fully solved
existence without hydrological cycle
by the abolition of manual scavenging only
Sol. (a)
2. There is a need to promote greater awareness
Passage says “water and biodiversity are on safe sanitation practices in urban areas
interdependent”. Options (c) and (d) are not wrong,
but they are not related to the question. Option (b) is Which of the statements given above is/are
an extreme one. Hence option (a) is correct, and is correct?
the best critical inference drawn. (a) 1 only (b) 2 only
(c) Both I and 2 (d Neither 1 nor 2
Passage - 4 Sol. (b)
In the last decade, the banking sector has been Statement 1 is wrong as nothing is guaranteed
restructured with a high degree of automation and regarding “fully solved”. Statement 2 is obviously
products that mainly serve middle-class and upper implied in the passage. Hence option (b) is correct.
middle-class society. Today there is need for a new
Passage - 6
agenda for the banking and non-banking financial
services that does not exclude the common man To understand the nature and quantity of Government
proper for man, it is necessary to attend to his character.
4. Which one of the following is the message that is As nature created him for social life, she fitted him for the
essentially implied in the above passage? station she intended. In all cases she made his natural
(a) Need for more automation and more products wants greater than his individual powers. No one man
of bank is capable, without the aid of society, of supplying his
(b) Need for a radical restructuring of our entire own wants; and those wants, acting upon every
public finance system individual, impel the whole of them into society.
(c) Need to integrate banking and non-banking
6. Which among the following is the most logical and
institutions
rational inference that can be made from the above
(d) Need to promote financial inclusion passage ?
Sol. (d) (a) Nature has created a great diversity in human
The last line of passage “Today there is a need for a society
new ........ that does not exclude the common man” (b) Any given human society is always short of
implies that need to promote “financial inclusion”. its wants
Also, the passage speaks of existing products (c) Social life is a specific characteristic of man
serving the upper middle and middle classes. Hence, (d) Diverse natural wants forced man towards
(d) is the best answer. social system

CSAT by Manjul Kumar Tiwari Sir


Previous Year Solved Papers 99
Sol. (d) (a) Organic farming should immediately replace
Option (a) is not mentioned anywhere even mechanised and chemical dependant
indirectly. Option (b) is not correct as individuals agricultural practices all over the world
may be short of fulfiling their own wants, but society (b) It is imperative for us to modify our land use
may be able to (that is why people come together in practices in order to mitigate climate change.
a society). Option (c) is an obvious truth. But it may (c) There are no technological solutions to the
be true for other animals also. Option (d) is the best problem of greenhouse gas emissions
inference. (d) Tropical areas are the chief sites of carbon
sequestration
Passage - 7
Sol. (b)
The nature of the legal imperatives in any given state
corresponds to the effective demands that state Option (a) being an extreme one, can be rejected.
encounters, and that these, in their turn, depend, in a Option (b) is clearly spoken of. Option (c) is wrong.
general way, upon the manner in which economic power Best answer is (b).
is distributed in the society which the state controls.
9. A person climbs a hill in a straight path from point
7. The statement refers to: ‘O’ on the ground in the direction of north-east
(a) the antithesis of Politics and Economics and reaches a point ‘A’ after travelling a distance
(b) the interrelationship of Politics and Economics of 5 km. Then, from the point ‘A’ he moves to point
(c) the predominance of Economics over Politics ‘B’ in the direction of north-west. Let the distance
(d) the predominance of Politics over Economics AB be 12 km. Now, how far is the person away
Sol. (b) from the starting point ‘O’?
Structure of legal systems in a society depends on (a) 7 km (b) 13 km
what people demand over time. Demands depend (c) 17 km (d) 11 km
on the manner of distribution of economic power in Sol. (b)
the society. State controls the society. So, the OB2 = OA2 + AB2
relationship of politics (State) and economics is OB2 = 52 + 122 = 25 + 144
being studied. Best answer is (b). The passage is
not making out any one of these are dominant over
the other.

Passage - 8
About 15 percent of global greenhouse gas emissions
come from agricultural practices. This includes nitrous
oxide fertilizers; methane from livestock, rice production,
and manure storage; and carbon dioxide (CO2) from
burning biomass, but this excludes CO2 emissions from
soil management practices, savannah burning and OB = 169 .
deforestation. Foresty and use, and land-use change
account for another percent of greenhouse gas emissions This means OB = 13 km
each ear, three quarters of which come from tropical Hence, obtion (b) is correct.
deforestation. The remainder is largely from draining and
10. An agricultural field is in the form of a rectangle
burning tropical peatland. About the same amount of
having length x1 meters and breadth x2 meters (x1
carbon is stored in the world’s peatlands as is stored in
and x2 are variable). If x1 + x2 = 40 meters, then the
the Amazon rainforest.
area of the agricultural field will not exceed which
8. Which among the following is the most logical and one of the following values?
rational inference that can be made from the above (a) 400 sq m (b) 300 sq m
passage? (c) 200 sq m (d) 80 sq m

CSAT by Manjul Kumar Tiwari Sir


100 CSAT : 2016

Sol. (a) (a) 8 (b) 9


Area of the field will be maximum when both sides (c) 10 (d) 11
of rectangle are equal. Therefore x1 = x2 = 20 meters Sol. (b)
(because x1 + x2 = 40 is given). Max. Area = 20 × 20 Person is standing on first step and middle step is 4
= 400 sq. m. step ahead. Therefore middle step is the 5th step.
11. The sum of the ages of 5 members comprising a Now top step will also be 4 step ahead of the middle
family, 3 years ago was 80 years. The average age step.
of the family today is the same as it was 3 years Top step = 5 + 4 = 9 steps
ago, because of an addition of a baby during the
Total no. of steps in the ladder = 9.
intervening period. How old is the baby ?
Direction for the following 3 (three) items: Consider the
(a) 6 months (b) 1 year
given information and answer the three items that follow.
(c) 2 years (d) 2 years and 6 months
When three friends A, B and C met, it was found that
Sol. (b) each of them wore an outer garment of a different colour.
The average age of family 3 years ago was 80/5 In random order, the garments are: jacket, sweater and
= 16 years tie; and the colours are: blue, white and black. Their
The sum of the ages of 5 members today will be surnames in random order Kumar and Singh.
80 + (3 × 5) = 80 + 15 = 95 years Further, we know that :
Now, the average age of family including new baby 1. neither B nor Ribeiro wore a white sweater
will be (95 + N)/6, which is same as 3 years ago i.e. 2. C wore a tie
16 years. N is the age of baby now 3. Singh’s garment was not white
Therefore, (95 + N)/6 = 16 => 95 + N = 16 × 6
4. Kumar does not wear a jacket
N = 96 – 95 = 1 year
5. Ribeiro does not like to wear the black colour
12. The total emoluments of two persons are the same,
6. Each of the friends wore only one outer
but one gets allowances to the extent of 65% of his
garment of only one colour
basic pay and the other gets allowances to the
extent of 80% of his basic pay. The ratio of the basic 14. What is C’s surname ?
pay of the former to the basic pay of the latter is: (a) Ribeiro
(a) 16 : 13 (b) 5 : 4 (b) Kumar
(c) 7 : 5 (d) 12 : 11 (c) Singh
Sol. (d) (d) Cannot be determined
Let the basic pay of former person be b1 and latter Sol. (a)
be b2. Information can be summarized as:
Therefore, the salary of former person
= b1 + 65% of b1 = 1.80b2
Now, total emoulments of two persons are same.
Hence, 1.65b1 = 1.80b2
b1 180 12
  From the above table, the answer is clearly visible.
b2 165 11
15. What is the colour of the tie?
13. A person is standing on the first step from the (a) Black
bottom of a ladder. If he has to climb 4 more steps (b) Blue
to reach exactly the middle step, how many steps (c) White
does he ladder have? (d) Cannot be determined

CSAT by Manjul Kumar Tiwari Sir


Previous Year Solved Papers 101
Sol. (b) 18. A person walks 12 km due north, then 15 km due
Infomation can be summarized as: east, after that 19 km due west and then 15 km due
south. How far is he from the starting point?
(a) 5 km (b) 9 km
(c) 37 km (d) 61 km
Sol. (a)
Let person walks the path OABCD as shown below.
From the above table, the answer is clearly visible.

16. Who wore the sweater?


(a) A
(b) B
(c) C
(d) Cannot be determined
Sol. (a)
Information can be summarized as:

His distance from starting point will be Do

DO  32  4 2  25 = 5 kms

19. A cube has all its faces painted with different


From the above table, the answer is clearly visible.
colours. It is cut into smaller cubes of equal sizes
17. AB is a vertical trunk of a huge tree with A being the such that the side of the small cube is one-fourth
point where the base of the trunk touches the the big cube. The number of small cubes with only
ground. Due to a cyclone, the trunk has been broken one of the sides painted is:
at C which is at a height of 12 meters, broken part is (a) 32 (b) 24
partially attached to the vertical portion of the trunk (c) 16 (d) 8
at C. If the end of the broken part B touches the Sol. (b)
ground at D which is at a distance of 5 meters from Only one side painted will be 4 small cubes on each
A, then the original height of the trunk is: face.
(a) 20 m (b) 25 m
(c) 30 m (d) 35 m
Sol. (b)

Therefore, 4 × 6 = 24 such small cubes

20. Ram and Shyam work on a job together for four


days and complete 60% of it. Ram takes leave then
and Shyam works for eight more days to complete
the job. How long would Ram take to complete the
entire job alone?
(a) 6 days (b) 8 days
(c) 10 days (d) 11 days
By the Pythagorus theorem,
Sol. (c)
CD2 = AC2 + AD2 = 122 + 52 => = CD = 13 m
Ram and Shyam can together complete 60% of work
The original height of the trunk = AC + CD in 4 days, therefore Ram and Shyam's one day work
= 12 + 13 = 25 m will be

CSAT by Manjul Kumar Tiwari Sir


102 CSAT : 2016

1 1 60 1 102, 112, ..... 192 (Total 10)


  
R S 100 4 202, 212, ..... 292 (Total 10)
1 1 3 Between 100 and 300, the number of numbers
  ... (1)
R S 20 beginning with 2 will be
Remaining 40% of work is complete by Shyam alone 200, 201, 202, ..... 299 (Total 100) of which 10 end
in 8 days. with 2 so subtract that.
Therefore, 100% of work can be completed by Shyam So final answer = 100 - 10 + 10 + 10 = 110.
in (8/40) × 100 = 20 days
Hence, answer (a) is correct.
1 1 3
   Directions for the following 8 (eight) items:
R S 20
Read the following five passages and answer the items
that follow the passages. Your answer to these items
1 3 1 2 1
    should be based on the passages only.
R 20 20 20 10

Ram alone can complete entire job in 10 days. Passage - 1


As we look to 2050, when we will need to feed two billion
21. A military code writes SYSTEM as SYSMET and more people, the question of which diet is best has taken
NEARER as AENRER. Using the same code, on new urgency. The foods we choose to eat in the coming
FRACTION can be written as: decades will have dramatic ramifications for the planet.
(a) CARFTION (b) FRACNOIT Simply put, a diet that revolves around meat and dairy a
(c) NOITCARF (d) CARFNOIT way of eating that is on the rise throughout the
Sol. (d) developing. world, will take a greater toll on the world’s
In given code, first half of the word and second half resources than one that revolves around unrefined
of the word are reversed. grains, nuts, fruits and vegetables.

FRACTION will be written as CARFNOIT. 24. What is the critical message conveyed by the above
passage?
22. If R and S are different integers both divisible by 5,
(a) Our increasing demand for foods sourced
then which of the following is not necessarily true?
from animals puts a greater burden on our
(a) R - S is divisible by 5
natural resources
(b) R + S is divisible by 10
(c) R x S is divisible by 25 (b) Diets based on grains, nuts, fruits and
vegetables are best suited for health in
(d) R2 + S2 is divisible by 5
developing countries
Sol. (b)
(c) Human beings change their food habits from
If R and S are multiple of 5, then R + S may or may
time to time irrespective of the health concerns
not be divisible by 10. To solve such questions,
(d) From a global perspective, we still do not
always take actual values and check. For example,
know which type of diet is best for us
if R = 20 and S = 15. You will see that only (b) is
correct. Sol. (a)
The passage is not about developing countries
23. How many numbers are there between 100 and alone. It is about the fact that growth of population
300 which either begin with or end with 2? in developing countries and their choice of meat
(a) 110 (b) 111 and milk as food will affect the entire world’s
(c) 112 (d) None of the above resources. Hence, options (c) and (d) are wrong.
Sol. (a) Option (b) speaks only about developing nations,
Between 100 and 300, the number of numbers hence ruled out. Best answer is option (a). It is the
ending with 2 will be most critical message conveyed.

CSAT by Manjul Kumar Tiwari Sir


Previous Year Solved Papers 103
Passage - 2 (c) no part of national output should be produced
All humdigest mother’s milk as infants, but until cattle and consumed without passing through
began being domesticated 10,000 years ago, children once commercial channels
weaned no longer needed to digest milk. As a result, they (d) a part of the national output being produced
stopped making the enzyme lactase, which breaks down and consumed without passing through
the sugar lactose into simple sugars. After humbegan commercial channels is a sign of
herding cattle, it became tremendously advantageous to underdevelopment
digest milk, and lactose tolerance evolved independently Sol. (b)
among cattle herders in Europe, the middle East and Only option(b) reflects the the idea of Author.
Africa. Groups not dependant on cattle, such as the Option (a) may be enticing, but it is an extrapolation
Chinese and Thai, remain lactose intolerant. not mentioned in passage.
25. Which among the following is the most logical Passage - 4
assumption that can be made from the above An increase in human-made carbon dioxide in the
passage? atmosphere could initiate a chain reaction between plant
(a) About 10,000 years ago, the domestication of and microorganisms that would unsettle one of the
animals took place in some parts of the world largest carbon reservoirs on the planet soil In a study, it
(b) A permanent change in the food habits of a was found that the soil, which contains twice the amount
community can bring about a genetic change of carbon present in a plants and Earth’s atmosphere
in its members combined, could become increasingly volatile people add
more carbon dioxide to the atmosphere. This is largely
(c) Lactose tolerant people only are capable of
because of increased plant growth. Although a
getting simple sugars in their bodies
greenhouse gas and a pollutant, carbon dioxide also
(d) People who are not lactose tolerant cannot
supports plant growth. As trees and other vegetation
digest any dairy product
flourish in a carbon dioxide-rich future, their roots could
Sol. (b) stimulate microbial activity in soil that may in turn
The passage speaks about the historical background accelerate the decomposition of soil carbon and its relsase
of different societies in terms of their dependence into the atmosphere as carbon dioxide.
on cattle and milk, and consequent genetic changes.
27. Which among the following is the most logical
Options (c) and (d) are very limited, hence ruled
corollary to the above passage?
out. Option (a) is too direct, and not an assumption.
(a) Carbon dioxide is essential for the survival of
Option (b) is the best answer.
microorganisms and plants
Passage - 3 (b) Humare solely responsible for the release of
“The conceptual difficulties in National Income carbon dioxide into the atmosphere
comparisons between underdeveloped and (c) Microorganisms and soil carbon are mainly
industrialised countries are particularly serious because responsible for the increased plant growth
a part of the national output in various underdeveloped (d) Increasing green cover could trigger the
countries is produced without passing through the release of carbon trapped in soil
commercial channels.” Sol. (d)

26. In the above statement, the author implies that: Option (b) is clearly wrong because of “solely
(a) the entire national output produced and responsible”. Various natural processes like
volcanoes do so too. Option (c) is overlooking the
consumed in industrialized countries passes
interdependence of these factors. Option (a) is not
through commercial channels
wrong, but is a fact, not a corollary (a proposition
(b) the existence of a non-commercialized sector
that follows from one already proved). Option (d) is
in different underdeveloped countries renders
the best corollary. As green cover grows, more
the national income comparisons over carbon trapped in soil will be released.Hence option
countries difficult (d) is correct.

CSAT by Manjul Kumar Tiwari Sir


104 CSAT : 2016

Passage - 5 So options (a), (c) and (d) are wrong. Best answer is
Historically, the biggest Challenge to world agriculture option (b). Surely, the passage does mention that
has been to achieve a balance between demand for and price affordability by poor, and marketing
supply of food. At the level of individual countries, the regulation is important.
demand-supply balance can be a critical issue for a closed
29. According to the above passage, the biggest
economy, especially if it is a populous economy and its
challenge to world agriculture is:
domestic agriculture is not growing sufficiently enough
(a) to find sufficient land for agriculture and to
to ensure food supplies, on an enduring basis; it is not so
expand food processing industries
much and not always, of a constraint for an open, and
(b) to eradicate hunger in underdeveloped
growing economy, which has adequate exchange
countries
surplues to buy food abroad. For the world as a whole,
(c) to achieve a balance between the production
Supply-demand balance is always an inescapable
of food and non-food items
prerequisite for warding off hunger and starvation.
(d) to achieve a balance between demand for and
However, global availability of adequate supply does
supply of food
not necessarily mean that food would automatically
move from countries of surplus to of deficit if the latter Sol. (d)
lack in purchasing power. The uneven distribution of Options (a) and (c) can be ruled out right away.
Inoger, starvation, under or malnourishment, etc., at the Option (b) mentions a fact that is not a challenge to
world-level, thus owes itself to the presence of empty- world agriculture, but to its management. Option
pocket hungry mouths, overwhelmingly confined to the (d) is the key idea. Hence option (d) is the correct
underdeveloped economies. Inasmuch as ‘a two-square answer.
meal’ is of elemental significance to basic human
30. According to the above passage, which of the
existence, the issue of worldwide supply` of food has
following helps/help in reducing hunger and
been gaining significance, in recent times, both because
starvation in the developing economies ?
the quantum and the composition of demand has been
1. Balancing demand and supply of food
undergoing big changes, and because, in recent years,
2. Increasing imports of food
the capailities individual countries to generate
uninterrupted chain of food supplies have come under 3. creasing purchasing power of the poor
strain. Food production, marketing and prices, especially 4. Changing the food consumption patterns and
price-affordability by the poor in the developing world, practices
have become global issues that need global thinking and Select the correct answer using the code given
global solutions. below:
(a) 1 only
28. According to the above passage, which of, the (b) 2, 3 and 4 only
following are the fundamental solutions for the (c) 1 and 3 only
world food security problem? (d) 1, 2, 3 and 4
1. Setting up more agro-based industries Sol. (c)
2. Improving the price affordability by the poor
Statement 4 is not mentioned as a definite policy
3. Regulating the conditions of marketing
goal. So options (b) and (d) are eliminated. Both 1
4. Providing food subsidy to one and all
and 3 are clearly stated. Hence option (c) is the
Select the correct answer using the code given
correct answer.
below:
(a) 1 and 2 (b) 2 and 3 only 31. The issue of worldwide supply of food has gained
(c) 1, 3 an 4 only (d) 1, 2, and 4 importance mainly because of:
Sol. (b) 1. overgrowth of the population worldwide
The passage does not talk about setting up more 2. sharp decline in the area of food production
agro based industries. So statement 1 is wrong. 3. limitation in the capabilities for sustained
Similarly, 4 is wrong as subsidies are not mentioned. supply of food

CSAT by Manjul Kumar Tiwari Sir


Previous Year Solved Papers 105
Select the correct answer using the code given (a) 1/3 (b) 1/2
below: (c) 1/4 (d) 1/6
(a) 1 and 2 only (b) 3 only
Sol. (a)
(c) 2 and 3 only (d) 1, 2 and 3
Volume of underground tank = 5.5 × 4 × 6 = 132 m3
Sol. (b)
Volume of cylindrical overhead tank
Options 1 and 2 are not stated directly. Only
statement 3 is relevant to passage. Hence option (b) 22 2
2
is the correct answer. = r h   2  7  88 m3
7
32. Four-digit numbers are to be formed using the digits Volume left after filling overhead tank
1, 2, 3 and 4; and none of these four digits are 132 – 88 = 44
repeated in any manner. Further,
Ratio of portions left to that of origin will be
1. 2 and 3 are not to immediately follow each
other 44 1
2. 1 is not to be immediately followed by 3 
132 3
3. 4 is not to appear at the last place
4. 1 is not to appear at the first place 34. In a class of 60 students, where the number of girls
How many different numbers can be formed? is twice that of boys, Kamal, a boy, ranked
(a) 6 (b) 8 seventeenth from the top. If there are 9 girls ahead
(c) 9 (d) None of the above of Kamal, the number of boys in rank after him is:
Sol. (a) (a) 13 (b) 12
From the given conditions, for the four positions (c) 7 (d) 3
available : Sol. (b)
1 cannot come at the first place. So 2, 3 and 4 can The number of girls is twice that of boys in a class of
appear there. 60 students. Hence number of girls is 40 and boys
4 cannot come at the last place. So 1, 2 and 3 can is 20.
appear there. Now, Kamal is seventeenth from the top. Therefore
2 and 3 cannot immediately follow each other. So there are 16 students ahead of Kamal out of which
23 and 32 is not allowed. 9 girls, and so 7 are boys (16 - 9 = 7).
1 cannot be immediately followed by 3. So 13 is not Therefore, 20 – 7 – 1 (himself) = 12 boys will be
allowed. ranked after Kamal. Hence option (b) is the correct
Let us list the possible numbers now - answer.
2431 — possible, does not violate any condition 35. A and B walk around a circular park. They start at
2143 — possible, does not violate any condition 8 a.m. from the same point in the opposite
directions. A and B walk at a speed of 2 rounds
3142 — possible, does not violate any condition
per hour and 3 rounds per hour respectively. How
3412 — possible, does not violate any condition
many times shall they cross each other after
3421 — possible, does not violate any condition 8:00 a.m. and before 9.30. a.m.?
4312 — possible, does not violate any condition (a) 7 (b) 6
Hence, answer is (a). Total 6 numbers are possible. (c) 5 (d) 8
Sol. (a)
33. A cylindrical overhead tank of radius 2 m and
Let the length of track be D.
height 7 m is to be filled from an underground
tank of size 5.5m x 4m x 6m. How much portion of A walks at a speed of 2 rounds per hour.
the underground tank is still filled with water after So, one round is covered in 30 min.
filling the overhead tank completely?
Distance = D

CSAT by Manjul Kumar Tiwari Sir


106 CSAT : 2016

So, speed of A = distance/time = D/30 Sol. (a)


B walks at a speed of 3 rounds per hour. Average monthly income = Rs.10,000
So, total monthly income of the family of 5 =
So, one round is covered in 20 min
Rs.50,000
Distance = D
If one person’s income increases by Rs.1,20,000 per
So, speed of B = distance/time = D/20 year, his/her monthly income increases by
Rs.10,000.
Since, they are moving opposite to each other
So, total monthly income now becomes = Rs.60,000
Relative speed = Speed of A + Speed of B
So, average monthly income now becomes 60/5 =
Hence, they will cross each other after together they
Rs.12,000.
have made one full round of D distance
Hence, answer is (a).
Time = Distance/Relative speed
38. In a race, a competitor has to collect 6 apples which
Hence, they will cross each other after
are kept in a straight line On a track and a bucket
D 20  30  D 20  30  D is placed at the beginning of the track which is a
  = 12 min.
 D D  200  300 500 starting point. The condition is that the competitor
  
 30 20  can pick only one apple at a time, run back with it
Between 8 to 9:30 AM, i.e. in 90 min they will cross and drop it in the bucket. If he has to drop all the
each other 90/12 times, i.e. 7.5 (7 times). apples in the bucket, how much total distance he
has to run if the bucket is 5 meters from the first
36. W can do 25% of a work-in 30 days, X can do 1/4 apple and all other apples are placed 3 meters
of the work in 10 days, Y can do 40% of the work in apart?
40 days and Z can do 1/3 of the work in 13 days.
(a) 40 m (b) 50 m
Who will complete the work first?
(a) W (b) X (c) 75 m (d) 150 m
(c) Y (d) Z Sol. (d)
Sol. (d)
W can do 25% work in 30 days
So, W will do whole work in 30 × 4 = 120 days
Similarly, X will do whole work in 10 × 4 = 40 days
Y can do 40% work in 40 days
So, Y will do the whole work in 40 × (100/4) To pick and drop each apple competitor has to travel
= 100 days double its distance from bucket
Z can do 1/3 of work in 13 days Hence, total distance travelled will be

Therefore, Z will do whole work in 13 × 3 = 39 days = 2(5 + 8 + 11 + 14 + 17 + 20) = 2 × 75 = 150 min.

Therefore, Z will complete the work first. 39. A round archery target of diameter 1 m is marked
with four scoring regions from the centre outwards
37. The average monthly income of a person in a as red, blue, yellow and white. The radius of the
certain family of 5 is Rs. 10,000. What will be the red band is 0.20 m. The width of all the remaining
average monthly income of a person in the same bands is equal. If archers throw arrows towards
family if the income of one person increased by Rs. the target, what is the probability, that the arrows
1,20,000 per year? fall in the red region of the archery target?
(a) Rs. 12,000 (b) Rs. 16,000 (a) 0.40 (b) 0.20
(c) Rs. 20,000 (d) Rs. 34,000 (c) 0.16 (d) 0.04

CSAT by Manjul Kumar Tiwari Sir


Previous Year Solved Papers 107
Sol. (c) day or to society at large. In other words, how should
Total area of archery target = r = 0.25 
2 internal and external accountability be reconciled?
2 Internal accountability is sought to be achieved by
Area of red band =   0.2   0.04 
internal performance monitoring, official supervision by
Probability that the arrows fall in red region bodies like the —Central-Vigilance Commission-and-
Area of red band 0.04 4 16 Comptroller and Auditor—General, and judicial review
=     0.16
Total area 0.25 25 100 of executive decisions. Articles 311 and 312 of the Indian
Constitution provide job security and safeguards to the
civil services, especially the All India Services. The
framers of the Constitution had envisaged that provision
of these safeguards would result in a civil service that is
not totally subservient to the political executive but will
have the strength to function in larger public interest.
The need to balance internal and external accountability
is thus built into the Constitution. The issue is where to
draw the line. Over the years, the emphasis seems to have
tilted in favour of greater internal accountability of the
civil services to the political leaders of the day who in
turn are expected to be externally accountable to the
society at large through the election process. This system
40. A person allows 10% discount for cash payment for seeking accountability to Society has not worked out,
from the marked price of a toy and still he makes a and has led to several adverse consequences for
10% gain. What is the cost price of the toy which is governance.
marked Rs. 770?
Some special measures can be considered for improving
(a) Rs. 610 (b) Rs. 620
accountability in civil services. Provisions of articles 311
(c) Rs. 630 (d Rs. 640
and 312 should be reviewed and laws and regulations
Sol. (c)
framed to ensure external accountability of civil services.
Selling price of toy = 0.9 × marked price = 0.9 × 770
The proposed Civil Services Bill seeks to address some of
Now SP = 1.10 CP (As 10% gain is there)
these requirements. The respective roles of professional
SP 0.90  770 9 civil services and the political executive should he
CP =    770 = 9 × 70
1.10 1.10 11 defined so that professional managerial functions and
management of civil services are depoliticized. For this
Hence, CP = 630
purpose, effective statutory civil service boards should
Directions for the following 6 (six) items: be created at the centre and in the states. Decentralization
Read the following two passages and answer the items and devolution of authority to bring government and
that follow each passage. Your answers to these items decision making closer to the people also helps to
should be based on the passages only. enhance accountability.

Passage - 1 41. According to the passage, which of the following


Accountability, or the lack of it, in governance generally, factor/factors led to the adverse consequences for
and civil services, in particular, is a major factor governance/public administration?
underlying the deficiencies in governance and public 1. Inability of civil services to strike a balance
between internal and external
administration. Designing an effective framework for
accountabilities
accountability has been a key element of the reform
agenda. A fundamental issue is whether civil services 2. Lack of sufficient professional training to the
officers of All India Services
should be accountable to the political executive of the
CSAT by Manjul Kumar Tiwari Sir
108 CSAT : 2016

3. Lack of proper service benefits in civil services (a) Better job security and safeguards.
4. Lack of Constitutional provisions to define (b) Supervision by Central Vigilance
the respective roles of professional civil Commission.
services vis-a-vis political executive in this (c) Judicial review of executive decisions.
context (d) Seeking accountability through enhanced
Select the correct answer using the code given participation by people in decision making
below : process.
(a) 1 only (b) 2 and 3 only Sol. (d)
(c) 1 and 4 only (d) 2, 3 and 4 The key word is internal here. Only option (d) is
Sol. (c) correct.
Theme - Civil services and accountability issues - - Passage - 2
Only option (d) sounds sensible, complete and not
extreme. Option (a) is wrong. Option (b) is not In general, religious traditions stress our duty to god, or
mentioned anywhere. Option (c) is wrong. Hence to some universal ethical principle. Our duties to one
option (d) is correct. another derive from these. The religious concept of rights
is primarily derived from our relationship to this divinity
42. With reference to the passage, the following or principle and the implication it has on our other
assumptions have been made : relationships. This correspondence between rights and
1. Political executive is an obstacle to the duties is critical to any further understanding of justice.
accountability of the civil services to the But, for justice to be practiced; rights and duties cannot
society
remain formal abstraction. They must be grounded in a
2. In the present framework of Indian polity, the
community (common unity) bound together by a sense
political executive is no longer accountable
of common union (communion). Even as a personal
to the society
virtue, this solidarity is essential to the practice and
Which of these assumptions is/are valid?
understanding of justice.
(a) 1 only (b) 2 only
(c) Both 1 and 2 (d) Neither 1 nor 2 45. With reference to the passage, the following
Sol. (a) assumptions have been made :
Statement 1 is correct, and is not explicitly stated, 1. Human relationships are derived from their
but assumed. So it becomes the correct assumption. religious traditions
2. Human beings can be duty bound only if they
43. Which one of the following is the essential
believe in god
message implied by this passage?
3. Religious traditions are essential to practice
(a) Civil services are not accountable to the society
they are serving and understand justice
(b) Educated and enlightened persons are not Which of these assumption(s) is/are valid?
taking up political leadership (a) 1 only (b) 2 and 3 only
(c) The framers of the Constitution did not (c) 1 and 3 only (d) 1, 2 and 3
envisage the problems being encountered by Sol. (a)
the civil services The last line makes Assumption 3 invalid. So all
(d) There is a need and scope for reforms to options are eliminated, except (a) which is the best
improve the accountability of civil services answer.
Sol. (d)
46. Which one of the following is the crux of this
Option (d) is clearly implied in the paragraph.
passage?
Option (a) is a fact stated (not implied). Option (b)
is not stated. Clearly, option (d) is the correct answer. (a) Our duties to one another derive from our
religious traditions
44. According to the passage, which one of the
following is not a method of enhancing internal (b) Having relationship to the divine principle is
accountability of civil services? a great virtue

CSAT by Manjul Kumar Tiwari Sir


Previous Year Solved Papers 109
(c) Balance between and duties is crucial to the Sol. (d)
delivery of justice in a society
Option (a) is wrong. Option (c) is wrong as per
(d) Religious concept of rights is primarily
given statements. Option (b) is incomplete. Hence,
derived from our relationship to god
option (d) is best.
Sol. (c)
The main message of this passage is “balance 49. A person X was driving in a place where all roads
between rights and duties is crucial to the delivery ran either north-south or east-west, forming a grid.
of justice in a society.” Which is reflected in option Roads are at a distance of 1 km from each other in
(c). a parallel. He started at the intersection of two
47. A ate grapes and pineapple; B ate grapes and roads, drove 3 km north, 3 km west and 4 km south.
oranges; C ate oranges, pineapple and apple; D ate Which further route could bring him back to his
grapes, apple and pineapple. After taking fruits, B starting point, if the same route is not repeated?
and C fell sick. In the light of the above facts, it can (a) 3 km east, then 2 km south
be said that the cause of sickness was: (b) 3 km east, then 1 km north
(a) Apple (b) Pineapple (c) 1 km north, then 2 km west
(c) Grapes (d) Oranges (d) 3 km south, then 1 km north
Sol. (d) Sol. (b)
A simple diagram will help solve this.

B and C fell sick, and both of them ate oranges.


So, it must be the oranges which made them sick.
Correct answer is option (d). Since roads are either North-South or, East-West, it
48. Consider the following statements. can be seen that person X can reach his original
1. The rate of population growth is increasing location by first travelling 3 km east and then 1 km
in the country north as given in option (b).
2. The death rate is declining faster in the
50. Consider the following statement:
country compared to birth rate
3. The birth rate is declining faster in the country “We shall go either for a picnic or for trekking”.
compared to death rate
Which of the following, if true, would falsify this
4. Rural-urban migration is taking place
claim?
regularly in the country
(a) We go for a picnic but not for trekking
Which one of the following conclusions may be
(b) Activities such as picnic and trekking are
true in the light of the above facts?
encouraged by the health authorities
(a) The rate of population growth is increasing
(c) We go for trekking and not for picnic
due to rural-urban migration
(d) We do not go either for picnic or for trekking
(b) The rate of population growth is increasing
Sol. (d)
due to decline in death rate only
The statement meant that one of those two actions
(c) The rate of population growth is increasing
were bound to happen - either picnic, or trekking.
due to increase in birth rate only
(d) The rate of population growth is increasing Option (d) negates both. Options (a) and (c) are
due to faster decline in death rate than in birth correct - do not falsify the claim. Option (b) is
rate. irrelevant. Hence, best answer is option (d).

CSAT by Manjul Kumar Tiwari Sir


110 CSAT : 2016

51. There were 50 faculty member comprising 30 males Sol. (b)


and the rest females. No male faculty member knew The possible arrangements are
music, but many of the female faculty members
did. The Head of the institution invited six faculty
members to a tea party by draw of lots. At the party
is was discovered that no members knew music.
The conclusion is that:
(a) the party comprised male faculty members
Hence D is seated next to B and E is seated next to
only
A. (b)
(b) the party comprised only those female faculty
members who could not give renderings in 53. There are five hobby clubs in a college —
music photography, yachting, chess, electronics and
(c) the party comprised both male and female gardening. The gardening group meets every
faculty members second day, the electronics group meets every third
day, the chess group meets every fourth day, the
(d) nothing can be said about the gender
yachting group meets every fifth day and the
composition of the party
photography group meets every sixth day. How
Sol. (d) many times do all the five groups meet on the same
Question does not say anything about the gender day within 180 days?
composition of the group invited for party. Further, (a) 5 (b) 18
we do not know which specific female faculty knew (c) 10 (d) 3
music. So either all in party were males, or some Sol. (d)
males and some females who did not know music 5 hobby groups - photography, yachting, chess
were present in the party. All options have some electronis and gardening.
problem except (d) which is definitely correct. Gardening - Meets every second day hence =
90 times in 180 days.
52. Five people A, B, C, D and E are seated about a
Electronis group meets every third day =
round table. Every chair is spaced equidistant from
60 times / 180
adjacent chairs.
Chess group meets every 4th day = 45 times / 180
(i) C is seated next to A
Yachting group meets every 5th day =
(ii) A is seated two seats from D 36 times / 180
Photography group meets every 6th day =
(iii) B is not seated next to A
30 times / 180
On the basis of above information, which of the We have to find highest common factor of these
following must be true? 5 numbers 90, 60, 45, 36 and 30. It is 3. Hence option
1. D is seated next to B (d) is correct answer.
2. E is seated next to A 54. There are some nectar-filled flowers on a tree and
3. D and C are separated by two seats some bees are hovering on it. If one bee lands on
each flower, one bee will be left out. If two bees land
Select the correct answer using the code given on each flower, one flower will be left out. The
below: number of flowers and bees respectively are:
(a) 1 only (a) 2 and 4 (b) 3 and 2
(b) 1 and 2 only (c) 3 and 4 (d) 4 and 3
(c) 3 only Sol. (c)
(d) Neither 1 nor 2 nor 3 Only option (c) satisfies the conditions.

CSAT by Manjul Kumar Tiwari Sir


Previous Year Solved Papers 111
Directions for the following 5 (five) items: Consider the Sol. (b)
following information and answer the five items that
Person Profession Marital status Gender
follow:
P Student Unmarried M/F
There are five persons in a group — P, Q, R, S and T. The Q Lawyer Unmarried M
group has one doctor, one lawyer and one artist. P and S R Artist Married F
are unmarried students. T is a man married to one of the S Student Unmarried M/F
group members. Q is the brother of P and is neither doctor T Doctor Married M
nor artist. R is not doctor.
T is a married man.
For Q. 55 to Q. 59
58. Who is the lawyer?
This was the easiest 5 question set, which should have
(a) P (b) Q
been solved by all. (c) R (d) S
Straight 5 questions cracked! Sol. (b)

P and S are unmarried students (hence not professionals), Person Profession Marital status Gender
T a married man (so there is a female in this group), Q is P Student Unmarried M/F
brother of P, R is not a doctor. Q is neither a doctor nor an Q Lawyer Unmarried M
artist, so he must be a lawyer. R must be an Artist, so T R Artist Married F
should be a doctor (because P and S are students). S Student Unmarried M/F
T Doctor Married M
55. Who is the doctor?
(a) T (b) P 59. Who of the following is definitely a man?
(c) Q (d) R (a) P (b) S
Sol. (a) (c) Q (d) None of the above
Sol. (c)
Person Profession Marital status Gender
Person Profession Marital status Gender
P Student Unmarried M/F
P Student Unmarried M/F
Q Lawyer Unmarried M
Q Lawyer Unmarried M
R Artist Married F
R Artist Married F
S Student Unmarried M/F
S Student Unmarried M/F
T Doctor Married M
T Doctor Married M
56. Who is the artist? Q is the “brother” of P.
(a) P (b) Q
60. There is an order of 19000 quantity of a particular
(c) R (d) T
product from a customer. The firm produces 1000
Sol. (c)
quantity of that product per day out of which 5%
Person Profession Marital status Gender are unfit for sale. In how many days will the order
P Student Unmarried M/F be completed?
Q Lawyer Unmarried M (a) 18 (b) 19
R Artist Married F (c) 20 (d) 22
S Student Unmarried M/F Sol. (c)
T Doctor Married M Firm produces 1000 quantity per day of which 5%
are unfit.
57. Who is the spouse of R? So fit quantity = 1000 - 50 = 950 per day.
(a) P (b) T So effectively the firm is producing 950 quantities /
(c) Q (d) S day.

CSAT by Manjul Kumar Tiwari Sir


112 CSAT : 2016

Hence, it would take 20 days to produce 19000 1. Lack of sustainable supply of biomass
quantities. 2. Biomass production competes with food
(20 x 950 = 19000) production 3. Bio-energy may not always be
Directions for the following 5 (five) items: low carbon on a life-cycle basis

Read the following two passages and answer the items Select the correct answer using the code given
that follow each passage. Your answers to these items below:
should be based on the passages only. (a) 1 and 2 only (b) 3 only

Passage - 1 (c) 2 and 3 only (d) 1, 2 and 3

Biomass as fuel for power, heat, and transport has the Sol. (d)
highest mitigation potential of all renewable sources. It All the three constraints are mentioned in the
comes from agriculture and forest residues as well as passage. Hence, correct answer is (d).
from energy crops. The biggest challenge in using 62. Which of the following can lead to food security
biomass residues is a long-term reliable supply delivered problem?
to the power plant at reasonable costs; the key problems
1. Using agricultural and forest residues as
are logistical constraints and the costs of fuel collection.
feedstock for power generation
Energy crops, if not managed properly, compete with
food production and may have undesirable impacts on 2. Using biomass for carbon capture and storage
food prices. Biomass production is also sensitive to the 3. Promoting the cultivation of energy crops
physical impacts of a changing climate. Select the correct answer using the code given
Projections of the future role of biomass are probably below:
overestimated, given the limits to the sustainable biomass (a) 1 and 2 only (b) 3 only
supply, unless breakthrough technologies substantially
(c) 2 and 3 only (d) 1, 2 and 3
increase productivity. Climate-energy models project that
biomass use could increase nearly four-fold to around Sol. (b)
150 — 200 exajoules, almost a quarter of world primary The fifth line of first paragraph says “Energy crops
energy in 2050. However the maximum sustainable compete with food production and have an impact
technical potential of biomass resources (both residues on prices.” Hence only statement 3 is correct.
and energy crops) without disruption of food and forest 63. In the context of using biomass, which of the
resources ranges from 80 — 170 exajoules a year by 2050, following is/are the characteristic/characteristics
and only part of this is realistically and economically of the sustainable production of biofuel?
feasible. In addition, some climate models rely on
1. Biomass as a fuel for power generation could
biomass-based carbon capture and storage, an unproven
meet all the primary energy requirements of
technology, to achieve negative emissions and to buy
the world by 2050
some time during the first half of the century.
2. Biomass as a fuel for power generation does
Some liquid biofuels such as corn-based ethanol, mainly
not necessarily disrupt food and forest
for transport, may aggravate rather than ameliorate
resources
carbon emissions on a life-cycle basis. Second generation
biofuels, based on ligno-cellulosic feedstocks — such as 3. Biomass as a fuel for power generation could
straw, bagasse, grass and wood — hold the promise of help in achieving negative emissions, given
sustainable production that is high-yielding and emit certain nascent technologies
low levels of greenhouse gases, but these are still in the R Select the correct answer using the code given
& D stage. below:

61. What is/are the present constraint/constraints in (a) 1 and 2 only (b) 3 only
using biomass as fuel for power generation? (c) 2 and 3 only (d) 1, 2 and 3

CSAT by Manjul Kumar Tiwari Sir


Previous Year Solved Papers 113
Sol. (c) (c) Monocropping with high-yield varieties is the
Statement 1 is wrong. So options (a) and (d) are only way to ensure food security to millions
eliminated. Statement 2 is correct. Within limits, (d) Green revolution can pose a threat to
biomass as a fuel does not disrupt food and forest biodiversity in food supply and food security
in the long run
resources. Statement 3 can be correct is those
unproven nascent technologies work out. Hence Sol. (d)
best answer is (c). “We are witnessing a dangerous dwindling of
biodiversity in our food supply. The green
64. With reference to the passage, following revolution is a mixed blessing”. Hence, the most
assumptions have been made : logical and critical inference is option (d). Option
1. Some climate-energy models suggest that the (a) is not wrong, but that is a fact stated. Option (b)
use of biomass as a fuel for power generation is also a fact. Option (c) may or may not be correct.
helps in mitigating greenhouse gas emissions
66. A class starts at 11:00 am and lasts till 2:27 pm.
2. It is not possible to use biomass as a fuel for
Four periods of equal duration are held during
power generation without disrupting food
this interval. After every period, a rest of 5 minutes
and forest resources
is given to the students. The exact duration of each
Which of these assumptions is/are valid? period is:
(a) 1 only (b) 2 only (a) 48 minutes (b) 50 minutes
(c) Both 1 and 2 (d) Neither 1 nor 2 (c) 51 minutes (d) 53 minutes
Sol. (a) Sol. (a)
Assumption 1 is implicit in passage. Hence option Class starts at 11:00 am and lasts till 2:27 pm.
(a) is correct. Assumption 2 is valid only if done
Total time = 207 minutes
improperly, not always.
Total rest time = 3x5 = 15 minutes
Passage - 2
Effective time to be divided = 207 - 15 = 192 minutes
We are witnessing a dangerous dwindling of biodiversity
Duration of one period = 192/4 = 48 minutes
in our food supply. The green revolution is a mixed
blessing. Over time farmers have come to rely heavily on Hence, (a).
broadly adapted, high yield crops to the exclusion of
67. Four friends A, B, C and D need to cross a bridge.
varieties adapted to the local conditions. Monocropping
A maximum of two persons can cross it at a time. It
vast fields with the same genetically uniform seeds helps
boost yield and meet immediate hunger needs. Yet high- is night and they just have one lamp. Persons that
yield varieties are also genetically weaker crops that cross the bridge must carry the lamp to find the
require expensive chemical fertilizers and toxic way. A pair must walk together at the speed of
pesticides. In our focus on increasing the amount of food slower person. After crossing the bridge, the person
we produce today, we have accidentally put ourselves at having faster speed in the pair will return with the
risk for food shortages in future. lamp each time to accompany another person in
the group. Finally, the lamp has to be returned at
65. Which among the following is the most logical and
the original place and the person who returns the
critical inference that can be made from the above
lamp has to cross the bridge again without lamp.
passage?
To cross the bridge, the time taken by them is as
(a) In our agricultural practices, we have become
follows : A: 1 minute, B: 2 minutes, C: 7 minutes
heavily dependent on expensive chemical
and D: 10 minutes. What is the total minimum time
fertilizers and toxic pesticides only due to
green revolution required by all the friends to cross the bridge?
(b) Monocropping vast fields with high-yield (a) 23 minutes (b) 22 minutes
varieties is possible due to green revolution (c) 21 minutes (d) 20 minutes

CSAT by Manjul Kumar Tiwari Sir


114 CSAT : 2016

Sol. (a) 70. Anita’s mathematics test had 70 problems carrying


Minimum time would be achieved when the fastest equal marks i.e., 10 arithmetic, 30 algebra and 30
person would be assigned to return the lamp, who geometry. Although she answered 70% of the
in this case is A. arithmetic, 40% of the algebra and 60% of the
Time taken by A & B = 2 minutes + 1 minute return geometry problems correctly, she did not pass the
time of A = 3 minutes test because she got less than 60% marks. The
Time taken by A & C = 7 minutes + 1 minute return number of more questions she would have to
time of A = 8 minutes answer correctly to earn a 60% passing marks is:
Time taken by A & D = 10 minutes + 1 minute return (a) 1 (b) 5
time of A = 11 minutes
(c) 7 (d) 9
Time taken by A alone = 1 minute return time
without lamp = 1 minute Sol. (b)
So, Total minimum time will be 3 + 8 + 11 + 1 = 23 Let every question carry 1 mark. So total number of
minutes. marks is 70.
68. 30g of sugar was mixed in 180 ml water in a vessel Therefore passing marks = 60% of 70 = 42 marks.
A, 40 g of sugar Was mixed in 280 ml of water in
Anita answered 70% of Arithmetic (10 Q) = 7
vessel B and 20 g of sugar was mixed in 100 ml of
water in vessel C. The solution in vessel B is 40% of Algebra (30 Q) = 12
(a) sweeter than that in C 60% of Geometry (30 Q) = 18
(b) sweeter than that in A So, Anita’s total marks = 7 + 12 +18 = 37 marks
(c) as sweet as that in C which is 5 short of 42.
(d) less sweet than that in C
So 5 more marks are required to pass the exam.
Sol. (d) Hence (b).
Content of sugar in A = 30g/180 ml = 1/6 g/ml
Content of sugar in B = 40g/280 ml = 1/7 g/ml 71. In a class, there are 18 very tall boys. If these
Content of sugar in C = 20g/100 ml = 1/5 g/ml constitute three-fourths of the boys and the total
number of boys is two-thirds of the total number of
Hence vessel C is the sweetest, followed by vessels
students in the class, what is the number of girls
A and B. Hence.
in the class?
69. In aid of charity, every student in a class (a) 6 (b) 12
contributes as many rupees as the number of
(c) 18 (d) 21
students in that class. With the additional
contribution of Rs. 2 by one student only, the total Sol. (b)
collection is Rs. 443. Then how many students are Let number of girls in the class be X
there in the class?
Number of boys in the class will be 2X (2/3 being
(a) 12 (b) 21 boys, total being X + 2X = 3X)
(c) 43 (d) 45
Number of very tall boys = 3/4 * 2X = 18
Sol. (b)
So X = 12.
Total collection = Rs 443.
Hence (b).
Total collection of the class - additional
contribution = 443 - 2 = Rs 441 72. Consider the following statements:
Every student contributed an amount equal to total 1. Either A and B are of the same age or A is
number of students. older than B

Let total number of students in the class be N. So, N 2. Either C and D are of the same age or D is
2 = 441 older than C

Hence, N = 21. 3. B is older than C

CSAT by Manjul Kumar Tiwari Sir


Previous Year Solved Papers 115
Which of the following conclusions can be drawn 74. Which one of the following boxes contains the
from the above statements? golf ball?
(a) A is older than B (a) F (b) E
(b) B and D are of the same age (c) D (d) None of the above
(c) D is older than C
Sol. (b)
(d) A is older than C
Sol. (d) Direction for the following 3 (three) items: Consider
A > = B, D > = C, B > C. This implies that A > C. the given-formation and answer the three items that
Hence A is older than C. follow.

73. The monthly average salary paid to all the Six boxes A, B, C, D, E and F have been painted with six
employees of a company was Rs. 5000. The different colours viz., violet, indigo, blue, green, yellow
monthly average salary paid to male and female and orange and arranged from left to right (not
employees was Rs. 5200 and Rs. 4200 respectively. necessarily either kept or painted with the colours in the
Then the percentage of males employed in the same order). Each box contains a ball of any one of the
company is following six games: cricket, hockey, tennis, golf, football
(a) 75% (b) 80% and volleyball (not necessarily in the same order). The
(c) 85% (d) 90%
golf ball is in violet box and is not in the box D. The box
Sol. (b) A which contains tennis ball is orange in colour and is
Let the total number of male employees be M & that at the extreme right. The hockey ball is neither in box D
of female be F. nor in box E. The box C having cricket ball is painted
Total salary (males) = Rs 5200 x M green. The hockey ball is neither in the box painted blue
Total salary (females) = Rs 4200 x F nor in the box painted yellow. The box C is fifth from
So, we have 5200M + 4200F = 5000 (M+F) right and next to box B. The box B contains volleyball.
Solving, we get 200M = 800F The box containing the hockey ball is between the boxes
containing golf ball and volleyball.
Hence, M = 4F.
Percentage of males employed = 4F/5F *100 = 80 %. 75. Which of the following statements is/are correct?
Direction for the following 3 (three) items: Consider (a) D is painted yellow
the given -formation and answer the three items that
(b) F is painted indigo
follow.
Six boxes A, B, C, D, E and F have been painted with six (c) B is painted blue
different colours viz., violet, indigo, blue, green, yellow (d) All of the above
and orange and arranged from left to right (not
necessarily either kept or painted with the colours in the Sol. (b)
same order). Each box contains a ball of any one of the Direction for the following 3 (three) items: Consider
following six games: cricket, hockey, tennis, golf, football
the given -formation and answer the three items that
and volleyball (not necessarily in the same order). The
follow.
golf ball is in violet box and is not in the box D. The box
A which contains tennis ball is orange in colour and is Six boxes A, B, C, D, E and F have been painted with six
at the extreme right. The hockey ball is neither in box D different colours viz., violet, indigo, blue, green, yellow
nor in box E. The box C having cricket ball is painted and orange and arranged from left to right (not
green. The hockey ball is neither in the box painted blue necessarily either kept or painted with the colours in the
nor in the box painted yellow. The box C is fifth from same order). Each box contains a ball of any one of the
right and next to box B. The box B contains volleyball. following six games: cricket, hockey, tennis, golf, football
The box containing the hockey ball is between the boxes and volleyball (not necessarily in the same order). The
containing golf ball and volleyball. golf ball is in violet box and is not in the box D. The box

CSAT by Manjul Kumar Tiwari Sir


116 CSAT : 2016

A which contains tennis ball is orange in colour and is Sol. (c)


at the extreme right. The hockey ball is neither in box D Journey takes 42 hours (1 day and 18 hours). So a
nor in box E. The box C having cricket ball is painted train starting at 6 AM from one end can take return
green. The hockey ball is neither in the box painted blue journey on third day 6 AM from other end.
nor in the box painted yellow. The box C is fifth from Hence two trains each will required from each end
right and next to box B. The box B contains volleyball. is a dailyservice is required.
The box containing the hockey ball is between the boxes
Hence total number of trains required will be 4. (c)
containing golf ball and volleyball.
79. A piece of tin is in the form of a rectangle having
76. The football is in the box of which colour?
length 12 cm and width 8 cm. This is used to
(a) Yellow construct a closed cube. The side of the cube is:
(b) Indigo (a) 2 cm (b) 3 cm
(c) 4 cm (d) 7 cm
(c) Cannot be determined as data are inadequate
Sol. (c)
(d) Blue Surface area of constructed cube = Area of rectangle
Sol. (c) Hence, 6a2 = 12*8

77. Two numbers X and Y are respectively 20% and So, a = 4 cm. (c)
28% less than a third number Z. By what
80. In a. question paper there are five questions to be
percentage is the number Y less than the number
attempted and answer to each question has two
X? choices - True (T) or False (F). It is given that no
(a) 12% (b) 10% two candidates have given the answers to the five
questions in an identical sequence. For this to
(c) 9% (d) 8%
happen the maximum number of candidates is:
Sol. (b) (a) 10 (b) 18
Let third number Z be 100. (c) 26 (d) 32
Sol. (d)
Therefore X will be 80 & Y will be 72.
List the total number of cases which are distinct
Hence Y is 10% less than X. (b) from each other.
78. A daily train is to be introduced between station A Such cases will be 2 x 2 x 2 x 2 x 2 = 25 (basic
and station B starting from each end at 6 AM and counting property)
the journey is to be completed in 42 hours. What is In all these cases, no two candidates have marked
the number of trains needed in order to maintain answers identically.
the Shuttle Service? Hence, Maximum number of candidates is 25 = 32

(a) 2 (b) 3 This means that is there be a 33rd candidate, who


will mark answers matching exactly 1 of the existing
(c) 4 (d) 7 32 candidates. (d)



CSAT by Manjul Kumar Tiwari Sir


Previous Year Solved Papers 117

Previous Year
CSAT : 2017 Solved Papers

Passage-1 to denizens of our large cities. Option (d) is


An air quality index (AQI) is a way to combine wrong as the passage does not talk about it.
measurements of multiple air pollutants into a single Best answer is (c)
number or rating. This index is ideally kept constantly
Passage-2
updated and available in different places. The AQI is
Productive jobs are vital for growth and a good lob is the
most useful when lots of pollution data are being
best form of inclusion. More than half of our population
gathered and when pollution levels are normally, but
depends on agriculture, but the experience of other
not always, low. In such cases, if pollution levels spike
countries suggests that the number of . people dependent
for a few days, the public can quickly take preventive
on agriculture will have to shrink if per capita incomes
action (like staying indoors) in response to an air quality
in agriculture are to go up substantially. While industry
warning. Unfortunately, that is not urban India. Pollution
is creating jobs, too many such jobs are low-productivity
levels in many large Indian cities are so high that they non-contractual jobs in the unorganized sector, offering
remain well above any health or regulatory standard for low incomes, little protection, and no benefits. Service
large part of the year. If our index stays in the Red/ jobs are relatively of high productivity, but employment
Dangerous’ region day after day, there is not much any growth in services has been slow in recent years.
one can do, other than getting used to ignoring it.
2. Which among the following is the most logical and
1. Which among the following is the most logical and rational inference that can be made from the above
rational inference that can be made from the above passage?
passage? (a) We must create conditions for the faster
(a) Our governments are not responsible enough growth of highly productive service jobs to
to keep our cities pollution free. ensure employment growth and inclusion.
(b) There is absolutely no need for air quality (b) We must shift the farm workers to the highly
indices in our country. productive manufacturing and service sectors
to ensure the economic growth and inclusion.
(c) Air quality index is not helpful to the residents
(c) We must create conditions for the faster
of many of our large cities.
growth of productive jobs outside of
(d) In every city, public awareness about agriculture even while improving the
pollution problems should increase. productivity of agriculture.
Sol. (c) (d) We must emphasize the cultivation of high-
We neet to find the most logical and rational yielding hybrid varieties and genetically
inference (a conclusion on the basis of evidence and modified crops to increase the per capita
logic given). Option (a) is not entirely wrong, but income in agriculture.
cannot be inferred from this passage! Option (b) is Sol. (c)
wrong as the reverse of that is true – we need these Option (a) sounds good, but the passage gives a
indices even more, and with better implementation. matter of fact situation that service jobs are not
Option (c) is the best, as these indices are not helpful growing fast. Option (b) is wrong as such jobs are

CSAT by Manjul Kumar Tiwari Sir


118 CSAT : 2017

low productivity in India, not high. Option (c) is says, though starting with landscape-based
the best that can be inferred logically – this is what approach that encourages biodiversity, etc. Option
we should do now as a nation to absorb agri (b) is wrong as it’s the reverse of what the passage
workforce, and improve agriculture itself. Option tells. Option (c) is wrong as the first statement itself
(d) is wrong, as other nations’ experience suggests talks about “outside protected areas”. Option (d) is
otherwise. Best answer is (c). wrong as this is not discussed. Best answer is (a).

Passage-3 Passage-4

A landscape-scale approach to land use can encourage The medium term challenge for Indian manufacturing is
greater biodiversity outside cdected areas. During to move from lower to higher tech sectors, from lower to
hurricane ‘Mitch’ in 1998, farms using ecoagricultural higher value-added sectors, and from lower to higher
practices suffered 58 percent, 70 percent and 99 percent productivity sectors. Medium tech industries are
less damage in Honduras, Nicaragua and Guatemala, primarily capital intensive and resource processing; and
respectively, than farms using conventional techniques. high tech industries are mainly capital and technology
In Costa, vegetative windbreaks and fencerows boosted intensive. In order to push the share of manufacturing in
farmers’ income from pasture and coffee while also overall GDP to the projected 25 per cent, Indian
increasing birdopen in browser PRO version Are you a manufacturing needs to capture the global market in
developer? Try out the HTML to PDF API pdfcrowd.com sectors showing a rising trend in demand. These sectors
diversity. Bee pollination is more effective when are largely high technology and capital intensive.
agricultural fields are closer to natural or seminatural
4. Which among the following is the most logical and
habitat, a finding that matters because 87 percent of the
rational inference that can be made from the above
world’s 107 leading crops depend on animal pollinators.
passage
In Costa Rica, Nicaragua and Colombia silvopastoral
(a) India’s GDP displays high value-added and
systems t tint integrate trees with pastureland are
high productivity levels in medium tech and
improving the sustainability of cattle production, and
resource processing industries.
diversifying and increasing farmers’ income.
(b) Promotion of capital and technology intensive
3. Which among the following is the most logical and manufacturing is not possible in India.
rational inference that can be made from the above (c) India should push up the public investments
passage? and encourage the private investments in
(a) Agricultural practices that enhance research and development, technology
biodiversity can often increase farm output upgradation and skill development.
and reduce the vulnerability to disasters. (d) India has already gained a great share in
(b) All the countries of the world should be global markets in sectors showing a rising
encouraged to replace ecoagriculture with trend in demand.
conventional agriculture. Sol. (c)
(c) Ecoagriculture should be permitted in Option (a) is factually wrong, as per the passage.
protected areas without destroying the Medium tech ones are not high value-added ones.
biodiversity there. Option (b) is not wrong as that is what is being
(d) The yield of food crops will be very high if suggested. Option (c) is the only one going with the
ecoagricultural practices are adopted to passage. Option (d) is wrong as we are yet to achieve
cultivate them. that. Best answer is (c).
Sol. (a)
Passage-5
The passage tells us that land use (in farming) must
be as per the physical landscape properties, if Over the last decade, Indian agriculture has become more
disaster impacts are to be contained. Option (a) is robust with record production of food grains and oilseeds.
correct, as it says “agri practices that enhance Increased procurement, consequently, has added huge
biodiversity can increase output”. Passage also of food grains in the granaries. India is one of the world’s

CSAT by Manjul Kumar Tiwari Sir


Previous Year Solved Papers 119
top producers of rice, wheat, milk, fruits and vegetables. Sol. (b)
India is still home the quarter of all undernourished The passage tells us that if the Centre grows weak,
people in the world. On an average, almost half of the the Union of India will break! Option (a) is not the
total expenditure of nearly half of the households is idea being discussed. Option (b) is clearly correct.
on food. Option (c) is reverse of the idea. Option (d) is not
5. Which among the following is the most logical discussed. Hence, best answer is (b).
corollary to the above passage Passage-7
(a) Increasing the efficiency of farm to-fork value
chain is necessary to reduce the poverty and Really I think that the poorest he that is in England has a
malnutrition. life to live, as the greatest he, and therefore truly, I think it
is clear that every in an that is to live under a government
(b) Increasing the agricultural productivity will
ought first by his own consent to put himself under the
automatically eliminate the poverty and
government, and I do think that the poorest man in
malnutrition in India.
England is not at all bound in a strict sense to that
(c) India’s agricultural productivity is already government that he has not had a voice to put himself
great and it is not necessary to increase it under.
further.
(d) Allocation of more funds for social welfare 7. The above statement argues for
and poverty alleviation programmes will (a) distribution of wealth equally to all
ultimately eliminate the poverty and (b) rule according to the consent of the governed
malnutrition in India. (c) rule of the poor
(d) expropriation of the rich
Sol. (a)
Sol. (b)
The passage tells us that our record production is
Paragraph argues for the condition that those who
matched with other negative outcomes. We are asked
submit to a government must have had their voice
the most logical corollary (a logic that flows from
while the process (of submitting/surrendering) was
this logic). Option (a) of course follows, as only then on. Hence, options (a), (c) and (d) are wrong, and
will malnutrition and poverty reduce. Option (b) is best answer is (b).
wrong, as nothing happens automatically. Option
(c) is wrong as “productivity” is not “production”. 8. The average rainfall in a city for the first four days
Option (d) is wrong as it is not mentioned anywhere. was recorded to be 0.40 inch. The rainfall on the
Best answer is (a). last two days was in the ratio of 4 : 3. The average
of six days was 0.50 inch. What was the rainfall
Passage-6 on the fifth day?
(a) 0.60 inch (b) 0.70 inch
The States are like pearls and the Centre is the thread
(c) 0.80 inch (d) 0.90 inch
which turns them into a necklace; if the read snaps, the
Sol. (c)
pearls are scattered.
Let the rainfall of the nth day be x(n).
6. Which one of the following views corroborates the
above statement? So, according to first data, and using average
formula, we get :
(a) A strong Centre and strong States make the
federation strong. [x(1) + x(2) + x(3) + x(4)] / 4 = 0.4 inch.

(b) A strong Centre is a binding force for national So, x(1) + x(2) ...x(4) = 1.6 inch ...(1)
integrity. Also, rainfall of 5th day to 6th day bear a ratio of
(c) A strong Centre is a hindrance to State 4 : 3.
autonomy. So, they will be x(5) = 4x, and x(6)= 3x.
(d) State autonomy is a prerequisite for a So according to second data – [ x(1) + x(2) + x(3) +
federation. x(4) + x(5) + x(6) ] / 6 = 0.5 inch

CSAT by Manjul Kumar Tiwari Sir


120 CSAT : 2017

So, x(1) + x(2) + ....+ x(5) + x(6) = 3 inch ...(2) 11. Who of the following belongs to Hyderabad?
(a) B
Now put equation (1) in equation (2). That gives
(b) E
x(5)+ x(6) = 3 - 1.6 = 1.4 inch.
(c) Neither B nor E
Now put values of x(5) and x(6) according to ratio (d) Cannot be determined as data are inadequate
given. So, 4x + 3x = 1.4. Or, x = 0.2. So x(5) = 4x = 0.8 Sol. (b)
inch. Answer is (c).
Let us first try to assign the lecturers their residence
Directions for the following 3 (three) items : Consider places :
the given information and answer the three items that
D – Shillong, F – Srinagar (commerce), C – Delhi
follow. (sociology), G – Chennai(stats)
A, B, C, D, E, F and G are Lecturers from different cities— Now let us see case of A – He/She is not from
Hyderabad, Delhi, Shillong, Kanpur, Chennai, Mumbai Hyderabad (given) so only possible places are
and Srinagar (not necessarily in the same order) who Mumbai or Kanpur.
participated in a conference. Each one of them is
But the Lecturer from Kanpur is specialized in
specialized in a different subject, viz., Economics, Geography. So A is from Mumbai (he is specialized
Commerce, History, Sociology, Geography, Mathematics in Economics).
and Statistics (not necessarily in the same order).Further
So, the answer of Q.10 is (b) Mumbai.
1. Lecturer from Kanpur is specialized in Geography
Now from the remaining subjects : As A-Economics,
2. Lecturer D is from Shillong C-Sociology, F-Commerce, G-Stats, hence only
3. Lecturer C from Delhi is specialized in Sociology possible subjects are History, Geography and Maths.
4. Lecturer B is specialized in neither History nor Now as B is not specialized in either History or
Mathematics Mathematics.
He is Specialized in Geography. So answer of Q.9
5. Lecturer A who is specialized in Economics does
is (a).
not belong to Hyderabad
Now, A- Mumbai , B-Kanpur (given), C- Delhi, D-
6. Lecturer F who is specialized in Commerce belongs
Shillong, F-Srinagar, G-Chennai. So the only one
to Srinagar
remaining is E from Hyderabad. So the answer of
7. Lecturer G who is specialized in Statistics belongs Q.11 is (b).
to Chennai
12. In a school, there are five teachers A, B, C, D and E.
9. Who is specialized in Geography?
A and B teach Hindi and English. C and B teach
(a) B English and Geography. D and A teach
(b) D Mathematics and Hindi. E and B teach History
and French. Who teaches maximum numb, of
(c) E
subjects?
(d) Cannot be determined as data are inadequate (a) A (b) B
Sol. (a) (c) D (d) E
Sol. (b)
10. To which city does the Lecturer specialized in
We have to prepare a simple table, as shown below.
Economics belong?
It is an easy question and can be cracked in a minute.
(a) Hyderabad
(b) Mumbai
(c) Neither Hyderabad nor Mumbai
(d) Cannot be determined as data are inadequate
Sol. (b) As B teaches the most subjects.

CSAT by Manjul Kumar Tiwari Sir


Previous Year Solved Papers 121
13. A 2-digit number is reversed. The larger of the two Sol. (a)
numbers is divided by It smaller one. What is the
largest possible remainder?
(a) 9 (b) 27
(c) 36 (d) 45
Sol. (d)
Logically, since the largest remaineder is asked,
start from option (d). If we get 45 as the remainder it
is the final solution (biggest amongst options). Now
according to basic mathematics, any number which
is a divisor has to be greater than the remainder. So
if we want 45 as remainder we need to pick a no
greater than 45 (which will be divisor). So first we
will try in the range of 46 to 49. On picking 49 we Let xy plane be ceiling. So, the yz plane be Wall 1,
see that its reverse is 94, and 94/49 gives remainder and the zx plane will be Wall 2. The insect is
45. Hence our answer is 45. Hence option (d). somewhere in air.
Now, distance from Wall1 = 1 m., and distance from
14. The monthly incomes of X and Y are in the ratio of Wall2 = 8 m. Also, distance from P = 9 m.
4 : 3 and their monthly expenses are in the ratio of
Now let its distance from ceiling be z m.
3: 2. However, each saves Rs. 6,000 per month.
So by applying distance formula between two
What is their total monthly income?
points, “((x-x1)2 + (y-y1)2+ (z-z1)2 )
(a) Rs. 28,000 (b) Rs. 42,000
We will now get, distance from Point P (0, 0, 0) as
(c) Rs. 56,000 (d) Rs. 84,000
“((1-0)2 + (8-0)2 + (z-0)2) = 9
Sol. (b)
Square this on both sides, to get : 1 + 64 + z2 = 81. So
Let monthly income of X = 4p, and let then monthly z = 4. Hence the distance from the ceiling is 4 metres.
income of Y= 3p (ratio is given 4 : 3). Answer.(a).
Now let monthly expenditure of X= 3q. Then Directions for the following 3 (three) items : Consider
monthly expenditure of Y= 2q. Each of them saves the given information and answer the three items that
Rs.6000 p/m. follow.
So since Savings= Income – Expenditure, Eight railway stations A, B, C, D, E, F, G and H are
connected either by two-way passages or one-way
so 4p – 3q=6000 ...(1)
passages. One-way passages are from C to A, E to G, B to
and 3p – 2q = 6000 ...(2) F, D to H, G to C, E to C and H to G. Two-way passages
Subtracting both equations, we get 4p – 3p – 3q + 2q are between A and E, G and B, F and D, and E and D.
= 0. We get, p = q.
16. While travelling from C to H, which one of the
So putting p = q in equation (1), we get, p = q = 6000.
following stations must be passed through?
Now total monthly income = 4p + 3q = 7p = (a) G (b) E
7 x 6000 = Rs.42,000. Hence, answer is (b). (c) B (d) F
Sol. (b)
15. Two walls and a ceiling of a room meet at right
angles at a point P. A fly is in the air 1 m from one 17. In how many different ways can a train travel from
wall, 8 m from the other wall and 9 m from the F to A without passing through any station more
point P. How many meters is the fly from the than once?
ceiling? (a) 1 (b) 2
(a) 4 (b) 6 (c) 3 (d) 4
(c) 12 (d) 15 Sol. (d)

CSAT by Manjul Kumar Tiwari Sir


122 CSAT : 2017

18. If the route between G and C is closed, which one So total number of cases = 7. Hence, answer is (d) –
of the following stations need not be passed None of the above.
through while travelling from H to C?
20. What is the total number of digits printed, if a book
(a) E (b) D
containing 150 pages is to numbered from 1 to 150?
(c) A (d) B
(a) 262 (b) 342
Sol. (c)
(c) 360 (d) 450
Let us jot down all the possible portals. (Key – ‘–’
Sol. (b)
means one-way, ‘=’ means two-way)
Pages pritned are from 1 to 150.
Total digits from pages 1 to 9 = 9.
Total digits from pages 10 to 99 = 90 x 2 = 180.
Total digits from pages 100 to 150 = 51 x 3 = 153.
Hence, sum of all digits = 342.

Directions for the following 8 (eight) items :


Read the following seven passages and answer the items
that follow the passages. Your answers to these items
should be based on the passages only.
Now for Q.16 we have to go from C to H. If we want
to leave C there is only one possibility, that is, go Passage-1
to A. Disruption of traditional institutions, identifications and
Now from A there is only one possibility that is to loyalties is likely to lead to ambivalent situations. It is
go to E. So for every travel we have to make from C possible that some people may renew their identification
to H we will pass through E. Answer to Q.16 is (b). with traditional groups whereas others align themselves
Now for Q.17 the following paths are possible : F– with new groups and symbols emergent from processes
D–E–C–A, or F–D–H–G–C–A, or F–D–E–A, or F– of political development. In addition, political
D–E–G–C–A. development tends to foster group awareness of a variety
of class, tribe, region, clan, language, religion, occupation
So there are 4 ways of going from F to A without
and others.
repeating a station. So answer to Q.17 is (d).
For Q.18 we will stop the route G–C. Now 21. Which among the following is the best explation
possibilities for travelling from H to C are as follows: of the above passage?
H–G–E–C and H–G–B–F–D–E–C. So nowhere we (a) Political development is not a unilinear
have to travel through A. So answer to Q.18 is (c). process for it involves both growth and decay.
(b) Traditional societies succeed in resisting
19. There are certain 2-digit numbers. The difference positive aspects of political development.
between the number and the one obtained on (c) It is impossible for traditional societies to
reversing it is always 27. How many such break away from lingering loyalties.
maximum 2-digit numbers are there? (d) Sustenance of traditional loyalties is
(a) 3 (b) 4 conducive to political development.
(c) 5 (d) None of the above Sol. (a)
Sol. (d) Option (a) is not wrong, but is the best explanation
Let the numbers be xy and yx. So their values will as it talks of both growth (new identities emerging)
be (10x + y) and (10y + x). and decay (old dying). Option (b) is wrong as they
Now (10x + y) – (10y – x) = 27 (given)  9x – 9y = 27 do not succeed in doing so always (which is what
 x – y = 3. the option says). Option (c) is wrong due to the word
So possible combinations are 9-6, 8-5, 7-4, 6-3, 5-2, “impossible”. Option (d) is nowhere mentioned or
4-1, 3-0. explained. Best answer is (a).

CSAT by Manjul Kumar Tiwari Sir


Previous Year Solved Papers 123
Passage-2 Passage-3

There has been a significant trend worldwide towards We live in digital times. The digital is not just something
regionalism in government, resulting in a widespread we use strategically and specifically to do a few tasks.
transfer of powers downwards towards regions and Our very perception of who we are, how we connect to
communities since 1990s. This process, which involves the world around us, and the ways in which we define
the creation of new political entities and bodies at a sub- our domains of life, labour and language are hugely
structured by the digital technologies. The digital is
national level and an increase in their content and
everywhere and; like air, invisible. We live within digital
powers, is known as devolution. Devolution has been
systems, we live with intimate gadgets, we interact
characterized as being made up of three factors—political
through digital media, and the very presence and
legitimacy, decentralization of authority and
imagination of the digital has dramatically restructured
decentralization of resources. Political legitimacy here our lives. The digital, far from being a tool, is a condition
means a mass demand from below for the and context that defines the shapes and boundaries of
decentralization process, which is able to create a our understanding of the self, the society, and the
political force for it to take place. In many cases, structure of governance.
decentralization is initiated by the upper tier of
23. Which among the following is the most logical and
government without sufficient political mobilization for
essential message conveyed by the above passage?
it at the grassroots level, and in such cases the
(a) All problems of governance can be solved by
decentralization process often does not fulfil its objectives. using digital technologies.
22. Which among the following is the most logical, (b) Speaking of digital technologies is speaking
rational and critical inference that can be made of our life and living.
(c) Our creativity and imagination cannot be
from the above passage?
expressed without digital media.
(a) Emergence of powerful mass leaders is (d) Use of digital systems is imperative for the
essential to create sub-national political existence of mankind in future.
entities and thus ensure successful devolution Sol. (b)
and
The passage speaks of how human beings have
(b) The upper tier of government should impose become literally one with digital media tools.
devolution and decentralization on the Without passing a value judgement, it talks about
regional communities by law or otherwise. the intense connection between mankind and
(c) Devolution, to be successful, requires a digital. Hence, the best answer is (b), while options
democracy in which there is free expression (a), (c) and (d) can be rejected for being unrelated to
of the will of the people at lower level and the passage, too extreme, and too futuristic,
respectively. Best answer is (b).
their active participation at the grassroots
level. Passage-4
(d) For devolution to take place, a strong feeling The IMF has pointed out that the fast growing economies
of regionalism in the masses is essential. of Asia face the risk of falling into ‘middle-income trap’.
Sol. (c) It means that average incomes in these countries, which
till now have been growing rapidly, will stop growing
Option (d) sounds negative, and can be eliminated.
beyond a point—a point that is well short of incomes in
Option (b) is factually wrong as per the passage, as
the developed West. The IMF identifies a number of
that leads to failure of devolution. Option (a) is not causes of middle-income trap—none of which is
wrong, but when compared to Option (c), not too surprising—from infrastructure to weak institutions, to
good. Best answer is option (c) as it beautifully leas than favourable macroeconomic conditions. But the
captures the essence of the process of successful broad, overall cause, says IMF, is a collapse in the growth
devolution in long run. Best answer is (c) of productivity.

CSAT by Manjul Kumar Tiwari Sir


124 CSAT : 2017

24. Which among the following is the most logical, Sol. (c)
rational and critical inference that can be made Option (a) is wrong it focuses only on rural to urban
from the above passage? migration which is not the main issue. Option (b) is
(a) Once a country reaches middle-income stage,
not our choice as it is too broad a statement. Option
it runs the risk of falling productivity which
(c) is best, as it focuses on inclusive innovation and
leads to stagnant incomes.
R&D for creating an egalitarian society (India).
(b) Falling into middle-income trap is a general
Option (d) is wrong as it even in low growth nations,
characteristic of fast growing economies.
(c) There is no hope at all for emerging Asian people can migrate to cities (and that is not the core
economies to sustain the growth momentum. concern in this question). Best answer is (c).
(d) As regards growth of productivity, the
Passage-6
performance of Asian economies is not
satisfactory. Climate change is likely to expose a large number of
Sol. (a) people to increasing environmental risks forcing them to
Option (b) is wrong due to a broad generalization migrate. The international community is yet to recognize
being made (“…is a characteristic of fast-growing this new category of migrants. There is no consensus on
economies”) whereas the passage is talking of some the definition and status of climate refugees owing to the
Asian economies only. Option (c) is extreme, and distinct meaning the term refugees carry under
hence rejected. Option (d) is wrong as not “all Asian international laws. There are still gaps in understanding
economies” are showing productivity problems. how climate change will work as the root cause of
Hence, best answer is option (a), though it is worded migration. Even if there is recognition of climate refugees,
in a twisted manner that can confuse the cause- who is going to provide protection? More emphasis has
effect relationship, but others are definitely not been given to international migration due to climate
correct. Best answer is (a). change. But there is a need to recognize the migration of
Passage-5 such people within the countries also so that their
An innovative India will be inclusive as well as problems can be addressed properly.
technologically advanced, improving the lives of all 26. Which among the following is the most logical and
Indians. Innovation and R&D can mitigate increases in rational assumption that can be made from the
social inequality and relieve the pressures created by above passage?
rapid urbanization. The growing divergence in
(a) The world will not be able to cope with large
productivity between agriculture and knowledge-
scale migration of climate refugees.
intensive manufacturing and services threatens to
(b) We must find the ways and means to stop
increase income inequality. By encouraging India’s R&D
further climate change.
labs and universities to focus on the needs of poor people
(c) Climate change will be the most important
25. Which among the following is the most logical and reason for the migration of people in the future.
rational assumption that can be made from the (d) Relation between climate change and
above passage? migration is not yet properly understood.
(a) Innovation and R&D is the only way to reduce
Sol. (d)
rural to urban migration.
(b) Every rapidly growing country needs to Option (a) is wrong as it is drawing a final inference
minimize the divergence between (not be able to cope) without adequate information.
productivity in agriculture and other sectors. Option (b) is generically correct but this passage is
(c) Inclusive innovation and R&D can help create specifically about climate refugees (and migrants),
an egalitarian society. not stopping climate change. Option (c) is an
(d) Rapid urbanization takes place only when a assumption. Option (d) is the essence of the passage
country’s economic growth is rapid. (which can be inferred). Best answer is (d).

CSAT by Manjul Kumar Tiwari Sir


Previous Year Solved Papers 125
Passage-7 28. Which of the following statements is/are correct
regarding biopesticides?
Many farmers use synthetic pesticides to kill infesting
insects. The consumption of pesticides in some of the 1. They are not hazardous to human health.
developed countries is touching 3000 grams/hectare. 2. They are persistent in environment.
Unfortunately, there are reports that these compounds
possess inherent toxicities that endanger, the health of 3. They are essential to maintain the biodiversity
the farm operators, consumers and the environment. of any ecosystem.
Synthetic pesticides are generally persistent in Select the correct answer using the code given
environment. Entering in food chain they destroy the below.
microbial diversity and cause ecological imbalance.
(a) 1 only (b) 1 and 2 only
Their indiscriminate use has resulted in development of
resistance among insects to insecticides, upsetting of (c) 1 and 3 only (d) 1, 2 and 3
balance in nature and resurgence of treated populations. Sol. (a)
Natural pest control using the botanical pesticides is safer
Statement 2 is wrong (synthetic ones are persistent,
to the user and the environment because they break down
not biopesticides). Statement 3 is not correct, as it
into harmless compounds within hours or days in the
speaks of “biopesticides” not plants having such
presence of sunlight. Plants with pesticidal properties
properties. So only 1 is correct. So best answer is
have been in nature for millions of years without any ill
option (a).
or adverse effects on the ecosystem. They are easily
decomposed by many microbes common in most soil. 29. Certain 3-digit numbers following characteristics:
They help in the maintenance of biological diversity, of All the three digits are different.
predators and the reduction of environmental
contamination and human health hazards. Botanical The number is divisible by 7.
pesticides formulated from plants are biodegradable and The number on reversing the digits is also divisible
their use in crop protection is a practical sustainable by 7.
alternative.
How many such 3-digit numbers are there?
27. On the basis of the above passage, the following
(a) 2 (b) 4
assumptions have been made:
1. Synthetic pesticides should never be used in (c) 6 (d) 8
modem agriculture. Sol. (b)
2. One of the aims of sustainable agriculture is
Divide the numbers into those divisible by 7, and
to ensure minimal ecological imbalance.
falling between successive hundreds, as shown in
3. Botanical pesticides are more effective as
table. Now, remember, any number that has two
compared to synthetic pesticides.
similar digits is cancelled out (condition 1). And,
Which of the assumptions give above is/are
when you find the first possible number (in this
correct?
case 168), then you automatically find the second
(a) 1 and 2 only (b) 2 only
(reverse of 168, i.e. 861). So we have 2 numbers. Then
(c) 1 and 3 only (d) 1, 2 and 3
keep checking and you will find 259 is the next. So
Sol. (b) 592 is also there. Hence 4 in all so far. You will soon
The passage talks about the sensitive issue of see the pattern that when you check a number, its
pesticides, ecological balance and sustainability. reverse must be seen in this table itself. So, for
Statement 1 is extreme – “Synthetic pesticides example, if you see 427, then its reverse will be 724
should never be used in modern agriculture”. This and must be in the table. It is not. So 427 anyway
is nowhere mentioned, and we can conclude at the can never be a possible solution. Elimination
most that its use should be minimized. Hence 1 is becomes very easy. and the maximum such numbers
wrong. So only option (b) is correct. will be 4 only. Answer is (b).

CSAT by Manjul Kumar Tiwari Sir


126 CSAT : 2017

Sol. (d)
A least expected question from basic statistics!
When the distribution is Symmetric, it means Mean
= Mode = Median. So (a) is wrong. According to
Pearson Mode Skewness, if Mode > Median > Mean,
then the data is skewed to the left, so in this case it
is skewed to the left. So (d) is correct.

33. The age of Mr. X last year was the square of a


number and it would be the cube of a number next
year. What is the least number of years he must
30. Examine the following statements:
wait for his age to become the cube of a number
All colours are pleasant.
again?
Some colours are pleasant.
(a) 42 (b) 38
No colour is pleasant.
Some colours are not pleasant. (c) 25 (d) 16
Sol. (b)
Give that statemet 4 is true, what can be definitely
concluded? Focus on this year’s age of Mr X first. We should
(a) 1 and 2 are true. (b) 3 is true. think of a number which on subtracting 1 gives a
(c) 2 is false. (d) 1 is false. square number and on adding 1 gives a cube
Sol. (d) number. It should also be a relevant number as it is
the age of a person! Start by recalling the squares
If some colours are not pleasant (Statement 4), then
and cubes of 1, 2, 3, 4 …. and you will soon find that
option (d) i.e. statement 1, that “All colours are
26 is such a number. On subtracting 1 it gives 25
pleasant” is definitely wrong. Hence answer is (d).
(square of 5), and on adding 1 it gives 27 (cube of 3).
31. How many numbers are there between 99 and 1000 Now we have to find the least number of years he
such that the digit 8 occupies the units place? has to wait for his age to be a cube of a number
(a) 64 (b) 80 again. As 27 is 3’s cube, we will have to look at 4’s
(c) 90 (d) 104 cube that is 64. So the number of years he has to
Sol. (c) wait is 64 – 26 (his current age) = 38 years. Hence
We have to find the numbers in which “8” will answer is (b).
occupy the unit’s place. All the numbers will be 3
34. P works thrice as fast as Q, whereas P and Q
digit numbers of the form _ _ 8, in which the first
together can work four times as fast as R. If P, Q
place can be filled by 9 ways (by any one of
and R together work on a job, in what ratio should
1,2,3,4,5,6,7,8,9) and the second place can be filled
they share the earnings?
by 10 ways by 10 digits (1,2,3,4,5,6,7,8,9,0). So total
(a) 3 : 1 : 1 (b) 3 : 2 : 4
number of ways by the principle of counting is 10 x
9 = 90 ways. Hence answer is (c). (c) 4 : 3 : 4 (d) 3 : 1 : 4
Sol. (a)
32. If for a sample data
Let P work with speed 3x, and let Q work with
Mean < Median < Mode
speed x. Now work speed of both p and q taken
then the distribution is
together = 4x. This is faster than R’s work by 4 times,
(a) symmetric so R’s work speed = 4x/4 = x. So now finally P’s
(b) skewed to the right speed = 3x, Q’s speed = x, and R’s speed = x. So
(c) neither symmetric nor skewed they should share earnings in the ratio 3 : 1 : 1.
(d) skewed to the left Answer is (a).

CSAT by Manjul Kumar Tiwari Sir


Previous Year Solved Papers 127
35. Consider the following relationships among 37. If 2 boys and 2 girls are to be arranged in a row so
members of a family of six persons A, B, C, D, E that the girls are not next to each other, how many
and F: possible arrangements are there?
The number of males equals that of females. (a) 3 (b) 6
(c) 12 (d) 24
A and E are sons of F.
Sol. (c)
D is the mother of two, one boy and one girl.
Possible cases are – GBBG – 4 cases (as boys and
B is the son of A. girls can exchange seats within the group making
There is only one married couple in the family at it 2 x 2), BGBG – 4 cases, GBGB – 4 cases, and TOTAL
present. = 4 + 4 + 4 = 12 cases. Hence answer is (c). This
question should have been done correctly.
Which one of the following inferences can be
drawn from the above? 38. The outer surface of a 4 cm x 4 cm x 4 cm cube is
(a) A, B and C are all females. painted completely in red. It is sliced parallel to
(b) A is the husband of D. the faces to yield sixty four 1 cm x 1 cm x 1 cm
(c) E and F are children of D. small cubes. How many small cubes do not have
(d) D is the daughter of F. painted faces?
Sol. (b) (a) 8 (b) 16
(c) 24 (d) 36
A and E are sons of F (mother or father). B is the son
Sol. (a)
of A (father, as he is the “son” of F). D is the mother
of a boy and a girl. Option (a) is wrong as B is the As we divide a painted cube into 64 smaller cubes,
son. Option (c) is wrong as E and F are not siblings there are 4 layers of cubes formed which have 16
(given). We are left with options (b) and (d). If you cubes each arranged like a cuboid. Now, the middle
assume (b) is correct, then (d) is wrong as A and D 2 cuboids have 4 cubes each which are completely
won’t be siblings. Hence, our final answer is (b). [ F uncoloured, so the correct answer here is 4 + 4 = 8.
is the grandmother in the family, whose sons are A Hence best answer is (a). This question should have
and E, and A’s wife is D, and B and C are children been done correctly.
of A and D. Females are F, D and C. Males are A, B
and E (condition 1) ]. Answer is (b).

36. A bag contains 20 balls. 8 balls are green, 7 are


white and 5 are red. What is Hie minimum number
of balls that must be picked up from the bag
blindfolded (without replacing any of it) to be
assured of picking at least one ball of each colour?
(a) 17 (b) 16 39. Consider the following:
(c) 13 (d) 11 A, B, C, D, E, F, G and H are standing in a row
Sol. (b) facing North.
There are 8 balls that are green and 7 that are white. B is not neighbour of G.
8 G + 7 W + 5 R =20. Now, if we pick 8 + 7 = 15 balls,
F is to the immediate right of G and neighbour of E.
there remains a chance that 8/8 are green and 7/7
are white, that is all 15 are green and white. But if G is not at the extreme end.
we pick 15 + 1 = 16 balls, there cannot be more than A is sixth to the left of E.
15 balls that are green and white, so we will get at
H is sixth to the right of C.
least 1 red ball. So, 16 is the answer when we are
sure of picking at least one ball of each colour Which one of the following is correct in respect of
(G, W, R). the above?

CSAT by Manjul Kumar Tiwari Sir


128 CSAT : 2017

(a) C is to the immediate left of A. 41. What does the above passage imply?
(b) D is immediate neighbour of B and F. (a) Resource-based conflicts are always
politically motivated.
(c) G is to the immediate right of D.
(b) There are no political solutions to resolve
(d) A and E are at the extreme ends. environmental and resource based conflicts.
Sol. (c) (c) Environmental issues contrilaite to resource
stresses and politic ril conflict.
The correct arrangement is (from left to right)
(d) Political conflict based on identity or ideology
A–C–B–D–G–F–E–H. First, since A is sixth to the
cannot be resolved.
left of E, it can be either A x x x x x E x or it can be x
Sol. (c)
A x x x x x E. Now, since H is sixth to the right of C,
it can be either C x x x x x H x or it can be x C x x x x Option (a) says that “such conflicts are always
x H. Since F is to the immediate right of G, and politically motivated”. The passage does not say so
neighbor of E, we have G F E. Combining it all – it merely says that “… have rarely been overt
together, we get A–C–B–D–G–F–E–H. So check all (open)”. The word rarely does not mean never.
options and G is to the immediate right of D. Hence, Hence, answer is not option (a). Option (b) is again
(c) is correct. an extreme option as it talks of “there are no political
solutions”. That isn’t mentioned hence rejected.
40. In a certain code, ‘256’ means ‘red colour chalk’,
Option (c) is correct. Option (d) is an extreme
‘589’ means ‘green colour flower’ and ‘254’ means
‘white colour chalk’. The digit in the code that statement not borne out from this limited passage.
indicates `white’ is So rejected. Best answer is (c).

(a) 2 (b) 4 Passage–2


(c) 5 (d) 8
The man who is perpetually hesitating which of the two
Sol. (b) things he will do first, will do neither. The man who
Since 2 is in ‘red colour chalk’, so it must mean resolves, but suffers his resolution to be changed by the
something from this set. So 2 does not mean white. first counter Suggestion of a friend—who fluctuates from
Similarly 5 is in ‘red colour chalk’ so it couldn’t opinion to opinion and veers from plan to plan-can never
mean white. Now 8 isn’t in ‘white colour chalk’ so accomplish anything. He will at best be stationary and
8 can’t mean white. So only remaining possibility probably retrograde in all. It is only the man who first
is option (b). consults wisely, then resolves firmly and then executes
his purpose with inflexible perseverance, undismayed
Directions for the following 8 (eight) items: Read the by those petty difficulties which daunt a weaker spirit—
Following eight passages and answer the items that that can advance to eminence in any line.
follow the passages. Your answers to these items should
42. The keynote that seems to be emerging from the
be based on the passages only.
passage is that
(a) we should first consult wisely and then
Passage-1
resolve firmly
What climate change will undeniably do is cause of (b) we should reject suggestions of friends and
amplify events that hasten the reduction of resources. remain unchanged
Competition over these diminising resources would (c) we should always remain broad-minded
ensue in the form of political or even violent conflict. (d) we should be resolute and achievement-
Resource based conflicts have rarely been overt and are oriented
thus difficult to isolate. Instead they take on veneers that Sol. (a)
appear more politically palatable. Conflicts over Option (b) because is wrong, and (c) as it is too
resources like water are often cloaked in the guise of vague. Options (a) and (d) look close. The reason
identity or ideology. (a) is answer because the passage delves in-depth

CSAT by Manjul Kumar Tiwari Sir


Previous Year Solved Papers 129
on the issue of suggestions and wavering mind. (b) People have to perceive toilet use and pit-
Hence (a) comes closest as “we should first consult emptying as clean and not polluting.
wisely and then resolve firmly” captures that (c) People cannot change their old habits.
keynote idea. Best answer is (a). (d) People have neither civic sense nor sense of
Passage-3 privacy.
Sol. (b)
During the summer in the Arctic Ocean, sea ice has been
The most crucial message conveyed is option (b), as
melting earlier and faster, and the winter freeze has been
the other 3 options are too negative and reject the
coming later. In the last three decades, the extent of
possibility of any solution to the social problem.
summer ice has declined by about 30 per cent. The
Best answer is (b).
lengthening period of summer melt threatens to
undermine the whole Arctic food web, atop which stand Passage-5
polar bears. In the last two decades, the world’s gross domestic
43. Which among the following is the most crucial product (GDP) has increased 50 percent, whereas
message conveyed by the above passage? inclusive wealth has increased by a mere 6 percent. In
(a) Climate change has caused Arctic summer to recent decades, GDP-driven economic performnce, has
be short but temperature to be high. only harmed inclusive wealth like human capital; and
(b) Polar bears can be shifted to South Pole to natural capital like forests, land and water. While the
ensure their survival. world’s human capital which stands at 57 percent of
(c) Without the presence of polar bears, the food total inclusive wealth grew by only 8 percent, the natural
chains in Arctic region will disappear. which is 23 percent of total inclusive wealth declined by
30 per cent worldwide in the last two decades.
(d) Climate change poses a threat to the survival
of polar bears. 45. Which of the following is the most crucial inference
Sol. (d) from the above passage?
The most crucial message is obvious – the polar (a) More emphasis should be laid on the
bears will face a true existential threat due to climate development of natural capital.
change (and Arctic ice reduction etc.). Option (c) (b) The growth driven by GDP only is neither
makes a big claim, not necessarily true. Option (b) desirable nor sustainable.
is desirable, but impractical and not mentioned! (c) The economic performance of the countries of
Option (a) is reverse of what’s being said. Best the world is not satisfactory.
answer is (d).
(d) The world needs more human capital under
Passage-4 the present circumstances.
Sol. (b)
Why do people prefer open defecation and not want
toilets or, if they have them, only use them sometimes? Option (c) is wrong. Options (a) and (d) are not
Recent research has shown two critical elements : ideas wrong, but are not the key ideas. The key idea
of purity and pollutions, and not wanting pits or septic is that we have taken the wrong route to
tanks to fill they have to be emptied. These are the issue prosperity, and only option (b) resonated with
that nobody wants to talk about, but if we want to that. Answer is (b).
eradicate the practice of open defection, they have to be
Passage-6
confronted and dealt properly.
By 2020, when the global economy is expected to run
44. Which among the following is the most crucial short of 56 million young people, India, with its youth
message conveyed by the above passage? surplus of 47 million, could fill the gap. It is in this context
(a) The ideas of purity and pollutions are so that labour reforms are often cited as the way to unlock
deep-rooted that they cannot be removed from double-digit growth in India. In 2014, India’s labour force
the minds of the people. was estimated to be about 40 per cent of the population,

CSAT by Manjul Kumar Tiwari Sir


130 CSAT : 2017

but 93 per cent of this force was in unorganized sector. Passage-8


Over the last decade, the compound annual growth rate
There is no harm in cultivating habits so long as they are
(CAGR) of employment has slowed to 0.5 per cent, with
not injurious. Indeed, most of us are little more than
about 14 million jobs created during last year when the
bundle of habits. Take away our habbits and the
labour force increased by about 15 million.
residuum would hardly be worth bothering about. We
46. Which of the following is most rational inference could not get on without them. They simplify the
from the above passage? mechanism of life. They enable us to do a multitude of
(a) India must control its population growth so
things automatically, which, if we had to give fresh and
as to reduce its unemployment rate.
original thought to them each time, would make existence
(b) Labour reforms are required in India to make
an impossible confusion.
optimum use of its vast labour force
productively 48. The author suggests that habits
(c) India is poised to achieve the double-digit (a) tend to make our lives difficult
growth very soon (b) add precision to our lives
(d) India is capable of supplying !It, skilled
(c) make it easier for us to live
young people to oil, countries.
(d) tend to mechanize our lives
Sol. (b)
Sol. (c)
Options (a) and (c) get ruled out – (a) because it is
beyond the scope of this passage, and (c) as it is an The idea of passage is that man is nothing but all
assumption. (d) is not the prime focus here, but (b) habits taken together. These make life regular and
says we have to change the labour laws to achieve automatic, thereby making it easy to live, which
faster growth. Which is main focus of the passage otherwise would be impossible. Hence, option (a)
hence answer is (b). is wrong. Option (b) is not mentioned. Option (d) is
not exactly mentioned so (automatically does not
Passage-7 always mean “mechanized”. Best answer is
The very first lesson that should be taught to us when option (c).
are old enough to understand it, is that complete freedom Directions for the following 2 (two) items: Consider the
from the obligation to work is unnatural, and ought to be given information and answer the two items that follow.
illegal, as we can escape our share of the burden of work
only by throwing it on someone else’s shoulders. Nature No supporters of ‘party X’, who knew Z and supported
ordains that the human race shall perish of famine if it his campaign strategy, agreed for the alliance with ‘party
stops working. We cannot escape from this tyranny. The Y’; but some of them had friends in ‘party Y’.
question we have to settle is how much leisure we can
49. With reference to the above information, which one
afford to allow ourselves.
among the following statements must be true?
47. The main idea of the passage is that (a0 Some supporters of ‘party Y’ did not agree for
(a) it is essential for human beings to work the alliance with the `party X’.
(b) there should be a balance between work and (b) There is at least one supporter of ‘party Y’
leisure who knew some supporters of ‘party X’ as a
(c) working is a tyranny which we to face friend.
(d) human’s understanding of the nature of work (c) No supporters of ‘party X’ supported Z’s
is essential campaign strategy.
Sol. (a) (d) No supporters of ‘party X’ knew Z.

Option (b) is not the main idea, but a final Sol. (b)
suggestion. Option (a), (c) and (d) are all correct The line “but some of them had friends in ‘party
points. But it is only in option (a) that main idea of Y’” clearly indicates that statement (b) is correct.
passage is being reflected. Best answer is (b).

CSAT by Manjul Kumar Tiwari Sir


Previous Year Solved Papers 131
50. With reference to the above information, consider 53. Four tests—Physics, Chemistry, Mathematics and
the following statement Biology are to be conducted on four consecutive
1. Some supporters of ‘party X’ knew Z days, not necessarily in the same order. The Physics
2. Some supporters of ‘party X’, who opposed test is held before the test which is conducted after
Z’s campaign strategy, knew Z. Biology. Chemistry is conducted exactly after two
3. No supporters of ‘party X’ supported Z’s tests are held. Which is the last test held?
campaign strategy. (a) Physics (b) Biology
Which of the statements given above, is/are not (c) Mathematics (d) Chemistry
correct? Sol. (c)
(a) 1 only (b) 2 and 3 only The correct sequence is Biology – Physics –
(c) 3 only (d) 1, 2 and 3 Chemistry – Mathematics. Best answer is (c).
Sol. (c)
Statement 1 is correct. Statement 2 is also correct 54. The sum of income of A and B is more than that of
Statement 3 incorrect. Hence, best answer is (c). C and D taken together. The sum of income of A
and C is the same as that of B and D taken together.
(note that we are asked the not correct ones).
Moreover, A earns half as much as the sum of the
51. If second and fourth Saturdays and ,all the income of B and D. Whose income is the highest?
Sundays are taken as only holidays for an office, (a) A (b) B
what would be the minimum number of possible (c) C (d) D
working days of any month of any year?
Sol. (b)
(a) 23 (b) 22
Given that the sum of incomes of A and B is more
(c) 21 (d) 20
than that of C and D. So, A + B > C + D. Similarly, A
Sol. (b)
+ C = B + D. Also given that A = (B + D)/2, so since
Since minimum number of working days is asked,
A + C is equal to B + D, hence C = (B + D)/2.
start with the smallest month possible. That would
Substituting values of A and C in terms of B and D,
be a February, in a non-leap year, with 28 days in
in the equation A + B > C + D, we get : 3B/2 + D/2
all. So the first day be Sunday, then the 8th, 15th
and 22nd will be Sundays as well. For the second > 3D/2 +B/2 Which gives us B > D. So the correct
and fourth Saturdays, we will have only 14th and answer is B, option (b). Also note that since A and C
28th. Hence total holidays will be 4 + 2 = 6. Working are equal, those two options get eliminated anyway.
days = 22. Best answer is (b). Any guesswork should be only between B and D.

52. If there is a policy that 1/3rd of population of a 55. Consider the following:
community has migrated every year from one Statement:
place, to some other place, what is the leftover Good voice is a natural gift but one has to keep
population of that community after the sixth year, practising to improve and well in the field of music.
if there is no further growth in the population
Conclusions:
during this period?
I. Natural gifts need nurturing and care.
(a) 16/243rdpart of the population
II. Even though one’s voice is not good, one can
(b) 32/243rd part of the population
keep practising.
(c) 32/729 thpart of the population
(d) 64/729th part of the population Which one of the following is correct, in respect of
Sol. (d) the above statement and conclusions?
As per the policy, population left after first year= (a) Only conclusion I follows from the statement.
2/3 of original, as 1/3 has left. Now apply a simple (b) Only conclusion II follows from the statement.
logic using ratio. Population left after second year= (c) Either conclusion I or conclusion II follows
[2/3]^2 of original. And so on. So, population left from the statement.
after 6th year= [2/3]^6 of original, which is equal (d) Neither couclusion I nor conclusion II follows
to 64/729 part of the population. Best answer is (d). from the statement.

CSAT by Manjul Kumar Tiwari Sir


132 CSAT : 2017

Sol. (a) 58. 15 students failed in a class of 52. After removing


Conclusion I definitely follows as that is the main the names of failed students, a merit order list has
assertion given. Conclusion II is incorrect, as it says been prepared in which the position of Ramesh is
“Even though one’s voice is not good, one can keep 22nd from the top. What is his position from the
practicing”. It is an assumption we will be making. bottom?
Hence best answer is (a). (a) 18th (b) 17th
(c) 16th (d) 15th
56. There are three pillars X, Y and Z of different Sol. (c)
heights. Three spiders A, B and C start to climb on
Total students in merit list= 37 (= 52 – 15). Ramesh’s
these pillars simultaneously. In one chance, A
position in these 37 students = 22nd, and so,
climbs on X by 6 cm but slips down 1 cm. B climbs
students behind him= 15 students (37 – 22). So, his
on Y by 7 cm but slips down 3 cm. C climbs on Z by
position from bottom = 16th (15 + 1). Answer is (c).
6.5 cm but slips down 2 cm. If each of them requires
40 chances to reach the top of the pillars, what is
the height of the shortest pillar?
(a) 161 cm (b) 163 cm
(c) 182 cm (d) 210 cm
Sol. (b)
Spiders A, B and C are climbing up the pillars X, Y
59. Consider the following:
and Z. In one climb, A goes up X by 6 cm, and falls
A+ B means A is the son of B.
by 1 cm. So net in one climb = 5 cm. Similarly, B goes A - B means A is the wife of B.
up Y by 7 cm, and falls by 3 cm. So net in one climb
What does the expression P + R - Q mean?
= 4 cm, and for C on Z, it is net 4.5 cm in one climb.
(a) Q is the son of P.
Given that all three took 40 climbs (chances) and
(b) Q is the wife of P.
reached the top. So they touched the top in the 40th
(c) Q is the father of P.
climb, and we should do the “net” calculation till (d) None of the above
the 39th for all three. For spider A, in 39 chances it Sol. (c)
will go up by 5 x 39 cms, B will go up by 4 x 39 cms,
P + R means P is son of R. R – Q means R is wife of
and C will go up by 4.5 x 39 cms. Obviously, for
Q. So (P + R – Q) means Q is the father of P. Answer
spider B, the distance is going to be the smallest. So
(c).
the total length for pole Y = (4 x 39) + 7 = 163 cms. A
common mistake made here is that we simply 60. Gopal bought a cell phone and sold it to Ram at
multiply net distance with 40, which we cannot (as 10% profit. Then Ram wanted to sell it back to
once the spider reaches the top, the question would Gopal at 10% loss. What will be Gopal’s position
be solved). Best answer is option (b). if he agreed?
(a) Neither loss nor gain
57. “Rights are certain advantageous conditions of (b) Loss 1%
social well-being indispensable to the true (c) Gain 1%
development of the citizen.” (d) Gain 0.5%
In the light of this statement, which one of the Sol. (c)
following is the correct understanding of rights? Let original price of phone = Rs 100.
(a) Rights aim at individual good only. Gopal sold at 10% profit. So selling price = Rs.110.
(b) Rights aim at social good only. Ram agreed to sell at a 10% loss so he will sell for
(c) Rights aim at both individual and social good. Rs 110 – 10% of 110 = Rs.110 – 11 = Rs 99.
(d) Rights aim at individual good devoid of social So total profit of Gopal = Rs 100 – Rs 99 = Re 1. But
well-being. this profit was made on his cost price (which is
Sol. (c) 100). So total % profit= 1 %. Answer is (c).

CSAT by Manjul Kumar Tiwari Sir


Previous Year Solved Papers 133
Directions for the following 7 (seven) items: Passage-3
Read the following seven passages and answer the items
In a democratic State, where a high degree of Political
that follow the passages. Your answer to these items
maturity of the people obtains, the conflict between the
should be based on the passages only.
will of the sovereign law-making body and the organized
Passage-1 will of the people seldom occurs.
We have hard work ahead. There is no resting for any of
63. What does the above passage imply?
us till we redeem our pledge in full fill we make all the
people of India what destiny intends them to be. We are (a) In a democracy, force is the main phenomenon
citizens of a great country, on the verge of bold advance, in the actual exercise of sovereignty.
and we have to live up to that high standard. All of us, to (b) In a mature democracy, force to a great extent
whatever religion we may belong, are equally the children is the main phenomenon in the actual exercise
of India with, equal rights, privileges and obligations. of sovereignty.
We cannot encourage communalism or narrow (c) In a mature democracy, use of force is
mindedness, for no nation can be great whose people are irrelevant in the actual exercise of sovereignty.
narrow in thought or action. (d) In a mature democracy, force is narrowed
61. The challenge the author of the above passage down to a marginal phenomenon in the
throws to the public is to achieve actual exercise of sovereignty.
(a) a high standard of living, progress and Sol. (d)
privileges Option (a) is wrong as the reverse is mentioned.
(b) equal privileges, fulfilment of destiny and Option (b) too is wrong! So we easily reduce options
political tolerance from 4 to 2. Check (c). It speaks of “irrelevance of
(c) spirit of adventure and economic parity force” and (d) talks of “a marginal phenomenon”.
(d) hard work, brotherhood and national unity Refer passage. It mentions “seldom occurs”. Hence,
Sol. (d) option (d) narrowly wins over (c). Best answer is
(d).
Passage-2
“The individual, according to Rousseau, puts his person Passage-4
and all his power in common under the supreme direction
of the General Will and in our corporate capacity we A successful democracy depends upon widespread
receive each member as an indivisible part of the whole.” interest and participation in politics, in which voting is
an essential part. To deliberately refrain from taking such
62. In the light of the above passage, the nature of an interest, and from voting, is a kind of implied anarchy,
General Will is best described as it is to refuse one’s political responsibility while enjoying
(a) the sum total of the private wills of the the benefits of a free political society.
individuals
(b) what is articulated by the elected 64. This passage relates to
representatives of the individuals (a) duty to vote
(c) the collective good as distinct from private (b) right to vote
wills of the individuals
(c) freedom to vote
(d) the material interests of the community
(d) right to participate in politics
Sol. (c)
Option (a) is wrong as the literal sum of all private Sol. (a)
wills is not General Wills (nowhere mentioned as Option (b) is wrong as “refraining from vote” is
an arithmetic truth). Option (b) is irrelevant being discussed. Similarly, (c) is also wrong. We
(elections etc.). Option (c) is best – collective good are left with (a) and (d), and the tone of the passage
distinct from the private wills. Option (d) is not clearly indicates a certain political duty of citizens.
explicitly mentioned. Best answer is (c). Hence, best answer is (a).

CSAT by Manjul Kumar Tiwari Sir


134 CSAT : 2017

Passage-5 Sol. (b)


In a free country, the man who reaches the position of Statement 1 is wrong due to the word “always” – a
leader is usually one of outstanding, character and ability. generalization. Statement 2 is correct as it is clearly
Moreover, it is usually possible to foresee that he will mentioned. Best answer is (b).
reach such a position, since early in life one can see his
qualities of character. But this is not always true in the Passage-7
case of a dictator; often he reaches his position of power There is more than a modicum of truth in the assertion
through chance, very often through the unhappy state of that “a working knowledge of ancient history is necessary
his country. to the intelligent interpretation of current events”. But
the sage who uttered these words of wisdom might well
65. The passage seems to suggest that have added something on the benefits of studying
(a) a leader foresees his future position particularly the famous battles of history for the lessons
(b) a leader is chosen only by a free country they contain for those of us who lead or aspire to
(c) a leader must see that his country is free from leadership. Such a study will reveal certain qualities and
despair attributes which enabled the winners to win—and
(d) despair in a country sometimes leads to certain deficiemcies which caused the losers to lose and
dictatorship the student will see that the same pattern recurs
Sol. (d) consistently, again and again, throughout the centuries.
One can easily eliminate option (a) as the leader’s 67. With reference to the above passage, the following
ability is foresee his ascent is not being explicitly assumptions have been made:
discussed. Option (b) is factually wrong. Option (c) 1. A study of the famous battles it history would
is not being discussed. Option (d) is clearly help us understand the modern warfare.
mentioned. Best answer is (d). 2. Studying the history is essential fin anyone
who aspires to be a leader.
Passage-6
The greatest blessing that technological progress has in Which of these assumptions is/are valid?
store for mankind is not, of course, an accumulation of (a) 1 only (b) 2 only
material possessions. The amount of these that can be (c) Both 1 and 2 (d) Neither 1 nor 2
effectively enjoyed by one individual in one lifetime is Sol. (d)
not great. But there is not the same narrow limit to the The Statement 1 goes wrong as it talks of
possibilities of the enjoyment of leisure. The gift of leisure “understand modern warfare” whereas the passage
may be abused by people who have had no experience of does not. Statement 2 is not correct. Hence best
making use of it. Yet the creative use of leisure by a answer is (d). Another very easy question.
minority in societies has been the mainspring of all
68. Suppose the average weight of 9 persons is 50 kg.
human progress beyond the primitive level.
The average weight of the first 5 persons is 45 kg,
66. With reference to the above passage, the following whereas the average weight of the last 5 persons is
assumptions have been made : 55 kg. Then the weight of the 5th person will be
1. People always see the leisure time as a gift (a) 45 kg (b) 47.5 kg
and use it for acquiring more material (c) 50 kg (d) 52.5 Kg
possessions. Sol. (c)
2. Use of leisure by some people to produce new Let weight of nth person be x(n). So, we have [ x(1) +
and original things has been the chief source x(2) .... + x(9) ] / 9 = 50. (average)
of human progress. Hence, x(1) + x(2) ..... + x(9) = 450 ………………..(1)
Which of these assumptions is/are valid? Now given that average weight of first 5 persons is
(a) 1 only (b) 2 only [ x(1) + x(2) ..... + x(5) ] / 5 = 45.
(c) Both 1 and 2 (d) Neither 1 nor 2 Hence x(1) + x(2) ..... + x(5) = 225 ………………..(2)

CSAT by Manjul Kumar Tiwari Sir


Previous Year Solved Papers 135
And given that [ x(5) + ..... + x(9) ] / 5 = 55. Hence, (b) The two watches show the identical time
x(5) + ...... + x(9) = 275 ………………..(3) again on completion of 90 days.
Now subtract both equations 2 and 3 from the 1st (c) The two watches show the identical time
equation. It will give x(5) = 50 kg. Answer is (c). again on completion of 120 days.
(d) None of the above statements correct.
69. In a group of six women, there are four tennis Sol. (d)
players, four postgraduates in Sociology, one
In 1 day (24 hours), first clock gains 2 min and the
postgraduate in Commerce and three bank
second loses 2 min. So net gap is 4 minutes per day.
employees. Vimala and Kamla are the bank
It is a 24 hour time format, so the two clocks (which
employees while Amala and Komala are
started with showing identical times) will next show
unemployed. Komala and Nirmala are among the
identical time when the net gap will become 1 full
tennis players. Amala, Kamla, Komala and
day (= 24 hours x 60 minutes = 1440 minutes). If 4
Nirmala are postgraduates in Sociology of whom
minutes are gained per day, then 1440 minutes will
two are bank employees. If Shyamala is a
be gained in 1440 / 4 = 360 days. Hence, answer is
postgraduate in Commerce, who among the
(d).
following is both a tennis player and a bank
employee? 72. In a city, 12% of households earn less than Rs.
(a) Amala (b) Komala 30,000 per year, 6% households earn more than
(c) Nirmala (d) Shyamala Rs. 2,00,000 per year, 22% households earn more
Sol. (c) than Rs. 1,00,000 per year and 990 households
It is given that Amala and Komala are unemployed earn between Rs. 30,000 and Rs. 1,00,000 per year.
so automatically options (a) and (b) are wrong. Also How many households earn between Rs. 1,00,000
it is given that Komala and Nirmala are amongst and Rs. 2,00,000 per year?
tennis players, so the only possible answer is (c) (a) 250 (b) 240
Nirmala. Best answer is (c). (c) 230 (d) 225
Sol. (b)
70. P = (40% of A) + (65% of B) and Q = (50% of A) +
Total households are 100%, then if 12% are below
(50% of B), where A is greater than B. In this context,
Rs.30000, X % are between 30,000 and 1,00,000, and
which of the following statements is correct?
22% are above 1,00,000. So, if we add them, we get :
(a) P is greater than Q.
12 + X + 22 = 100 => X = 66%.
(b) Q is greater than P.
(c) P is equal to Q. This 66% is given to us as 990 households. So the
(d) None of the above can be concluded with total population = (990 x 100) / 66 = 1500
certainty. households. Now 22% of 1500 are above
Rs.1,00,000. But 6% are above Rs.2,00,000. So
Sol. (d)
households between Rs.1,00,000 and Rs.2,00,000
The exact relation between A and B is required to
are 16% of 1500 ( = 22% - 6%), which is 240. That
find relation between P and Q. As the numerical
gives us the answer = 240 households.
values of A and B will diverge, the answers will
start to change from initial assumptions. Best 73. A clock strikes once at 1 o’clock, twice at 2 o’clock
answer is (d). and thrice at 3 o’clock, and so on. If it takes 12
seconds to strike at 5 o’clock, what is the time taken
71. A watch loses 2 minutes in every 24 while another
by it to strike at 10 o’clock?
watch gains 2 minutes, in 24 hours. At a particular
(a) 20 seconds (b) 24 seconds
instant, the two watches showed an identical time.
(c) 28 seconds (d) 30 seconds
Which of the following statements is correct if 24-
Sol. (b)
hour clock is
(a) The two watches show the identical time 74. Consider the given statement and the two
again on completion of 30 days. conclusions that follow:

CSAT by Manjul Kumar Tiwari Sir


136 CSAT : 2017

Statement: 77. There is a milk sample with 50% water in it. If


Morning walk is good for health. 1/3rd of this milk is added to equal amount of
pure milk, then water in the new mixture will fall
Conclusions:
down to
All healthy people go for morning walk. (a) 25% (b) 30%
Morning walk is essential for maintaining good (c) 35% (d) 40%
health. Sol. (a)
(a) 1 only (b) 2 only Let the weight of Mixture = 100 g. So amount of
(c) Both 1 and 2 (d) Neither 1 nor 2 milk in mixture = 50 g, and amount of water in
Sol. (d) mixture = 50 g.
“Morning walk is good for health”, it cannot mean We then took one-third of “this milk” i.e. 100 g
that “All healthy people go for morning walk”. mixture (= 100/3 g). That has (100/3) / 2 water,
Morning walk is not the only essential requirement. and (100/3) / 2 milk. We added 100/3 pure milk
Hence Conclusion 1 is wrong. Hence options (a) to it.
and (c) are wrong. Focus on Conclusion 2. The word So finally we have a mixture equal to – 100/3 milk
“essential” is what makes it wrong, as that is not + (100/6) water + (100/6) milk = 400 / 6 g mixture.
mentioned. Best answer is (d). So amount of Water in this final mixture= [ 100 / 6
] / [ 400 / 6 ] = 1 / 4 = 25 %. So final percent of water
75. There are thirteen 2-digit consecutive odd
in the mixture= 25%. Answer is (a).
numbers. If 39 is the mean of the first five such
numbers, then what is the mean of all the thirteen 78. There are 4 horizontal and 4 vertical lines, parallel
numbers? and equidistant to one another on a board. What
(a) 47 (b) 49 is the maximum number of rectangles and squares
(c) 51 (d) 45 that can be formed?
Sol. (a) (a) 16 (b) 24
Let the 13 consecutive 2-digit odd numbers be 2n+1, (c) 36 (d) 42
2n+3 so on. (because 2n+1 = an odd number). Sol. (c)
So, for the first 5 numbers, we get [ 2n+1 + 2n+3 .... + There are 4 horizontal lines and 4 vertical lines. So
2n+9 ] / 5 = 39. => 2n+1 ..... + 2n+9 = 195 which to make a rectangle, we have to select any 2 vertical
gives n = 17. lines and any 2 horizontal lines. That can be done
So the series is 35, 37, 39 onwards till 59. This series’ in 4C2 x 4C2 ways, that is a total of 36 ways. Answer
mean is 47 (Sum divided by 13). Answer is (a). is (c).

76. Six boys A, B, C, D, E and F play a game of cards. 79. A freight train left Delhi for Mumbai at an average
Each has a pack of 10 cards. F borrows 2 cards speed of 40 km/hr. Two hours later, an express
from A and gives away 5 to C who in turn gives 3 train left Delhi for Mumbai, following the freight
to B while B gives 6 to D who passes 1 to E. Then train on a parallel track at an average speed of 60
the number of cards possessed by D and E is equal km/hr. How far from Delhi would the express train
to the number of cards possessed by meet the freight train?
(a) A, B and C (b) B, C and F (a) 480 km (b) 260 km
(c) A, B and F (d) A, C and F (c) 240 km (d) 120 km
Sol. (b) Sol. (c)
The final tally of cards with each boy will be : A-8, The distance travelled by the Freight Train in the
B-7, C-12, D-15, E-11, F-7. So total sum of cards of D time the second train has just started, is 2 x 40 = 80
and E is 26. Which means F + B + C = 26 Cards. So km. Now the express train is moving with average
the correct answer is (b). speed 60 km/hr.

CSAT by Manjul Kumar Tiwari Sir


Previous Year Solved Papers 137
So the relative speed = 20 kmph. Now it has to cover (a) Randhir (b) Neha
80 km (effectively) with a relative speed of 20 kmph. (c) Sonal (d) Data are inadequate
That means 4 hours. Now the actual distance Sol. (d)
travelled by the express train in these 4 hours = 4 x As Neha has obtained more marks than Randhir
60 km = 240 km. Hence, answer is (c). (given) so Randhir cannot be the answer. Sonal has
80. In a test, Randhir obtained more marks than the obtained more marks than Shankar, but her relation
total marks obtained by Kunal and Debu. The total with Randhir is not given. Also no sufficient
marks obtained by Kunal and Shankar are more information is given about her relation with both
than those of Randhir. Sonal obtained more marks Kunal and Debu. So we can come to the conclusion
than Shankar. Neha obtained more marks than that Data are inadequate. Answer is (d).
Randhir. Who amongst them obtained highest
marks? 

CSAT by Manjul Kumar Tiwari Sir


138 CSAT : 2018

Previous Year
CSAT : 2018 Solved Papers

1. Consider the following three-dimensional figure: What is the number at ? in the above pattern?
(a) 17 (b) 19
(c) 21 (d) 23
Sol. (a)
1. 8+4-6=6
2. 10 + 6 - 5 = 11
3. 15 + 8 - 7 = 16
4. 13 + 8 - 4 = 17
The first number and the third number in each
column is first added, and then the second number
How many triangles does the above figure have? is subtracted from the total obtained. Thus, for the
(a) 18 (b) 20
first column, we have 8 + 4 = 12, and subtracting 6
(c) 22 (d) 24 from it, we get 12 - 6 = 6. This is the last number in
Sol. (b) the first column. Repeat this in all columns, and in
Five triangles are on top the end, we get 13 + 8 = 21, and 21 - 4 = 17.
Five triangles are in bottom
4. How many diagonals can be drawn by joining the
The middle layer is as shown with 10 triangles. vertices of an octagon?
So, total = 5 + 5 + 10 = 20 (a) 20 (b) 24
(c) 28 (d) 64
2. Consider the following sum:
Sol. (a)
• + 1• + 2• + •3 + •1 = 21•
In the above sum, • stands for
(a) 4 (b) 5
(c) 6 (d) 8
Sol. (d)
The given expression can be expanded as:
x + 10 + x + 20 + x + 10(x) + 3 + 10(x) + 1 = 210 + x
On solving, we get: x = 8.
Here we used the concept of face value and place
value - a number 2• can be 25 = 20 + 5 = 2x10 + 5.
3. Consider the following pattern of numbers :
Each point on the octagon can be connected to
(8 - 3) points.
So each point can form 5 diagonals.
So we get 40 in all.

CSAT by Manjul Kumar Tiwari Sir


Previous Year Solved Papers 139
However this includes similar pairs, i.e. 1-6, 6-1, 6. A train 200 metres long is moving at the rate of
2-4, 4-2 etc. 40 kmph. In how many seconds will it cross a
So we divide this by 2. man standing near the railway line?
(a) 12 (b) 15
So, answer is 20.
(c) 16 (d) 18
5. The figure drawn below gives the velocity graphs Sol. (d)
of two vehicles A and B. The straight line OKP
Length = 200m
represents the velocity of vehicle A at any instant,
whereas the horizontal straight line CKD Speed = 40 km/h
represents the velocity of vehicle B at any instant. So time in seconds 9 = d/t
In the figure, D is the point where perpendicular
from P meets the horizontal line CKD such that 200 m
Time  18sec
PD= 1/2 LD : 40 km/h

Directions for the following 4 (four) items :


Read the following four passage’s and answer the items
that follow. Your answers to these items should be based
on the passages only.
Passage-1
Global population was around 1.6 billion in 1990—
today it is around 7.2 billion and growing. Recent
estimates on population growth predict a global
population of 9.6 billion in 2050 and 10.9 billion in 2100.
What is the ratio between the distances covered by
Unlike Europe and North America, where only three to
vehicles A and B in the time interval OL?
four per cent of population is engaged in agriculture,
(a) 1 : 2 (b) 2 : 3
around 47 per cent of India’s population is dependent
(c) 3 : 4 (d) 1 : 1
upon agriculture. Even if India continues to do well in
Sol. (c)
the service sector and the manufacturing sector picks
Area of velocity-time graph gives diplacement.
up, it is expected that around 2030 when India overtakes
China as the world’s most populous country, nearly 42
per cent of India’s population will still be predominantly
dependent on agriculture.

7. Which of the following is the most logical and


rational inference that can be made from the above
passage?
(a) Prosperity of agriculture sector is of critical
importance to India.
(b) Indian economy greatly depends on its
agriculture.
(c) India should take strict measures to control
its rapid population growth.
(d) India’s farming communities should switch
over to other occupations to improve their
economic conditions.
Sol. (a)

CSAT by Manjul Kumar Tiwari Sir


140 CSAT : 2018

Passage-2 (d) In today’s culture, scientific views are given


Many pathogens that cause foodborne illnesses are more importance than spiritual views.
unknown. Food contamination can occur at any stage Sol. (b)
from farm to plate. Since most cases of food poisoning
Option (a) is clearly incorrect as it is not mentioned.
go unreported, the true extent of global foodborne
Option (d) is incorrect because nowhere in the
illnesses is unknown. Improvements in international
monitoring have led to greater public awareness, yet the passage is it written that scientific views are
rapid globalization of food production increases currently been given more importance than
consumers’ vulnerability by making food harder to cultural ones. The passage speaks of society
regulate and trace. “We have the world on our plates”, considering science and faith incompatible.
says an official of WHO. The passage states that the author “finds it odd
that even in today’s culture there seems to be an
8. Which of the following is the most logical corollary
to the above passage? assumption that scientific and cultural views are
(a) With more options for food come more risks. incompatible”. This infers that the author is in
(b) Food processing is the source of all foodborne disagreement with the said fact and so option (b)
illnesses. is the best answer.
(c) We should depend on locally produced food The passage does state that there are some
only. fundamental questions which science is currently
(d) Globalization of food production should be unable to give a reasonable answer to, but it
curtailed. doesn’t mean that science is not capable of
Sol. (a) answering them, it may just be a matter of time.
Option (b) and (c) are clearly incorrect as they are Passage-4
not mentioned anywhere. Though I have discarded much of past tradition and
Option (d) may be rejected as the passage, though custom, and am anxious that India should rid herself of
criticizing globalization of food production, does all shackles that bind and contain her and divide her
not imply that globalization should be curtailed. people, and suppress vast numbers of them, and prevent
Passage-3 the free development of the body and the spirit; though I
I am a scientist, privileged to be somebody who tries to seek all this, yet I do not wish to cut myself off from that
understand nature using the tools of science. But it is past completely. I am proud of that great inheritance that
also clear that there are some really important questions has been and is, ours and I am conscious that I too, like
that science cannot really answer, such as : Why is there all of us, am a link in that unbroken chain which goes
something instead of nothing? Why are we here? In those back to the dawn of history in the immemorial past of
domains, I have found that faith provides a better path to India.
answers. I find it oddly anachronistic that in today’s
culture there seems to be a widespread presumption that 10. The author wants India to rid herself of certain
scientific and spiritual views are incompatible. past bonds because
(a) he is not able to see the relevance of the past
9. Which of the following is the most logical and (b) there is not much to be proud of
rational inference that can be made from the above (c) he is not interested in the history of India
passage?
(d) they obstruct her physical and spiritual
(a) It is the faith and not science that can finally
growth
solve all the problems of mankind.
Sol. (d)
(b) Science and faith can be mutually
complementary if their proper domains are The use of words like “shackles” and “bind” and
understood. “suppress” clearly show the author’s view that
(c) There are some very fundamental questions certain past bonds obstruct “her physical and
which cannot be answered by either science spiritual growth” (people and India considered
or faith. synonymously). Option (d) is the best answer.

CSAT by Manjul Kumar Tiwari Sir


Previous Year Solved Papers 141
Options (b) and (c) are incorrect because the author 13. What was the approximate ratio of sheet steel and
has shown interest,appreciation and pride in scrap steel imports in the first three months of the
India’s past on a whole. year?
Option (a) is incorrect for the same reason. (a) 1 : 1 (b) 1.2 : 1
Directions for the following 3 (three) items: (c) 1.4 : 1 (d) 1.6 : 1
The following three items are based on the graph given Sol. (b)
below which shows imports of three different types of Amount of sheet steel imported = 40 + 37 + 36 =
steel over a period of six months of a year. Study the 113 thousand tons
graph and answer the three items that follow. Amount of scrap steel imported = 32 + 34 + 32 = 98
thousand tons
So the ratio will be 113 / 98 = 1.2 approximately
(the closest option)
Hence, answer is (b).

Directions for the following 3 (three) items: Rotated


positions of a single solid are shown below. The various
faces of the solid are marked with different symbols like
dots, cross and line. Answer the three items that follow
the given figures.
The figures in the brackets indicate the average cost per
ton over six months period.

11. By how much (measured in thousands of tons)


did the import of sheet steel exceed the import of
coil steel in the first three months of the year?
(a) 11 (b) 15
(c) 19 (d) 23
Sol. (c)
Amount of sheet steel imported = 217 thousand 14. What is the symbol on the face oppsite to that
tons containing a single dot?
This is the total of all months = 40 + 37 + 36 + 36 + (a) Four dots (b) Three dots
34 + 34 = 217 thousand tons (c) Two dots (d) Cross
Sol. (b)
So Cost = 217 × 256 × 1000 = $ 55,552,000
Hence, answer is (c).

12. What was the approximate total value (in $) of


sheet steel imported over the six months period?
(a) 45,555 (b) 50,555
(c) 55,550 (d) 65,750
Sol. (c)
Sheet steel imports = 40 + 37 + 36 thousand tons
= 113 TT One dot is opposite to three dots.
Coil steel imports = 30 + 31 + 33 thousand tons
15. What is the symbol on the face opposite to that
= 94 TT
containing two dots?
So the difference = 113 - 94 = 19 TT
(a) Single dot (b) Three dots
The question requires you to read the graph in a
fine manner. (c) Four dots (d) Line

CSAT by Manjul Kumar Tiwari Sir


142 CSAT : 2018

Sol. (c) is an integral part of the right to education. If the


education process lacks quality, children are being
denied their right. The Right of Children to Free and
Compulsory Education Act lays down that the
curriculum should provide for learning through
activities, exploration and discovery. This places an
obligation on us to change our perception of children as
passive receivers of knowledge, and to move beyond the
Two dots is opposite to four dots. convention of using textbooks as the basis of
examinations. The teaching-learning process must
16. What is the symbol on the face opposite to that become stress-free; and a massive programme for
containing cross? curricular reform should be initiated to provide for a
(a) Single dot (b) Two dots child-friendly learning system that is more relevant and
(c) Line (d) Four dots empowering. Teacher accountability systems and
Sol. (c) processes must ensure that children are learning. And
that their right to learn in a child- friendly environment
is not violated. Testing and redesigned to ensure that
these do not force children to struggle between school
and tuition centres, and bypass childhood.

17. According to the passage, which of the following


is/are of paramount importance under the Right
to Education?
(1) Sending of children to school by all parents
Cross is opposite to line. (2) Provision of adequate physical infrastructure
Directions for the following 4 (four) items: in schools
Read the following passage and answer the four items (3) Curricular reforms for developing child-
that follow. Your answers to these items should be based friendly learning system
on the passage only.
Passage Select the correct answer using the code given
It is no longer, enough for us to talk about providing for below.
universal access to education. Making available (a) 1 only (b) 1 and 2 only
schooling facilities is an essential prerequisite, but is (c) 3 only (d) None of the above
insufficient to ensure that all children attend school and Sol. (c)
participate in the learning process. The school may be The thrust of the passage is about fundamental
there, but children may not attend or they may drop out changes that go beyond mere infrastructure and
after a few months. Through school and social mapping, attendance. It says that the Right to Education
we must address the entire gamut of social, economic, focuses not on changing the mindset of parents,
cultural and indeed linguistic and pedagogic issues, but focuses on changing the quality of the
factors that prevent children from weaker sections and education that disadvantaged groups receive. It
disadvantaged groups, as also girls, from regularly focuses on intangible changes like curricular
attending and complementing elementary education. reforms, teacher’s accountability etc. rather than
The focus must be on the poorest and most vulnerable the actual physical infrastructure present at
since these groups are the most disempowered and at
schools. It focuses on teaching-learning process.
the greatest risk of violation or denial of their right to
Statement 1 is not correct. Statement 2 also is
education.
The right to education goes beyond free and compulsory incorrect. Only 3 fits in.
education to include quality education for all. Quality So best answer is (c) - 3 only.

CSAT by Manjul Kumar Tiwari Sir


Previous Year Solved Papers 143
18. With reference to the above passage, the following Sol. (c)
assumptions have been made : We are asked the “essential message” in the
(1) The Right to Education guarantees teachers’ paragraph.
accountability for the learning process of Option (a) is a fact, not the essential message.
children.
Option (b) is another fact, though a praiseworthy
(2) The Right to Education guarantees 100% one, but not the essential message.
enrolment of children in the schools.
Option (c) is correct, as that is what the passage is
(3) The Right to Education intends to take full trying to communicate - Quality Education.
advantage of demographic dividend.
Option (d) is not the main message.
Which of the above assumptions is/are valid?
(a) 1 only (b) 2 and 3 only 21. If LSJXVC is the code for MUMBAI, the code for
DELHI is
(c) 3 only (d) 1, 2 and 3
(a) CCIDD (b) CDKGH
Sol. (a) (c) CCJFG (d) CCIFE
19. According to the passage, which one of the Sol. (a)
following is critical in bringing quality in Code is a letter back for first letter, two back for
education? second letter, three back for the third letter, and so
(a) Ensuring regular attendance of children as on.
well as teachers in school So, correct code for DELHI is CCIDD.
(b) Giving pecuniary benefits to teachers to
22. If RAMON is written as 12345 and DINESH as
motivate them
675849, then HAMAM will be written as
(c) Understanding the socio-cultural background
(a) 92233 (b) 92323
of children
(c) 93322 (d) 93232
(d) Inculcating learning through activities and
Sol. (b)
discovery
Sol. (d) 23. If X is between -3 and -1, and Y is between -1 and 1,
then X2 - Y2 is in between which of the following?
Option (d) is the best answer as the passage says
(a) -9 and 1 (b) -9 and -1
that ensuring regular attendance is required, but
(c) 0 and 8 (d) 0 and 9
not sufficient in ensuring quality education.
However inculcating learning through activities Sol. (d)
and discoveries falls under the Act’s objectives to If X is between -3, -1
enhance the learning process and make it less Then X2 is between 1 to 9
dependent on stressful books and examinations.
If Y is between -1, 1
Options (a) and (b) could be eliminated easily. Then Y2 is between 0, 1
Option (c) is not the major argument.
X2 – Y2 is between 0, 9.
20. What is the essential message in this passage?
(a) The Right to Education now is a 24. X and Y are natural numbers other than 1, and Y
Fundamental Right. is greater than X. Which ofthe following represents
(b) The Right to Education enables the children the largest number?
of poor and weaker sections of the society to (a) XY (b) X/Y
attend schools. (c) Y/X (d) (X + Y)/XY
(c) The Right to Free and Compulsory Education Sol. (a)
should include quality education for all. y>x>1
(d) The Government as well as parents should x x x
 as 1
ensure that all children attend schools. y y y

CSAT by Manjul Kumar Tiwari Sir


144 CSAT : 2018

y y sub-humid ecosystems, even in the humid tropics, the


xy  as  y and xy > y impact could be most dramatic. Impoverishment of
x x
human-impacted terrestrial ecosystems may exhibit
xy x 1 1 1 itself in a variety of ways: accelerated erosion as in the
xy > as xy   as   2 mountain regions of the country, salinization of land as
xy y x y x
in the semi-arid and arid ‘green revolution’ areas of the
But xy > 2 country, e.g., Haryana and western Uttar Pradesh, and
site quality decline—a common phenomenon due to
Directions for the following 2 (two items): general decline in tree cover and monotonous
Read the following information and answer the two monoculture of rice/wheat across the Indian plains. A
items that follow: major consequence of deforestation is that it relates to
The plan of an office block for six officers A, B, C, D, E adverse alterations in the hydrology and related soil and
and F is a follows: Both B and CV occupy office to the nutrient losses. The consequences of deforestation
right of the corridor (a one enters the office block) and A invariably arise out of site degradation through erosive
occupies on the left of the corridor. E and F occupy office losses. Tropical Asia, Africa and South America have
on opposite sides of the corridor but their offices do not the highest levels of erosion. The already high rates for
face each other. The offices of C and D face each other. E the tropics are increasing at an alarming rate (e.g.,
does not have a corner office. F’s office is further down through the major river systems—Ganga and
the corridor than A’s, but on the same side. Brahmaputra, in the Indian context), due to deforestation
and ill-suited land management practices subsequent
25. If E sits in his office and faces the corridor, whose
to forest clearing. In the mountain context, the declining
office is to his left?
moisture retention of the mountain soils, drying up of
(a) A (b) B
the underground springs and smaller rivers in the
(c) C (d) D
Himalayan region could be attributed to drastic changes
Sol. (c)
in the forest cover. An indirect consequence is drastic
Final Plan
alteration in the upland-lowland interaction, mediated
through water. The current concern the tea planter of
Assam has is about the damage to tea plantations due
to frequent inundation along the flood-plains of
Brahmaputra, and the damage to tea plantation and the
consequent loss in tea productivity is due to rising level
of the river bottom because of siltation and the changing
course of the river system. The ultimate consequences of
26. Who is/are F’s immediate neighbour/ site desertification are soil degradation, alteration in
neighbours? available water and its quality, and the consequent
(a) A only (b) A and D decline in food, fodder and fuel-wood yields essential
(c) C only (d) B and C for the economic well-being of rural communities.
Sol. (a)
27. According to the passage, which of the following
Directions for the following 7 (seven) items: are the consequences of decline in forest cover?
Read the following four passages and answer the items (1) Loss of topsoil
that follow. Your answers to these items should be based (2) Loss of smaller rivers
on the passages only. (3) Adverse effect on agricultural production
Passage-1 (4) Declining of groundwater
‘Desertification’ is a term used to explain a process of Select the correct answer using the code given
decline in the biological productivity of an ecosystem, below.
leading to total loss of productivity. While this (a) 1, 2 and 3 only (b) 2, 3 and 4 only
phenomenon is often linked to the arid, semi-arid and (c) 1 and 4 only (d) 1, 2, 3 and 4

CSAT by Manjul Kumar Tiwari Sir


Previous Year Solved Papers 145
Sol. (d) (1) Desertification is a phenomenon in tropical
The passage talks about the human impact on areas only.
water bodies and general ecosystems, due to (2) Deforestation invariably leads to floods and
agricultural and other practices. desertification.
All the four statements are mentioned. Loss of Which of the above assumptions is/are valid?
topsoil is a great concern. Smaller rivers drying (a) 1 only (b) 2 only
out (in Himalayan region) is mentioned. Drying
(c) Both 1 and 2 (d) Neither 1 nor 2
up of underground springs and rivers leading to
damage to tea plantations, leading to poor Sol. (d)
agricultural production too is mentioned. Assumption 1 is incorrect as desertification also
Soil degradation, soil erosion and soil water occurs in sub-tropical and temperate regions.
retention capacity all indicate that the
Assumption 2 is also incorrect.
groundwater too is adversely affected.
Correct answer is (d) - 1, 2, 3 and 4. Passage-2

28. Which of the following is/are the correct A diversity of natural assets will be needed to cope with
inference/ inferences that can be made from the climate change and ensure productive agriculture,
passage? forestry, and fisheries. For example, crop varieties are
(1) Deforestation can cause changes in the course needed that perform well under drought, heat, and
of rivers. enhanced CO2. But the private-sector and farmer-led
(2) Salinization of land takes place due to human process of choosing crops favours homogeneity adapted
activities only. to past or current conditions, not varieties capable of
(3) Intense monoculture practice in plains is a producing consistently high yields in warmer, wetter, or
major reason for desertification in Tropical drier conditions. Accelerated breeding programmes are
Asia, Africa and South America. needed to conserve a wider pool of genetic resources of
Select the correct answer using the code given existing crops, breeds, and their wild relatives. Relatively
below. intact ecosystems, such as forested catchments,
(a) 1 only mangroves, wetlands, can buffer the impacts of climate
(b) 1 and 2 only change. Under a changing climate, these ecosystems are
(c) 2 and 3 only themselves at risk, and management approaches will
(d) None of the above is a correct inference need to be more proactive and adaptive. Connections
Sol. (a) between natural areas, such as migration corridors, may
Statement 1 is mentioned, hence a correct inference be needed to facilitate species movements to keep up with
can be drawn. the change in climate.

Statement 2 is incorrect because it says that the 30. With reference to the above passage, which of the
salinization of land can take place only by human following would assist us in coping with the
intervention, something that the passage does not climate change?
claim.
(1) Conservation of natural water sources
Statement 3 is incorrect as the passage only talks
(2) Conservation of wider gene pool
about intense monoculture practices in India and
does not specify its ill effects in Africa and South (3) Existing crop management practices
America. (4) Migration corridors
Hence, answer is (a) - 1 only. Select the correct answer using the code given
below.
29. With reference to ‘desertification’, as described in
the passage, the following assumptions have been (a) 1, 2 and 3 only (b) 1, 2 and 4 only
made : (c) 3 and 4 only (d) 1, 2, 3 and 4

CSAT by Manjul Kumar Tiwari Sir


146 CSAT : 2018

Sol. (b) Sol. (c)


Another passage on climate change related issues. “Fascination with glitter” and “shopping mall”
Statement 2 is clearly wrong, as it goes against the clearly states that the passage is trying to tell us to
question asked - “which of these will assist in reduce our consumerism. Our top challenge is
coping with climate change?” Existing crop people and their aspirations, and they are guided
management practices (Statement 3) are directly by consumerism - hence (c) is best.
against the idea of coping up with climate change. Option (a) is not chosen as it is not explicitly
mentioned.
Three options have 3 - (a), (c) and (d). All can be
Option (b) is wrong.
eliminated!
Option (d) is incorrect as “knowledge of biological
Answer is (b).
systems” is different from understanding the fact
You can quickly check if that is indeed correct - it that the Earth is “a biological system”.
is - (b) - 1, 2 and 4 only.
Passage-4
31. With reference to the above passage, the following Some people believe that leadership is a quality which
assumptions have been made: you have at birth or not at all. This theory is false, for the
(1) Diversification of livelihoods acts as a coping art of leadership can be acquired and can indeed be
strategy for climate change. taught. This discovery is made in time of war and the
(2) Adoption of mono-cropping practice leads to results achieved can surprise even the instructors. Faced
the extinction of plant varieties and their wild with the alternatives of going left or right, every soldier
relatives. soon grasps that a prompt decision either way is better
than an endless discussion. A firm choice of direction
Which of the above assumptions is/are valid?
has an even chance of being right while to do nothing
(a) 1 only (b) 2 only
will be almost certainly wrong.
(c) Both 1 and 2 (d) Neither 1 nor 2
33. The author of the passage holds the view that
Sol. (c)
(a) leadership can be taught through war
Both assumptions are valid. experience only
(b) leadership can be acquired as well as taught
Passage-3
(c) the results of training show that more people
Today, the top environmental challenge is a combination
acquire leadership than are expected
of people and their aspirations. If the aspirations are more
(d) despite rigorous instruction, very few leaders
like the frugal ones we had after the Second World War,
are produced
a lot more is possible than if we view the planet as a
Sol. (b)
giant shopping mall. We need to get beyond the
fascination with glitter and understand that the planet The passage says that the theory that people are
works as a biological system. born with leadership qualities (or not at all) is false
and that leadership can be naturally acquired or
32. Which of the following is the most crucial and can be taught. So the best possible answer is (b).
logical inference that can be made from the above Option (a) is wrong as that is not the only way.
passage?
Option (c) is wrong as any comparison of numbers
(a) The Earth can meet only the basic needs of
is not provided.
humans for food, clothing and shelter.
Option (d) is not mentioned anywhere.
(b) The only way to meet environmental
challenge is to limit human population. 34. A number consists of three digits of which the
(c) Reducing our consumerism is very much in middle one is zero and their sum is 4. If the number
our own interest. formed by interchanging the first and last digits is
(d) Knowledge of biological systems can only greater than the number itself by 198, then the
help us save this planet. difference between the first and last digits is

CSAT by Manjul Kumar Tiwari Sir


Previous Year Solved Papers 147
(a) 1 (b) 2 Sol . (c)
(c) 3 (d) 4
Sol. (b)

37. If Pen < Pencil, Pencil < Book and Book > Cap,
then which one of the following is always true?
(a) Pen > Cap (b) Pen < Book
(c) Pencil = Cap (d) Pencil > Cap
35. A solid cube of 3 cm side, painted on all its faces, is
Sol . (b)
cut up into small cubes of 1 cm side. How many of
Pen < Pencil
the small cubes will have exactly two painted
faces? Pencil < Book
(a) 12 (b) 8 So, Pen < Book.
(c) 6 (d) 4
38. A bookseller sold ‘a’ number of Geography
Sol. (a)
textbooks at the rate of Rs. x per book, ‘a + 2’
number of History textbooks at the rate of
Rs. (x + 2) per book and ‘a - 2’ number of
Mathematics textbooks at the rate of Rs. (x - 2) per
book. What is his total sale in Rs. ?
(a) 3x + 3a (b) 3ax + 8
(c) 9ax (d) x3a 3
Sol. (b)
Sale from Geography = Rs. ax
From History = ax + 2a + 2x + 4
From Maths = ax – 2a – 2x + 4
So, Total Sale = Rs. 3ax + 8.
4 cubes on each face, and 12 in total.
39. A bag contains 15 red balls and 20 black balls.
36. While writing all the numbers from 700 to 1000, Each ball is numbered either 1 or 2 or 3. 20% of the
how many numbers occur in which the digit at red balls are numbered 1 and 40% of them are
hundred’s place is greater than the digit at ten’s numbered 3. Similarly, among the black balls, 45%
place, and the digit at ten’s place is greater than are numbered 2 and 30% are numbered 3. A boy
the digit at unit’s place? picks a ball at random. He wins if the ball is red
(a) 61 (b) 64 and numbered 3 or if it is black and numbered 1 or
(c) 85 (d) 91 2. What are the chances of his winning?

CSAT by Manjul Kumar Tiwari Sir


148 CSAT : 2018

(a) 1/2 (b) 4/7 Sol. (c)


(c) 5/9 (d) 12 /13 Let the total marks to pass the x.
Sol. (b)
40
Probability of picking a random ball is x = 30 + 30 [Marks gain + Marks left to pass]
100
Red = 15/35, [ within the 15 red balls : Red with
number 1 = 20% of 15 = 3/15, Red with number 3  x = 150.
= 40% of 15 = 6/15 ]. 42. 19 boys turn out for playing hockey. Of these, 11
Black ’! 20/35, [ within the 20 black balls : Black are wearing hockey shirts and 14 are wearing
with number 2 = 45% of 20 = 9/20, Black with hockey pants. There are no boys without shirts
number 3 = 30% of 20 = 6/20 ]. and pants. What is the number of boys wearing
So, we have to find probability of red balls with full uniform?
number 3 or black balls with numbers 1 and 2 So (a) 3 (b) 5
probability is (c) 6 (d) 8
So probability is Sol. (c)

 15 6   20 5   20 9  4 Total number of boys for playing hockey = 19


       =
 35 15   35 20   35 20  7 14 are wearing hockey pants
Ò! 5 are not wearing hockey pants
40. Two persons, A and B are running on a circular
Ò! 5 are only wearing shirts
track. At the start, B is ahead of A and their
positions make an angle of 30° at the centre of the Now, 11 are wearing hockey shirts out of which 5
circle. When A reaches the point diametrically are only wearing shirts.
opposite to his starting point, he meets B. What is 6 people (= 11 - 5) are in full uniform, i.e. both
the ratio of speeds of A and B, if they are running hockey shirts and hockey pants.
with uniform speeds? Directions for the following 6 (six) items:
(a) 6 : 5 (b) 4 : 3
Read the information given below and answer t he six
(c) 6 : 1 (d) 4 : 2
items that follow.
Sol. (a)
A, B, C and D are students. They are studying in four
different cities, viz., P, Q, R and S (not necessarily in that
order). They are studying in Science college, Arts college,
Commerce college and Engineering college (not
necessarily in that order), which are situated in four
different States, viz., Gujarat, Rajasthan, Assam and
Kerala (not necessarily in that order). Further, it is given
that—
Ratio of angles covered by A & B = Ratio of their (i) D is studying in Assam
Speeds (ii) Arts college is located in city S which is in
180 6 Rajasthan
 Answer = =
150 5 (iii) A is studying in Commerce college
41. A student has to get 40% marks to pass in an (iv) B is studying in city Q
examination. Suppose he gets 30 marks and fails (v) Science college is located in Kerala
by 30 marks, then what are the maximum marks
in the examination? 43. A is studying in
(a) 100 (b) 120 (a) Rajasthan (b) Gujarat
(c) 150 (d) 300 (c) City Q (d) Kerala

CSAT by Manjul Kumar Tiwari Sir


Previous Year Solved Papers 149
Sol. (b) 47. Which one of the following statements is correct
regarding Engineering college?
Student City College State
(a) C is studying there.
A P/R Commerce Gujarat
(b) B is studying there.
B Q Science Kerala (c) It is located in Gujarat.
C S Arts Rajasthan (d) D is studying there.
D P/R Engineering Assam Sol. (d)
A is studying in Gujarat.
Student City College State
44. Science college is located in A P/R Commerce Gujarat
(a) city Q (b) city S B Q Science Kerala
(c) city R (d) city P C S Arts Rajasthan
Sol. (a) D P/R Engineering Assam
The final arrangement is:
Only option (d) is correct.
Student City College State
48. Which one of the following statements is correct?
A P/R Commerce Gujarat
(a) Engineering college is located in Assam.
B Q Science Kerala
(b) City Q is situated in Assam.
C S Arts Rajasthan (c) C is studying in Kerala.
D P/R Engineering Assam (d) B is studying in Gujarat.
Science college is located in city Q. Sol. (a)

45. C is studying in Student City College State


(a) Science college (b) Rajasthan A P/R Commerce Gujarat
(c) Gujarat (d) city Q B Q Science Kerala
Sol. (b) C S Arts Rajasthan
The final arrangement is: D P/R Engineering Assam
Student City College State Only option (a) is correct.
A P/R Commerce Gujarat
Directions for the following 8 (eight) items
B Q Science Kerala
Read the following eight passages and answer the items
C S Arts Rajasthan
that follow. Your answers to these items should be based
D P/R Engineering Assam
on the passages only.
C is studying in Rajasthan. Passage-1
All actions to address climate change ultimately involve
46. Which one of the following statements is correct?
costs. Funding is vital in order for countries like India to
(a) D is not studying in city S.
design and implement adaptation and mitigation plans
(b) A is studying in Science college.
and projects. The problem is more severe for developing
(c) A is studying in Kerala.
countries like India, which would be one of the hardest
(d) Engineering college is located in Gujarat.
hit by climate change, given its need to finance
Sol. (a)
development. Most countries do indeed treat climate
The final arrangement is:
change as real threat and are striving to address it in a
Student City College State more comprehensive and integrated manner with the
A P/R Commerce Gujarat limited resources at their disposal.
B Q Science Kerala 49. With reference to the above passage, the following
C S Arts Rajasthan assumptions have been made :
D P/R Engineering Assam (1) Climate change is not a challenge for
Only option (a) is correct. developed countries.

CSAT by Manjul Kumar Tiwari Sir


150 CSAT : 2018

(2) Climate change is a complex policy issue and Sol. (d)


also a development issue for many countries. The passage talks about the health hazards posed
(3) Ways and means of finance must be fount to by cooking with biomass and coal and reports that
enable developing countries to enhance their there are more than 10 lakh premature deaths each
adaptive capacity. year due to household pollution via cooking fuels.
Which of the above assumptions is/are valid? So option is (d).
(a) 1 and 2 only (b) 3 only
Option (a) can be ruled out - it is not given.
(c) 2 and 3 only (d) 1, 2 and 3
Option (b) is not explained in the passage.
Sol. (c)
Option (c) is irrelevant to the given content in the
The passage doesn't claim that climate change is a
passage.
challenge for developed countries. So option (a) is
Passage-3
incorrect.
Scientific knowledge has its dangers, but so has every
It states that climate change indeed is a complex
great thing. Over and beyond the dangers with which it
policy especially for developing countries are hit
threatens the present, it opens up as nothing else can,
the hardest by climate change as they need to
the vision of a possible happy world; a world without
continue their financial development.
poverty, without war, with little illness. Science,
The passage states that funding is required whatever unpleasant consequences it may have by the
especially for countries like India if they are to way, is in its very nature a liberator.
adapt and find a possible solution to increase their
adaptive capacity. So statements 2 and 3 are correct. 51. Which one of the following is the most important
Passage-2 implication of the passage?
(a) A happy world is a dream of science.
Cooking with biomass and coal in India is now
(b) Science only can build a happy world, but it
recognized to cause major health problems, with women
is also the only major threat.
and children in poor populations facing the greatest risk.
(c) A happy world is not possible without
There are more than 10 lakh premature deaths each year
science.
from household air pollution due to polluting cooking
(d) A happy world is not at all possible with or
fuels with another 1.5 lakh due to their contribution to
without science.
general outdoor air pollution in the country. Although
the fraction of the Indian population using clean cooking Sol. (a)
fuels, such as LPG, natural gas and electricity, is slowly Option (a) reflected in passage
rising, the number using polluting solid fuels as their The passage doesnot talk about other ways hence,
primary cooking fuel has remained static for nearly 30 due cant surely know the option (c).
years at about 70 crore.
Passage-4
50. Which of the following is the most crucial and The Arctic’s vast reserves of fossil fuel, fish and minerals
logical inference that can be made from the above are now accessible for a longer period in a year. But unlike
passage? Antarctica, which is protected from exploitation by the
(a) Rural people are giving up the use of polluting Antarctic Treaty framed during the Cold War and is not
solid fuels due to their increasing awareness subject to territorial claims by any country, there is no
of health hazards. legal regime protecting the Arctic from industrialization,
(b) Subsidizing the use of clean cooking fuels will especially at a time when the world craves for more and
solve the problem of India’s indoor air
more resources. The distinct possibility of ice-free summer
pollution.
has prompted countries with Arctic coastline to scramble
(c) India should increase its import of natural
for great chunks of the melting ocean.
gas and produce more electricity.
(d) Access to cooking gas can reduce premature 52. Which one of the following is the most important
deaths in poor households. implication of the passage?

CSAT by Manjul Kumar Tiwari Sir


Previous Year Solved Papers 151
(a) India can have territorial claims in the Arctic So option (c) is the best answer.
territory and free access to its resources.
Option (a) is wrong as that is a strong policy
(b) Melting of summer ice in the Arctic leads to
recommendation, not mentioned.
changes in the geopolitics.
(c) The Arctic region will solve the world’s future Option (b) is wrong as that is another strong
problem of resource crunch. recommendation, not mentioned!
(d) The Arctic region has more resources than Option (d) is not to be chosen as it is a mere fact,
Antarctica. not a crucial message conveyed.
Sol. (b)
Passage-6
Option (a) nowhere mentioned in the passage.
India’s educational system is modelled on the mass
Option (c) is incorrect as the passage does not talk
education system that developed in the 19th century in
about the resource hunger of the world getting
Europe and later spread around the world. The goal of
solved by Arctic resources.
the system is to condition children as ‘good’ citizens
Option (d) is wrong as comparison of quantum of and productive workers. This suited the industrial age
resources of Arctic and Antarctica is not given. that needed the constant supply of a compliant
The passage clearly says that there is no legal workforce with a narrow set of capabilities. Our
regime protecting the Arctic, and an ice-free educational institutes resemble factories with bells,
summer could be the catalyst for geo-political uniforms and batch-processing of learners, designed to
changes as countries try to scramble for parts of get learners to conform. But, from an economic point of
the arctic. view, the environment today is very different. It is a
complex, volatile and globally interconnected world.
Passage-5
Being a member of the WTO, India is bound by the 54. With reference to the above passage, the following
agreements that have been signed and ratified by its assumptions have been made:
members, including itself. According to Article 6 of the (1) India continues to be a developing country
Agriculture Agreement, providing minimum support essentially due to its faulty education system.
prices for agricultural products is considered distorting
(2) Today’s learners need to acquire new-age
and is subject to limits. The subsidy arising from ‘minimal
skill-sets.
supports’ cannot exceed 10 per cent of the value of
agricultural production for developing countries. PDS (3) A good number of Indians go to some
in India entails minimum support prices and public developed countries for education because
stockholding of food grains. It is possible that, in some the educational systems there are a perfect
years, the subsidy to producers will exceed 10 per cent of reflection of the societies in which they
the value of agricultural production. function.
Which of the above assumptions is/are valid?
53. What is the crucial message conveyed by the above
passage? (a) 1 and 3 only (b) 2 only
(a) India should revise its PDS. (c) 2 and 3 only (d) 1, 2 and 3
(b) India should not be a member of WTO.
Sol. (b)
(c) For India, food security collides with trade.
(d) India provides food security to its poor. This passage talks about the gross delinking which
our education system has, with today’s competitive
Sol. (c)
markets. It tells us that our educational institutions,
The passage talks about the problem India, as a
made by the Britishers, haven’t reformed with time
WTO member, is facing. On the one hand, it has to
and resemble factories whose only job is to produce
uphold the agreements it has signed as a member
conforming students.
of WTO, but on the other it also has to look out for
the food security of its underprivileged citizens, So the most relevant option is (b). Today’s learners
which often clashes with WTO’s norms. need to acquire new-age skills-sets.

CSAT by Manjul Kumar Tiwari Sir


152 CSAT : 2018

Statement 1 is incorrect as it does not explicitly 56. Which of the following is the most logical, rational
mention or even imply that our education system and crucial message given by the passage?
is a reason for our country still being a developing
(a) Preserving crop genetic diversity is an
nation.
insurance against the effects of climate
Statement 3 is not given anywhere. change.

Passage-7 (b) Despite great risks, monoculture is the only


The practice of dieting has become an epidemic; everyone way to ensure food security in the world.
is looking out for a way to attain that perfect body. We (c) More and more genetically modified crops
are all different with respect to our ethnicity, genetics, only can save the world from impending
family history, gender, age, physical and mental and shortages of food.
spiritual health status, lifestyles and preferences. Thereby
(d) Asia and North America will be worst
we also differ in what foods we tolerate or are sensitive
sufferers from climate change and the
to. So we really cannot reduce so many complexities into
consequent shortage of food.
one diet or diet book. This explains the failure of diets
across the world in curbing obesity. Unless the reasons Sol. (a)
for weight gain are well understood and addressed and This passage only talks about the risks posed by
unless habits are changed permanently, no diet is likely monoculture and climate change related risks. So
to succeed. option (b) and (c) are ruled out.

55. What is the most logical and rational inference Although the passage only mentions the negative
that can be made from the above passage? effects of climate change that could happen in
(a) Obesity has become an epidemic all over the North America and Asia, this does not mean that
world. they will be the worst sufferers of food shortage.
(b) A lot of people are obsessed with attaining a Hence we rule out (d).
perfect body. So only correct option left is (a).
(c) Obesity is essentially an incurable disease.
(d) There is no perfect diet or one solution for 57. A shopkeeper sells an article at Rs. 40 and gets
obesity. X% profit. However, when he sells it at Rs. 20, he
faces same percentage of loss. What is the original
Sol. (d)
cost of the article?
The main inference that can be drawn from the
passage is that every individual has a slightly (a) Rs. 10 (b) Rs. 20
different body style which leads to lots of (c) Rs. 30 (d) Rs. 40
complexities. A diet cannot take into consideration
Sol. (c)
these complexities; that is why most diets fail to
work out. The passage urges people to understand x
CP  CP = Rs. 40
the reasons for weight gain and changing habits. 100
So the correct answer is (d). x
CP  CP   Rs. 20
100
Passage-8 On solving CP = Rs. 30
Monoculture carries great risks. A single disease or pest
can wipe out swathes of the world’s food production, an 58. There are 24 equally spaced points lying on the
alarming prospect given that its growing and wealthier circumference of a circle. What is the maximum
population will eat 70% more by 2050. The risks are number of equilateral triangles that can be drawn
magnified by the changing climate. As the planet warms by taking sets of three points as the vertices?
and monsoon rains intensify, farmlands in Asia will (a) 4 (b) 6
flood. North America will suffer more intense droughts,
and crop diseases will spread to new latitudes. (c) 8 (d) 12

CSAT by Manjul Kumar Tiwari Sir


Previous Year Solved Papers 153
Sol. (c) 61. Consider the following graph:

Which one of the following statements is not correct


According to a property of circles, angle subtended
with reference to the graph given above?
at centre is double than that at the circumference.
(a) On 1st June, the actual progress of work was
So, we need 60° (for condition of equilateral less than expected.
triangles) (b) The actual rate of progress of work was the
So, we need 120° at centre. greatest during the month of August.
According to symmetry, we can get (c) The work was actually completed before the
24/[(360)/120] = 8 equilateral triangles. expected time.
(d) During the period from 1st April to 1st
59. Consider the sequence given below:
September, at no time was the actual progress
4/12/95, 1/1/96, 29/1/96, 26/2/96, ....
more than the expected progress.
What is the next term of the series?
Sol. (d)
(a) 24/3/96 (b) 25/3/96
(c) 26/3/96 (d) 27/3/96 Option (d) is incorrect hence it is our answer.
Sol. (b) There is a brief window of time from mid-August
We are adding 28 days to each date. to 1st September where actual progress was more
So adding 28 days to 26-02-1996 which is a leap than expected progress.
year, we get 25-03-1996. 62. For a sports meet, a winners’ stand comprising
60. Twelve equal squares are placed to fit in a rectangle three wooden blocks is in the following form:
of diagonal 5 cm. There are three rows containing
four squares each. No gaps are left between
adjacent squares. What is the area of each square?
(a) 5/7 sq cm (b) 7/5 sq cm
(c) 1 sq cm (d) 25/12 sq cm
Sol. (c)
There are six different colours available to choose
from and each of the three wooden blocks is to be
painted such that no two of them has the same
colour. In how many different ways can the
winners’ stand be painted?
(a) 120 (b) 81
(c) 66 (d) 36
Sol. (a)
By Pythagoras theorem, we have
(5)2 = (3x)2 + (4x)2
25 = 25x2
x=1
So area of squares are = x2 = 1.

CSAT by Manjul Kumar Tiwari Sir


154 CSAT : 2018

The Stand can be painted in 6 × 5 × 4 = 120 ways 65. Average hourly earnings per year (E) of the
in which no color will be clashing. workers in a firm are represented in figures A and
Direction for the following 2 (two) items: B as follows :
63. Looking at the graph, it can be inferred that from
1990 to 2010

(a) population growth rate has increased


(b) population growth rate has decreased
(c) growth rate of population has remained stable
(d) population growth rate shows no trend
Sol. (d)

64. With reference to the above graph, consider the


From the figures, it is observed that
following statements considering 1970 as base
(a) the values of E are different
year:
(b) ranges (i.e., the difference between the
(1) Population has stabilized after 35 years.
maximum and the minimum) of E are different
(2) Population growth rate has stabilized after
(c) slopes of the graphs are same
35 years.
(d) rates of increase of E are different
(3) Death rate has fallen by 10% in the first 10
Sol. (c)
years.
25  20
(4) Birthrate has stabilized after 35 years. Slope of first Graph = =5
2014  2013
Which of the above are the most logical and
25  20
rational statements that can be made from the Slope of second Graph = =5
2014  2013
above graph?
(a) 1 and 2 only (b) 1, 2 and 3 66. Consider the figure given below:
(c) 3 and 4 (d) 2 and 4
Sol. (d)
If we observe carefully from 1990 to 2010, we can
see that first there is a decrease in both birth rate
and death rate and then they are stabilized. So
option (d) is the best answer.
Population growth rate has been stabilized after
35 years as the difference between the birth rate
and the death rate has been some from
approximately 2000. Option (d) is best answer.

CSAT by Manjul Kumar Tiwari Sir


Previous Year Solved Papers 155
To fit the question mark, the correct answer is (a) 2000 (b) 2500
(c) 3000 (d) 3500
(a) (b) Sol. (a)
To avoid loss, SP  CP.
Minimum case SP = CP
So the selling price should be equal to cost price.
(c) (d)
Selling price of 2000 pieces = 2000 × 350
= Rs. 7 lakh
Sol. (a) Cost Price of 2000 pieces = Rs. 7 lakh
There is a pattern. So best answer is (a).

68. A lift has the capacity of 18 adults or 30 children.


How many children can board the lift with 12
adults?
(a) 6 (b) 10
(c) 12 (d) 15
67. Consider the following figures A and B :
Sol. (b)

18 adults = 30 children
30
1 adult = children
18
So with 12 adults 6 more adults can board
30
i.e., 12+  6   22 total people
18
So, out of 22 total people, 10 are children

69. A person bought a refrigerator worth Rs. 22,800


with 12.5% interest compounded yearly. At the end
of first year he paid Rs. 8,650 and at the end of
second year Rs. 9,125. How much will he have to
pay at the end of third year to clear the debt?
(a) Rs. 9,990 (b) Rs. 10,000
(c) Rs. 10,590 (d) Rs. 11,250
Sol. (d)
Total interets in first year is 12.5% of 22800
= Rs. 2850.
Debt at end of first year is Rs. 22800 + 2850
= Rs. 25650.
After paying 8650, new principle is Rs. 17000.
Now, 12.5% of that is 2125
The manufacturing cost and projected sales for a So, debt at end of second year = 19125.
product are shown in the above figures A and B
respectively. What is the minimum number of So, he is left with debt Rs. 10000 + 12.5% of 10,000
pieces that should be manufactured to avoid a loss? i.e. total Rs. 11,250

CSAT by Manjul Kumar Tiwari Sir


156 CSAT : 2018

70. Consider the following figures :

With reference to the above graphs, which of the


In the figures (I) to (VI) above, some parts are shown following are the most logical and rational
to change their positions in regular directions. inferences that can be made?
Following the same sequence, which of the figures (1) Over the last two and a half decades, the
given below will appear at (VII) stage? dependency ratio for country B has decreased.
(2) By the end of next two and a half decades, the
dependency ratio of country A will be much
less than that of country B.
(3) In the next two decades, the work-force relative
to its total population will increase in country
B as compared to country A.
Select the correct answer using the code given
Sol. (b) below.
(a) 1 and 2 only (b) 2 and 3 only
(c) 1 and 3 only (d) 1, 2 and 3
Sol. (c)

 moving anti-clockwise
moving clockwise

71. Consider the following graphs. The curves in the


72. Lakshmi, her brother, her daughter and her son
graphs indicate different age groups in the
are badminton players. A game of doubles is about
populations of two countries A and B over a period
to begin:
of few decades:
1. Lakshmi’s brother is directly across the net
from her daughter.
2. Her son is diagonally across the net from the
worst player’s sibling.
3. The best player and the worst player are on
the same side of the net.
Who is the best player?
(a) Her brother (b) Her daughter
(c) Her son (d) Lakshmi

CSAT by Manjul Kumar Tiwari Sir


Previous Year Solved Papers 157
Sol. (a)

73. The graph given below indicates the changes in


key policy rates made by the Central Bank several
times in a year:

74. With reference to the above table, which of the


following is/are the most logical and rational
inference! inferences that can be made?
(1) Higher per capita income is generally
associated with higher Tele-density.
(2) Higher GDP growth rate always ensures
higher per capita income.
(3) Higher GDP growth rate does not necessarily
ensure higher Tele density.
Select the correct answer using the code given
Which one of the following can be the most likely below.
reason for the Central Bank for such an action?
(a) 1 only (b) 2 and 3
(a) Encouraging foreign investment
(c) 1 and 3 (d) 3 only
(b) Increasing the liquidity
Sol. (c)
(c) Encouraging both public and private savings
(d) Anti-inflationary stance
Sol. (d)
Whenever the Central Bank resists policy rates, it
is a way to suck out liquidity from the system.
Normally this is due to the central bank
expectation of rising inflation. Hence a
continuously rising repo rate indicates an anti-
inflationary stance. Hence option d is correct.

Directions for the following 2 (two) items :


The following table gives the GDP growth rate and Tele-
density is data of different States of a country in a
particular year. Study the table and answer the two items
that follow.

CSAT by Manjul Kumar Tiwari Sir


158 CSAT : 2018

See the states 15 and 16. State 15 has a higher GDP (1) During the given beriod, the revenue from
growth rate but less per capita income. Direct Taxes as percentage of gross tax
So, statement 2 which says that higher GDP revenue has increased while that of Indirect
growth rate always ensure higher per capita Taxes decreased.
income, is wrong. (2) The trend in the revenue from Excise Duty
demonstrates that the growth of
Statement 1 says that higher per capita income is
manufacturing sector has been negative
associated with higher tele density.
during the given period.
75. With reference to the above table, the following Select the correct answer using the code given
assumptions have been made: below.
(1) Nowdays, prosperity of an already high (a) 1 only (b) 2 only
performing State cannot be sustained without (c) Both 1 and 2 (d) Neither 1 nor 2
making further large investments in its Sol. (a)
telecom infrastructure. (1) is correct (2) is incorrect from the graph.
(2) Nowadays, a very high Tele-density is the
77. If x - y = 8, then which of the following must be
most essential condition for promoting the
true?
business and economic growth in a State.
1. Both x and y must be positive for any value of
Which of the above assumptions is/are valid? x and y.
(a) 1 only (b) 2 only 2. If x is positive, y must be negative for any value
(c) Both 1 and 2 (d) Neither 1 nor 2 of x and y.
3. If x is negative, y must be positive for any value
Sol. (d) of x and y.
Statement 1 tries to connect higher GDP growth Select the correct answer using the code given
rate only with further large investments in telecom. below.
That is wrong. Hence statement 1 is ruled out. (a) 1 only (b) 2 only
Statement 2 seems to be statement 1 in a reworded (c) Both 1 and 2 (d) Neither 1 nor 2 nor 3
format. It is wrong. Hence answer is option (d). Sol. (d)
76. The following graph indicates the composition of x = 10 x = 5 x = -1
our tax revenue for a period of two decades: y =2 y = –3 y = ± 9
If x > 0
y can be both > 0 or < 0
If x < 0
y<0

Directions for the following 3(three) items:


Read the following two passages and answer the name
that follow. Your answers to these name should be based
on the passages only.

Passage-1
The quest for cheap and plentiful meat has resulted in
farms where more and more animals are squeezed into
smaller lots in cruel and shocking conditions. Such
With reference to the above graph, which of the practices have resulted in many of the world’s health
following is/are the most logical and rational pandemics such as the avian flu. Worldwide, livesotck
inference/ inferences that can be made? increasingly raised in cruel, cramped conditions, where

CSAT by Manjul Kumar Tiwari Sir


Previous Year Solved Papers 159
animals spend their short live under artificial light, 79. Which of the following is the most rational and
pumped full of antibiotics and growth hormones, until crucial message given by the passage?
the day they are slaughtered. Meat production is water (a) The conflict between man and wildlife cannot
intensive. 15000 litres of water is needed for every be resolved, no matter what efforts we make.
kilogram of meat compared with 3400 liters for rice, 3300
(b) Safe wildlife corridors between protected
litres for eggs and 256 for a kilogram of potatoes.
areas is an essential aspect of conservation
78. What is the most rational and crucial message efforts.
given by the passage?
(c) India needs to declare more protected areas
(a) Mass production of meat through industrial
and set up more tiger reserves.
farming is cheap and is suitable for providing
protein nutrition to poor countries. (d) India’s National Parks and Tiger Reserves
(b) Meat-producing industry violates the laws need to be professionally managed.
against cruelty to animals. Sol. (b)
(c) Mass production of meat through industrial
The passage focuses on the problems with
farming is undesirable and should be stopped
relocation of wild animals, especially animals like
immediately.
tigers as they tend to wander towards their original
(d) Environmental cost of meat production is
place of living.
unsustainable when it is produced through
industrial farming. This puts the tigers at risk of human hunting and
Sol. (d) also puts the native human population at risk. So
the solution offered here is the presence of wildlife
The passage focuses on mainly two aspects :
corridors in which wild animals can safely travel.
1. Cruelty to animals which are slaughtered in
So, option (b).
factory farms, and
2. High environmental cost of maintaining such 80. With reference to the above passage, the following
farms instead of, say, organic free roam poultry assumptions have been made:
or other methods. (1) The strategy of conversation of wildlife by
Option (a) is the opposite of what is being said. relocating them from one protected area to
Option (b) is correct but it is not the most crucial another is not often successful.
message being given. (2) India does not have suitable legislation to save
Option (c) is an extreme statement, and not a the tigers, and its conservation efforts have
recommendation being given. failed which forced the tigers to live outside
Option (c) is an extreme statement, and not a protected areas.
recommendation being given. Which of the above assumptions is/are valid?

Passage-2 (a) 1 only (b) 2 only


A male tiger vas removed from Pench Tiger Reserve and (c) Both 1 and 2 (d) Neither 1 nor 2
was relocated in Parma National Park. Later, this tiger
Sol. (a)
trekked toward his home 250 miles away. The trek of
this solitary tiger highlights a crisis. Many wildlife India does have suitable legislation that help in
reserves exist as islands of fragile habitat in a vast sea of saving the tiger population. Further, that is not the
humanity, yet tigers can range over a hundred miles, topic of discussion in this passage. Hence,
seeking prey, mates and territory. Nearly a third of statement 2 is ruled out.
India’s tigers live outside tiger reserves, a situation that However, due to the absence of wildlife corridors
is dangerous for both human and animal. Prey and tigers in the country, relocating of wild animals is often
can only disperse if there are recognized corridors of unsuccessful.
land between protected areas to allow unmolested
passage.


CSAT by Manjul Kumar Tiwari Sir


160 CSAT : 2019

Previous Year
CSAT : 2019 Solved Papers

1. If every alternative letter of the English alphabet (a) 1 (b) 2


from B onwards (including B) is written in lower (c) 3 (d) None
case (small letters) and the remaining letters are Sol. (a)
capitalized, then how is the first month of the In the given sequence 1, 5, 7, 3, 5, 7, 4, 3, 5, 7, there
second half of the year written? is only one such 5 that is not preceded immediately
(a) JuLY (b) jULy by 3, but immediately followed by 7. It is the one in
(c) jUly (d) jUlY bold – 1, 5, 7, 3, 5, 7, 4, 3, 5, 7.
Sol. (d)
4. A joint family consists of seven members A, B, C,
Rearranging Alphabets yield letters as –
D, E, F and G with three females. G is a widow and
AbCdEfGhIjKlMnOpQrStUvWxYz. So, July will
sister-in-law of D’s father F. B and D are siblings
be written jUlY.
and A is daughter of B. C is cousin of B. Who is E?
2. Sunita cuts a sheet of paper into three pieces. (1) Wife of F
Length of first piece is equal to the average of the (2) Grandmother of A
three single digit odd prime numbers. Length of (3) Aunt of C
the second piece is equal to that of the first plus Select the correct answer using the code given
one-third the length of the third. The third piece is below:
as long as the other two pieces together. The length (a) 1 and 2 only (b) 2 and 3 only
of the original sheet of paper is (c) 1 and 3 only (d) 1, 2 and 3
(a) 13 units (b) 15 units Sol. (d)
(c) 16 units (d) 30 units All are correct – E is the wife of F, grandmother of
Sol. (d) A, and aunt of C. The information given in question
The only three single digit odd prime numbers can be interpreted by the following diagram
are 3, 5 and 7. So the length of the first piece
= (3+ 5 + 7)/3 = 5. Now let the length of the second
piece = X units.
Then by the condition given in question we have
– Length of the third piece = X + 5. Length of the
second piece = X = 5 + (X + 5)/3.
Solve this to get X = 10. So, length of the first piece
= 5, second piece = 10 and third piece = 15. So Hence answer is (d).
length of the original sheet = 5 + 10 + 15 = 30 units.
Ans.(d) 5. Each face of a cube can be painted in black or white
colours. In how many different ways can the cube
3. In the sequence 1, 5, 7, 3, 5, 7, 4, 3, 5, 7, how many be painted?
such 5s are there which are not immediately (a) 9 (b) 10
preceded by 3 but are immediately followed by 7? (c) 11 (d) 12
CSAT by Manjul Kumar Tiwari Sir
Previous Year Solved Papers 161
Sol. (b) Sol. (c)
There are many cases possible. Basic assumption The rule of divisibility of 3 is that the sum of all
is that all faces are identical and not identified by the digits of the number should be divisible by 3.
different numbers. We have 4 + 2 + 5 + 2 + 7 + 4 + 6 + B = 30 + B, which
0 sides White : 1 way (all sides Black) is completely divisible by three. So B can take value
as 0, 3, 6, or 9. Hence 4 values are possible for B.
1 side White : 1 way (all other sides Black)
Ans.(c)
2 sides White : 2 ways (1 way with adjacent sides
Directions for the following 3 (three) items:
White, and 1 way with opposite sides White)
Read the following information and answer the three
3 sides White : 2 ways (1 way where three White
items that follow:
sides have same corner, and 1 way where opposite
sides are White and one center side is White) Six students A, B, C, D, E and F appeared in several tests.
Either C or F scores the highest. Whenever C scores the
4 sides White : 2 ways (same as 2 sides Black)
highest, then E scores the least. Whenever F scores the
5 sides White : 1 way (same as 1 side Black) highest, B scores the least.
6 sides White : 1 way (same as 0 sides Black)
In all the tests they got different marks; D scores higher
So total=1+1+2+2+2+1+1 = 10 ways. than A, but they are close competitors; A scores higher
than B; C scores higher than A.
6. How many triplets (x, y, z) satisfy the equation x +
y + z = 6, where x, y and z are natural numbers? 9. If F stands second in the ranking, then the position
(a) 4 (b) 5 of B is
(c) 9 (d) 10 (a) Third (b) Fourth
Sol. (d) (c) Fifth (d) Sixth
x, y and z are natural numbers. So, x, y and z will 10. If B scores the least, the rank of C will be
be greater than zero. It also means that minimum 1 (a) Second (b) Third
will be there at all places in (x, y, z). (c) Fourth (d) Second or third
So we can conclude that x’ + y’ + z’ = 6 – (1 + 1 + 1)
= 3, where x’, y’, and z’ can take value 0 too. 11. If E is ranked third, then which one of the
following is correct?
Now this is a problem of distributing three objects
(a) E gets more marks than C
at three places (i.e. with two partitions). This can
(b) C gets more marks than E
be done in 5!/(3!×2!) = 10 ways.
(c) A is ranked fourth
7. If $ means ‘divided by’; @ means ‘multiplied by’; # (d) D is ranked fifth
means ‘minus’, then the value of 10#5@1$5 is Solution for Q.9 to Q.11:
(a) 0 (b) 1 Let’s first number the conditions given in question. There
(c) 2 (d) 9 are eight conditions in all.
Sol. (d) i. Six students A, B, C, D, E and F appeared in
After changing the expression as per the several tests.
conditions given in the question, we get 10#5@1$5 ii. Either C or F scores the highest.
= 10 - 5 × 1 ÷ 5.
iii. Whenever C scores the highest, then E scores
Using BODMAS, we first get 1 / 5 = 1/5. Then we
the least.
get 5 x 1/5 = 1. Then we get 10 – 1 = 9. Hence, we
iv. Whenever F scores the highest, B scores the least.
have answer 9. Ans.(d)
v. In all the tests they got different marks;
8. An 8-digit number 4252746B leaves remainder 0
vi. D scores higher than A, but they are close
when divided by 3. How many values of B are
competitors;
possible?
(a) 2 (b) 3 vii. A scores higher than B;
(c) 4 (d) 6 viii. C scores higher than A

CSAT by Manjul Kumar Tiwari Sir


162 CSAT : 2019

Sol. (c) (c) Weight of Rohan is least


If F stands second, then by condition (ii), C will be (d) ‘Whose weight is least’ cannot be determined
first. If C is first (i.e., score highest) by condition Solution for Q. 12 to 13:
(iii), E scores the least. And using conditions v, vi, Sol. (d)
vii and viii, we got the following order - C – F – D – Let weight of Sohan = S, weight of Mohan = M and
A – B – E. Hence B stood fifth. weight of Rohan = R.
Sol. (d) Then by the condition given we have :
B scores last means C is not on the first position 2S < M, even double of S is less than M. So, S < M.
(condition iii). So on the first position there must
2S < R, even double of S is less than R. So S < R.
be F (condition ii). Now out of six positions first
So, we can conclude that S is less than M and R.
and last positions are fixed. F _ _ _ _ B.
But we do not know the order relation between M
The middle four positions will be filled by E, C, D
and R. Again, 2R > M, means either R = M or R > M
and A. In which C, D and A will come in following
or R < M. So we cannot conclude relation between
order (conditions v, vi, vii and viii) – F _ C _ D _ A
R and M.
_ B In this arrangement E can occupy four
2R > S is obvious as we have concluded R > S
positions. In every case C can be either occupy
above. So not give any fruitful information. Ans.(d)
second or third place. Ans.(d)
Sol. (b)
Sol. (b)
It is clear that weight of Sohan is least.
E is ranked third, it means it is not ranked last, it
means C is not first, it means F is first, it means B is Directions for the following 7 (seven) items:
last (using conditions given in question). So we Read the following five passages and answer the
have following arrangement - items that follow each passage. Your answers to
F_E__B these items should be based on the passages only.
Now by using conditions v to viiii we have final Passage — 1
order – India’s economic footprint, given its population, still
remains small compared to the US, the European Union
FCEDAB
or China. It has much to learn from other economies, yet
Sol. (b) must implement solutions that fit its unique
Directions for the following 2 (two) items: circumstances. India especially needs an effective long-
Read the following statements SI and S2 and answer the term regulatory system based on collaboration rather
two items that follow: than the ‘ current top-down approach. Regulations seek
S1: Twice the weight of Sohan is less than the weight of desirable outcomes yet are repeatedly used as political
Mohan or that of Rohan. tools to push one agenda or another. Often, regulations
S2: Twice the weight of Rohan is greater than the weight fail to consider impacts on jobs and economic growth —
of Mohan or that of Sohan. or less restrictive alternatives. Regulations may be used
to protect local markets at the expense of more widely
12. Which one of the following statements is correct? shared prosperity in the future. Additionally, regulations
(a) Weight of Mohan is greatest inevitably result in numerous unintended consequences.
(b) Weight of Sohan is greatest In today’s hyper competitive global economy,
(c) Weight of Rohan is greatest regulations need to be viewed as “weapons” that seek
(d) Whose weight is greatest’ cannot be cost-justified social and environmental benefits while
determined improving the economic well-being of most citizens.
13. Which one of the following statements is correct? 14. Which one of the following is the most logical,
(a) Weight of Mohan is least rational and crucial inference that can be derived
(b) Weight of Sohan is least from the above passage ?

CSAT by Manjul Kumar Tiwari Sir


Previous Year Solved Papers 163
(a) A better regulatory system will help India of a modified sugar (sialylated oligosaccharides). This
achieve the size of economy appropriate to its is not utilized by the baby for its own nutrition. However,
population. the bacteria constituting the infant’s microbiome thrive
(b) In a competitive global economy, India must on this sugar which serves as their food. Malnourished
use regulations strategically. mothers have low levels of this sugar in their milk.
(c) Regulations in India do not favour its Consequently, the microbiomes of their infants fail to
integration with today’s hyper competitive mature. That in turn, leads to malnourished babies.
global economy.
16. Which one of the following is the most logical,
(d) Job creation and economic growth should be rational and crucial inference that can be derived
dominant considerations in developing from the above passage ?
India’s regulatory system. (a) If malnourished condition in children is
Sol. (b) caused by gut bacteria, it cannot be treated.
Option (b) comes close to being the most logical (b) The guts of malnourished babies should be
inference. inoculated with mature microbiomes.
15. On the basis of the above passage, the following (c) Babies of malnourished mothers should be
assumptions have been made : fed with dairy milk fortified with sialylated
oligosaccharides instead of mother’s milk.
(1) In today’s global economy, regulations are not
(d) Research on benign effects of gut bacteria on
effectively used to protect local markets.
nutrition has policy implications.
(2) social and environmental concerns are
Sol. (d)
generally ignored by the governments across
the world while implementing the Option (a) is wrong. If the cause is found, treatment
regulations. is possible. Option (b) seems too far-fetched, and
is not mentioned anywhere (inoculation etc.)
Which of the above assumptions is/are valid ?
is not mentioned anywhere (inoculation etc.)
(a) 1 only (b) 2 only
Option (c) sounds good, if it is technically correct.
(c) Both 1 and 2 (d) Neither 1 nor 2
The problem with it is “…instead of mother’s
Sol. (d) milk”. Option (d) is correct. It is a logical, rational
The first assumption is invalid. and crucial inference regarding policy choices to
The second assumption is wrong as the last part be made regarding nutrition.
of the para tells us the opposite – governments
17. On the basis of the above passage, the following
focusing more on social and environmental
assumptions have been made :
concerns than economic ones (the balance is
(1) Processed probiotic foods are a solution to
missing). So 2 is wrong.
treat the children suffering from malnutrition
Hence 1 only (option (a)) seems correct. due to immature gut bacteria composition.
Passage — 2 (2) The babies of malnourished mothers generally
In a study, scientists compared the microbiomes of poorly tend to be’ malnourished. Which of the above
nourished and well nourished infants and young assumptions is/are valid?
children. Gut microbes were isolated from faecal Which of the above assumptions is/are valid?
samples of malnourished and healthy children. The (a) 1 only (b) 2 only
microbiome was “immature” and less diverse in (c) Both 1 and 2 (d) Neither 1 nor 2
malnourished children compared to the better developed Sol. (b)
“mature” microbiome found in healthy children of the Option 1 is incorrect.
same age. According to some studies, the chemical Assumption 2 is clearly valid as seen in the passage
composition of mother’s milk has shown the presence (the last two sentences).

CSAT by Manjul Kumar Tiwari Sir


164 CSAT : 2019

Passage — 3 Which of the above assumptions is/are valid ?


Temperatures have risen nearly five times as rapidly on (a) 1 and 2 only (b) 3 only
the Western Antarctic Peninsula than the global average (c) 2 and 3 only (d) 1, 2 and 3
over the past five decades. Researchers have now found Sol. (b)
that melting glaciers are causing a loss of species Assumption 1 cannot be made on the basis of this
diversity among benthos in the coastal waters off the para, as only one example is cited. So options (a)
Antarctic Peninsula, Impacting an entire seafloor and (d) are ruled out. Assumption 2 is clearly
ecosystem. They Believe increased levels of suspended wrong, as the reverse is cited. So option (c) is ruled
sediment in water to be the cause of the dwindling out. Hence, (b) is correct. Assumption 3 is
biodiversity in the coastal region. mentioned in the para.
18. On the basis of the above passage, the following Passage — 5
assumptions have been Made : Food varieties extinction is happening all over the world
(1) Regions of glaciers warm faster than other — and it is happening fast. For example, of the 7,000
regions due to global warming. apple varieties that were grown during the nineteenth
(2) Global warming can lead to seafloor
century, fewer than o hundred remain. In the Philippines,
sedimentation in some areas.
thousands of varieties of rice once thrived; now only up
(3) Melting glaciers can reduce marine
to a hundred are grown there. In China, 90 percent of
biodiversity in some areas.
the wheat varieties cultivated just a century ago have
Which of the above assumptions is/are valid?
disappeared. Farmers in the past painstakingly bred and
(a) 1 and 2 only (b) 3 only
developed crops well suited to the peculiarities of their
(c) 2 and 3 only (d) 1, 2 and 3
local climate and environment. In the recent past, our
Sol. (c)
heavy dependence on a few high yielding varieties and
Only two assumptions are valid. technology-driven production and distribution of food
Passage — 4 is causing the dwindling of diversity in food crops. If
A research team examined a long-term owl roost. Owls some mutating crop disease or future climate change
prey on small mammals and the excreted remains of decimates the few crop plants we have come to depend
those meals that accumulated over the time, provide us on to feed our growing population, we might desperately
an insight into the composition and structure of small need some of those varieties we have let go extinct.
mammals over the past ‘millennia. The research 20. On the basis of the above passage, the following
suggested that when the Earth went through a period of assumptions have been made :
rapid warming about 13,000 years ago, the small
(1) Humans have been the main reason for the
mammal community was stable and resilient. But, from
large scale extinction of plant species.
the last quarter of the nineteenth century, human-made
changes to the environment had caused an enormous (2) Consumption of food mainly from locally
drop in biomass and energy flow. This dramatic decline cultivated crops ensures crop diversity.
in energy flow means modern ecosystems are not (3) The present style of production and
adapting as easily as they did in the past. distribution of food will finally lead to the
19. On the basis of the above passage, the following problem of food scarcity in the near future.
assumptions have been made : (4) Our food security may depend on our ability
(1) Global warming is a frequently occurring to preserve the locally cultivated varieties of
natural phenomenon. crops.
(2) The impending global warming will not Which of the above assumptions are valid ?
adversely affect small mammals.
(a) 1 and 3 (b) 2 and 4
(3) Humans are responsible for the loss of the
Earth’s natural resilience. (c) 2 and 3 (d) 1 and 4

CSAT by Manjul Kumar Tiwari Sir


Previous Year Solved Papers 165
Sol. (b) 23. If the numerator and denominator of a proper
If you check assumption 3, it is not necessarily fraction are increased by the same positive
valid always, but only when a crisis (mutating crop quantity which is greater than zero, the resulting
disease or future climate change) happens. Hence, fraction is
options (a) and (c) assumption 1 talks are false (a) always less than the original fraction
about “extinction of plant species” but the passage (b) always greater than the original fraction
is about “food varieties”. Assumption 2 is valid. (c) always equal to the original fraction
So 2 and 4 are right, and answer is option (b). (d) such that nothing can be claimed definitely
Sol. (b)
21. What is X in the sequence 132, 129, 124, 117, 106,
We know the basic property of ratios that if
93, X ?
Numerator < Denominator (which is the case of
(a) 74 (b) 75
proper fraction), then the new fraction is greater
(c) 76 (d) 77
than the original one if same positive quantity is
Sol. (c) added to both numerator and denominator.
The series is formed reducing by consecutive prime That is, a/b < (a + x)/(b + x) .
no. 3, 5, 7, 11, 13 ... i.e., by prime numbers starting
with 3. So, the next prime will be 17. The value of X 24. What is X in the sequence 4, 196, 16, 144, 36, 100,
will be 93 – 17 = 76. Ans.(c) 64, X ?
(a) 48 (b) 64
22. A wall clock moves 10 minutes fast in every 24 (c) 125 (d) 256
hours. The clock was set right to show the correct Sol. (b)
time at 8:00 a.m. on Monday. When the clock shows
The series can be seen as 4 (=22), 196 (=142), 16
the time 6:00 p.m. on Wednesday, what is the
(= 42), 144 (=122), 36 (=62), 100 (=102), 64 (=82), X
correct time ?
(?2).
(a) 5:36 p.m. (b) 5:30 p.m.
22, 142, 42, 122, ...etc.
(c) 5:24 p.m. (d) 5:18 p.m.
Sol. (a) Now observe every alternate term, value of X will
be square of 8 i.e., 64. Ans.(b)
In 24 hours, the correct clock moves 24 × 60 = 1440
minutes, but the incorrect clock will move 1440 + 25. In a group of 15 people; 7 can read French, 8 can
10 = 1450 min. So now we have basic relation read English while 3 of them can read neither of
between the correct and incorrect clock that in the these two languages. The number of people who
time correct clock moves 1440 minutes, incorrect can read exactly one language is
clock moves 1450 minutes.. (a) 10 (b) 9
Now by the condition given in question the (c) 5 (d) 4
incorrect clock has moved 24 + 24 + 10 = 58 hours Sol. (b)
(i.e. 58×60 minutes). So, n(A  B) = 12, n(A) = 7, n(B) = 8.
If the incorrect clocks moves 1450 minutes, correct And we have n(A  B) = n(A) + n(B) - n(A )B) By
clock moves 1440 minutes. putting the values, we have 12 = 7 + 8 - n(A ) B).
If the incorrect clock moves 1 minute, correct clock So, n(A ) B) = 3.
moves 1440/1450 minutes.
So, total number of people who can read exactly
But in our case incorrect clock has moved 58 hrs × one language = (7 - 3) + (8 - 3) = 4 + 5 = 9.
60 min /hr = 3480 minutes.
So, if the incorrect clock moved 3480 minutes, the
correct clock will have moved (1440×3480)/1450
= 3456 minutes.
Converting 3456 min into hours we have 3456/60
= 573/5 or 57 hours 36 minutes. Ans.(a)

CSAT by Manjul Kumar Tiwari Sir


166 CSAT : 2019

26. A printer numbers the pages of a book starting Again if the weight of the Father is F, we have C1 +
with 1 and uses 3089 digits in all. How many pages C2 + F = 52 × 3 = 156 ... (2)
does the book have ? Now it is given that the weight of the father = F
(a) 1040 (b) 1048 = 60.
(c) 1049 (d) 1050
By putting this value in equation (2), we have
Sol. (c) C1 + C2 = 156 - 60 = 96.
Digits required to print one digit numbers (1 to 9)
Again by using equation (1) we have M = 150 - 96
= 9×1 = 9 = 54 kg.
Digits required to print two digit numbers (10 to
99) = 90 × 2 = 180 29. Suppose you have sufficient amount of rupee
Digits required to print three digit numbers (100 to currency in three denominations : Rs. 1, Rs. 10 and
999) = 900 × 3 = 2700. Rs. 50. In how many different ways can you pay a
So, upto 999 pages we have 2700 + 180 + 9 = 2889 bill of Rs. 107 ?
digits. (a) 16 (b) 17
Now from here onwards each number will use 4 (c) 18 (d) 19
digits and we are remaining with 3089 - 2889 = Sol. (c)
200 digits. Case I: We use TWO Rs. 50 notes - One possibility
So 200/4 = 50 more numbers are there. i.e., 999 + - 50(×2) + 1(×7) = 107
50 = 1049 pages in the book. Ans.(c)
Case II: We use ONE Rs. 50 note -
27. Consider the following sequence that follows some Six possibilities (by adding Rs.10 note from none
arrangement : to five) –
c_accaa_aa_bc_b Case III: We use no Rs. 50 note (Only Rs. 10 and 1
The letters that appear in the gaps are notes) - Ten possibilities –
(a) abba (b) cbbb Case IV: We use only Rs. 1 note - One possibility -
(c) bbbb (d) cccc 1 (×107) = 107.
Sol. (b) So, total 18 possibilities.
There are 15 characters in the sequence. So we
should find sequences of 5 characters each in it. 30. ‘A’ started from his house and walked 20 m
By applying the options given, we find that correct towards East, where his friend B joined him. They
sequence is ccacc aabaa bbcbb when we use option together walked 10 m in the same direction. Then
(b) – c-b-b-b. ‘A’ turned left while ‘B’ turned right and travelled
2 m and 8 m respectively. Again ‘B’ turned left to
28. A family has two children along with their parents. travel 4 m followed by 5 m to his right to reach his
The average of the weights of the children and their office. ‘A’ turned right and travelled 12 m to reach
mother is 50 kg. The average of the weights of the his office. What is the shortest distance between
children and their father is 52 kg. If the weight of the two offices?
the father is 60 kg, then what is the weight of the (a) 15 m (b) 17 m
mother? (c) 19 m (d) 20 m
(a) 48 kg (b) 50 kg Sol. (b)
(c) 52 kg (d) 54 kg
Sol. (d)
Average of weight of two children and their mother
(i.e. total 3 members) = 50. So, the sum of the weight
of two children and mother = C1 + C2 + M = 50 × 3
= 150 ... (1) (where C1 and C2 are the weights of
two children and M is the weight of the Mother)

CSAT by Manjul Kumar Tiwari Sir


Previous Year Solved Papers 167
Using Pythagoras Theorem in triangle, PQR It means first day of 2015 will be X+7 day. (As
there is one odd day in one ordinary year)
Shortest Distance = Hypotenuse = (150)2  8 2 )
After every 7 days, the same day appears. So, the
first day of 2015 will be same as 2009.
= (225  64)  289 = 17
Since both are ordinary years. We can use the
Sol. (b) calendar of 2009 in 2015. Ans.(d)
31. Consider two statements S1 and S2 followed by a 33. Number 136 is added to 5B7 and the sum obtained
question: is 7A3, where A and B are integers. It is given that
S1: p and q both are prime numbers. 7A3 is exactly divisible by 3. The only possible
S2: p + q is an odd integer. value of B is
Question: Is pq an odd integer? (a) 2 (b) 5
Which one of the following is correct ? (c) 7 (d) 8
(a) S1 alone is sufficient to answer the question
Sol. (d)
(b) S2 alone is sufficient to answer the question
(c) Both S1 and S2 taken together are not sufficient It is given 136 + 5B7 = 7A3.
to answer the question Add the unit’s numbers to get 6 + 7 = 13. So, carry
(d) Both S1 and S2 are necessary to answer the over 1.
question => 1 + 3 + B = 1A => 1 + 3 + B = 10 + A => B – A
Sol. (b) = 6.
1. odd × odd = odd; 2. odd×even = even; 3. even × Which means if A = 0, B = 6; if A = 1, B = 7, if A = 2,
even = even; 4. odd + odd = even; 5. even + even = B = 8 and if A = 3, B = 9.
even and 6. even + odd = odd.
But given 7A3 is completely divisible by 3. So, as
By above rules p×q = even, if both p and q are even per rules of divisibility, 7 + A + 3 = 10 + A should
or they form a pair of odd and even numbers and also be completely divisible by 3.
p×q = odd, if p and q both are odd.
So the possible values of A are 2, 5 and 8.
State 2 is sufficient to answer the question.
(12 / 15 / 18 divisible by 3) Out of these, only
32. Which year has the same calendar as that of 2009? 2 satisfies both the conditions so A = 2, so B = 8.
(a) 2018 (b) 2017 Ans.(d)
(c) 2016 (d) 2015
Directions for the following 7 (seven) items:
Sol. (d)
Read the following six passages and answer the items
To have the same calendar two things should be that follow each passage. Your answers to these items
matched. First – the first day of the year and second should be based on the passages only.
– the year type i.e., ordinary or leap. Let the first
day of year 2009 is X day. It means first day of Passage — 1
2010 will be X+1 day, as there is one odd day in What stands in the way of the widespread and careful
one ordinary year. adoption of ‘Genetic Modification (GM)’ technology is
It means first day of 2011 will be X+2 day. (As an `Intellectual Property Rights’ regime that seeks to
there is one odd day in one ordinary year) create private monopolies for such technologies. If GM
technology is largely corporate driven, it seeks to
It means first day of 2012 will be X+3 day. (As
maximize profits and that too in the short run. That is
there is one odd day in one ordinary year)
why corporations make major investments for herbicide-
It means first day of 2013 will be X+5 day. (As tolerant and pest-resistant crops. Such properties have
there are TWO odd days in one LEAP year) only a short window, as soon enough, pests and weeds
It means first day of 2014 will be X+6 day. (As will evolve to overcome such resistance. This suits the
there is one odd day in one ordinary year); and corporations. The National Farmers Commission

CSAT by Manjul Kumar Tiwari Sir


168 CSAT : 2019

pointed out that priority must be given in genetic Passage — 2


modification to the incorporation of genes that can help Most invasive species are neither terribly successful nor
impart resistance to drought, salinity and other stresses. very harmful. Britain’s invasive plants are not
34. Which one of the following is the most logical, widespread, not spreading especially quickly, and often
rational and crucial message conveyed by the above less of a nuisance than vigorous natives such as bracken.
passage? The arrival of new species almost always increases
biological diversity in a region; in many cases, a flood of
(a) Public research institutions should take the newcomers drives no native species to extinction. One
lead in GM technology and priorities the reason is that invaders tend to colonise disturbed
technology agenda. habitats like polluted lakes and post-industrial
(b) Developing countries should raise this issue wasteland, where little else lives. They are nature’s
in WTO and ensure the abolition of Intellectual opportunists.
Property Rights.
36. Which one of the following is the most logical and
(c) Private corporations should not be allowed rational inference that can be made from the above
to do agribusiness in India, particularly the passage?
seed business. (a) Invasive species should be used to
(d) Present Indian circumstances do not favour rehabilitate desert areas and wastelands of a
the cultivation of genetically modified crops. country.
(b) Laws against the introduction of foreign
Sol. (a)
plants are unnecessary.
Option (b) is not relevant to this passage. Option
(c) Sometimes, the campaigns against foreign
(c) is an extreme, and is not stated thus, in the
plants are pointless.
passage. Option (d) is wrong, as the passage
indicates otherwise (last part). Best answer is (d) Foreign plants should be used to increase the
option (a), as the passage criticizes the approach biodiversity of a country.
of private corporations, and indicates what India Sol. (c)
really needs. (which private firms won't do, and Except option (c): remaining options are logical
hence only public enterprises can or have to). derivations of aspects mentioned in the passage.
35. On the basis of the above passage, the following
Passage — 3
assumptions have been made:
Diarrhoeal deaths among Indian children are mostly
(1) The issue of effects of natural calamities on due to food and water contamination. Use of
agriculture is not given due consideration by contaminated groundwater and unsafe chemicals in
GM technology companies. agriculture, poor hygiene in storage and handling of
(2) In the long run, GM technology will not be food items to food cooked and distributed in unhygienic
able to solve agricultural problems arising due surroundings; there are myriad factors that need
to global warming. regulation and monitoring. People need to have
awareness of adulteration and ways of complaining to
Which of the above assumptions is/are valid? the relevant authorities. Surveillance of food-borne
(a) 1 only (b) 2 only diseases involves a number of government agencies and
entails good training of inspection staff. Considering
(c) Both 1 and 2 (d) Neither 1 nor 2
the proportion of the urban population that depends on
Sol. (a) street food for its daily meals, investing in training and
education of street vendors is of great significance.
Assumption 1 is correct, as that is clearly
mentioned in the passage. Assumption 2 is too 37. On the basis of the above passage, the following
broad, as it assumes that GM technology will assumptions have been made:
“never” be able to do it, which may be wrong. (1) Food safety is a complex issue that calls for a
Hence, option (a) is best. multipronged solution.

CSAT by Manjul Kumar Tiwari Sir


Previous Year Solved Papers 169
(2) Great investments need to be made in in the organised sector was growing at 2 percent per
developing the manpower for surveillance year. As the economy began to grow at 7 - 8 percent per
and training. year, the rate of growth of employment in the organised
(3) India needs to make sufficient legislation for sector actually declined to 1 percent per year.
governing food processing industry.
39. The above passage seems to imply that
Which of the above assumptions is/are valid? (1) most of modern economic growth is based on
(a) 1 and 2 only (b) 3 only technological progress.
(c) 1 and 3 only (d) 1, 2 and 3
(2) much of modern Indian economy does not
Sol. (a) nurture sufficient symbiotic relationship with
Only two assumptions are valid. labour-intensive, natural resource-based
livelihoods.
Passage - 4
(3) service sector in India is not very labour-
The interests of working and poor people have
intensive.
historically been neglected in the planning of our cities.
(4) literate rural population is not willing to enter
Our cities are increasingly intolerant,’, unsafe and
organised sector.
unlivable places for large numbers of citizens and yet
Which of the statements given above are correct?
we continue to plan via the old ways - the static
Development Plan — that draws exclusively from (a) 1 and 2 only (b) 3 and 4 only
technical expertise, distanced from people’s live (c) 1, 2 and 3 only (d) 1, 2, 3 and 4
experiences and needs, and actively excluding large Sol. (c)
number of people, places, activities and practices that Statement 4 is not mentioned anywhere. So,
are an integral part of the city. options (b) and (d) are ruled out. Statements 1, 2, 3
are reflected in the passage.
38. The passage seems to argue
(a) against the monopoly of builders and the Passage — 6
interests of elite groups. India has banking correspondents, who help bring
(b) against the need for global and smart cities. people in the hinterland into the banking fold. For them
(c) in favour of planning cities mainly for to succeed, banks cannot crimp on costs. They also cannot
working class and poor people. afford to ignore investing in financial education and
(d) in favour of participation of peoples’ groups literacy. Banking correspondents are way too small to
in city planning. be viewed as a systemic risk. Yet India’s banking
Sol. (d) regulator has restricted them to serving only one bank,
perhaps to prevent arbitrage. ‘Efforts at banking outreach
Options (a) and (b) are clearly wrong. Option (c) is
may succeed only if there are better incentives at work
not wrong, as that is one logic given right at the
for such last-mile workers and also those providers who
start. But option (d) is best as it covers option (c) as
ensure not just basic bank accounts but also products
well. If we start involving peoples’ groups in city
such as accident and life insurance and micro pension
planning, then automatically we will have the
schemes.
interests of working class and poor people taken
care of. In fact, much more than that is possible 40. Which one of the following is the most logical,
then. rational and crucial inference that can be derived
from the above passage?
Passage — 5 (a) Efforts to bring people in India’s hinterland
A vast majority of Indians are poor, with barely 10 into the banking system are not successful.
percent employed in the organised sector. We are being (b) For meaningful financial inclusion, India’s
convinced that vigorous economic growth is generating banking system needs more number of
substantial employment. But this is not so. When our banking correspondents and other such last-
economy was growing at 3 percent per year, employment mile workers.

CSAT by Manjul Kumar Tiwari Sir


170 CSAT : 2019

(c) Meaningful financial inclusion in India 42. A solid cube is painted yellow, blue and black such
requires that banking correspondents have that opposite faces are of same colour. The cube is
diverse skills then cut into 36 cubes of two different sizes such
(d) Better banking outreach would be impossible that 32 cubes are small and the other four cubes
unless each banking correspondent is are Big. None of the faces of the bigger cubes is
allowed to serve a number of banks painted blue. How many cubes have only one face
Sol. (c) painted?
(a) 4 (b) 6
Option (c) is reflected in the passage.
(c) 8 (d) 10
Options (a), (b) and (d) also wrong as that
conclusion has not been drawn anywhere in this Sol. (c)
passage. By the condition described in the question the cube
41. The number of times the digit 5 will appear while can be divided as shown below. Our desired cubes
writing the integers from 1 to 1000 is are marked with R. So a total of 4 and 4 = 8 cubes
(a) 269 (b) 271 will have only one face painted.
(c) 300 (d) 302
Yellow
Sol. (c)
From 1 to 1000, the numbers in which 5 can occur
could be of one digit, two digits or three digits.
Case I – If the number is of one digit – 5 will appear
only one time, i.e. in 5.
Case II – If the number is of two digits – then
(a) There is only one 5, this can happen in two
ways _5 and 5_. In the first case (_5) the blank
Place can be filled in 8 ways(as 0 and 5 cannot
appear at that place), while in the second case (5_)
the blank place can be filled in 9 ways (5 cannot
appear there). Total 9 + 8 = 17 ways.
(b) There are two 5s. In this case only ONE 43. A and B are two heavy steel blocks. If B is placed
possibility. on the top of A, the weight increases by 60%. How
much weight will reduce with respect to the total
Case III – If the number is of three digits – then
weight of A and B, if B is removed from the top of
(a) Only one 5. Then, 5 can occupy three positions.
A?
5 _ _ or _ 5 _ or _ _ 5. In the first case (5_ _), remaining
(a) 60% (b) 45.5%
two positions can be filled in 9 way each. So total
(c) 40% (d) 37.5%
9 × 9 = 81 possibilities. In the second case (_ 5 _)
first position can be filled in 8 ways and last Sol. (d)
position can be filled in 9 ways. So total 9 × 8 = 72 Let the weight of A be 100 kg.
possibilities. Same will be true for the third (_ _ 5) So, the combined weight of A + B will 160 kg. Out
case. So total 72 possibilities. of this 160 kg, 60 kg is reduced now.
(b) Only two 5. This can be done in three ways 55_ So problem now becomes “60 is what percent of
or 5_5 or _55. In first (55_) and second (5_5) case it 160 ?
can be filled in 9 ways each. While in the third
So, 60×100/160 = 37.5%. Ans. (d)
case (_55) it can be filled in 8 ways. So total 9 + 9 +
8 = 26 possibilities. 44. Mr ‘X’ has three children. The birthday of the first
(c) All three digits are 5. This can be done in only child falls on the 5th Monday of April, that of the
ONE way. i.e, 555. second one falls on the 5th Thursday of November.
So, total = 1 + 17 + 1 + 81 + 72 + 72 + 26 + 1 = 271. On which day is the birthday of his third child,
Ans.(b) which falls on 20th December?

CSAT by Manjul Kumar Tiwari Sir


Previous Year Solved Papers 171
(a) Monday (b) Thursday (a) 18 (b) 24
(c) Saturday (d) Sunday (c) 32 (d) 36
Sol. (b) Sol. (d)
November is a 30-day month. In a 30-day month, Select any two of the first set of 4 lines. That can be
all the seven days of week occur 4 times (7 × 4 = 28) done in 4C2 ways.
and two days of the week occur 5 times. These two Now select any two of the second set of 4 lines.
days will be the first two or the last two days of the That can also be done in 4C2 ways.
month. It means if a certain day is occurring 5 times
So the total number of ways of doing it = 4C2 x 4C2
in a month, it must be on those first two or last two
= 6 x 6 = 36 ways. Ans. (d)
days. Now the second child’s birthday is on the
47. In a school every student is assigned a unique
fifth Thursday, so it has to be in the end of
identification number. A student is a football player
November.
if and only if the identification number is divisible
That means either 29 November is Thursday or 30
by 4, whereas a student is a cricketer if and only if
November is Thursday.
the identification number is divisible by 6. If every
If 29 November is Thursday then, December 20 number from 1 to 100 is assigned to a student, then
will be a Thursday. how many of them play cricket as well as football?
If 30 November is Thursday then, December 20 (a) 4 (b) 8
will be Wednesday. This is not given in the options.
(c) 10 (d) 12
Hence, answer has to a Thursday. Ans.(b)
Sol. (b)
45. Consider the following Statements and The required number should be completely
Conclusions: divisible by both 4 and 6. That means it should be
Statements: divisible by LCM of 4 and 6, which is 12. Such 8
1. Some rats are cats. numbers are possible which are completely
2. Some cats are dogs. divisible by 12. They are 12, 24, 36, 48, 60, 72, 84
3. No dog is a cow. and 96. Ans.(b)
Conclusions:
I. No cow is a cat. 48. When a runner was crossing the 12 km mark, she
II. No dog is a rat. was informed that she had completed only 80% of
III. Some cats are rats. the race. How many kilometres was the runner
Which of the above conclusions is/are drawn from supposed to run in this event?
the statements? (a) 14 (b) 15
(a) I, II and III (b) Only I and II (c) 16 (d) 16.5
(c) Only III (d) Only II and III Sol. (b)
Sol. (c) 12 km is 8/10 = 4/5 of the whole race. So whole
The conclusion of ‘Some rats are cats’ is ‘Some race must be 15 km.
cats are rats’. So conclusion III is valid. No In this question 12 km is 80% of the total race.
conclusion can be drawn in terms of rats and dogs  12 km = 0.8 R  R = 12 / 0.8 = 15.
as statement I and II both are starting with ‘some’. So total race will be of 15 km. Ans.(b)
So, conclusion II is not valid. That eliminates all
options but (c). 49. Raju has Rs. 9000 with him and he wants to buy a
The only conclusion in terms of cat and cow will mobile handset; but he finds that he has only 75%
be ‘Some cats are not cows’. So conclusion I is not of the amount required to buy the handset.
valid. Ans.(c). Therefore, he borrows 2000 from a friend. Then
(a) Raju still does not have enough amount to
46. The number of parallelograms that can be formed buy the handset.
from a set of four parallel lines intersecting another (b) Raju has exactly the same amount as required
set of four parallel lines, is to buy the handset.

CSAT by Manjul Kumar Tiwari Sir


172 CSAT : 2019

(c) Raju has enough amount to buy the handset (1) At least 30 Indian participants are vegetarian.
and he will have 500 with him after buying (2) At least 10 Indian participants are non-
the handset. vegetarian.
(d) Raju has enough amount to buy the handset
Select the correct answer using the codes given
and he will have 1000 with him after buying
below:
the handset.
(a) 1 only (b) 2 only
Sol. (a)
(c) Both 1 and 2 (d) Neither 1 nor 2
In this question Rs. 9000 is 75% of the cost of mobile
Sol. (c)
phone. So total cost of mobile phone is Rs. 12000
( = 9000 / 0.75). Now, as he borrows Rs. 2000, he Let’s try to maximise the number of Indian-
will be still short of Rs.1000 to buy the phone. Vegetarians. Out of 70 Indians, all vegetarians (i.e,
Ans.(a) 60) can be Indians. So, at least 10 Indians will be
there who will be non-vegetarians. This number
50. In 2002, Meenu’s age was one-third of the age of can increase depending on the number of
Meera, whereas in 2010, Meenu’s age was half the Vegetarian-Indians.
age of Meera. What is Meenu’s year of birth?
Let’s try to minimise the number of Indian-
(a) 1992 (b) 1994
Vegetarians. For that we have maximise the
(c) 1996 (d) 1998
number of non-Indian-Vegetarians. Out of 30 Non-
Sol. (b)
Indians, at max all can be vegetarian. Still 30
Let Meenu’s age in 2002 be M and Meera’s age in
vegetarians remain which will fall under Indian
2002 be X.
category. So, at least 30 Indians will be there who
In 2002, Meenu = 1/3 of Meera.  M = X/3 will be vegetarians. Hence both statements are
In 2010, Meenu = 1/2 of Meera. So, M + 8 correct. Ans.(c)
= (X + 8) / 2 Directions for the following 8 (eight) items :
Solving these two, we get M = 8. So, in 2002, Meenu Read the following seven passages and answer the items
was 8 years old. So she was born in 1994. that follow each passage. Your answers to these items
should be based on the passages only.
51. Rakesh and Rajesh together bought 10 balls and
10 rackets. Rakesh spent 1300 and Rajesh spent Passage — 1
1500. If each racket costs three times a ball does,
Political theorists no doubt have to take history of
then what is the price of a racket?
injustice, for example, untouchability, seriously. The
(a) Rs. 70 (b) Rs. 90
concept of historical injustice takes note of a variety of
(c) Rs. 210 (d) Rs. 240
historical wrongs that continue into the present in some
Sol. (c)
form or the other and tend to resist repair. Two reasons
Let the cost of each ball is Rs. X. Then cost of each might account for resistance to repair. One, not only are
racket will be 3X. the roots of injustice buried deep in history, injustice itself
Cost of 10 balls = 10X, and cost of 10 rackets = 30X. constitutes economic structures of exploitation,
So total cost = 10X + 30X = 40X. ideologies of discrimination and modes of representation.
By the condition given in question, we have Two, the category of historical injustice generally extends
across a number of wrongs such as economic deprivation,
40X = 1300 + 1500 or 40X = 2800 or X = 70. Price of
social discrimination and lack of recognition. This
each racket = Rs. 210. Ans.(c)
category is complex, not only because of the overlap
52. In a conference, out of a total 100 participants, 70 between a number of wrongs, but because one or the other
are Indians. If 60 of the total participants are wrong, generally discrimination, tends to acquire partial
vegetarian, then which of the following statements autonomy from others. This is borne out by the history of
is/are correct? repair in India.

CSAT by Manjul Kumar Tiwari Sir


Previous Year Solved Papers 173
53. What is the main idea that we can infer from the will give them a firm ground to start from and their culture
passage ? will lead them as deep as philosophy and as high as art.
(a) Untouchability in India has not been taken Together it will impart meaning to human existence.
seriously by political theorists.
55. On the basis of the above passage, the following
(b) Historical injustice is inevitable in any society
assumptions have been made :
and is always beyond repair.
(1) A society without well educated people
(c) Social discrimination and deprivation have cannot be transformed into a modern society.
their roots in bad economies. (2) Without acquiring culture, a person’s
(d) It is difficult, if not impossible, to repair every education is not complete.
manifestation of historical injustice. Which of the above assumptions is/are valid ?
Sol. (d) (a) 1 only (b) 2 only
Option (b) is clearly ruled out due to “… beyond (c) Both 1 and 2 (d) Neither 1 nor 2.
repair”. The passage nowhere mentions that. Sol. (b)
Option (c) is not mentioned in this fashion. Option Only one assumption is valid.
(a) sounds correct, but is not our choice because
Passage — 3
the first statement can also mean that political
Soil, in which nearly all our food grows, is a living
theorists have taken it seriously. Option (d) is the
resource that takes years to form. Yet it can vanish in
best.
minutes. Each year 75 billion tonnes of fertile soil is lost
54. On the basis of the above passage, the following to erosion. That is alarming — and not just for food
assumptions have been made : producers. Soil can trap huge quantities of carbon dioxide
(1) Removal of economic discrimination leads to in the form of organic carbon and prevent it from escaping
removal of social discrimination. into the atmosphere.
(2) Democratic polity is the best way to repair 56. On the basis of the above passage, the following
historical wrongs. assumptions have been made :
Which of the above assumptions is/are valid ? (1) Large scale soil erosion is a major reason for
(a) 1 only (b) 2 only widespread food insecurity in the world.
(c) Both 1 and 2 (d) Neither 1 nor 2. (2) Soil erosion is mainly anthropogenic.
Sol. (d) (3) Sustainable management of soils helps in
combating climate change.
The passage clearly says that historical injustice
Which of the above assumptions is/are valid ?
spans across many categories like economic
(a) 1 and 2 only (b) 3 only
deprivation, social discrimination and lack of
(c) 2 and 3 only (d) 1, 2 and 3
recognition. So assumption 1 is not right.
Sol. (b)
Assumption 2 is not explicitly stated anywhere.
Hence both are invalid. Assumption 1 is wrong as the link between soil
erosion and food insecurity is not established in
Passage — 2 the passage. The word “alarming” does not mean
Education plays a great transformatory role in life, that food insecurity has already arrived.
particularly so in this rapidly changing and globalizing Assumption 2 is incorrect. It is not mentioned
world. Universities are the custodians of the intellectual anywhere in the passage that man is responsible
capital and promoters of culture and specialized for soil erosion. Assumption 3 is surely correct as
knowledge. Culture is an activity of thought, and seen in the last line.
receptiveness to beauty and human feelings. A merely Passage — 4
well informed man is only a bore on God’s earth. What
Inequality is visible, even statistically measurable in
we should aim at is producing men who possess both many instances, but the economic power that drives it is
culture and expert knowledge. Their expert knowledge invisible and not measurable... Like the force of gravity,

CSAT by Manjul Kumar Tiwari Sir


174 CSAT : 2019

power is the organising principle of inequality, be it of (c) Shifting ma’ r crops to new croplands will
income, or wealth, gender, race, religion and region. Its lead to a gr t increase in the total area under
effects are seen in a pervasive manner in all spheres, but cultivation and thus an increase in overall
the ways in which economic power pulls and tilts visible agricultural production.
economic variables remain invisibly obscure,
(d) Climate change is the most important factor
57. On the basis of the above passage, the following affecting the agricultural economy in the
assumptions have been made : future.
(1) Economic power is the only reason for the Sol. (a)
existence of inequality in a society.
Option a is true.
(2) Inequality of different kinds, income, wealth,
etc, reinforces power. Option (b) is not mentioned. Option (c) is wrong
as it is not discussed in this manner.
(3) Economic power can be analysed more
through its effects than by direct empirical Passage — 6
methods. A bat’s .wings may look like sheets of skin. But
Which of thy above assumptions is/are valid ? underneath, a bat has the same five fingers as an
(a) 1 and 2 only (b) 3 only orangutan or a human, as well as a wrist connected to
(c) 1 and 3 only (d) 1, 2 and 3 the same cluster of wrist bones connected to the same
long bones of the arm. What can be more curious than
Sol. (b)
that the hand of a man, formed for grasping, that of a
Assumption 1 is not correct due to the phrase “only
mole for digging, the leg of the horse, the-paddle of the
reason”. Assumption 2 is wrong as it is inverting
porpoise, and the wing of the bat, should all be
the relationship – it is power that reinforces and
constructed on the some pattern ?
drives inequality. Assumption 3 is correct as seen
in first statement. 59. Which one of the following is the most logical,
scientific and rational inference that can be made
Passage — 5
from the above passage ?
Climate change may actually benefit some plants by
lengthening growing seasons and increasing carbon (a) Different species having similar structure of
dioxide. Yet other effects of a warmer world, such as more hands is an example of biodiversity.
pests, droughts, and flooding, will be less benign. How (b) Limbs being used by different species for
will the world adapt? Researchers project that by 2050, different kinds of work is an example of
suitable croplands for four commodities — maize, biodiversity.
potatoes, rice and wheat — will shift, in some cases (c) Man and the aforementioned animals having
pushing farmers to plant new crops. Some farmlands similar structure of limbs is an example of
may benefit from warming, but others won’t. Climate coincidence in evolution.
alone does not dictate yields; political shifts, global
(d) Man and the aforementioned animals have a
“demand, and agricultural practices will influence how
shared evolutionary history.
farms fare in the future.
Sol. (d)
58. Which one of the following is the most logical and
rational inference that can be made from) the above Option (a) is not mentioned specifically. Option
passage ? (b) again talks about “biodiversity” but limbs
(a) Farmers who modernize their methods and being used for different purpose is not biodiversity!
diversify their fields will be. in an Option (c) sound correct, but is not, as “coincidence
advantageous position in future. in evolution” is not explicitly mentioned. Option
(b) Climate change will adversely affect the crop (d) is best as “a shared evolutionary history” makes
diversity. it a “very curious situation”.

CSAT by Manjul Kumar Tiwari Sir


Previous Year Solved Papers 175
Passage — 7 (3) The first (I) and the fifth (V) floors are painted
Around 56 million years ago, the Atlantic Ocean had red.
not fully opened and animals, perhaps including our To ensure that any two consecutive floors have
primate ancestors, could walk from Asia to North different colours
America through Europe and across Greenland. Earth (a) Only statement 2 is sufficient
was warmer than it is today, but as the Palaeocene epoch (b) Only statement 3 is sufficient
gave way to Eocene, it was about to get much warmer (c) Statement 1 is not sufficient, but statement 1
still —rapidly and radically. The cause was a massive along with statement 2 is sufficient
geologically sudden release of carbon. During this period (d) Statement 3 is not sufficient, but statement 3
called Palaeocene - Eocene Thermal Maximum or PETM, along with statement 2 is sufficient.
the carbon injected into the atmosphere was roughly Sol. (b)
the amount that Would be injected today if humans
Precondition - five floors are painted with 4
burned all the Earth’s reserves of coal, oil and natural
different colours.
gas. The PETM lasted for about 1,50,000 years, until the
excess carbon was reabsorbed. It brought on drought, Statement 1 – The middle three floors are painted
floods, insect plagues and a few extinctions. Life on Earth in different colours does not gurantee any two
survived — indeed, it prospered — but it was drastically consecutive floors of different colours as the colour
different. of “I and II” or “IV and V” can be same.
Statement 2 – Second (II) and the fourth (IV) floors
60. Based on the above passage, the following
are painted in different colours does not guarantee
assumptions have been made:
any two consecutive floors of different colours as
(1) Global warming has a bearing on the planet’s
the colour of I and II or II and III, III and IV or IV
biological evolution.
and V can be same.
(2) Separation of land masses causes the release
of huge quantities of carbon into the Statement 3 will ensure that any two consecutive
atmosphere. floors have different colours as there are only four
(3) Increased warming of Earth’s atmosphere can colours to be used. The remaining three floors will
change the composition of its flora and fauna. have different colours. Ans.(b)
(4) The present man-made global warming will
62. P, Q and R are three towns. The distance between
finally lead to conditions similar to those
P and Q is 60 km, whereas the distance between P
which happened 56 million years ago.
and R is 80 km. Q is in the West of P and R is in the
Which of the assumptions given above are valid? South of P. What is the distance between Q and R?
(a) 1 and 2 (b) 3 and 4 (a) 140 km (b) 130 km
(c) 1 and 3 (d) 2 and 4 (c) 10 km (d) 100 km
Sol. (c) Sol. (d)
Assumption 1 is clearly valid. Assumption 2 is
too far-fetched, and cannot be deduced.
Assumption 3 is valid as clearly stated.
Assumption 4 again is too far-fetched as the
amounts involved are too high.

61. A five-storeyed building with floors from I to V is


painted using four different colours and only one
colour is used to paint a floor. Using Pythagoras theorem –
Consider the following statements:
(1) The middle three floors are painted in
Distance between Q and R = (0800 2  0600 2 )
different colours.
= (6400  3600) = 10000  100
(2) The second (II) and the fourth (IV) floors are
painted in different colours. = 100 km. Ans.(d)

CSAT by Manjul Kumar Tiwari Sir


176 CSAT : 2019

63. All members of a club went to Mumbai and stayed (3) Cricket players do not play football, (False)
in a hotel. On the first day, 80% went for shopping
This cannot be concluded from the above data.
and 50% went for sightseeing, whereas 10% took
It is not mentioned if the cricketer plays
rest in the hotel. Which of the following
football or not.
conclusion(s) can be drawn from the above data?
(1) 40% members went for shopping as well as Hence, option (d) is correct.
sightseeing. 65. The ratio of a two-digit natural number to a number
(2) 20% members went for only shopping. formed by reversing its digits is 4: 7. The number
Select the correct answer using the code given of such pairs is
below: (a) 5 (b) 4
(a) 1 only (b) 2 only
(c) 3 (d) 2
(c) Both 1 and 2 (d) Neither 1 nor 2
Sol. (a) Sol. (b)
By the Venn Diagram, we get Let the ten’s digit of two-digit number ab be ‘a’
and unit’s digit be ‘b’. So the number will be of the
form 10a + b.
After reversing the digits the number will be
10b + a.
By the condition given in question we have
(10a+b)/(10b+a) = 4/7 which means a/b = 1/2
So by putting the values, total possible pairs (12,
Only statement (1) is correct. Ans.(a)
21), (24, 42), (36, 63), (48, 84).
64. In a school, 60% students play cricket. A student Thus, four pairs are possible. Ans.(b)
who does not play cricket, plays football. Every
66. In an examination, A has scored 20 marks more
football player has got a two-wheeler. Which of
than B. If B has scored 5% less marks than A, how
the following conclusions cannot be drawn from
much has B scored?
the above data?
(1) 60% of the students do not have two-wheelers. (a) 360 (b) 380
(2) No cricketer has a two-wheeler.
(c) 400 (d) 420
(3) Cricket players do not play football.
Select the correct answer using the code given Sol. (b)
below: Check options directly.
(a) 1 and 2 only (b) 2 and 3 only Start with (a). If B is 360, A will be 380. Now, 5% of
(c) 1 and 3 only (d) 1, 2 and 3
380 = 19. So B will become 380 – 19 = 361. Hence
Sol. (d) this option is wrong (B is 360, not 361).
1. 60% of the students do not have two-wheelers,
If B is 380, A will be 400. Now, 5% of 400 = 20. So B
(False)
will be 400 – 20 = 380. Hence (b) is correct. You do
This cannot be concluded fromthe above data.
not need to check options (c) and (d) at all.
It is not mentioned if the cricketer has got two
wheeler or not. Moreover, it is given that every 67. Seeta and Geeta go for a swim after a gap of every
fotobal player has got a two wheeler but it 2 days and every 3 days respectively. If on 1st
will be only 40% of the student count. January both of them went for a swim together,
(2) No cricketer has a two-wheeler, (False) when will they go together next?
This cannot be concluded from the above data.
(a) 7th January (b) 8th January
It is not mentioned if the cricketer has got two
wheeler or not. (c) 12th January (d) 13th January

CSAT by Manjul Kumar Tiwari Sir


Previous Year Solved Papers 177
Sol. (d) If y = 39, x can take various values 25, 26, 27.... (not
Seeta goes to swim every 3rd day and Geeta goes necessarily integers) In that case y – x will take
values like 14, 13, 12, 11 ..... 0, - 1 etc.
to swim every 4th day. They will go to swim
together every 12th day (LCM of 3 and 4 is 12). So, Similarly, if y = 38, values of y – x will be 13, 12, 11,
on the 12th day after January 1, they will go ..... 0, -1 etc. and so on.
together to swim i.e., on January 13. Ans.(d) So, we can say that value of y – x is less than or
68. X, Y and Z are three contestants in a race of 1000 equal to 15 in all cases.
m. Assume that all run with different uniform Hence, option (c) is correct.
speeds. X gives Y a start of 40 m and X gives Z a 70. Ena was born 4 years after her parents’ marriage.
start of 64 m. If Y and Z were to compete in a race of Her mother is three years younger than her father
1000 m, how many metres start will Y give to Z? and 24 years older than Ena, who is 13 years old.
(a) 20 (b) 25 At what age did Ena’s father get married?
(a) 22 years (b) 23 years
(c) 30 (d) 35
(c) 24 years (d) 25 years
Sol. (b)
Sol. (b)
In the 1000 m race, X gives Y 40 m start. That means Ena’s present age = 13
Y starts race 40 m ahead of X.
Ena’s mother’s present age = 13 + 24 = 37.
In the 1000 m race, X gives Z 64 m start. That means
Ena’s father’s present age = 37 + 3 = 40.
Z starts race 64 m ahead of X
Now at present Ena is 13 years old and her parents
got married 4 years before she was born.
So, Ena’s parents got married 13 + 4 = 17 years
earlier.
So at the time of marriage her father would be of 40
So, this means Y gives a lead of 24 m in a 1000 – 40 – 17 = 23 years old.
= 960 m race.
Hence, option (b) is correct.
So, by unitary method, lead given by Y in 1000 m 71. Rakesh had money to buy 8 mobile handsets of a
= 24/960×1000 = 25 m. Ans.(b)
specific company. But the retailer offered very good
69. If x is greater than or equal to 25 and y is less than discount on that particular handset. Rakesh could
or equal to 40, then which one of the following is buy 10 mobile handsets with the amount he had.
always correct? What was the discount the retailer offered?
(a) x is greater than y (a) 15% (b) 20%
(b) (y - x) is greater than 15 (c) 25% (d) 30%
(c) (y - x) is less than or equal to 15 Sol. (b)

(d) (x - y) is greater than or equal to 65 Discount = Marked Price – Selling Price. Let the
Marked Price of 10 articles = Rs. 100.
Sol. (c)
So, Marked Price of 8 articles = Rs. 80. (so each
Given that x is greater than or equal to 25. Also, y
was marked at Rs.10)
is less than or equal to 40.
Let’s try to find the various values of y – x. So, as per the question, Rakesh purchased 10
mobile phones for Rs. 80
If y = 40, x can take various values like 25, 26, 27,
....... (not necessarily integers – not given in So, the selling price of 10 mobile phones = Rs. 80
question) In that case y – x will take values like 15, (so each bought at Rs.8)
14, 13, 12,......0, - 1 etc. The discount is 2 rupees on 10 which is 20%.

CSAT by Manjul Kumar Tiwari Sir


178 CSAT : 2019

[Also, discount % = Discount × 100/ MP Sol. (d)


= 20 × 100/100 = 20%]. Option (a) is irrelevant to the passage given. Option
Hence, option (b) is correct. (b) is also irrelevant as “India” is not discussed in
the passage. Option (c) is not correct as this is not
72. The average marks of 100 students are given to be the theme of this passage. Best answer is option
40. It was found later that marks of one student (d).
were 53 which were misread as 83. The corrected
mean marks are Passage — 2
(a) 39 (b) 39.7 With the digital phenomenon restructuring most social
(c) 40 (d) 40 sectors, it is little surprise that global trade negotiations
are now eyeing the digital area in an attempt to pre-
Sol. (b)
emptively colonise it. Big Data is freely collected or mined
Given, average marks of 100 students = 40. from developing countries, and converted into digital
So, total marks of 100 students = 40 × 100 = 4000. intelligence in developed countries. This intelligence
But in this total the error is 83 – 53 = 30, more than begins to control different sectors and extract monopoly
the actual total. rents. A large foreign company providing cab service,
So, Actual total = 4000 – 30 = 3970. for instance, is not a work of cars and drivers, it is digital
intelligence about commuting, public transport, roads,
So the correct Mean = 3970/100 = 39.70. Ans.(b)
traffic, city events, personal behavioural characteristics
Directions for the following 8 (eight) items: of commuters and driver and so on.
Read the following six passages and answer the items
74. Which one of the following is the most logical and
that follow each passage. Your answers to these items
rational corollary to the above passage?
should be based on the passages only.
(a) Globalization is not in the interests of India
Passage — 1 as it undermines its socio-economic
Low-end IoT (Internet of Things) devices are cheap structures.
commodity items: addressing security would add to the (b) India should be careful to protect its digital
cost. This class of items is proliferating with new sovereignty in global trade talks.
applications; many home appliances, thermostats, (c) India should charge monopoly rents from
security and monitoring devices and personal multinational companies in exchange for Big
Data.
convenience devices are part of the IoT. So are fitness
(d) The loss of Big Data from India is proportional
trackers, certain medical implants and computer-like
to the degree/value of its foreign trade.
devices in automobiles. The IoT is expected to expand
Sol. (b)
exponentially — but new security challenges are
daunting. Options (a), (c) and (d) are wrong. The passage is
clearly talking about how digital data ownership
73. Which one of the following statements is the most is now driving trade and business advantages, and
logical and rational inference that can be made how Indian data in the hands of foreign firms is
from the above passage? not a good idea (for India). Clearly, (b) represents
(a) Development of enabling technologies in the best corollary (i.e. a guidance for the future).
India can be a big boost to its manufacturing
sector. Passage — 2
(b) India is not yet fully ready to adopt IoT in With the digital phenomenon restructuring most social
view of the imminent security challenges. sectors, it is little surprise that global trade negotiations
(c) Life becomes more comfortable with the are now eyeing the digital area in an attempt to pre-
development of cheap low-end IoT devices. emptively colonise it. Big Data is freely collected or mined
(d) As we go digital, we must recognise the huge from developing countries, and converted into digital
threat to Internet security from some IoT intelligence in developed countries. This intelligence
devices. begins to control different sectors and extract monopoly

CSAT by Manjul Kumar Tiwari Sir


Previous Year Solved Papers 179
rents. A large foreign company providing cab service, Sol. (a)
for instance, is not a work of cars and drivers, it is digital
Options (b), (c) and (d) are incorrect. Option (b) is
intelligence about commuting, public transport, roads,
wrongly interpreting the passage – the poor may
traffic, city events, personal behavioural characteristics
not have contributed to climate change, but they
of commuters and driver and so on.
may have a role in mitigating it now. Option (c) is
75. Which of the following is most definitively implied a big assumption being made, and may be wrong.
by the above passage? Option (d) is not mentioned anywhere thus. The
(a) 7 Big Data is the key resource in the digital use of “only” makes it wrong. Best choice is option
space. (a).
(b) Big economies create Big Data. 77. The above passage implies that
(c) Access to Big Data is the prerogative of (1) There is a potential problem of food insecurity
developed countries. in India.
(d) Access to and possession of Big Data is a
(2) India will have to strengthen its disaster
characteristic of developed countries.
management capabilities.
Sol. (a)
Which of the above assumptions is/are valid?
This is the central idea. Option (b) is a mere fact.
Option (c) is wrong – the developed countries have (a) 1 only (b) 2 only
no such right! Option (d) is not entirely correct. (c) Both 1 and 2 (d) Neither 1 nor 2

Passage — 3 Sol. (c)

The rural poor across the world, including India, have Both assumptions seem valid. Assumption 1 is
contributed little to human-induced climate change, yet indicated as small farms produce a lot of food, and
they are on the frontline in coping with its effects. Farmers are under threat. Assumption 2 is a clear message
can no longer rely on historical averages for rainfall and for India, from the tone of the passage.
temperature, and the more frequent and extreme weather
events, such as droughts and floods, can spell disaster. Passage – 4
And there are new threats, such as sea level rise and the A changing climate, and the eventual efforts of
impact of melting glaciers on water supply. How governments (however reluctant) to deal with it, could
significant are small farms? As many as two billion have a big impact on investors’ returns. Companies that
people worldwide depend on them for their food and produce or use large amounts of fossil fuels will face
livelihood. Small-holder farmers in India produce 41 higher taxes and regulatory burdens. Some energy
percent of the country’s food grains, and other food items producers may find it impossible to exploit their known
that contribute to local and national food security. reserves, and be left with “stranded assets” — deposits
of oil and coal that have to be left in the ground. Other
76. What is the most logical and rational Corollary to industries could be affected by the economic damage
the above passage? caused by more extreme weather — storms, floods, heat
(a) Supporting small farmers is an important part waves and droughts.
of any agenda regarding environmentally
sustainable development. 78. On the basis of the above passage, the following
assumptions have been made:
(b) Poor countries have little role to play in the
mitigation of global warming. Governments and companies need to be
(c) Due to a large number of farmer households, adequately prepared to face the climate change.
India will not have food security problem in Extreme weather events will reduce the economic
the foreseeable future. growth of governments and companies’in future.
(d) Only small-holder farmers in India can ensure Ignoring climate change is a huge risk for
food security. investors.

CSAT by Manjul Kumar Tiwari Sir


180 CSAT : 2019

Which of the above assumptions is/are valid? strongly worded, and is not mentioned. In fact, a
(a) 1 and 2 only (b) 3 only word of support for private schools is seen in the
(c) 1 and 3 only (d) 1, 2 and 3 end of the passage. So we rule out assumption 3.
Sol. (c) We are now left with options (a) and (b).
Assumption 1 is not correct. Hence, (b) is our
Assumption 3 is definitely correct. So option (a) is
answer.
ruled out. Assumption 2 is worded is a strange
way – “economic growth of governments” does Passage – 6
not sound right. So we reject it. So option (a) and A majority of the TB infected in India are poor and lack
(d) are gone. We are left with option (c) only. sufficient nutrition, suitable housing and have little
understanding of prevention. TB then devastates
Passage – 5 families, makes the poor poorer, particularly affects
Access to schooling for those coming of school age is women and children, and leads to ostracisation and loss
close to universal, but access to quality exhibits a sharp of employment. The truth is that even if TB does not kill
gradient with socio-economic status. Quotas for the them, hunger and poverty will. Another truth is that
weaker sections in private schools is a provision deep-seated stigma, lack of counselling, expensive
introduced by the Right of Children to Free and treatment and lack of adequate support from providers
Compulsory Education Act, 2009. The quotas have and family, coupled with torturous side-effects
imposed a debate on issues of social integration and demotivate patients to continue treatment — with
equity in education that private actors had escaped by disastrous health consequences.
and large, The idea of egalitarian education system with
80. Which one of the following is the most logical,
equality of opportunity as its primary goal appears to
rational and crucial message conveyed by the above
be outside the space that private school principals
passage?
inhabit. Therefore, the imposition of the quotas has led
(a) TB is not a curable disease in Indian
to resistance, sometimes justified.
circumstances.
79. With reference to the above passage, the following (b) Curing TB requires more than diagnosis and
assumptions have been made: medical treatment.
(1) Making equality of opportunity a reality is (c) Government’s surveillance mechanism is
the fundamental goal of the Indian education deficient; and poor people have no access to
system. treatment.
(2) The present Indian school system is unable (d) India will be free from diseases like TB only
when its poverty alleviation programmes are
to provide egalitarian education.
effectively and successfully implemented.
(3) Abolition of private schools and
establishment of more government schools is Sol. (b)
the only way to ensure egalitarian education. Option (a) is wrong. It is now here mentioned thus,
Which of the above assumptions is/are valid? with an air of fatality. Option (c) is wrong as poor
(a) 1 and 2 only (b) 2 only do have access to treatment though it seems to be
not working out well. The phrase “…have no
(c) 2 and 3 only (d) 3 only
access” is wrong. Option (d) goes tangentially into
Sol. (b)
a different direction. Best answer is (b).
Assumption 2 is surely correct, as per the passage.
So option (d) is ruled out. Assumption 3 is too


CSAT by Manjul Kumar Tiwari Sir


Previous Year Solved Papers 181

Previous Year
CSAT : 2020 Solved Papers

1. In the sum We can see that by subtracting 1 from each term,


  1  5      1  1   we will get a sequence of primes.
for which digit does the symbol  stand? 14 – 1 = 13; 18 – 1 = 17; 20 – 1 = 19; 24 – 1 = 23;
30 -1 = 29; 32 – 1 = 31.
(a) 2 (b) 3
(c) 4 (d) 5 So the next term will be 38 – 1 = 37 which is also a
prime number.
Sol. (b)
So the correct option is (c).
Checking options becomes easy for such type of
problems. 4. One page is torn from a booklet whose pages are
Option (a) will give 2 + 12 + 52 + 22 + 21 = 109 numbered in the usual manner starting from the
(which is not 122). So option (a) is wrong. first page as 1. The sum of the numbers on the
Option (b) gives us 3 + 13 + 53 + 33 + 31 = 133. remaining pages Is 195. The torn page contains
which of the following numbers?
So option (b) is correct.
(a) 5, 6 (b) 7, 8
2. If you have two straight sticks of length 7.5 feet (c) 9, 10 (d) 11, 12
and 3.25 feet, what is the minimum length can you Sol. (b)
measure?
We know that the sum of n consecutive natural
(a) 0.05 foot (b) 0.25 foot
numbers = n(n+1)/2.
(c) 1 foot (d) 3.25 feet
Natural numbers are integers starting with 1.
Sol. (b)
Now according the information given in question
When we have to measure the “minimum length”,
our value of n(n+1)/2 should be greater than 195.
it is the length common to both the given sticks. So
it will be the HCF. Because one page is torn, so the sum is less than
Correct answer is HCF of 7.5 and 3.25, which is what it should be.
0.25. If we take n = 19 we will get n(n+1)/2 = 19 × 20/2
= 190 which is less than 195.
3. A simple mathematical operation in each number
So taking next number let’s assume n = 20.
of the sequence 14, 18, 20, 24, 30, 32, ..... results in a
sequence with respect to prime numbers. Which In this case :
one of the following is the next number in the n(n+1)/2 = 20×21/2 = 210. But two consecutive
sequence? numbers were removed, so 210 – 195 = 15. It means
(a) 34 (b) 36 that the sum of the two consecutive torn pages
(c) 38 (d) 40 should be 15. Only option (b) matches.
Sol. (c)
5. Consider the following arrangement that has some
The given sequence is: 14, 18, 20, 24, 30, 32, …. We
missing letters:
are told that by applying an operation, we get
prime numbers from this series. abab_b_bcb_dcdcded_d

CSAT by Manjul Kumar Tiwari Sir


182 CSAT : 2020

The missing letters which complete the 7. Two statements S1 and S2 are given below followed
arrangement are by a Question:
(a) a, b, c, d (b) a, b, d, e S1: There are not more than two figures on any
(c) a, c, c, e (d) b, c, d, e page of a 51-page book.
Sol. (c)
S2: There is at least one figure on every page.
Observing the sequence and by putting various Question: Are there more than 100 figures in that
options you can find that the correct sequence is book?
ababa bcbcb cdcdc deded. You have to find a
Which one of the following is correct in respect of
repeated pattern. Only (c) gives that pattern.
the above Statements and the Question?
By putting (a), we would get – ababa bbbcb cdcdc
(a) Both S1 and S2 are sufficient to answer the
deddd – there is no pattern in it
Question, but neither S1 alone nor S2 alone is
By putting (b), we would get – ababa bbbcb ddcdc sufficient to answer the Question.
deded – there is no pattern in it (b) S1 alone is sufficient to answer the Question.
By putting (d), we would get – ababb bcbcb ddcdc (c) S1 and S2 together are not sufficient to answer
deded – there is no pattern in it the Question.

6. Let A3BC and DE2F be four-digit numbers where (d) S2 alone is sufficient to answer the Question.
each letter represents a different digit greater than Sol. (c)
3. If the sum of the numbers is 15902, then what is Using S1 alone, number of figures on any page
the difference between the values of A and D? could be 0, 1 or 2. So minimum 0 (if every page will
(a) 1 (b) 2 contain 0 figures) and maximum 102 figures (if
(c) 3 (d) 4 every page will contain 2 figures) are possible. So
Sol. (c) S1 is not sufficient to answer the question
uniquely.
A 3 B C
Using S2 alone, number of figures on any single
+ D E 2 F
page could 1, 2, 3, … up to infinity. So it is also not
1 5 9 0 2 sufficient to answer the question alone.
Each letter represents different digits. Even combining both we can have either 1 or 2
Unknown digits are 6 (A, B, C, D, E, F). Each digit figures on any page. Which gives us minimum 51
is greater than 3 (possible digits are 4, 5, 6, 7, 8, 9). (if every page will contain 1 figure) and maximum
So each of these digits (4-9) will be used. We have 102 figure (if every page will contain 2 figures). So
to find A – D (or D – A). it can’t be answered uniquely even by combining
S1 and S2.
Here, C + F is ending with 2. So, C and take two
values 8 or 7. If C is 8 then F is 4 and if C is 7 then 8. Consider the following data:
F is 5. In any case B + 2 + 1 (carry) is ending with 0
so B = 7. It means C can’t take value 7, so C = 8 and
F = 4.

Now F + 3 + 1 (carry) is ending with 9 so E = 5.


Hence there is no carry forward in the next term
i.e. A + D = 15 and remaining digits are 9 and 6.
Thus 9+ 6 = 6 + 9 = 15. What is the value of x in the above table?
So the difference between A and D is 3. So correct (a) 7.8 (b) 7.6
option is (c). (c) 7.4 (d) 7.2

CSAT by Manjul Kumar Tiwari Sir


Previous Year Solved Papers 183
Sol. (a) (a) 15th July is a Sunday if the year is a leap year.
Let the number of boys = b and number of girls = g. (b) 15th July is a Sunday if the year is not a leap
So by the condition given in question we have year.
9g + 8b = 8.8 (b + g). That gives us ratio b:g = 1:4. It (c) 12th July is a Sunday if the year is a leap year.
means that if there are p boys, then number of girls (d) 12th July is not a Sunday if the year is a leap
will be 4p, where p is any positive integer. year.
Sol. (c)
Again by the condition given in question
Let’s calculate what day of week will be on July
8  4p  7 p 12 (Considering Jan 12 is Sunday and it is a
4p  p = X non-leap year).
Total number of odd days between Jan 12 and July
8 × 4p + 7×p = X (4p + p)
12 = 19 (remaining day of Jan) + 28 (Feb) +
 32p + 7p = 5p × X  39p = 5pX  39 = 5X or X
31 (March) + 30 (April) + 31 (May) + 30 (June) +
= 39/5 or X = 7.8
12 (July) = 181.
9. A family of two generations consisting of six Now, 181 when divided by 7 gives remainder of 6.
members P, Q, R, S, T and U has three males and So final odd days = 6.
three females. There are two married couples and
So If Jan 12 is Sunday July 12 will be Saturday (in
two unmarried siblings. U is P’s daughter and Q
a non-leap year).
is R’s mother-in-law. T is an unmarried male and
S is a male. Which one of the following is correct? In a leap year July 12 will be Sunday as one extra
(a) R is U’s husband. (b) R is S’s wife. day will be added of Feb 29.
(c) S is unmarried. (d) None of the above
11. A man walks down the backside of his house
Sol. (b) straight 25 meters, then turns to the right and walks
Given – 1. Two generations; 2. Six members P, Q, 50 meters again; then he turns towards left and
R, S, T and U. Three males, and three females. Two again walks 25 meters. If his house faces to the
married couples (which means out of these four East, what is his direction from the starting point?
two will be male and two will be female). Two (a) South-East (b) South-west
unmarried siblings. (c) North-East (d) North-west
Logical Conclusions - Out of two unmarried Sol. (d)
siblings one will be male and one will be female. Here is the diagram representing the movement –
There are only two generations. Let the older 15m
generation be G1 and younger generation be G2. Final
Point
P and Q will be part of G1 because of being a parent 50m
and mother-in-law. West
Front side of
East
25m the house
U, R and T (unmarried male) will be part of G2.
Back side of
Considering all the information we can get the house
following order (circle representing male and Starting point
triangle representing female – From the diagram given above, it is clear that he
Husband Wife Mother G1 will be in North-West direction from the starting
P Q
Son point.
Mother in law
Daughter
T G2
Son Daughter in law 12. Two Statements are given followed by two
S
Husband Wife
R
U Conclusions:
Statements:
10. If in a particular year 12th January is a Sunday, All numbers are divisible by 2.
then which one of the following is correct? All numbers are divisible by 3.

CSAT by Manjul Kumar Tiwari Sir


184 CSAT : 2020

Conclusions: Above argument can be drawn in the two ways. In


I. All numbers are divisible by 6. both the cases you can see that in some cases all
II. All numbers are divisible by 4. dogs can be blacks but then in some cases it is not.
Hence conclusion I not follows. Similarly ‘every
Which of the above Conclusions logically follows/
dog will be black’ if the dotted circle will be
follow from the two given Statements?
congruent to the circle representing dog. So in this
(a) Only Conclusion-I
case conclusion II does not follow. To be a valid
(b) Only Conclusion-II
conclusion it should be true in every case. Hence
(c) Neither Conclusion-I nor conclusion-II
both the conclusions are not valid.
(d) Both Conclusions-I and Conclusion-II
14. Consider the following sequence of numbers:
Sol. (a)
5 1 4 7 3 9 8 5 7 2 6 3 1 5
Given statements are – first, all numbers are
8 6 3 8 5 2 2 4 3 4 9 6
divisible by 2 and second, all numbers are divisible
by 3. If a number is divisible by 3 and 2 then it will How many odd numbers are followed by the odd
surely be divisible by 6 (which is 2 x 3), but will number in the above sequence?
not be necessarily divided by 4. (a) 5 (b) 6
For example – 6, 18, 30 … are all such numbers. (c) 7 (d) 8
Hence only conclusion 1 is true (logically follows). Sol. (b)
13. Two Statements are given followed by two Given sequence is – 5 1 4 7 3 9 8 5 7 2 6 3 1 5 8 6 3
Conclusions: 8 5 2 2 4 3 4 9 6.
Statements: It happens six times when an odd number is
All cats are dogs. followed by another odd number. See the bold
All cats are black. digits (51, 73, 39, 57, 31 and 15).
Conclusions: 15. A is 16th from the left end in a row or boys and V is
I. All dogs are black. 18th from the right end. Q is 11th from A towards
II. Some dogs are not black. the right and 3rd from V towards the right end.
How many boys are there in the row?
Which of the above Conclusions logically follows/
follow from the two given Statements disregarding (a) 40
commonly known facts? (b) 41
(a) Only Conclusion-I (c) 42
(b) Only Conclusion-II
(d) Cannot be determined due to insufficient data
(c) Neither Conclusion-I nor Conclusion-II
(d) Both Conclusions-I and Conclusion-Il Sol. (b)
Sol. (c) A is 16th from left so 15 students will be there to
Neither of the conclusions will be true for the given A’s left. G is 11th to right from A and 3rd to right
syllogism argument. To understand let’s have look from V. So there will be 7 students between A and
on the following Euler/Venn diagrams. The circle V and 2 students between V and G. Consequently
represents Cats, Dogs and Black (dotted circle). there will 14 students to the right of G. So total
number of Students will 15 + 7 + 2 + 14 + A(1) +
V(1) + G (1) = 41.
16. Three Statements S1, S2 and S3 are given below
followed by a Question:
S1: C is younger than D, but older than A and B.
S2: D is the oldest.
S3: A is older than B.

CSAT by Manjul Kumar Tiwari Sir


Previous Year Solved Papers 185
Question: Sol. (a)
Who among A, B, C and D is the youngest? Let the side of P = a, so the side of Q = 2a.
Which one of the following is correct in respect of Volume of P (x) = a3 volume of Q (y) = 8a3
the above Statements and the Question? Also, It is given that,
(a) S1 alone is sufficient to answer the Question.
Mass of P (m) = u × X = u × a3 ….. (1) and
(b) S1 and S2 together are sufficient to answer
Mass of Q (n) = v × y = v × 8a3 …..(2)
the Question.
(c) S2 and S3 together are sufficient to answer It is also given that Mass of Q (n) = 2 × mass of
the Question. P (m).
(d) S1 and S3 together are sufficient to answer So, by using equations 1 and 2 we have
the Question. v × 8a3 = 2 × u × a3 4v = u.
Sol. (d) So option (a) is correct.
Question asked is ‘Who among A, B, C and D is
the youngest?’ 19. The average age of a teacher and three students is
20 years. If all the three students are of same age
Option (a): S1 alone is not sufficient. Because there
and the difference between the age of the teacher
is no clarity about A and B as to who will be
and each student is 20 years, then what is the age
younger.
of the teacher?
Option (b): S1 and S2 together will also be not
sufficient as again there will be not clarity about A (a) 25 years (b) 30 years
and B as who will be younger. (c) 35 years (d) 45 years
Option (c): Combination S2 and S3 will not be Sol. (c)
sufficient as it does not provide any information Let the age of Teachers is T years and the three
about C.
students is a years (Given age of all three student
Option (d): Combination S1 and S3 will give order is same). Then according to the information given
D > C > A > B. So, the youngest person will be B. in question we have

17. How many integers are there between 1 and 100 (T + 3a)/4 = 20 or T + 3a = 80 ….(1)
which have 4 as a digit but are not divisible by 4? T – a = 20 ….(2) (assuming that teachers age will
(a) 5 (b) 11 be greater than the students)
(c) 12 (d) 13
By solving equations 1 and 2 simultaneously we
Sol. (c) have a = 15 and T = 35. Option (c) is correct.
Between 1 and 100 there are 19 such integers which
have 4 as a digit – 4, 14, 24, 34, 40, 41, 42, 43, 44, 45, 20. A person bought a car and sold it for Rs. 3, 00,000.
46, 47, 48, 49, 54, 64, 74, 84 and 94. If he incurred a loss of 20%, then how much did he
Out of these, only 7 integers – 4, 24, 40, 44, 48, 64 spend to buy the car?
and 84 are divisible by 4. (a) Rs. 3, 60,000 (b) Rs. 3, 65,000
(c) Rs. 3, 70,000 (d) Rs. 3, 75,000
So correct answer will be 19 – 7 = 12.
Sol. (d)
18. Let x, y be the volumes; m, n be the masses of two Since loss is 20%, so 80% of CP = 3,00,000. => 0.8
metallic cubes P and Q respectively. Each side of CP = 300000 So CP = 3,75,000.
Q is two times that of P and mass of Q is two times
that of P. Let u = m / x and V = n / y. which one or Directions for the following 6 (six) items: Read the
the following is correct? following five passages and answer the items that follow.
(a) u = 4v (b) u = 2v Your answers to these items should be based on the
(c) v = u (d) v = 4u passages only.

CSAT by Manjul Kumar Tiwari Sir


186 CSAT : 2020

Passage — 1 without actual availability of security that guarantees


In India, over the last decade or so, labour has been that private force cannot be used to thwart this right. The
departing agriculture, but is only going to construction role of the State, therefore, is not just to abstain from
and unregistered manufacturing which are not markedly preventing rightful free expression, but also to actively
better jobs. Services, where labour tends to be most ensure that private parties are not able to block it.
productive, are not generating the additional jobs the
22. On the basis of the above passage, the following
country needs. India will need 24 million or so jobs over
assumptions have been made:
the next decade. The new sector, e-commerce, can at best
1. State should have some institutions to ensure
close only half the jobs gap. Only those sectors that drive
its appropriate role in a digital society.
domestic demand such as health and education can
2. State should ensure that private parties do
comfortably fill the other half.
not violate the citizens’ right to privacy.
21. Which one of the following is best Implied in the 3. Digital economy is not compatible with the
passage? idea of not violating the citizens’ privacy.
(a) Strong measures need to be taken to reduce Which of the above assumptions is/are valid?
the rural to urban migration of labour. (a) 1 and 2 (b) 3 Only
(b) The working condition in construction and
(c) 1 and 3 (d) 2 only
unregistered manufacturing needs to be
Sol. (a)
improved.
(c) Service sector has been reducing the problem Statement 3 is clearly wrong, as it is not assumed
of unemployment. anywhere in the passage. So options (b) and (c)
(d) Increased social sector spending is are ruled out.
imperative for large-scale job creation. Look at the options – we need to just check if
Sol. (d) Statement 1 is right or not. It is indeed correct
Clearly option (d) is the answer. because “to actively ensure” is possible only
through institutional mechanisms (not one-off or
Option (a) is wrong as the passage does not talk
ad hoc responses). Hence only (a) seems correct.
about reducing labour movement out of rural areas.
The passage talks about ‘right to privacy’, then it
Option (b) is wrong as that is not the core idea in
provides insight on meaningfulness of right with
the passage (“best implied”).
an example on ‘right to free expression’. At the
Option (c) is just opposite to what is given and is
end of the passage there is mention of ‘role of state’
evident from ‘Services, where labour tends to be
in ensuring the ‘right to free expression’ through
most productive, are not generating the additional
acts of omission and commission.
jobs the country needs.’ It is given that ‘Only those
sectors that drive domestic demand such as health Passage — 3
and education (social sector) can comfortably fill
One of the biggest ironies around water is that it comes
other half’ so (d) is true.
from rivers and other wetlands. Yet it is seen as divorced
Option (d) is slightly indirect but ultimately means from them. While water is used as a resource, public policy
what is being asked. Health and education sectors does not always grasp that it is a part of the natural
comprise the social sector, and large investments ecosystem. Efforts at engineering water systems are thus
there can generate many jobs. efforts at augmenting water supply rather than
Passage — 2 strengthening the capacities of ecological systems.

In India, the current focus on the right to privacy is based 23. Which one of the following is the most logical and
on some new realities of the digital age. A right is a rational inference that can be made from the above
substantive right only if it works in all situations, and passage?
for everyone. A right to free expression for an individual (a) Rivers and other wetlands should be
about her exploitation, for instance, is meaningless protected under Ramsar Convention.

CSAT by Manjul Kumar Tiwari Sir


Previous Year Solved Papers 187
(b) Engineering water systems should be (d) You should know that investing your money
modernized and further augmented. is a risky business.
(c) Wetlands need to be reinforced as more than Sol. (a)
just open sources or water. Option (c) can be eliminated as that’s not
(d) Water supply should not be free of cost so as mentioned anywhere.
to prevent its misuse or overuse. Option (d) is worded rather loosely and can be
Sol. (c) eliminated.
Passage does not talk about ‘Ramsar Convention’ Option (b) would be eliminated due to the words
neither does it talk about ‘cost of its supply’, so “to generate wealth”.
options (a) and (d) are ruled out. We are left with Option (a) now. At the end, the
Passage talks about ‘strengthening the capacities passage talks about diversifying the assets for
of ecological systems’. For this you have to give lower risk [‘All these extreme forms of risks we
more respect to the wetlands, from where the water know will not manifest at the same time’].
is sourced, and that can be done by considering
Passage — 5
them more than just open sources of water. So
option (c) is correct. Although most of the Genetically Modified (GM) crops
Option (b) is not the most logical inference as cultivated now are genetically engineered for a single
modernization of water systems does not trait, in future, crops genetically engineered for more than
one trait will be the norm. Thus, biotechnology’s role in
necessarily mean respecting natural ecosystems.
agriculture and the regulation or the same cannot be
Passage — 4 understood solely in the context of the current generation
Asset allocation is the most important investment of GM crops. Instead, there is a need to take a
decision we will ever make, and sadly, most of us do not comprehensive look, taking into account various aspects,
give that decision the importance it deserves. We are including socio-economic impacts, so that the potential
adamant about seeking predictability with our future. or the technology can be harnessed while minimizing
We tend to think of investing in risky assets as extremely negative impacts. Given the importance of biotechnology
volatile and value eroding. We also dislike fluctuating
in developing varieties that can help in climate change
returns and the loss of control of investment. We think
mitigation and adaptation, not using biotechnology as a
our money is best left idle unproductive but safe. There is
part of the climate change action plan cannot be an option.
no asset that is risk-free. We could lose our jobs, our
Domestic regulation of biotechnology cannot be viewed
homes can lose value, our banks can go bankrupt, our
in isolation of trade policy and obligations under various
bonds can default, the government can collapse and
companies we chose fondly may cease to exist. But we international treaties and conventions.
cannot live life assuming that all these extreme events 25. With reference to the above passage, the following
are waiting to happen, and all at the same time. All these assumptions have been made:
extreme forms or risks we know will not manifest at the
1. Biotechnology regulation is an evolving
same time.
process.
24. Which one of the following statements best implies 2. Participation of people is needed in policy
the suggestion given by the author of the passage? decisions regarding biotechnology
(a) Distribute your wealth across different kinds regulation.
of assets so that your risks would be
3. Biotechnology regulation should take into
minimized.
account socio-economic aspects in decision-
(b) Risk-taking behaviour should be a necessary
making.
component of your Personality if you want to
generate wealth. 4. Wider involvement of political executive in
(c) While making investments, find a trustworthy biotechnology regulation improves its
asset management organization which effectiveness in dealing with the country’s
would manage your wealth for you. trade Policies and international obligations.

CSAT by Manjul Kumar Tiwari Sir


188 CSAT : 2020

Which of the above assumptions are valid? The breaking down fields 10 zeros.
(a) 1, 2 and 4 only (b) 1 and 3 only = 1 × 5 × 10 × (3 × 5) × (2×10) × (5×5) × (3×10) ×
(c) 2, 3 and 4 only (d) 1, 2, 3 and 4 (7×5) × (4×10) × (9×5) × (5×10) × (11×5) × (6×10)
Sol. (b)
28. Let XYZ be a three-digit number, where (x + y + Z)
This passage has two questions and hence must is not a multiple of 3. Then (XYZ + YZX + ZXY) is
be attempted. not divisible by
Statement 1 is correct. It is evolving constantly. (a) 3 (b) 9
So option (c) is eliminated. (c) 37 (d) (X + Y + Z)
Statement 3 is correct and is reflected in the Sol. (b)
passage. XYZ is a 3-digit number not divisible by 3.
So, option (b) is correct. We know that expanded form of XYZ will be 100
X + 10Y + Z. (basic decimal number system)
26. Which one of the following statements best implies
So, XYZ + YZX + ZXY = (100 X + 10Y + Z) +
the crux of the passage?
(100 Y + 10Z + X) + (100 Z + 10X + Y)
(a) Precautionary principle is not given
importance in current debate on developing = 111X + 111Y + 111Z = 111 (X + Y + Z)
GM crops. (note 3 and 37 are prime numbers and X + Y + Z is
(b) Biotechnology is not currently used in climate not a multiple of 3 (given), so it will not be multiple
change mitigation and adaptation of 9 too)
mechanisms. So, the correct option is (b).
(c) Biotechnology’s role is not confined to the
29. Let p, q, r and s be natural numbers such that
current priorities of developing GM crops.
p - 2016 = q + 2017 = r – 2018 = s + 2019 which one
(d) The negative Impacts of not biotechnology are
of the following is the largest natural number?
properly understood.
(a) p (b) q
Sol. (c) (c) r (d) s
We are now asked the “crux of the passage”. Sol. (c)
The passage is talking mainly about what From the given relations, we have p – q = 4033;
biotechnology is being used for today, and what is r – q = 4035 which clearly means r > p > q. Also
can be and must be used for in the coming years. r – s = 4037 which means r > s.
Hence, option (c) best summarises the crux. Thus, r is the greatest of all. So the correct option
27. How many zeroes are there at the end of the is (c).
following product? 30. How many five-digit prime numbers can be
1  5  10  15  20  25  30 obtained by using all the digits 1, 2, 3, 4 and 5
 35  40  45  50  55  60 without repetition of digits?
(a) Zero (b) One
(a) 10 (b) 12
(c) Nine (d) Ten
(c) 14 (d) 15
Sol. (a)
Sol. (a)
A number is prime if it is divisible by none other
In a multiplication problem, 0 can be produced in
than 1 and itself.
two ways. First when a number is multiplied by
10 or second when 5 is multiplied by 2 or any even Now take the digits 1,2,3,4 and 5. Just add them
number. up once. We get 15 as the sum. Recall the rule of
divisibility by 3.
So to count the number of zeroes in the given
multiple break the product and count no. of 5 & “If the sum of the digits of a number is divisible by
2 & 10. 3 then number will be divisible by 3”.

CSAT by Manjul Kumar Tiwari Sir


Previous Year Solved Papers 189
So any number formed using these five digits will So, statement 1 is surely correct.It is definitely
be divisible by 3. So it cannot be prime. Hence zero. assumed in it. Why? When we are constantly
worried about the fickle nature of FDI (and local
Directions for the following 7 (Seven) items: Read the
private investment), and its negative relation with
following five passages and answer the items that follow.
employment generation, how can we rely on it as
Your answers to these items should be based on the
an economic policy? If 1 is correct, then options (a)
passages only.
and (b) are possible answers but not (c) or (d).
Passage — 1 Now between (a) and (b), we have to check 2 and 3.
Private investment in general is volatile. Foreign private Focus on 2 – India can do nothing to reduce
investment is more volatile because the available volatility in FDI because it is the inherent nature of
investment avenues are significantly greater (i.e., the FDI to seek higher returns (hence fickle – moves
entire world). Therefore, the responsibility of providing from here to there). Note the phrase “a necessary
employment cannot be left to Foreign Direct investment consequence”. So 2 is wrong. But 3 may be right,
(FDl). The current FDI inflows are volatile over time and and 5 is surely right (last part of the passage).
across sectors and regions, which is a necessary Why is 5 right? Education and health are
consequence of their search for the highest returns. The bottlenecks (in absorbing new technology) so
adverse consequences are unstable employment and an public investments will help.
accentuation of income and regional inequalities. A
Hence, correct answer is option (b). Also note that
probable positive consequence of foreign investment is
4 is not explicitly talked about anywhere.
the inflow of new technology and its subsequent
diffusion. However, the technology diffusion is not at all Passage — 2
certain because the existing state of physical and human Many opportunities to harness the highly skewed,
capital in India may prove inadequate for the diffusion. seasonal and spatial distribution of monsoon flows,
which occur in a four-month period from June to
31. With reference to the above passage, the following
September annually, have been lost. Since these few
assumptions have been made:
months account for most or the rainfall and consequent
1. Relying on foreign investment in the long run freshwater availability, the need for holding rainwater
is not an economically sound policy. in reservoirs, for subsequently releasing it for use over
2. Policies must be undertaken to reduce the year, is a necessity nobody can afford to overlook.
volatility in foreign private investment. Climate change will continue to affect weather conditions
3. Policies must be undertaken to strengthen and create water shortages and excesses. While millions
domestic private investment. suffer from droughts and floods, waters in the country’s
4. Public investment should be given priority many rivers flow unutilized, and are discharged into the
over private investment. sea every year.
5. Substantial public investment in education 32. With reference to the above passage, which of the
and health should be undertaken. Which of following could be the most rational and practical
the above assumptions is/are valid? implications for India?
(a) 1, 2 and 4 (b) 1, 3 and 5 1. Inter-linking of rivers should be undertaken.
(c) 2, 4 and 5 (d) 3 only
2. A network of dams and canals should be built
Sol. (b) across the country for proper distribution of
The passage is talking about the nature of private water.
investment and its fickle tendency. It explains why
3. Farmers should be provided easy loans for
employment generation cannot be left to foreign
digging borewells.
private investment, and even its positive effects
may get diluted due to inherent problems in Indian 4. Usage or water for agriculture should be
human capital. regulated by law.

CSAT by Manjul Kumar Tiwari Sir


190 CSAT : 2020

5. Distribution of river water among regions 3. Economic freedom has a positive impact on
should be regulated by the Union building up human capital.
Government. Which of the above assumptions is/are valid?
Select the correct answer using the code given (a) 1 only (b) 2 only
below. (c) 3 only (d) 1, 2 and 3
(a) 1 and 2 (b) 2, 4 and 5 Sol. (c)
(c) 1, 3 and 4 (d) 2, 3 and 5
Sol. (a) Passage-4
Passage is talking about shortages and excesses Our urban bodies cannot possibly ensure sustainable
of water due to unpredictability and nature of
delivery of water in our cities unless financing
Monsoon in India. It also talks about ‘unutilized
mechanisms are put in place. Water delivery requires
water which flows with the rivers’.
heavy investment in collecting it from a natural source,
Statement 3 is not an implication we can get from
treating it to make it potable, and laying a distribution
the passage. Eliminate it. So we are left with
network of pipes for delivery to the users. It also requires
options (a) and (b) only.
investments in sewerage infrastructure and sewage
Statements 1 and 2 indicate possible solutions to
treatment plants so that the sewers can carry the
tackle the problem of water discharge into oceans.
wastewater to these plants to ensure that no untreated
Inter-linking of rivers will allow usage of excess
sewage is discharged back into natural water bodies. If
water in deficit regions. And network of dams and
our cities were rich enough to meet the entire cost, water
canals will ensure timely harnessing of skewed
could be delivered free. They are not.
monsoon flows.
Hence, (a) is best. Also, statement 3 is wrong, so 34. What is the most logical and crucial message
(c) and (d) are ruled out. conveyed by the passage?
(a) Urban local bodies must recover costs through
Passage — 3
user charges.
People will invest in education whenever they are (b) Urban local bodies are not efficient enough to
granted the economic freedom to fully enjoy benefits meet the water requirements of our cities.
Again, this is for the obvious reason that the return on
(c) Water shortage in our cities is a perennial
education increases as the level of economic freedom
problem that cannot be solved.
rises. When people, thanks to lower tax rates, are allowed
(d) In view of the water crisis in our cities, there
to retain of the higher income that they gain from
is an urgent need to limit the population of
incremental level of education, it makes sense to invest
cities by adopting an upper limit of
in education. On the other hand, when the government
population size.
decides to tax the higher income of educated individuals
at even higher rates, it makes very little sense to invest in Sol. (a)
educating oneself further. The same incentives apply to Try this passage as it has two questions.
parents who decide on whether to invest in their Option (c) is clearly wrong, as it says “…cannot
children’s education. be solved”.
33. With reference to the above passage, the following Option (d) is wrong as it is nowhere mentioned in
assumptions have been made: the passage.
1. Lower tax rates in a country invariably
Efficiency is not being discussed in the passage,
translate into greater investments in higher
rather financial viability is. So (b) is not entirely
education.
correct.
2. Investment in the education of children
ensures their economic freedom. So option (a) is correct.

CSAT by Manjul Kumar Tiwari Sir


Previous Year Solved Papers 191
35. With reference to the above passage, the following 3. India needs to build value chains for high-
assumptions have been made: value agri-products like livestock and
1. Rich cities only can ensure sustainable horticulture.
delivery of water. 4. Higher global prices of agricultural
2. Sustainable delivery of water in cities means commodities are essential for India’s poverty
much more than supplying water to reduction.
households. Which of the above assumptions are valid?
Which of the above assumptions is/are valid? (a) 1 and 3 (b) 2 and 4
(a) 1 only (b) 2 only (c) 2 and 3 (d) 3 and 4
(c) Both 1 and 2 (d) Neither 1 nor 2 Sol. (c)
Sol. (b) Statement 2 is definitely correct (an assumption
Statement 1 is not correct, as “rich cities could made in the para) because good price incentives
supply water for free” is mentioned. We are talking indeed trigger agri investments (mentioned
of sustainability in statement 1. clearly). So options (b) and (c) are only possibilities.
Statement 3 is not mentioned anywhere, even
Statement 2 is correct as sustainable delivery
indirectly. So only option (b) is left.
includes many aspects other than just supplying
Statement 4 is not the focus, it was a passing
water to households.
statement.
Hence option (b) is right. So, option (c) is correct.
Passage-5 37. Which one of the following statements best reflects
the critical message of the passage?
In India, agriculture still engages about half of workforce,
(a) India should create large- scale off-farm rural
and about 85 per cent of its farms mall and marginal.
employment to reduce poverty in the near
Compared to China Vietnam, which have experienced
future.
fast structural and rural transformation, India’s story is
(b) India should create a large number of farmer
of slow transformation. As a result, poverty reduction in
producer companies
India was at a much slower pace during 1988—2014,
(c) Private investment in agriculture should be
compared to China and Vietnam. India’s poverty
given priority over public investment.
reduction was slow during 1988-2005, but during 2005-
(d) Inclusive agricultural growth is key to reduce
2012, it accelerated dramatically—almost three times
poverty in the near future.
faster than during the earlier period. What did India do
during this period? Research reveals that the relative Sol. (d)
price scenario changed significantly (by more than 50%) The passage does not talk about ‘off-farm rural
in favour of agriculture in the wake of rising global prices. employment’, ‘farmer produce companies’ or the
This boosted private investments in agriculture by more priority of private investment in agriculture over
than 5o%. As a result, agri-GDP growth touched 4/1% public. These are not mentioned anywhere at all,
during 2007-2012 as against 2.4% during 2002—2007. and it would be a pure assumption to consider
The net surplus or agri-trade touched $25 billion in 2013- them relevant.
2014: real farm wages rose by 7% per annum. All this led So options (a), (b) and (c) are ruled out.
to unprecedented fall in poverty. The passage clearly implies that the rate of poverty
36. With reference to the above passage, the following reduction was higher when the growth rate of agri-
assumptions have been made: GDP was more. Hence option (d) is correct.
1. Structural and rural transformation is
38. Two Statements S 1 and S2 are given below with
impossible when farms are mainly small arid
regard to four numbers P, Q, R and S followed by a
marginal.
Question:
2. A good price incentive can trigger investments
S1: R is greater-than P as well as Q.
in agriculture.
S2: S is not the largest one.

CSAT by Manjul Kumar Tiwari Sir


192 CSAT : 2020

Among four numbers P, Q, R and S which one is Using S2 alone there are two numbers 13 (= 4×3 +
the largest? 1) and 17 (= 4×4 + 1) are possible.
Which one of the following is correct in respect of Even after combining both you will get set of two
the above Statements and the Question? numbers i.e., 13 and 17. So correct answer is
(a) S1 alone is sufficient to answer the Question. option (d).
(b) S2 alone is sufficient to answer the Question. 40. Two Statements S1 and S2 are given below with
(c) S1 and S2 together are sufficient to answer regard to two numbers followed by a Question:
the Question, but neither S1 alone nor S2 alone S1: Their product is 21.
is sufficient to answer the Question. S2: Their sum is 10.
(d) S1 and s2 together are not sufficient to answer Question:
the Question.
What are the two numbers?
Sol. (c)
The question is ‘Among four numbers P, Q, R and Which one of the following is correct in respect of
S, which one is LARGEST?’ the above Statements and the Question?
(a) S1 alone is sufficient to answer the Question.
S1 alone is not sufficient to answer the question,
as it doesn’t talk about S. (b) S2 alone is sufficient to answer the Question.

S2 alone can’t be true as it says that S is not the (c) S1 and S2 together are sufficient to answer
largest, but doesn’t give any hint about R, P and Q. the Question, but neither S1 alone nor S2 alone
is sufficient to answer the Question.
By combining S1 and S2 we get – ‘there are four
(d) S1 and S2 together are not sufficient to answer
numbers P, Q, R and S; S cannot be largest; So
the Question.
largest number will be among P, Q and R but ‘R is
greater than P and Q’, so R is the largest. Sol. (c)
Hence, answer is (c). S1 alone gives you a × b = 21 (where a and b are
two numbers) which have multiple possibilities
39. Two Statements S1 and S2 are given below like 3×7 = 21, 1.5×14 = 21 or 6 × 3.5 = 21. (it is not
followed by a Question: mentioned that numbers are integers)
S1: n is a prime number.
So S1 alone is not sufficient.
S2: n leaves a remainder of 1 when divided by 4.
S2 alone will also not sufficient as it will give you
If n is a unique natural number between 10 and a + b = 10, and again multiple possibilities of a
20, then what is n? and b like (1, 9); (2, 8); (3, 7)… etc.

Which one of the following is correct in respect of By combining bothwe have a + b = 10 or b = 10 – a


the above Statements and the Question? …(1) and a × b = 21 …(2)
(a) S1 alone is sufficient to answer the Question. Putting the value of b in equation 2, we have a ×
(b) S2 alone is sufficient to answer the Question. (10 – a) = 21 =>a2 – 10a +21 = 0
(c) S1 and S2 together are sufficient to answer => a2 – 7a – 3a +21 = 0 => a(a – 7) – 3 (a – 7) = 0 =>a
the Question, but neither S1 alone nor S2 alone – 7) (a – 3) = 0
is sufficient to answer the Question. => If a = 7, b = 3 and If a = 3, b = 7.
(d) S1 and S2 together are not sufficient to answer
So both S1 and S2 are required to find the numbers.
the Question.
Option (c) is correct.
Sol. (d)
Neither of the S1 and S2 together or alone can give 41. What is the greatest length x such that 3 ½ m and
you the value of n. 8 ¾ m are integral multiples of x?
Using S1 alone you have four prime numbers (a) 1 ½ m (b) 1 1/3 m
between 10 and 20 -> 11, 13, 17, and 19. (c) 1 ¼ m (d) 1 ¾ m

CSAT by Manjul Kumar Tiwari Sir


Previous Year Solved Papers 193
Sol. (d) Sol. (d)
1 3 Growth rate = Birth rate – Death rate
We have to basically find the HCF of 3 and 8 .
2 4 Growth rate for 1911-1921 = 48.1 – 35.5 = 12.6
1 7 3 35 Growth rate for 1941-1951 = 39.9 – 27.4 = 12.5
3  and 8 
2 2 4 4 Growth rate for 1961-1971 = 41.1 – 18.9 = 22.2
7 35
So we have to find HCF of and Growth rate for 1971-1981 = 37.1 – 14.8 = 22.3
2 4
HCF of Numerators 43. The recurring decimal representation 1.272727...
HCF of fractions = is equivalent to
LCM of Denomenators
(a) 13/11 (b) 14/11
HCF of 7 and 35 7 3
=  1 (c) 127/99 (d) 137/99
LCM of 2 and 4 4 4
Sol. (b)
Alternate method: Given that X = 1.272727…
3 So 100 x = 127.272727…
Here X × Integer = 8 .
4  Subtract the first from second, to get – 99 X
1 = 126
3
So Integer = 2 . X = 126/99 = 14/11.
X
So the correct option is (c).
1
3
Working backward from options 2 is not an 44. What is the least four-digit number when divided
1 by 3, 4, 5 and 6 leaves a remainder 2 in each case?
1
2 (a) 1012 (b) 1022
integer.
(c) 1122 (d) 1222
So option (a) is not possible.
Sol. (b)
Similarly, options (b) and (c) are not possible.
Desired number = (Least multiple of four digit of
1 3
3 8 LCM of 3, 4 and 5) + 2
For option (d), 2 = 2 and 4 = 5. So the LCM of 3, 4, 5 and 6 is 60.
3 3
1 1
4 4 The least 4-digit number divisible by 60 is 1020.
So the correct option is (d). So with remainder 2 it will be 1020 + 2 = 1022.
Answer is (b).
42. Consider the following data:
45. In adult Population or a city, 40% men and 30%
women are married. What is the percentage of
married adult population if no man marries more
than one woman and no woman marries more than
one man; and there are no widows and widowers?
1
(a) 33 % (b) 34%
7
2
(c) 34 % (d) 35%
7
Sol. (c)
For which period was the natural growth rate
No man marries more than one woman and no
maximum?
woman marries more than one man. It means the
(a) 1911 - 1921 (b) 1941 - 1951
Number of married men = Number of married
(c) 1961 - 1971 (d) 1971 - 1981
women = 120 (let). Married adult population = 240.

CSAT by Manjul Kumar Tiwari Sir


194 CSAT : 2020

Number of men = 120×100/40 = 300 and Number Sol. (b)


of women 120 × 100/30 = 400. So, total population (10n + 1) will have sum of digits 2 for any whole
= 300 + 400 = 700. number n. Whole numbers are integers starting
Required percentage 240×100/700 = = 240/7 % with 0. Just put different value of n and observe
2 yourself.
= 34 % For n = 0, 100 + 1 = 1 + 1. So sum is 2.
7
For n = 1, 101 + 1 = 11. So sum is 1 + 1 = 2.
46. What is the remainder when 51 x 27 x 35 x 62 x 75
For n = 2, 102 + 1 = 100 + 1 = 101. So sum is 1 +
is divided by 100?
1 = 2.
(a) 50 (b) 25
So the correct option is (b).
(c) 5 (d) 1
Sol. (a) 49. In a class, there are three groups A, B and C. If one
When a number is divided by 100, last two digits student from group A and two students from group
of that number will always be remainder. For B are shifted to group C, then what happens to the
example 312/100 will give 12 as remainder or 5759 average weight of the students of the class?
will give 59 as remainder. (a) It increases.
So our question is basically reduced to find last (b) It decreases.
two digits of 51 × 27 × 35 × 62 × 75. (c) It remains the same.
However, present question can be solved by (d) No conclusion can be drawn due to
looking at only unit digit of the above insufficient data.
multiplication. Sol. (c)
In the multiplication of two numbers, the unit’s Class Average will remain same as nobody is
place of the resultant number always be the coming in or going out of the class. So average
product of the unit’s place digits of the number weight equilibrium will be same as before shifting.
which are multiplying. For example, the unit’s A change can happen only if someone is added to
digit of 51 × 27 will 1 × 7 = 7. the group(s) or someone leaves the group(s), not
In above we will get unit digit as 1 × 7 × 5 × 2 × 5 otherwise.
Only matching option is (a). 50. How many different sums can be formed with the
47. A sum of Rs. 2,500 is distributed among X, Y and denominations Rs. 50, Rs. 100, Rs. 200, Rs. 500
Z in the ratio 1/2 : 3/4 : 5/6 . What is the difference and Rs. 2,000 taking at least three denominations
between the maximum share and the minimum at a time?
share? (a) 16 (b) 15
(a) Rs. 300 (b) Rs. 350 (c) 14 (d) 10
(c) Rs. 400 (d) Rs. 450 Sol. (a)
Sol. (c) We have to choose at least three denominations
out of five. So we can either choose 3
Ratio 1/2 : 3/4 : 5/6 can be written as 6 : 9 : 10.
denominations out of 5 or choose 4 out of 5 or
The total amount is Rs. 2500. Clearly the shares
choose all 5.
are Rs. 600, Rs. 900 and Rs. 1000 respectively.
Mathematically it will be written as –
The difference between the greatest and the
Total number of ways = 5C3 + 5C4 + 5C5 = 10 + 5 +
smallest is Rs. 400. So the correct answer is (c).
1 = 16 ways.
48. For what value of it, the sum of digits in the number Manually, these combinations will be –
(10n + 1) is 2? (a) Three denomination combinations –
(a) For n= 0 only 50+100+200; 50+100+500; 50+100+2000;
(b) For any whole number n 50+200+500; 50+200+2000; 50+500+2000;
(c) For any positive integer n only 100+200+500; 100+200+2000; 100+500+2000;
(d) For any read number n 200+500+2000; TOTAL 10

CSAT by Manjul Kumar Tiwari Sir


Previous Year Solved Papers 195
(b) Four denomination combinations – 53. Consider the following statements:
50+100+200+500; 50+100+200+2000; 1. The minimum number of points of
50+100+500+2000; 50+200+500+2000; intersection of a square and a circle is 2.
100+200+500+2000; TOTAL 5 2. The maximum number of points of
(c) Five denomination combinations – intersection of a square and a circle is 8.
50+100+200+500+2000; TOTAL 1 Which of the above statements is/are correct?
So total = 10 + 5 + 1 = 16. (a) 1 only (b) 2 only
(c) Both 1 and 2 (d) Neither 1 nor 2
51. A person X can complete 20% of work in 8 days
and another person y can complete 25% of the same Sol. (b)
work in 6 days. If they work together, in how many As shown in figure, minimum numbers of points
days will 40% of the work be completed? of intersection of a square and a circle is 2 and
(a) 6 (b) 8 maximum number of points of intersection is 8.
(c) 10 (d) 12
Sol. (a)
X can complete 20% of the work in 8 days. So X
can complete the entire work in 40 days. Thus,
work done by X in 1 day = 1/40.
Y can complete 25% of the same work in 6 days. So
work entire work in 24 days and work done by Y
So option (b) is correct.
in 1 day = 1/24.
Work done by X and Y in 1 day = 1/40 + 1/24 = 8/ 54. A man takes half time in rowing a certain distance
120 = 1/15. downstream than upstream. What is the ratio or
the speed in still water to the speed of current?
That means X and Y can together complete the
(a) 1 : 2 (b) 2 : 1
entire work in 15 days.
(c) 1 : 3 (d) 3 : 1
So, time taken to finish 40% or 2/5th of the work =
Sol. (d)
15 × 2/5 = 6 days. So the correct option is (a).
52. A car travels from a place X to place Y at an average Try working backwards. For option (d), s = 3x, c =
speed of v km/hr from y to X at an average speed 1x. So downstream speed = 4x and upstream speed
of 2v km/hr, again from X to y at an average speed = 2x. Clearly for the same distance, ratio of times
of 3v km/hr and again from y to x at an average taken will be ratio of these speeds, and that comes
speed of 4v km/hr. Then the average speed of the out to 4x : 2x = 2:1. (upstream time : downstream
car for the entire journey time).
(a) is less than v km/hr
Hence option (d) is correct.
(b) lies between v and 2v km/hr
(c) lies between 2v and 3v km/hr 55. How many pairs of natural numbers are there such
(d) lies between 3v and 4v km/hr that the difference of whose squares is 63?
Sol. (b) (a) 3 (b) 4
(c) 5 (d) 2
Average Speed = Total distance ÷ Total time.
Sol. (a)
Let us assume that distance between X and Y be
‘a’. Then, total distance = a + a + a + a = 4a. Natural numbers are integers starting with 1.

And total time = Distance/Speed = a/v + a/2v + Let the two numbers be a and b then
a/3v +a/4v = 25a/12v a2 – b2 = 63 =>(a + b) (a – b) = 63. Now 63 = 9 × 7
So, average speed for the entire journey = 4a/ (25a/ = 21 × 3 = 63 × 1 So,
12v) = 48v/25 = 1 .92v. Case I: a + b = 9; a – b = 7. Here we will get a = 8
So the correct option is (b) i.e. between v and 2v. and b = 1

CSAT by Manjul Kumar Tiwari Sir


196 CSAT : 2020

Case II: a + b = 21; a – b = 3. Here we will get a = 12 (a) 10 kg (b) 15 kg


and b = 9. (c) 20 kg (d) 25 kg
Case III: a + b = 63; a – b = 1. Here we will get a = Sol. (a)
32 and b = 31 Let the weight of empty vessel = v and weight of
water = w, then by the conditions given we have
Since 63 has only these 3 pairs of factors, so the
correct option is (a). v + w = 40 …. (1) and
v + w/3 = 20 …. (2)
56. Which one of the following will have minimum
By solving equations (1) and (2) simultaneously
change in its value if s is added to both numerator
we have w = 30 and v = 10.
and the denominator of the fractions 2/3, 3/4,
4/5 and 5/6? 60. A frog tries to come out of a dried well 4.5 m deep
(a) 2/3 (b) 3/4 with slippery walls. Every time the frog jumps 30
(c) 4/5 (d) 5/6 cm, slides down 15 cm. what is the number of jumps
Sol. (d) required for the frog to come out of the well?
(a) 28 (b) 29
Out of all given fractions, the largest is 5/6 = 83.3%.
(c) 30 (d) 31
Rest all are smaller. So the largest will show the
Sol. (b)
minimum impact of this operation.
Total distance to be jumped by frog to come out =
57. A digit n > 3 is divisible by 3 but not divisible by 6. 4.5 m = 450 cm.
Which one of the following is divisible by 4?
Frog jumps 30 cm and slips 15 cm, which means
(a) 2n (b) 3n
that, in a jump he ascends 30 – 15 = 15 cm.
(c) 2n + 4 (d) 3n + 1
In 28 such jumps he will ascend to 28 × 15 = 420
Sol. (d)
cm.
A digit n > 3 is divisible by 3 but not divisible by
Now only 30 cm remain to come out, which the
6 is 9.
frog will jump in the next (29th) jump.
So n = 9. Of all given options 2n, 3n, 2n+4, and
In the last jump frog will not slip because he will
3n + 1, only the last one ( = 3×9 + 1 = 28 ) will be
be outside the well.
divisible by 4.
Directions for the following 6 (six) items: Read the
58. If 1 litre of water weighs 1 kg, then how many cubic
following five passages and answer the items that follow.
millimeters of water will weigh 0.1 gm?
Your answers to these items should be based on the
(a) 1 (b) 10
passages only.
(c) 100 (d) 1000
Sol. (c) Passage — 1
Given 1 litre = 1 kg. Spanish ships in the late 16th century first brought the
We know that 1 m3 (1 cubic metre) is exactly 1000 potato tuber from South America to Europe whereby in
Litres. the early 19th century, it bad become a reliable backup to
=>1000 litre = 109 cubic millimetres. cereal crops, particularly in the cold, rain-soaked soils of
(How? 1 m x 1 m x 1 m = 100 cm x 100 cm x 100 cm Ireland. The Irish were soon almost wholly dependent
= 1000 mm x 1000 mm x 1000 mm = 109 mm3) on the potato as their staple food. And they were planting
=>1 litre = 106 cubic millimetres = 1 kg = 1000 gm primarily one prodigious variety, the ‘Lumper’ potato,
whose genetic frailty would be cruelly exposed by the
So 0.1 gram = 102 mm3 = 100 cubic millimetres.
fungus ‘Phytophthora infestans’. In 1845, spores of the
59. A vessel full of water weighs 40 kg. If it is deadly fungus began spreading across the country,
one-third filled, its weight becomes 20 kg. What is destroying nearly all the Lumpers in its path. The
the weight of the empty vessel? resulting famine killed or displaced millions.

CSAT by Manjul Kumar Tiwari Sir


Previous Year Solved Papers 197
61. Which one or the following statements best reflects 4. The supply of safe drinking water and proper
the critical message or the passage? sanitation facilities to all should be ensured.
(a) For introducing any foreign plant into a 5. Authorities should ensure the vaccination as
country, the soil and climate conditions of that prescribed
country should be suitable.
(b) As a staple food of a country, tuber crops like Select the correct answer using the code given
potato cannot replace cereal crops. below.
(c) Some or the fungal infections or plants cannot (a) 1, 2, 3 and 4 (b) 2, 3, 4 and 5
be prevented or stopped from spreading (c) 1 only (d) 3 and 5 only
across large areas. Sol. (b)
(d) Relying on a homogeneous food source is not
desirable. Passage—3
Sol. (d) The pulse variety ‘Pusa Arhar 16’ has the potential to be
grown in the paddy-growing of Punjab, Haryana and
The passage does not talk about prevention of
fungal infections, so (c) is not correct. Passage does Uttar Pradesh and eventually in all of India. Its yield
not even talk about whether the tuber crops can (about 2000 kg/hectare) will be significantly greater than
replace the cereal crops or not, so (b) is not correct. those of the existing varieties and because its size will be
Passage gives the fundamental, critically important uniform, it will be amenable to mechanical harvesting,
message that “do not rely on a homogenous food an attractive feature for farmers in northern India who
source else a single disease can wipe out the currently use this technology for paddy. Most important,
entire food economy”. Arhar straw, unlike paddy straw, is green arid can be
ploughed back into the soil. In Paddy straw, the problem
Passage-2
is the high silica content, which does not allow for easy
India is at once among the fastest growing global decomposition. In the case of Arhar, the farmer, even after
economies and home to the largest number of combine harvesting, just needs to run a rotovator to cut
malnourished children in the world. There are regions the leftover straw into pieces, which can be ploughed
where malnutrition is not the exception but the norm. back and will decompose very fast. AU this is difficult
And across the country, malnutrition is the cause of with leftover paddy stalks that cannot be easily salvaged
death for roughly half the 1-3 million children who die or ploughed back. Farmers, therefore, choose the easiest
before their fifth birthday each year. Even those children option of simply burning it.
who survive suffer permanently from the damage that
has already been done to their bodies and minds from 63. Which of the following are the most rational
not getting enough of the right foods and nutrients. inferences that can be made from the passage?
Around 44 million children under 5 are stunted. That 1. Farmers’ income will be higher with pulse
makes it harder for them to learn in school and cultivation than with paddy cultivation.
subsequently earn a living as adults. Their lifetime 2. Pulse cultivation causes less pollution as
earnings potential is almost a quarter less than that or compared to paddy cultivation.
their healthy peers. 3. Pulse straw can be used to improve soil
quality.
62. With reference to the above passage, which of the
4. In the context of northern Indian agriculture,
following is/are the most rational and practical
paddy straw has no usefulness.
implication/ Implications?
5. Mechanized agriculture is the main cause for
1. India’s Public Distribution System should be
stubble burning.
monitored by the Union Government.
2. Girls should be encouraged to delay marriage Select the correct answer using the code given
and first pregnancy. below.
3. Mothers should be encouraged to breastfeed (a) 2, 3 and 5 (b) 1, 4 and 5
their children immediately after birth. (c) 2 and 3 only (d) 1 and 4 only

CSAT by Manjul Kumar Tiwari Sir


198 CSAT : 2020

Sol. (c) Passage-5


Statement 5 is not correct as it is mentioned that Economic liberalization in India was shaped largely by
paddy stalks are tough to collect or plough back the economic problems of the government than by the
into the ground, hence farmers choose to burn them economic priorities of the people or by the long-term
(so its not mechanized agriculture that’s the development objectives. Thus, there were limitations in
culprit). conception and design which have been subsequently
validated by experience. Jobless growth, persistent
So options (a) and (b) are ruled out.
poverty and rising inequality have mounted as problems
Now, statement 1 is not correct as we do not know since economic liberalization began. And all these years
really whether a direct comparison of pulse later, four quiet crises confront the economy: agriculture
cultivation and paddy cultivation is even possible. infrastructure, industrialization and education as
It’s not explicitly mentioned. constraints on the country’s future prospects. These
So options (b) and (d) are ruled out. problems must be resolved if economic growth has to be
sustained and transformed into meaningful
So finally we are left with option (c), which is 2
development.
and 3 only.
Passage-4 65. Which of the following is/are the most rational
and logical Inference/ Inferences that can be made
In India, authorities always look to store the maximum
from the passage?
amount of water in reservoirs during the monsoon
1. It is essential to rethink and redefine the
season, which is then used for irrigation and generation
economic role of the state in the quest for
of electricity during the summer months. It is an
development.
internationally accepted practice that the water level or 2. India has not made effective implementation
a reservoir should be kept below a certain level before the of its policies in social sectors nor made
onset of monsoon season. This is so that when monsoon sufficient investments in them.
rains come, there is space to store the excess rainwater
and also so that water can be released in a regulated Select the correct answer using the code given
manner. But the authorities store the maximum amount below.
of water in reservoirs even before the close of the monsoon, (a) 1 only (b) 2 only
only to ensure greater electricity generation and (c) Both 1 and 2 (d) Neither 1 nor 2
irrigation. Sol. (c)

64. With reference to the above passage, the following Theme is “Economic liberalization in India”. There
are two questions, so must be attempted.
assumptions have been made:
We are asked to find which on is a rational and
1. High risks involved in holding maximum
logical inference.
water in reservoirs are due to our over-
dependence on hydro power projects. Statement 1 speaks about rethinking the economic
role of State, in the quest for development. From
2. Storage capacity of dams should not be fully
the passage, it is clear that the Indian liberalization
used before or during monsoon season.
process was triggered by government’s economic
3. Role of dams in flood control is problems and not people’s problems, and that led
underestimated in India. to many design flaws, which are now visible in
Which of the above assumptions is/are valid? multiple shortcomings across sectors.

(a) 1 and 2 only (b) 2-only So, 1 does seem a rational inference. The State’s
economic role must be rethought and redefined if
(c) 3 only (d) 1, 2 and 3
we want proper development. Statement 2 is also
Sol. (d) reflected in passage.
All 3 statements are reflected in passage. So correct option is (c).

CSAT by Manjul Kumar Tiwari Sir


Previous Year Solved Papers 199
66. With reference to the above passage, the following discount of 10% it will be 80 – 8 (10% of 80) = 72.
assumptions have been made: And after services tax, it will become 72 – 7.2 (10%
1. India’s economy needs to be greatly integrated of 72) = = 64.8.
with global economy so as to create large Case 3 Let the price of the article = 100, after 10%
number of jobs and to sustain its growth service it will be of 90. After second discount of
momentum. 20% it will be 90 – 18 (20% of 90) = 72. And after
2. Economic liberalization would cause large the last discount of 10% it will become 72 – 7.2
economic growth which would reduce (10% of 72) = 64.8.
poverty and create sufficient employment in
68. The letters from A to Z are numbered from 1 to 26
the long run.
respectively. If GHI = 1578 and DEF = 912, then
Which of the above assumptions is/are valid?
what is ABC equal to?
(a) 1 only (b) 2 only
(a) 492 (b) 468
(c) Both 1 and 2 (d) Neither 1 nor 2 (c) 262 (d) 246
Sol. (d) Sol. (d)
A mere glance at both the statements 1 and 2 shows It is given that A to Z are numbered in same order
that both are not valid at all. as 1 to 26 (the word “respectively”).
The passage hasn’t talked about integration of Now, GHI must logically become 789. But we are
Indian economy with the global economy. It also given GHI = 1578.
hasn’t talked about potentially high growth being Note that 789 x 2 = 1578.
the driver of bringing economic liberalization in
Also given that DEF = 912. But 912 = 456 x 2 and
India (which the second statement is hinting at).
456 = DEF in the given coding scheme.
Hence 2 is also wrong.
So logic is “doubling of give number code”.
Hence answer is (d).
So correct answer would be ABC = 123 = 246.
67. A shop owner offers the following discount
69. What is the missing term in the following?
options on an article to a customer:
ACPQ : BESU:: MNGI: @
1. Successive discounts or 10% and 20%, and
(a) NPJL (b) NOJM
then pay a service tax of 10%
(c) NPIL (d) NPJM
2. Successive discounts of 20% and 10%, and
then pay a service tax of 10% Sol. (d)
3. Pay a service tax or 10% first, then successive ACPQ : BESU :: MNGI : @
discounts of 20% and 10% Here A + 1 = B; C + 2 = E; P + 3 = S; Q + 4 = U (So
Which one of the following is correct? ACPQ becomes BESU)
(a) 1 only is the best option for the customer.
Similarly, M + 1 = N; N + 2 = P; G + 3 = J; I + 4 = M.
(b) 2 only is the best option for the customer.
So the correct option is (d) as MNGI will become
(c) 3 only is the best option for the customer.
NPJM.
(d) All the options are equally good for the
customer. 70. What is the largest number among the following?
Sol. (d) (a) (1/2)–6 (b) (1/4)–3
All the options are equally good for the customer. (c) (1/3)–4 (d) (1/6)–2
Case 1 –Let the price of the article = 100, after first Sol. (c)
discount of 10% it will be of 90. After second (1/2)– 6 = 64;
discount of 20% it will be 90 – 18 (20% of 90) = 72. (1/4)– 3 = 64;
And after services tax, it will become 72 – 7.2 (10%
(1/3)– 4 = 81;
of 72) = 64.8.
Case 2 Let the price of the article = 100, after first (1/6)– 2 = 36;
discount of 20% it will be of 80. After second So the correct option is (c).

CSAT by Manjul Kumar Tiwari Sir


200 CSAT : 2020

Directions for the following 6 (six) items: Read the reducing emissions by 33 percent by 2030, and towards
following five passages and answer items that follow. this it has initiated a strong push towards a gas-based
Your answers to these items should be based on the economy and has also invested heavily in renewable
Passages only. energy. However, business houses are wary of investing
too heavily in renewable energy at a time when the
Passage — 1
technology is not yet ready.
Bank credit to the industrial sector has started shrinking.
Its decline has been a serious concern as credit growth is 72. Which one of the following is the most logical and
essential to revive investment. The problem’s Origins lie rational inference that can be made from the above
in the incomplete reforms of the last 25 years. An passage?
institutional change that should have followed the 1991 (a) India’s commitment to reduce emissions by
reforms should have been setting up of a resolution 33% is unlikely to be achieved.
corporation for banks. In a market economy with booms
(b) India should import gas rather than invest in
and busts, banks should be allowed to be set up and to
renewable resources.
fail. Today, we cannot shut down banks because there is
no proper system to shut them down. Weak loss-making (c) Getting renewable resources to market too
banks continue to need more capital. soon may be costly.
71. Which one of the following is the most logical and (d) India should put in more efforts in the
rational inference that can be made from the above exploration of natural gas.
passage?
Sol. (c)
(a) Indian banking system is not able to help the
country in its economic growth. The passage says that ‘India has potential for solar
(b) Economic reforms that started in 1991 have energy. We have decided to shift to renewable
not helped in improving the economy to sources of energy but they are costly and
expected levels. technologically not perfect. Indian government is
(c) India lacks the institutional mechanism to committed to reduce emission by 33% till 2030 and
deal with the failure of banks. for that it has made efforts in gas-based economy
(d) Encouraging the foreign investments in our and renewable energy fields. Business-houses are
industrial sector is a good alternative to this reluctant to invest in renewable energy sources
sector’s dependence on banks for credit. because of lack of technology.’
Sol. (c)
The passage does not provide comparison of gas
The passage is focused on the limitations of the over renewable sources, and definitely does not
way the Indian banking system is structured, as say that gas must be preferred over renewables. So
banks are not allowed to fail, and there is no formal (b) is clearly ruled out.
resolution process for that.
Similarly, option (d) sounds like a policy
Option (a) is too broad.
prescription that’s not intended, in the passage.
Option (b) is too generalized.
So between (a) and (c), it is a tough choice. We will
Option (c) is precise and the actual inference we
go with (c).
can draw.
Option (d) is irrelevant. 73. With reference to the above passage, the following
assumptions have been made:
Passage — 2
1. Governments often provide inefficient and
India has tremendous potential for solar energy. We all
costly subsidies for technologies that may not
realize that we have to stop burning fossil fuels to meet
be ready in the near future.
our energy needs. But certain renewable resources are
still going through their cost Curves and learning curves 2. India’s commitment of reducing emissions by
to get the required amount of output. The Indian 33% by 2030 shall be on the basis of gas-based
Government has strongly committed to its targets of economy.

CSAT by Manjul Kumar Tiwari Sir


Previous Year Solved Papers 201
Which of the above assumptions is/are valid? (c) Solid waste management should be
(a) 1 only (b) 2 only integrated with the maintenance of drainage
(c) Both 1 and 2 (d) Neither 1 nor 2 and sewerage networks.
Sol. (a) (d) Bad management of solid waste and
sewerage systems by our municipalities is the
74. With reference to the above passage, the following
reason for drinking water shortages in our
assumptions have been made:
cities.
1. Genome editing does not require the transfer
of genes from one plant to another. Sol. (c)
2. Through genome editing, the chosen genes The passage does not talk about ‘resources of
can be altered precisely in a manner akin to urban local bodies’ and ‘drinking water shortage’,
the natural process that helps plants to adapt so (b) and (d) are ruled out.
to the environmental factors. Between (a) and (c), option (c) is evident from ‘…
Which of the above assumptions is/are valid? when unscientific solid waste management
(a) 1 only (b) 2 only combines with poor drainage..’. So correct answer
(c) Both 1 and 2 (d) Neither 1 nor 2 is option (c).
Sol. (c) Passage — 5
Both the assumptions are valid. In Part III of the constitution which assures people certain
Statement 1 is evident from ‘Researchers use fundamental rights, Article 25 proclaims that “all persons
‘molecular scissors’ to dissect the genome and are equally entitled to freedom of conscience and the right
repair it..’. freely to profess, practice and propagate religion”. What
Statement 2 looks like the summary of the passage. people fail to notice is that this Proclamation is prefixed
It is correct. with the words “subject to public order, morality, health
Hence answer is (c). and to the other provisions of this Part”, Which set
conditions precedent for the legal protection of religious
Passage — 4 practices of any Community. The closing words of this
Many people understand the connection n solid waste prefatory rider in Article 25 virtually constitute a
management and health in terms of the consequences of subordination clause placing other fundamental rights
unattended heaps of garbage which become home for mentioned in Part Ill over and above the right to religious
flies and other Vermin. However, there is another aspect freedom. Among those other fundamental rights is the
that is not well-understood, that is, what happens when rights to equality before law and equal protection of laws-
unscientific solid waste management combines with poor assured at the outset and elaborated in later articles to
drainage and dumping of untreated sewage into drains mean, inter alia, that the State shall not deny equal
which are meant to carry storm water during rains. The protection of laws to any person or group of persons on
result is choked drains which are full of stagnant water the basis of religion alone.
breeding mosquitoes, resulting in the spread of water-
76. What is the most logical inference from the above
borne diseases.
passage?
75. In the context of India, which one of the following (a) State shall not interfere with the religious
statements best reflects the eritical message of the affairs of the Citizens.
passage? (b) Religious freedom under the Constitution is
(a) In India, the drainage networks are not open to State intervention
separate for sewerage and storm water. (c) Religious freedom of the citizens is not
(b) Urban local bodies do not have enough covered under fundamental rights.
resources and legislative authority to deal (d) Religious practices of any Community are
with the problems or waste management. immune to State laws.

CSAT by Manjul Kumar Tiwari Sir


202 CSAT : 2020

Sol. (b) So at the end of the 2nd operation the ratio of the
The passage wants to say that our constitution quantity of liquid A to the total quantity of liquid
provides the fundamental right of religious 2 2 2
 20  4   16   4  16
freedom with certain conditions. If it is harmful =      
 20   20   5  25
for the society in any way, the state can intervene.
Options (c) and (d) are both extreme and can be It means out of the total quantity of 25 units liquid
immediately rejected. A is 16 units and liquid B is 25 – 16 = 9 units.
Option (a) is not correct at all. Hence required ratio is 16 : 9.

77. How many different 5-letter words (with or 79. The average score of a batsman after his 50th
without meaning) can be constructed using all the innings was 46.4. After 60th innings, his average
letters of the word ‘DELHI’ so that each word has Score increases by 2.6. What was his average score
to start with D and end with I? in the last ten innings?
(a) 24 (b) 18 (a) 122 (b) 91
(c) 12 (d) 6 (c) 62 (d) 49
Sol. (d) Sol. (c)
D___I Average of 50 innings = 46.4.

We have three places and three letters E, H, L which Sum of 50 innings = 50 × 46.4 = 2320 …(i)
can be arranged in 3! = 6 ways. Average of 60 innings = 46.4 + 2.6 = 49

So the correct option is (d). This is a question from Sum of 60 innings = 60 × 49 = 2940 …(ii)
permutations and combinations. If you don’t know So runs scored in last ten innings = 2940 – 2320
the topic, you can try doing it manually as well = 620.
which may take some time. Average = 620/10 = 62. So the correct option is (c).

78. A bottle contains 20 litres of liquid A. 4 litres of 80. As a result of 25% hike in the price of rice per kg, a
liquid A is taken out of it d replaced by same person is able to purchase 6 kg less rice for Rs.
quantity of liquid B. Again 4 litre of the mixture is 1,200. What Was the Original price of rice per kg?
taken out and replaced by same quantity of liquid (a) Rs. 30 (b) Rs. 40
B. What is the ratio of quantity of liquid A to that of (c) Rs. 50 (d) Rs. 60
liquid B in the final mixture? Sol. (b)
(a) 4 : 1 (b) 5 : 1 Let the original price be P. So, 1.25 P is the new
(c) 16 : 9 (d) 17 : 8 price.
Sol. (c) Earlier quantity purchased = 1200 / P kg.
If y units of some other liquid is replaced in x units New quantity purchased = 1200 / 1.25P kg
of original liquid n times then the ratio of the
Difference = 6 kg = 1200/P – 1200/1.25P.
quantity of original liquid to the total quantity of
liquid after n operation = ((x-y)/x)n. => P = 40

Applying to our case x = 20, y = 4, n = 2. So, option (b) is correct.




CSAT by Manjul Kumar Tiwari Sir


Previous Year Solved Papers 203

Previous Year
CSAT : 2021 Solved Papers

Directions for the following 4 (four) items: Read the we are assuming that all men are not united by any
following four passages and answer the items that follow. Your shared common values or principles.
answer to these items should be based on the passages only. Option (d) is incorrect as people believe that they
Passage – 1 understand their own religion and on the basis of
this understanding only they tend to discard and
With respect to what are called denominations of religion, disapprove of the other denominations.
if everyone is left to be a judge of his own religion, there is
no such things as religion that is wrong; but if they are to Passage – 2
be a judge of each other’s religion, there is no such thing
It is certain, that seditions, wars, and contempt or breach
as a religion that is right, and therefore all the world is
of the laws are not so much to be imputed to the
right or all the world is wrong in the matter of religion.
wickedness of the subjects, as to the bad state of a
1. What is the most logical assumption that can be dominion. For men are not born fir for citizenship but
made from the passage given above? must be made so. Besides, men’s natural passions are
(a) No man can live without adhering to some everywhere the same; and if wickedness more prevails,
religious denomination. and more offences are committed in one commonwealth
(b) It is the duty of everyone to propagate one’s than in another, it is certain that the former has neither
religious denomination. enough pursued the end of unity, nor framed its laws
(c) Religious denominations tend to ignore the with sufficient forethought; and that, therefore, it has
unity of man. failed in making quite good its right as a commonwealth.
(d) Men do not understand their own religious
2. Which among the following is the most logical
denomination.
and rational inference that can be made from the
Sol. (c)
passage given above?
The passage tries to understand changes in (a) Seditions, wars, and breach of the laws are
meaning of religion changes based on the inevitable in every dominion.
perspective of the looker. By contrasting how we (b) It is not the people, but the sovereign who is
tend to see one’s religious denomination as the most responsible for all the problems of any
correct one. dominion.
Option (a) is incorrect: As the passage does not (c) That dominion is the best which pursues the
assume that an individual cannot survive without aim of unity and has laws for good
following any religious denomination. citizenship.
Option (b) is incorrect as the passage is concerned (d) It is impossible for men to establish a good
only with the interpretation of religion from different dominion.
perspectives but not with the propagation of Sol. (c)
religion. Option (a) is incorrect: The option is an assumption
Option (c) is correct as when we consider our that issues of sedition, wars, etc. are inevitable and
religion to be the correct one and all others as false, they should not be amputated to the wickedness of

CSAT by Manjul Kumar Tiwari Sir


204 CSAT : 2021

the subjects. So, it can not be an inference drawn Sol. (c)


based on the passage. Statement 1 is correct: Since the passage is talking
Option (b)is incorrect. As it lays the responsibility about the removal of inequalities, we can assume
for all the issues on the sovereign, which is not the that equalities are prerequisites for people to
most rational inference that can be drawn based on participate in social transactions and interactions
the information provided. with confidence.
Option (c) is correct: It has been mentioned that if Statement 4 is correct: The last lines of the passage
there are more crimes in dominion in comparison reflect the statement.
to another, it is due to laws which lack foresight, Hence, Option (c) is correct.
and the sovereign has not pursued unity. But if the
Passage – 4
State does so, it becomes the best State.
Option (d) is incorrect: This is not the most rational Aristocratic government ruins itself by limiting too
and logical inference as the ability of men or narrowly the circle within which power is confined;
oligarchic government ruins itself by the incautious
sovereign to establish good dominion is not being
scramble for immediate wealth. But even democracy ruins
discussed and cannot be inferred as well.
itself by excess of democracy. Its basic principle is the
Passage – 3 equal right of all to hold office and determine public
Inequality violates a basic democratic norm: the equal policy. This is, at first glance, a delightful arrangement;
it becomes disastrous because the people are not properly
standing of citizens. Equality is a relation that obtains
equipped by education to select the best rulers and the
between persons in respect of some fundamental
wisest courses. The people have no understanding and
characteristic that they share in common. Equality is,
only repeat what their rulers are pleased to tell them.
morally speaking, a default principle. Therefore, persons
Such a democracy is tyranny or autocracy. – Plato
should not be discriminated on grounds such as race,
caste, gender, ethnicity, disability, or class. These features 4. Which one of the following statements best reflects
of human condition are morally irrelevant. The idea that the crux of the passage given above?
one should treat persons with respect not only because (a) Human societies experiment with different
some of these persons some special features or talent, for forms of governments.
example skilled cricketers, gifted musicians, or literary
(b) Any form of government tends to deteriorate
giants, but because persons are human being, is by now by excess of its basic principle.
part of commonsense morality.
(c) Education of all citizens ensures a perfect,
3. With reference to the above passage, the following functional and sustainable democracy.
assumptions have been made: (d) Having a government is a necessary evil
1. Equality is a prerequisite for people to because tyranny is inherent in any form of
participate in the multiple transactions of government.
society from a position of confidence.
Sol. (b)
2. Occurrence of inequality is detrimental to the
survival of democracy. Option (a) is incorrect: Although this is reflected in
3. Equal standing of all citizens is an idea that the passage, it is not the most important or decisive
point or issue.
cannot actually be realised even in a
democracy. Option (b) is correct because two forms of
4. Right to equality should be incorporated government-oligarchies and democracy have been
into our values and day-to-day political discussed and it has been shown how they can turn
vocabulary. into tyrannies due to their excesses.
Which of the above assumptions are valid? Option (c) is incorrect because the author is not
(a) 1 and 2 only (b) 2 and 3 only advocating that democracy is more desirable and
(c) 1 and 4 only (d) 3 and 4 only not providing any recommendations to achieve it.

CSAT by Manjul Kumar Tiwari Sir


Previous Year Solved Papers 205
Although the author gives lack of proper education Using these 3 digits, we can make 3!, i.e. 6 three-
as one of the reasons as to why democracy does not digit numbers.
serve its intended purpose, it is not the crux of the So their sum, S = 369 + 396 + 639 + 693 + 936 + 963
passage. The reason being that such a conclusion = 3996
would ignore the first part of the discussion on
We can see that, S is divisible by both 74 and 9.
oligarchy.
Option (d) is incorrect because although it has been 7. There are two Classes A and B having 25 and 30
posited that any form of government can be students respectively. In Class-A the highest score
tyrannical, the author does not support the is 21 and lowest score is 17. In Class-B the highest
existence of any form of government. score is 30 and lowest score is 22. Four students
are shifted from Class-A to Class-B.
5. In a group of 120 persons, 80 are Indians and rest
Consider the following statements:
are foreigners. Further, 70 persons in the group
can speak English. The number of Indians who 1. The average score of Class-B will definitely
can speak English is decrease.

(a) 20 (b) 30 2. The average score of Class-A will definitely


increase.
(c) 30 or less (d) 30 or more
Which of the above statements is/are correct?
Sol. (d)
(a) 1 only (b) 2 only
Out of 120 persons, 80 are Indians and 40 are
(c) Both 1 and 2 (d) Neither 1 nor 2
foreigners.
Sol. (a)
Out of 120 persons, 70 can speak English, and the
rest cannot. Average = Sum of observations / Number of
observations
The maximum possible number of Indians who can
speak English is 70, i.e. if all the English-speaking However, we cannot be sure regarding the marks of
people are Indians. the 4 students that have been moved from class A.
They may have marks close to 21, which will lead
The minimum possible number of Indians who can
to decrease in the average marks of Class A. If they
speak English is 30, i.e. if all the foreigners speak
have marks close to 17, it will lead to increase in the
English.
average marks of Class A. Therefore, we cannot
So, English-speaking Indians will fall in the range
determine whether average of class A will increase
of 30 to 70.
or decrease. Statement 2 is incorrect.
6. Consider all 3-digit numbers (without repetition
8. Consider two Statements and a Question:
of digits) obtained using three non-zero digits
Statement-1: Priya is 4 ranks below Seema and is
which are multiples of 3. Let S be their sum.
31st from the bottom.
Which of the following is/are correct?
Statement-2: Ena is 2 ranks above Seema and is
1. S is always divisible by 74.
37th from the bottom.
2. S is always divisible by 9.
Question: What is Seema’s rank from the top in
Select the correct answer using the code given the class of 40 students?
below:
Which one of the following is correct in respect of
(a) 1 only (b) 2 only the Statements and the Question?
(c) Both 1 and 2 (d) Neither 1 nor 2 (a) Statement-1 alone is not sufficient to answer
Sol. (c) the Question
Three non-zero digits which are multiples of 3 are: (b) Statement-2 alone is not sufficient to answer
3, 6, and 9. the Question

CSAT by Manjul Kumar Tiwari Sir


206 CSAT : 2021

(c) Either Statement-1 alone or Statement-2 alone Sol. (c, d)


is sufficient to answer the Question From statement 1, we get:
(d) Both Statement-1 and Statement-2 are A & D > B, E & F
required to answer the Question As none of them is the heaviest, we can conclude
that C must be the heaviest. So, statement 1 alone is
Sol. (c)
sufficient.
From Statement 1, we get:
From Statement 2, we get:
C>A>D
Using statement 2 alone we cannot determine the
heaviest item.

10. In the English alphabet, the first 4 letters are written


in opposite order, and the next 4 letters are written
in opposite order and so on; and at the end Y and
Z are interchanged. Which will be the fourth letter
to the right of the 13th letter?
From Statement 2, we get: (a) N (b) T
(c) H (d) I
Sol. (b)
The English alphabet is:
ABCD EFGH IJKL MNOP QRST UVWX YZ
As per the question, the letters are arranged as
follows:
We know that, there are 40 students in the class.
DCBA HGFE LKJI PONM TSRQ XWVU ZY
Seema’s rank from the top can be calculated using
either of the statements. Hence, option (c) is correct. 13th letter in this arrangement is P.
And the 4th letter to its right is T.
9. Consider two Statements and a Question:
Statement-1: Each of A and D is heavier than each Directions for the following 4 (four) items : Read the
of B, E and F, but none of them is the heaviest. following four passages and answer the items that
follow. Your answers to these items should be based on
Statement-2: A is heavier than D, but is lighter
the passages only.
than C. Question: Who is the heaviest among A, B,
C, D and E? Passage – 1
India faces a challenging immediate future in energy and
Question: Who is the heaviest among A, B, C, D
climate policy-making. The problems are multiple :
and E?
sputtering fossil fuel production capability; limited access
Which one of the following is correct in respect of to electricity and modern cooking fuel for the poorest;
the Statements and the Question? rising fuel imports in an unstable global energy context;
(a) Statement-1 alone is sufficient to answer the continued electricity pricing and governance challenges
Question leading to its costly deficits or surplus supply; and not
least, growing environmental contestation around land,
(b) Statement-2 alone is sufficient to answer the
water and air. But all is not bleak: growing energy
Question
efficiency programmes; integrated urbanisation and
(c) Both Statement-1 and Statement-2 are
transport policy discussions; inroads to enhancing
required to answer the Question
energy access and security; and bold renewable energy
(d) Neither Statement-1 alone nor Statement-2 initiatives, even if not fully conceptualized, suggest the
alone is sufficient to answer the Question promise of transformation.

CSAT by Manjul Kumar Tiwari Sir


Previous Year Solved Papers 207
11. Which one of the following statements best reflects antibiotic growth promoters in the 1990s, the major pork
the critical message conveyed by the passage given exporters says it is producing more pigs – and the animals
above? get fewer diseases.
(a) India’s energy decision-making process is
12. Which one of the following statements best reflects
ever more complex and interconnected.
the critical message conveyed by the passage given
(b) India’s energy and climate policy is heavily
above ?
tuned to sustainable development goals.
(a) People should avoid consuming the products
(c) India’s energy and climate actions are not
of animal farming.
compatible with its broader social, economic
(b) Foods of animal origin should be replaced
and environmental goals.
with foods of plant origin.
(d) India’s energy decision-making process is
(c) Using antibiotics on animals should be
straightforward supply-oriented and ignores
banned.
the demand side.
(d) Antibiotics should only be used to treat
Sol. (a) diseases.
Option (a) is correct- As the passage talks about the Sol. (d)
challenges and prospects of energy and climate
Option (a) is incorrect- Passage suggests nothing
policy. The problems are multiple & the mention of
about what not to be consumed by the people.
myriad factors like sputtering fossil fuel production
Option (b) is incorrect- Passage doesn't talks about
capabilities, unstable global market, etc. illustrates
replacing animal origin foods with plant origin
how the energy decision-making process is complex
foods. It merely reflects the side effects of using
and interconnected.
antibiotics for other purposes than treating disease.
Option (b) is an incorrect-The statement only talks
Option (c) is incorrect-Passage doesn't talks about
about India’s energy policy tuned to sustainable
banning antibiotics on animals. It only mentions
development goals. It does not talk about the variety
the trend of minimizing the use of antibiotics on
of functions that are involved in energy and climate
animals given their ill effects on human health.
policymaking.
Option (d) is correct- Passage mentions the ill effects
Option (c) is an incorrect-This option only talks
of antibiotics as growth promoters and how some
about how climate actions are not compatible with
farming companies and also, Denmark -a major pork
social and economic goals. Thus, it does not reflect
exporter, banned antibiotic growth promoters but
the critical message of the passage which deals with
not its use as a drug to treat diseases.
the complexities of policymaking in the energy
sector and the few bright spots we have. Passage – 3
Option (d) is incorrect- As the passage considers Policy makers and media have placed the blame for
demand-side fluctuations as well by mentioning skyrocketing food prices on a variety of factors, including
rising fuel imports. In addition, it is very much high fuel prices, bad weather in key food producing
evident in the passage that the policy and decision- countries, and the diversion of land to non-food
making process is complex due to several factors production. Increased emphasis, however, has been
like import deficit, saturated production placed on a surge in demand for food from the most
capabilities, etc. populous emerging economies. It seems highly probable
that mass consumption in these countries could be well
Passage – 2
poised to create a food crisis.
There are reports that some of the antibiotics sold in the
market are fed to poultry and other livestock as growth 13. With reference to the above passage, the following
promoters. Overusing these substances can create assumptions have been made:
superbugs, pathogens that are resistant to multiple drugs 1. Oil producing countries are one of the reasons
and could be passed along humans. Mindful of that, some for high food prices.
farming companies have stopped using the drugs to make 2. If there is a food crisis in the world in the near
chickens gain weight faster. Since Denmark banned future, it will be in the emerging economies.

CSAT by Manjul Kumar Tiwari Sir


208 CSAT : 2021

Which of the above statements is/are valid? Statement 2 is incorrect- As the passage nowhere
(a) 1 only (b) 2 only mentions that rising GDP guarantees reasonable
(c) Both 1 and 2 (d) Neither 1 nor 2 distribution to all households.
Sol. (d) Hence, option (d) is correct.
Statement 1 is incorrect- The passage only mentions
that high fuel prices are leading to increasing food 15. Seven books P, Q, R, S, T, U and V are placed side
prices, it doesn't explicitly mention oil-producing by side. R, Q and T have blue covers and other
countries being one of the reasons for high fuel books have red covers. Only S and U are new books
prices. and the rest are old. P, R and S are law reports; the
Statement 2 is incorrect: The passage mentions that rest are Gazetteers. Books of old Gazetteers with
blue covers are
food crisis will be caused due to emerging
(a) Q and R (b) Q and U
economies’ tendency of mass consumption.
(c) Q and T (d) T and U
However, the passage only mentions emerging
economies as a causative agent rather than affected Sol. (c)
agent. The following table represents the information given
Hence, option (d) is correct. in the question:

Passage – 4
A central message of modern development economics is
the importance of income growth, by which is meant
growth in Gross Domestic Product (GDP). In theory,
rising GDP creates employment and investment
Since we have been asked about the books which
opportunities. As incomes grow in a country where the are old gazetteers with blue cover, therefore option
level of GDP was once low, households, communities, (c) is the correct answer.
and governments are increasingly able to set aside some
funds for the production of things that make for a good 16. Replace the incorrect term by the correct term in
life. Today GDP has assumed such a significant place in the given sequence 3, 2, 7, 4, 13, 10, 21, 18, 31, 28,
the development lexicon, that if someone mentions 43, 40 where odd terms and even terms follow the
“economic growth”, we know they mean growth in GDP. same pattern.
(a) 0 (b) 1
14. With reference to the above passage, the following (c) 3 (d) 6
assumptions have been made: Sol. (a)
1. Rising GDP is essential for a country to be a On separating the Odd and the Even Series, we
developed country. observe the following pattern:
2. Rising GDP guarantees a reasonable
ODD SERIES EVEN SERIES
distribution of income to all households.
3 2
Which of the above assumptions is/are valid?
3+4=7 0+4=4
(a) 1 only (b) 2 only 7 + 6 = 13 4 + 6 = 10
(c) Both 1 and 2 (d) Neither 1 nor 2 13 + 8 = 21 10 + 8 = 18
Sol. (d) 21 + 10 = 31 18 + 10 = 28
Statement 1 is incorrect- Passage mentions that in 31 + 12 = 43 28 + 12 = 40
theory, rising GDP creates employment and
On observing the given two series, we observe that
investment opportunities, and this, in turn, sets
the first term in EVEN SERIES should be replaced
aside funds necessary for the good life. Thus
by 0, therefore option (a) is the correct answer.
assuming only rising GDP cannot be said as
essential criteria for a developed country but 17. Following is a matrix of certain entries. The entries
income redistribution also plays a critical role as follow a certain trend row-wise. Choose the
mentioned in the passage. missing entry (?) accordingly.

CSAT by Manjul Kumar Tiwari Sir


Previous Year Solved Papers 209
Select the correct answer using the code given
below:
(a) 1 only
(b) 2 only
(a) 9B (b) 3A (c) Both 1 and 2
(c) 3B (d) 3C (d) Neither 1 nor 2
Sol. (c) Sol. (c)
From one row to another, the letters shift in a cyclic Distance between places A and B = 15 km
manner, i.e. BAC to CBA to ACB. As far as the Speed of X = 1.5 km/hr
numbers are concerned, they follow the following
So, Distance covered by X in 5 hours = 7.5 km
pattern.
Speed of Y in 1st hour = 1 km/hr
So, Distance covered by Y in 1st hour = 1 km
Similarly, Distance covered by Y in 2nd hour = 1.25
km
Distance covered by Y in 3rd hour = 1.5 km
So, the required alpha-numeric term is 3B. Distance covered by Y in 4th hour = 1.75 km

18. You are given two identical sequences in two rows: Distance covered by Y in 5th hour = 2 km
So, the total distance covered by Y in 5 hours = 1 +
1.25 + 1.5 + 1.75 + 2 = 7.5 km
Hence, both the given statements are true.
What is the entry in the place of C for the
Sequence-II? 20. A student appeared in 6 papers. The maximum
(a) 2.5 (b) 5 marks are the same for each paper. His marks in
(c) 9.375 (d) 32.8125 these papers are in the proportion 5 : 6 : 7 : 8 : 9 : 10.
Sol. (c) Overall he scored 60%. In how many number of
The inherent pattern in the two sequences has been papers did he score less than 60% of the maximum
represented below. marks?
(a) 2 (b) 3
Sequence I Sequence II
(c) 4 (d) 5
8 × 0.5 = 4 5 × 0.5 = 2.5 (A)
Sol. (b)
4 × 1.5 = 6 2.5 × 1.5 = 3.75 (B)
Let total marks in each subject be 100.
6 × 2.5 = 15 3.75 × 2.5 = 9.375 (C) Therefore, total marks for all 6 subjects = 600
15 × 3.5 = 52.5 Overall marks scored = 60% of 600 = 360
52.5 × 4.5 = 236.25 According to the question,
5x + 6x + 7x + 8x + 9x + 10x = 360
19. A person X from a place A and another person Y
or 45x = 360 or x = 8
from a place B set out at the same time to walk
So, marks in the given 6 subjects must be:
towards each other. The places are separated by a
5 × 8 = 40
distance of 15 km. X walks with a uniform speed
6 × 8 = 48
of 1.5 km/hr and Y walks with a uniform speed of
1 km/hr in the first hour, with a uniform speed of 7 × 8 = 56
1.25 km/hr in the second hour and with a uniform 8 × 8 = 64
speed of 1.5 km/hr in the third hour and so on. 9 × 8 = 72
Which of the following is/are correct? 10 × 8 = 80
1. They take 5 hours to meet. Hence, in 3 subjects the student has scored less than
2. They meet midway between A and B. 60% marks.

CSAT by Manjul Kumar Tiwari Sir


210 CSAT : 2021

Passage – 1 climatic conditions, it can be directly inferred that


Fig trees (genus Ficus) are considered sacred in India, fig trees can have a role in the seed dispersal of the
East Asia and Africa and are common in agricultural same. Hence, Statement 4 is correct.
and urban landscapes where other large trees are absent. Option (d) is correct- Based on the above deductions,
In natural forests, fig trees provide food for wildlife when this is the most appropriate answer.
other resources are scarce and support a high density
Passage – 2
and diversity of frugivores (fruit-eating animals). If
frugivorous birds and bats continue to visit to fig trees At the heart of agroecology is the idea that agroecosystems
located in sites with high human disturbances, sacred should mimic the biodiversity levels and functioning of
fig trees may promote frugivore abundance. Under natural ecosystems. Such agricultural mimics, like their
favourable microclimate, plenty of seedlings of other tree natural models, can be productive, pest-resistant,
species would grow around fig trees. nutrient conserving, and resilient to shocks and stresses.
In ecosystems there is no ‘waste’, nutrients are recycled
21. On the basis of the passage given above, the indefinitely. Agroecology aims at closing nutrient loops,
following assumption have been made: i.e., returning all nutrients that come out of the soil back
1. Fig trees can often be keystone species in to the soil such as through application of farmyard
natural forests. manure. It also harnesses natural processes to control
2. Fig trees can grow where other large woody pests and build soil fertility i.e., through intercropping.
species cannot grow. Agroecological practices include integrating trees with
3. Sacred trees can have a role in biodiversity livestock and crops.
conservation.
22. Consider the following:
4. Fig trees have a role in the seed dispersal of 1. Cover crops
other tree species. 2. Fertigation
Which of the above assumptions is/are valid? 3. Hydroponics
(a) 1 and 2 only (b) 3 only 4. Mixed farming
(c) 2 and 4 only (d) 1, 3 and 4 only 5. Polyculture
6. Vertical farming
Sol. (d)
Option (a) is incorrect- The passage mentions fig Which of the above farming practices can be
trees providing food for wildlife species and compatible with agroecology, as implied by the
supporting a density and diversity of frugivores passage?
where resources are scarce and absent. The passage (a) 1, 4 and 5 only
is without of any mention of large ‘woody’ species (b) 2, 3, 4 and 5 only
giving way for the growth of fig trees in their (c) 1, 2, 3 and 6 only
absence. Hence, statement 2 particularly of woody (d) 4 and 6 only
species can’t be inferred/ established from the Sol. (a)
passage. Option (a) is correct: The passage mentions that
Option (b) is incorrect- The passage mentions the farming practices compatible with agroecology
biodiversity prevailing around fig trees in the form include the integration of trees with livestock and
of frugivores, bats, animals, and humans living in crops. This boils down to practices of Cover
co-existence. Hence, the third statement holds as a cropping, Mixed farming, and Polyculture.
valid assumption. However, it is not the only valid These are all reflected in 1, 4, 5 statements.
assumption.
Passage – 3
Option (c) is incorrect- The passage mentions that
plenty of seedlings of other tree species are found to Computers increasingly deal not just with abstract data
grow around fig trees due to the visit of frugivorous like credit card details and databases, but also with the
birds and bats. Hence, under favorable micro- real world of physical objects and vulnerable human

CSAT by Manjul Kumar Tiwari Sir


Previous Year Solved Papers 211
bodies. A modern car is a computer on wheels; an brings down the rate of infectious diseases. But the entry
aeroplane is a computer on wings. The arrival of the of the market into this domain has created a false sense
“Internet of Things” will see computers baked into of security that gets conditioned and reinforced by the
everything from road signs and MRI scanners to onslaught of advertisements. Experience in Western
prosthetics and insulin pumps. There is little evidence Europe shows that along with personal hygiene, general
that these gadgets will be any more trustworthy than improvements in environmental conditions and
their desktop counterparts. Hackers have already proved components like clean water, sanitation and food security
that they can take remote control of internet connected have brought down infant/child death/infections rates
cars and pacemakers. considerably. The obsession with hand hygiene also
brings in the persisting influence of the market on
23. Which one of the following statements best reflects
personal health, overriding or marginalizing the negative
the most critical inference that can be made from
impact on ecology and the emergence of resistant germs.
the passage given above?
(a) Computers are not completely safe. 24. On the basis of the passage given above, the
(b) Companies producing the software do not following assumptions have been made:
take cyber security seriously. 1. People who are obsessed with personal
(c) Stringent data security laws are needed. hygiene tend to ignore the community
(d) The present trend of communication hygiene.
technologies will affect our lives in future. 2. Emergence of multi-drug resistant germs can
Sol. (a) be prevented by personal cleanliness.
3. Entry of the market in the domain of hygiene
Option (a) is correct:The computers though are
increases the risk of infectious diseases.
vulnerable to risks or threats of intrusion; the
4. Scientific and micro-level interventions are not
passage tries to establish the fact that they are not
sufficient to bring down the burden of
completely safe.
infectious diseases.
Option (b) is incorrect: Though the passage 5. It is community hygiene implemented
mentions security breaches by hackers but does not through public health measures that is
sufficiently establish the information that really effective in the battle against infectious
companies building software are not taking cyber diseases.
security seriously. Which of the above assumptions are valid?
Option (c) is incorrect: Due to the rising (a) 1 and 2 only
vulnerabilities and threats of data breaches in case (b) 3 and 4 only
of hackers taking over, the need for stringent data (c) 4 and 5 only
security laws can be indirectly inferred. However, (d) 1, 2 and 4 only
it is a fallout of the recent trends of communication
Sol. (c)
technologies and their complexities which is the
Option (a) is incorrect- Statement 1 is an extreme
more critical inference of the passage.
statement and the passage does not establish the
Option (d) is incorrect: Although option (d) seems
fact that the people obsessed with personal hygiene
correct it does not carry the apprehensive tone of tend to ignore community hygiene. Further, there is
the passage. no mention of multi-drug resistant germs rather
Hence Option (a) is correct. resistant germs which cannot be prevented from
just personal hygiene. Thus Statement 2 stands an
Passage – 4
invalid assumption.
A social and physical environment riddled with poverty, Option (b) is incorrect- It can be inferred from the
inequities, unhygienic and insanitary conditions passage that personal hygiene undoubtedly
generates the risk of infectious diseases. Hygiene has reduces the rate of infectious diseases. Further, the
different levels: personal, domestic and community entry of the market in this domain has created a
hygiene. There is no doubt that personal cleanliness false sense of security; it cannot be established that

CSAT by Manjul Kumar Tiwari Sir


212 CSAT : 2021

it increases the risk of infectious diseases per se. Case II :


Hence, statement 3 stands as an invalid
assumption.
Option (c) is correct- Experiences from western
Europe have established the direct role of
community hygiene through public measures
effective in reducing infectious diseases and death
rates. Hence Statement 5 stands a valid assumption.
Scientific and micro-level interventions referring to
personal and domestic hygiene not sufficient to Conclusion-I : From both diagrams, we can
bring down the burden of diseases can be directly conclude that “Some almirahs are tables” is
inferred from the essence of the passage. Thus defenitely true. Hence, Conclusion-I follows.
statement 4 stands a valid assumption.
Conclusion-II : From both diagrams, we can say
Option (d) is incorrect- Based on the above that “Some cats may not be chairs”.
deductions, statements 1 and 2 stand eliminated
Hence, Conclusion-II also follows.
leading to this option being incorrect though
statement 4 stands as a valid assumption. Hence, option (c) is the correct answer.

25. A Statement followed by Conclusion-I and 26. A boy plays with a ball, and he drops it from a
Conclusion-II is given below. You have to take the height of 1.5 m. Every time the ball hits the ground,
Statement to be true even if it seems to be at variance it bounces back to attain a height 4/5th of the
from the commonly known facts. Read all previous height. The ball does not bounce further
Conclusions and then decide which of the given if the previous height is less than 50 cm. What is
Conclusion(s) logically follows/follow from the the number of times the ball hits the ground before
Statement, disregarding the commonly known the ball stops bouncing?
facts. (a) 4 (b) 5
Statement: Some cats are almirahs. Some almirahs (c) 6 (d) 7
are chairs. All chairs are tables.
Sol. (b)
Conclusion-I: Certainly some almirahs are tables.
Given Height = 1.5 m = 1.5 × 100 cm = 150 cm
Conclusion-II: Some cats may not be chairs.
When ball hits the ground for the 1st time, its height
Which one of the following is correct?
= (4/5) × 150 = 120 cm When ball hits the ground
(a) Only Conclusion-I
for the 2nd time, its height = (4/5) × 120 = 96 cm
(b) Only Conclusion-II
When ball hits the ground for the 3rd time, its
(c) Both Conclusion-I and Conclusion-II
height = (4/5) × 96 = 76.8 cm
(d) Neither Conclusion-I nor Conclusion-II
When ball hist the ground for the 4th time, its
Sol. (c)
height = (4/5) × 76.8 = 61.44 cm When ball hits the
Based on the statements, we can have these
ground for the 5th time, its height = (4/5) × 61.44 =
scenarios:
49.15 cm
Case I :
Hence, option (b) is the correct answer.

27. Images of consonants of the English alphabet


(Capitals) are observed in a mirror. What is the
number of images of these which do not look like
their original shapes?
(a) 13 (b) 14
(c) 15 (d) 16

CSAT by Manjul Kumar Tiwari Sir


Previous Year Solved Papers 213
Sol. (b) These numbers have been listed below:
All the consonants of the english alphabet are: 703, 712, 721, 730, 802, 811, 820, 901 and 910
B, C, D, F, G, H, J, K , L, M, N, P, Q, R, S, T, V, W, X, Hence, there are 9 such integers in which the sum
Y, Z of the digits is 10. Hence, option (d) is the correct
So. there are a total of 21 consonants. answer.

The number of consonants whose images look like 30. A woman runs 12 km towards her North, then
them are: 6 km towards her South and then 8 km towards
H, M, T, V, W, X and Y her East. In which direction is she from her starting
Therefore, there are a total of 7 such consonants. point?
(a) An angle less than 45o South of East
Hence, the number of consonants whose images do
(b) An angle less than 45o North of East
not look like them = 21 – 7 = 14
(c) An angle more than 45o South of East
28. A bank employee drives 10 km towards South from (d) An angle more than 45o North of East
her house and turns to her left and drives another
Sol. (b)
20 km. She again turns left and drives 40 km, then
The path taken by the woman has been depicted
she turns to her right and drives for another 5 km.
below:
She again turns to her right and drives another
30 km to reach her bank where she works. What is
the shortest distance between her bank and her
house?
(a) 20 km (b) 25 km
(c) 30 km (d) 35 km
Sol. (b)
The route taken by the bank employee has been
depicted below:

We know that, the angle opposite to the larger side


is always greater.
Therefore, by looking at the diagram we can
conclude that the woman must be “At an angle less
than 45° North of East”.
Hence, option (b) is the correct answer.

Directions for the following 4 (four) items : Read the


From the diagram, the shortest distance between following four passages and answer the items that
follow. Your answers to these items should be based on
her house and her bank is 25 km.
the passage only.
Hence, option (b) is the correct answer.
Passage – 1
29. Integers are listed from 700 to 1000. In how many Researchers simulated street lighting on artificial
integers is the sum of the digits 10? grassland plots containing pea-aphids, sap-sucking
(a) 6 (b) 7 insects, at night. These were exposed to two different
(c) 8 (d) 9 types of light – a white light similar to newer commercial
Sol. (d) LED lights and an amber light similar to sodium street
We need to find out all the integers between 700 to lamps. The low intensity amber light was shown to
1000, in which sum of the digits is 10, e.g. 703  7 inhibit, rather than induce, flowering in a wild plant of
+ 0 + 3 = 10 the pea family which is a source of food for the pea-aphids

CSAT by Manjul Kumar Tiwari Sir


214 CSAT : 2021

in grasslands. The number of aphids was also as well as many plant-derived medicines. At least one-
significantly suppressed under the light treatment due third of the world’s agricultural crops depend upon
to the limited amount of food available. pollination. Bees are the most dominant taxa when it
comes to pollination and they are crucial to more than
31. Which one of the following statements best reflects
four hundred crops. Pollination is an essential service
the most critical inference that can be made from
that is the result of intricate relationship between plants
the passage given above?
and animals, and the reduction or less of either affects
(a) Low intensity light has more adverse effect
the survival of both. Effective pollination requires
on the plants as compared to high intensity
resources, such as refuges of pristine natural vegetation.
light.
(b) Light pollution can have a permanent adverse 32. On the basis of the passage given above, the
impact on an ecosystem. following assumptions have been made:
(c) White light is better for the flowering of plants 1. Sustainable production of India’s cereal food
as compared to the light of other colours. grains is impossible without the diversity of
(d) Proper intensity of light in an ecosystem pollinating animals.
is important not only for plants but for
2. Monoculture of horticultural crops hampers
animals too.
the survival of insects.
Sol. (b) 3. Pollinators become scarce in cultivated areas
Option (a) is incorrect: The passage mentions that devoid of natural vegetation.
low-intensity light inhibits the flowering of a 4. Diversity in insects induces diversity of
wildflower. However, it cannot be inferred that Low- plants.
intensity light has more adverse effects vis-vis High-
Which of the above assumptions is/are valid?
intensity light.
(a) 1 only (b) 2, 3 and 4 only
Option (b) is correct: It can be inferred from the
passage that light pollution can have permanent (c) 1 and 2 only (d) 3 and 4 only
adverse effects on the ecosystem. Sol. (d)
Option (c) is incorrect: The passage does not Statement 1 is incorrect: It is an extreme statement.
mention that white color is good for the flowering Though the diversity of pollinating animals is
of the plants. With this information, we cannot important for the Sustainable production of India’s
sufficiently infer that White light is better for the cereal food grains and can be a valid assumption
flowering of the plants vis-a-vis all the other lights from the passage the word impossible is making
of all the other colors. this statement incorrect.
Option (d) is incorrect: The passage mentions that Statement 2 is incorrect: It cannot be explicitly
low-intensity light inhibits the flowering of plants. assumed from the passage that the Monoculture of
Further, it highlights that the number of aphids was horticultural crops hampers the survival of insects.
also significantly suppressed under the light Also, the passage does not talk about hampering
treatment due to the limited amount of food agricultural diversity due to monoculture practices
available. However the main aim of the passage is leading to a reduction in pollinators.
comparison between different lights and their Statement 3 is correct: From the following lines of
effects. the passage “ Pollination is an essential service that
Hence, Option (b) is correct is the result of an intricate relationship between
plants and animals, and the reduction or loss of
Passage – 2 either affects the survival of both. Effective
Approximately 80 percent of all flowering plant species pollination requires resources, such as refuges of
are pollinated by animals, including birds and mammals, pristine natural vegetation” it can be assumed that
but the main pollinators are insects. Pollination is the pollinators become scarce in cultivated areas
responsible for providing us with a wide variety of food, devoid of natural vegetation.

CSAT by Manjul Kumar Tiwari Sir


Previous Year Solved Papers 215
Statement 4 is correct: From the following lines of “These climatic shifts will have an impact on the
the passage “ Pollination is an essential service that hydrological cycles in the region, lead to more run-
is the result of an intricate relationship between off and less recharge, and affect the groundwater
plants and animals, and the reduction or loss of tables.”, it can be stated that the climate changes
either affects the survival of both”, it can be assumed increase the criticality of water resources while
that diversity in insects induces diversity of plants. simultaneously threatening it is the main crux of
Hence Option (d) is correct. the passage.
Option (d) is incorrect: This is also not the crux of
Passage – 3
the passage, as it does not explicitly talk about
A study conducted on the impacts of climate change over adopting unsustainable livelihoods and risky
the Cauvery basin of Tamil Nadu using regional climate coping strategies by the farmers.
models showed an increasing trend for maximum and Hence Option (c) is correct.
minimum temperatures, and a decrease in the number of
Passage – 4
rainy days. These climatic shifts will have an impact on
the hydrological cycles in the region, lead to more run- Researchers were able to use stem cells to gauge the
neurotoxic effects of the environmental pollutant
off and less recharge, and affect the groundwater tables.
Bisphenol A (BPA). They used a combination of
Further, there has been an increase in the frequency of
biochemical and cell-based assays to examines the gene
droughts in the State. This has driven farmers to increase expression profile during the differentiation of mouse
dependency on groundwater resources to secure their embryonic stem cells upon treatment with BPA, a
crops. compound known to cause heart diseases, diabetes, and
developmental abnormalities in humans. They were able
33. Which one of the following statements best reflects
to detect and measure BPA toxicity towards the proper
the crux of the passage given above?
specification of primary germ layers, such as endoderm
(a) Development of regional climate models helps and ectoderm, and the establishment of neural
in choosing climate-smart agricultural progenitor cells.
practices.
(b) Heavy dependence on groundwater 34. On the basis of the passage given above, the
resources can be reduced by adopting dry- following assumptions have been made:
land cropping systems. 1. BPA may alter embryonic development in
(c) Climate changes increase the criticality of vivo.
water resources while simultaneously 2. Biochemical and cell-based assays are useful
in finding out treatments for pollution-
threatening it.
induced diseases.
(d) Climate changes cause the farmers to adopt
3. Embryonic stem cells could serve as a model
unsustainable livelihoods and risky coping
to evaluate the physiological effects of
strategies.
environmental pollutants.
Sol. (c)
Which of the above assumptions are valid?
Option (a) is incorrect: It is not the main crux of the (a) 1 and 2 only (b) 2 and 3 only
passage. The passage does not mention anything (c) 1 and 3 only (d) 1, 2 and 3
about climate-smart agricultural practices. Sol. (c)
Option (b) is incorrect: This is also not the main Statement 1 is correct: It can be assumed from the
crux of the passage. Though it mentions the heavy passage that the Bisphenol A (BPA). may alter
dependence on groundwater resources by the embryonic development in vivo from the line “
farmers due to the impact of climate change, it mouse embryonic stem cells upon treatment with
doesn’t talk about the use of sustainable agriculture BPA.
practices or adopting dry-land cropping systems. Statement 2 is incorrect: Though Biochemical and
Option (c) is correct: This is the main crux of the cell-based assays are useful in examining the gene
passage. From the following lines of the passage expression profile in embryonic stem cells upon

CSAT by Manjul Kumar Tiwari Sir


216 CSAT : 2021

treatment with Bisphenol A (BPA), it cannot be 37. From January 1, 2021, the price of petrol (in Rupees
explicitly assumed that they are useful in finding per litre) on mth day of the year is 80 + 0.1m, where
out treatments for pollution-induced diseases. m = 1, 2, 3, …, 100 and thereafter remains constant.
Statement 3 is correct: It can be assumed from the On the other hand, the price of diesel (in Rupees
passage that Embryonic stem cells could serve as a per litre) on nth day of 2021 is 69 + 0.15n for any n.
model to evaluate the physiological effects of On which date in the year 2021 are the prices of
environmental pollutants. these two fuels equal?
(a) 21st May (b) 20th May
Hence Option (c) is correct.
(c) 19th May (d) 18th May
35. If 32019 is divided by 10, then what is the remainder?
Sol. (b)
(a) 1 (b) 3
Price of the diesel on nth day of the year =
(c) 7 (d) 9
69 + 0.15n Price of the petrol on mth day of the year
Sol. (c)
= 80 + 0.1m Where m = 1 to 100. After which it
It’s given that: 32019 is divided by 10. Now,
remains constant.
31 = 3
4" Price of the petrol on and after 100th day =
32 = 9
80 + 0.1 × 100 = 80 + 10 = Rs. 90
33 = 27
Now, total number of days till 30th April = 31 + 28
34 = 81 + 31 + 30 = 120 days
35 = 243 20th May means 120 + 20 = 140 days
36 = 729 Price of the diesel = 69 + 0.15 × 140 = 90
Since, unit place of the power of 3 repeats after every
Therefore, we can say that on 20th May 2021, price
4 steps (i.e. it has a cyclicity of 4). Now, on dividing
of these two fuels will be equal.
2019 by 4 we get a remainder of 3.
Hence, option (b) is the correct answer.
Hence, 32019 will have the same last digit as that of
33, i.e. 7. 38. A biology class at high school predicted that a local
(33)/10 = 27/10 population of animals will double in size every
Hence, the remainder will be 7. 12 years. The population at the beginning of the
year 2021 was estimated to be 50 animals. If P
36. The number 3798125P369 is divisible by 7. What represents the population after n years, then which
is the value of the digit P? one of the following equations represents the model
(a) 1 (b) 6 of the class for the population?
(c) 7 (d) 9 (a) P = 12 + 50n (b) P = 50 + 12n
Sol. (b) (c) P = 50 (2)12n (d) P = 50 (2)n/12
Given: 3798125P369 is divisible by 7 Sol. (d)
Let’s express the given number in terms of triplets Population is getting doubled every 12 years, and
of digits, starting from the right, as follows. population in the year 2021 is 50 animals. So, after
(37) (981) (25P) (369) 12 years it will get doubled to 100 animals.
Now, 369 – 25P + 981 – 37 = 1350 – 25P – 37 = 1313 P = 50 × 2n/12 = 50 × 212/12 = 50 × 2 = 100 animals.
– 25P For all other options, this value comes out to be way
By dividing 1313 by 7 we get 4 as remainder. above 100.
25P in the above equation should be in between Hence, option (d) is the correct answer.
250 to 259. Now, 252 and 259 are divisible by 7. So,
39. In a class, 60% of students are from India and 50%
the number must be 252 + 4 = 256. Hence, value of
of the students are girls. If 30% of the Indian
P=6
students are girls, then what percentage of foreign
Hence, option (b) is the correct answer. students are boys?

CSAT by Manjul Kumar Tiwari Sir


Previous Year Solved Papers 217
(a) 45% (b) 40% By looking at the above diagrams, we can conclude
(c) 30% (d) 20% that conclusion-I and conclusion-II does not follow.

Sol. (d) Directions for the following 3 (three) items: Read the
Let total number of students in the class be 100. following two passages and answer the items that
Indian students = 60% of 100 = 60 follow. Your answers to these items should be based on
So, foreign students = 100 – 60 = 40 students the passages only.
Total number of girls students = 50% of 100 = 50
Passage-1
According to the question,
Medieval merchants risked the hazards of the Silk Road
Total number of Indian girl students = 30% of 60 =
to reach the markets of China; Portuguese caravels in the
18 students
15th century sailed beyond the bounds of the known
So, foreign girl students = 50 – 18 = 32
world, searching less for knowledge than for gold and
As total foreign students = 40
spices. Historically, the driver for opening frontiers has
So, foreign boy students = 40 – 32 = 8
always been the search for resources. Science and
So, percentage of boys among foreign students =
curiosity are weaker drivers. The only way to open up
(8/40) × 100 = 20%
space, whether the space of solar system or interstellar
Hence, option (d) is the correct answer.
space is to create an economic engine and that engine is
40. A Statement followed by Conclusion-I and resource extraction.
Conclusion-II is given below. You have to take the
41. Which one of the following statements best sums
Statement to be true even if it seems to be at variance
up the passage given above?
from the commonly known facts. Read all
(a) Wealth generation is the primary motive for
Conclusions and then decide which of the given
any human endeavour.
Conclusion(s) logically follows/follow from the
(b) Space, whether space in solar system or
Statement, disregarding the commonly known
interstellar space, will govern our future
facts.
economy.
Statement: Some radios are mobiles. All mobiles
(c) Human beings are motivated to explore
are computers. Some computers are watches.
new frontiers principally by economic
Conclusion-I: Certainly some radios are watches.
considerations.
Conclusion-II: Certainly some mobiles are
(d) Wealth generation is based on the risk-taking
watches.
behaviour of some men.
Which one of the following is correct?
(a) Only Conclusion-I Sol. (c)
(b) Only Conclusion-II Option (a) is incorrect: The passage talks about the
(c) Both Conclusion-I and Conclusion-II wealth generation as a strong motive for exploration
(d) Neither Conclusion-I nor Conclusion-II but not the primary motive for any endeavor. They
Sol. (d) can have other motives, like science and curiosity
though they are weak.
Based on the statements, we can have these
Option (b) is incorrect: The passage only talks about
scenarios:
the possibility of initial exploration based on the
economy. It does give any hint on the future.
Option (c) is correct: The passage talks about the
economic consideration as principal for exploration
like exploration for gold, spice, and Chinese market.
Option (d) is incorrect: The passage does talk about
explorations that involve risk, but it will be incorrect
to assume that wealth generation is always based
on risk-taking behavior. Wealth generation may or
may not involve risk-taking behavior.

CSAT by Manjul Kumar Tiwari Sir


218 CSAT : 2021

Passage-2 Option (c) is incorrect: The passage does not argue


for falsification of facts. It merely focuses on the
“........ most people would agree that telling deliberate lies
inclusion of lies in truth occasionally.
is wrong, except perhaps in certain special situations
Option (d) is incorrect: The passage does not argue
where more harm will be done by telling the truth. Even
for concealing the truth. The phrase “measure of
the most truthful people probably tell a good many more
falsehood” reflects that concealing truth will only
lies that might be regarded as semantic lies; their use of
be to some extent rather than complete concealment
words contains some measure of falsehood, more or less
of truth.
deliberate.”
44. A pie diagram shows the percentage distribution
42. The idea which the first part of the passage
of proteins, water and other dry elements in the
mention is
human body. Given that proteins correspond to
(a) agreement about telling lies.
16% and water corresponds to 70%. If both
(b) disagreement about telling lies.
proteins and the other dry elements correspond to
(c) disagreement about telling the truth.
p%, then what is the central angle of the sector
(d) disagreement about the harm in telling the
representing p on the pie diagram?
truth.
(a) 54° (b) 96°
Sol. (d) (c) 108° (d) 120°
Option (a) is incorrect: We can only imply option Sol. (c)
(a) hence it is not correct. Percentage of Other Dry Elements in the human
Option (b) is incorrect: The first part agrees on telling body = 100 – (Percentage of Proteins + Percentage
lies as telling the truth is not possible in every of Water) = 100 – (16 + 70) = 100 – 86 = 14%
situation. The following pie-chart represents the scenario
Option (c) is incorrect: Like option (b) the first option described in the question.
is more focused on the agreement.
Option (d) is correct: The passage talks about harm
in telling the truth.

43. Which one of the following habits is found more


often in good people?
(a) Mixing up the true and false
(b) Intentional mixing up of truth with the false.
(c) Falsification of facts
(d) Complete concealment of truth So, percentage of both Proteins and Other Dry
Elements, i.e. p = 16 + 14 = 30%
Sol. (b)
In a pie diagram, 100% corresponds to 360°.
Option (a) is correct: The passage talks about people
telling lies to reduce the harm from telling the truth. So, 30% will correspond to (360/100) × 30 = 108°
That’s why all people mix truth with false. That is, the central angle of the sector representing
However, it cannot be said in terms of good people p on the pie diagram = 108°
habits. 45. Joseph visits the club on every 5th day, Harsh visits
Option (b) is incorrect: The passage specifically on every 24th day, while Sumit visits on every 9th
highlights in the last line “their use of falsehood, day. If all three of them met at the club on a Sunday,
more or less deliberate”. Good people use lies more then on which day will all three of them meet
or less deliberately to reduce the harm from the truth. again?
Hence it’s their habit of intentionally mixing the (a) Monday (b) Wednesday
truth with false. Hence this option is correct. (c) Thursday (d) Sunday

CSAT by Manjul Kumar Tiwari Sir


Previous Year Solved Papers 219
Sol. (b) Sol. (b)
Joseph visits the club every 5th day. Present age of X is 42 years. Let the present age of Y
Harsh visits the club every 24th day. Sumit visits be y years. As per the question, 42 – y = 2y
the club every 9th day. Or 3y = 42
The next time they will meet again will be the LCM Or y = 14 years
of these time-periods.
So, at present the ages of X and Y are 42 and 14
LCM (5, 24, 9) = 360
respectively.
So, all the three will meet 360 days after Sunday.
Statement 1:
Now, we need not count 360 days. Rather we will
use the concept of odd days. Eight years ago, the ages of X and Y must have been
34 and 6 respectively.
Odd number of days in 360 = Remainder when 360
is divided by 7 = 3 So, they will meet again on We can see that the age of X was not 5 times the age
Sunday + 3 = Wednesday of Y. Hence, this statement is incorrect.
Statement 2:
46. The difference between a 2-digit number and the
After fourteen years, the ages of X and Y will be 56
number obtained by interchanging the positions
and 28 respectively.
of the digits is 54.
Consider the following statements: We can see that the age of X would indeed be two
1. The sum of the two digits of the number can be times the age of Y. Hence, this statement is correct.
determined only if the product of the two digits
48. If the price of an article is decreased by 20% and
is known.
then the new price is increased by 25%, then what
2. The difference between the two digits of the is the net change in the price?
number can be determined. (a) 0%
Which of the above statements is/are correct? (b) 5% increase
(a) 1 only (b) 2 only (c) 5% decrease
(c) Both 1 and 2 (d) Neither 1 nor 2 (d) Cannot be determined due to insufficient data
Sol. (b) Sol. (a)
Let the number be xy, i.e. 10x + y Let the initial price be Rs. 100
The number obtained on interchanging the New price on decreasing the original price by 20%
positions of the digits will be yx, i.e. 10y + x = 100 – 20% of 100 = 100 – 20 = Rs. 80
Difference between these two numbers = (10x + y) – Now, the final price on increasing the previous
(10y + x) = 9x – 9y = 9 (x – y) = 54 So, 9 (x – y) = 54 price by 25% = 80 + 25% of 80 = 80 + 20 = Rs. 100 So,
Or x – y = 6 there is no net change in price.
So, statement 2 is correct. 49. When a certain number is multiplied by 7, the
47. X said to Y, “At the time of your birth I was twice product entirely comprises ones only (1111...). What
as old as you are at present.” If the present age of X is the smallest such number?
(a) 15713 (b) 15723
is 42 years, then consider the following statements:
(c) 15783 (d) 15873
1. 8 years ago, the age of X was five times the age
of Y. Sol. (d)
2. After 14 years, the age of X would be two times As the number comprising of all 1’s is obtained on
the age of Y. multiplication by 7, so it means that 7 is the factor
Which of the above statements is/are correct? of that number.
(a) 1 only (b) 2 only Our answer will the smallest number comprising
(c) Both 1 and 2 (d) Neither 1 nor 2 of all 1’s that will be divisible by 7. So, let’s check.

CSAT by Manjul Kumar Tiwari Sir


220 CSAT : 2021

Is 1 divisible by 7? – No 51. With reference to the above passage, the following


Is 11 divisible by 7? – No assumptions have been made:
1. Modern democracies rely on the market forces
Is 111 divisible by 7? – No
to enable them to be welfare states.
Is 1111 divisible by 7? – No 2. Markets ensure sufficient economic growth
Is 11111 divisible by 7? – No necessary for democracies to be effective.
3. Government programmes are needed for those
Is 111111 divisible by 7? – Yes
left behind in economic growth.
So, 111111/7 = 15,873 Which of the above assumptions is/are valid ?
50. A man completes 7/8 of a job in 21 days. How (a) 1 and 3 only (b) 3 only
many more days will it take him to finish the job if (c) 2 and 3 only (d) 1, 2 and 3
quantum of work is further increased by 50% Sol. (b)
(a) 24 (b) 21 Statement 1 is incorrect: The passage mentions that
(c) 18 (d) 15 “there is a built-in tension between markets and
democracy.” The passage further goes on to
Sol. (d)
elaborate, how. Here “built-in” refers to the systemic
Initial units of work = 8 units aspects of each.
Out of which, 7 units are completed in 21 days. Statement 2 is incorrect: The last few lines of the
So, the number of days that man takes in completing passage invalidate the claim made in statement 2.
one unit of work = 21/7 = 3 days Now, amount of The passage mentions that “some people never get
new work = 8 + 50% of 8 = 8 + 4 = 12 units the opportunity to develop skills that markets
demand” and that “capitalism has always
Work remaining = 12 – 7 = 5 units
witnessed bursts of unemployment”. Therefore, we
So, the time taken by the man to complete the rest of can assume that democracies are not efficient if the
the work = 5 × 3 = 15 days market runs strong.
Directions for the following 2 (two) items: Read the Statement 3 is correct: As the markets cannot
following two passages and answer the items that ensure equity (as understood from the various lines
follow. Your answers to these items should be based on of the passage), government programs are needed
the passages only. to complement the market outcomes, to help those
who have not benefited by the “efficient distribution
Passage-1 of resources” of the markets.
Can a democracy avoid being a welfare state for long ? Hence, Option (b) is correct.
Why cannot mass welfare be left entirely to the markets ? Passage-2
There is a built-in tension between markets and
In our schools, we teach our children all that is there to
democracy. Markets do not work on a one- person-one- know about Physics, maths and history and what-have
vote principle as democracies do. What one gets out of you. But do we teach them about the bitter caste divide
the market place depends on one’s endowments, skills, that plagues the country, about the spectre of famine that
purchasing power and the forces of demand and supply. stalks large part of our land, about gender sensitivity,
Markets reward individual initiative and skill, and may about the possibility of atheism as a choice, etc.? Equally
also lift many from the bottom rungs of society, but some important, do we teach them to ask question, or do we
people never get the opportunity to develop skills that teach them only to passively receive our wisdom? From
the cocooned world of school, suddenly, the adolescent
markets demand; they are simply too poor and too
finds himself/herself in the unfettered world of
handicapped; or skill formation takes too long. By creating
university. Here he/she is swept up in a turmoil of ideas
jobs, markets may be able to help even unskilled people, and influences and ideologies. For someone who has
but capitalism has always witnessed bursts of been discouraged from asking questions and forming an
unemployment. opinion, this transition can be painful.

CSAT by Manjul Kumar Tiwari Sir


Previous Year Solved Papers 221
52. Which one of the following best reflects the central 53. There are three points P, Q and R on a straight line
idea of the passage given above ? such that PQ:QR = 3:5. If n is the number of possible
(a) School curriculum is not compatible with the values of PQ:PR, then what is n equal to ?
expectations of children and parents. (a) 1 (b) 2
(b) Emphasis on academic achievements does (c) 3 (d) 4
give time for development of personality and
skills. Sol. (b)
(c) Preparing the children to be better citizens We have to arrange three points P, Q, and R on a
should be the responsibility of the education straight line, such that PQ : QR = 3 : 5 Three points
system. can be arranged in 3!, i.e. 6 ways.
(d) To be a better citizen, the present world order
demands societal and life-coping skills in However, the number of possible values of PQ : PR
addition to academic content. will be only 2, i.e. n = 2. They have been represented
Sol. (d) below.

Option (a) is incorrect: The passage makes no


mention of the expectations of either parents or
children. Hence it is beyond the scope of the
passage.
Option (b) is incorrect: The time factor, i.e.,
allocation of time between academics and skill
development activities, is not discussed in the
passage.
Option (c) is close but incorrect: The passage does
not discuss the aspect of holistic education of 54. On a chess board, in how many different ways
children in terms of whose responsibility it should can 6 consecutive squares be chosen on the
be. Though it asserts that the schools do not fulfill diagonals along a straight path ?
this important role and it is important that these (a) 4 (b) 6
values and such education are not received (c) 8 (d) 12
straightaway at the time of college, but also at the
Sol. (b)
time of school, the author does not make any
conclusive judgments about where the ultimate On one diagonal, 6 consecutive squares can be
responsibility should lie. chosen in 3 ways.

Option (d) is the best answer: The passage explicitly


mentions, both the present world order demands,
as well as the societal and life-coping skills in
addition to academic content. The present world
order is mentioned in the lines, “But do we teach
them about the bitter caste divide that plagues the
country, about the specter of famine that stalks large
parts of our land, about gender sensitivity, about
the possibility of atheism as a choice, etc.?”. Societal
and life coping skills are discussed as “Equally
important, do we teach them to ask questions, or do
we teach them only to passively receive our
wisdom?”. Further, the author asserts “From the So, total number of ways of choosing 6 consecutive
cocooned world of school”, making the assertion squares on the diagonals along a straight path
made in the statement a valid one. =3+3=6

CSAT by Manjul Kumar Tiwari Sir


222 CSAT : 2021

55. In the series_b_a_ba_b_abab_aab; fill in the six Considering statement 2:


blanks (_) using one of the following given four We know that, 1 + 2 + 3 = 6
choices such that the series follows a specific order.
And 1 × 2 × 3 = 6
(a) bababa (b) baabba
So, Statement 2 is correct too.
(c) bbaabb (d) ababab
58. A cubical vessel of side 1 m is filled completely
Sol. (d) with water. How many millilitres of water is
The given series is: _b_a_ba_b_abab_aab contained in it (neglect thickness of the vessel) ?
The pattern being followed in the above series is: (a) 1000 (b) 10000
abb, then aab. Again abb, and then aab, and so on. (c) 100000 (d) 1000000
The blanks have been highlighted below:
Sol. (d)
a b b / a a b / a b b / a ab / ab b / aab
Volume of the cube = Side × Side × Side = 1 × 1 × 1
56. Using 2, 2, 3, 3, 3 as digits, how many distinct = 1 cubic meter Now, 1 cubic meter = 1000000
numbers greater than 30000 can be formed? mililiters
(a) 3 (b) 6 59. There are 6 persons arranged in a row. Another
(c) 9 (d) 12 person has to shake hands with 3 of them so that
Sol. (b) he should not shake hands with two consecutive
For the number to be greater than 30000, it must persons. In how many distinct possible
start with the digit 3. Also, as only 5 digits are given combinations can the handshakes take place?
to us, all must be used. (a) 3 (b) 4
3____ (c) 5 (d) 6
The 4 blanks have to be filled by two 2’s and two Sol. (b)
3’s. Number of ways to do so = 4!/(2! 2!) = 6
We just need to choose 3 persons out of 6, such that
These numbers are: 33322, 33232, 33223, 32332,
no two of them are together.
32323, and 32233.
Number of ways to choose 3 out of 6 people, without
57. Consider the following statements : any constraints = 6C3 = (6 × 5 × 4)/(3 × 2 × 1) = 20
1. The sum of 5 consecutive integers can be 100. Number of ways to choose 3 out of 6 people, such
2. The product of three consecutive natural that all of them are together = 4
numbers can be equal to their sum.
Number of ways to choose 3 out of 6 people, such
Which of the above statements is/are correct?
that two of them are together = 3 + 2 + 2 + 2 + 3 = 12
(a) 1 only
So, required answer = 20 – (4 + 12) = 4
(b) 2 only
60. An amount of money was distributed among A, B
(c) Both 1 and 2
and C in the ratio p : q : r. Consider the following
(d) Neither 1 nor 2
statements :
Sol. (c)
1. A gets the maximum share if p is greater than
Considering statement 1:
(q + r).
Let the 5 consecutive numbers be x-2, x-1, x, x+1,
2. C gets the minimum share if r is less than
x+2
(p + q).
Their sum = 5x = 100
Which of the above statements is/are correct?
Or x = 20
(a) 1 only (b) 2 only
So, the numbers are 18, 19, 20, 21, and 22. Statement
1 is correct. (c) Both 1 and 2 (d) Neither 1 nor 2

CSAT by Manjul Kumar Tiwari Sir


Previous Year Solved Papers 223
Sol. (a) Sol. (b)
Ratio of distribution of money among A, B and C is Option (a) is incorrect - The passage talks about the
p : q : r. Considering statement 1: luxury of universities that are choosing the online
If p > (q + r), then p is definitely the largest number. mode of education. However, the passage does not
talk about any inferiority of the conventional
So, A must have got the maximum share. Hence,
medium of pedagogy. Hence option (a) is incorrect.
statement 1 is correct. Considering statement 2:
Option (b) is correct - The passage talks about the
If r < (p + q), then r may or may not be the smallest
pricing of the education or problem of access to the
number. For example, 5 < (2 + 4)
education hence Option (b) is correct.
So, C may or may not have got the minimum share.
Option (c) is incorrect - It is not wholly represented
Hence, statement 2 is incorrect.
in the passage.
Directions for the following 2 (two) items : Read the Option (d) is incorrect- The passage does not specify
following two passages and answer the items that follow. Your public or private, universities or coaching
answers to these items should be based on the passages only. institutions. It only focuses on the betterment of the
Passage-1 education system with an online medium of
pedagogy.
The best universities like Harvard and MIT, despite
having the luxury of having some truly excellent teachers Passage-2
on their payroll, are increasingly embracing the “flipped Our cities are extremely vulnerable to climate change
classroom” format, where students listen to video lectures because of large concentrations of populations and poor
at home, and spend class time applying their knowledge, infrastructure. Moreover, population densities are
solving problems, discussing examples, etc. Professors increasing in them but we have not yet developed the
guide that discussion and fill in wherever necessary, systems to address climate change impacts. Our cities
explaining those bits that seem to be eluding the students contribute to 65 percent of the GDP, but there are not
and throwing in advanced ideas that happen to be enough facilities to cater to the needs of the people. It is
topical. These universities have made their video lectures important to address the issues of air quality, transport,
available free for anyone in the world. They are also etc., that are vital to identifying sustainable solutions.
encouraging colleges and universities all over the world We need to involve citizens in city planning and create
to integrate these online courses into their own pedagogy, an ecosystem that meets the needs of people.
picking the pieces that are appropriate for their needs
and building a package around them. 62. Which among the following is the most logical
and rational inference that can be made from the
61. Which one of the following statements best reflects
passage given above ?
the central idea of the passage given above?
(a) Our cities need to have well-defined
(a) Efficacy of universities would be better in
administrative set-up with sufficient
online mode of conducting classroom tuition
autonomy.
as compared conventional method.
(b) Availability of higher education can be made (b) Ever increasing population densities is a
easier and cheaper without diluting the hindrance in our efforts to achieve sustainable
content. development.
(c) We need not invest much in infrastructure (c) To maintain and develop our cities we need
related to higher education and yet develop to adopt sustainability related interventions.
better human and social capital. (d) Public-Private Partnership mode of
(d) Private sector institutions in higher education development is the viable long-term solution
as well as coaching institutes can take for the infrastructure and sustainability
advantage of this opportunity and thrive well. problems of India.

CSAT by Manjul Kumar Tiwari Sir


224 CSAT : 2021

Sol. (c) Sol. (a)


Option (a) is incorrect - The passage does not P scored 40 marks more than Q. So, marks of Q are
mention any administrative problem which is q, then the marks of P will be q + 40. Q scored 10%
caused due to lack of autonomy. The passage mostly less marks than P. That is, marks of Q are 90% of the
talks about the problem of climate change due to marks of P.
increasing population density and lack of So, q = 90% of (q + 40) Or 10q = 9q + 360
infrastructure. Hence option (a) is incorrect. Or q = 360
Option (b) is incorrect - Though the passage talks
So, Q scored 360 marks.
about the increasing population density, the core
aspect of the passage focuses on climate change, 65. A person P asks one of his three friends X as to
and in inference, we need to have a holistic how much money he had. X replied, “If Y gives me
perspective which is absent in option (b) hence it is Rs. 40, then Y will have half of as much as Z, but if
incorrect. Z gives me Rs. 40, then three of us will have equal
Option (c) is correct - The passage talks about the amount.” What is the total amount of money that
different problems for climate change caused due X, Y and Z have ?
to increasing population density and infrastructure. (a) Rs. 420 (b) Rs. 360
Hence to solve the problem we need to adopt (c) Rs. 300 (d) Rs. 270
sustainable intervention. hence the option (c) is
Sol. (b)
correct.
Option (d) is incorrect - Though the passage talks Let the amount of money with X, Y and Z be x, y
about the involvement of the public in decision and z. As per the question,
making the passage does not highlight the role of If Y gives me Rs. 40, then Y will have half of as
the private sector. Hence Public-private partnership much as Z.
can be not argued as an inference of the passage. That is, y – 40 = z/2
Hence option (d) is incorrect. Or z = 2y - 80 ...(i)
63. Jay and Vijay spent an equal amount of money to If Z gives me Rs. 40, then three of us will have equal
buy some pens and special pencils of the same amount.
quality from the same store. If Jay bought 3 pens That is, x + 40 = y = z – 40
and 5 pencils, and Vijay bought 2 pens and 7
So, x = y – 40
pencils, then which one of the following is correct?
(a) A pencil costs more than a pen. And z = y + 40 ...(ii)
(b) The price of a pencil is equal to that of a pen Using statements (i) and (ii), we get:
(c) The price of a pen is two times the price of a 2y – 80 = y + 40
pencil Or y = 120
(d) The price of a pen is three times the price of a
So, x = y – 40 = 120 – 40 = 80
pencil
And z = y + 40 = 120 + 40 = 160
Sol. (c)
So, money with X, Y and Z will be Rs. 80, Rs. 120,
Let the price of a pen and a pencil be Rs. a, and Rs.
b respectively. As per the question, and Rs. 160.

3a + 5b = 2a + 7b Or a = 2b Total amount of money with X, Y and Z = x + y + z


So, price of a pen is twice as that of a pencil. = 80 + 120 + 160 = Rs. 360

64. P scored 40 marks more than Q in an examination. 66. In a code language ‘MATHEMATICS’ is written
If Q scored 10% less marks than P, then how much as ‘LBSIDNZUHDR’. How is CHEMISTRY’
did Q score. written in that code language?
(a) 360 (b) 380 (a) DIDLHRSSX (b) BIDNHTSSX
(c) 400 (d) 420 (c) BIDLHTSSX (d) DGFLIRUQZ

CSAT by Manjul Kumar Tiwari Sir


Previous Year Solved Papers 225
Sol. (b) We know that, the student got 387 marks.
In a code language ‘MATHEMATICS’ is written as So, reduction from maximum marks = 450 – 387
‘LBSIDNZUHDR’. The pattern used in the above = 63 Hence, the number of incorrect responses
coding is as follows: = 63/7 = 9
M (13)  L (12), i.e. reduction of 1 69. Consider the following addition problem :
A (1)  B (2), i.e. increase of 1 3P+4P+PP+PP = RQ2; where P, Q and R are
T (20)  S (19), i.e. reduction of 1 different digits.

H (8)  I (9), i.e. increase of 1 What is the arithmetic mean of all such possible
sums?
And so on.
(a) 102 (b) 120
So, the code of CHEMISTRY will be: BIDNHTSSX (c) 202 (d) 220
67. At which one of the following times, do the hour Sol. (c)
hand and the minute hand of the clock make an 3P + 4P + PP + PP = RQ2
angle of 180° with each other? Or 30 + P + 40 + P + 10P + P + 10P + P = 100R + 10
(a) At 7:00 hours Q + 2 Or 24P + 70 = 100R + 10 Q + 2
(b) Between 7:00 hours and 7:05 hours Or 20P + 70 + 4P = 100R + 10 Q + 2
(c) At 7:05 hours
The unit digit of the resultant is 2. It will be obtained
(d) Between 7:05 hours and 7:10 hours
when 4 is multiplied by P. So, P must be 3, or 8. If P
Sol. (d)
= 3, then:
Angle of 180° means that they must be directly
24P + 70 = 24 × 3 + 70 = 72 + 70 = 142
opposite to each other.
If P = 8, then:
This will happen a little time after 7:05 hours.
24P + 70 = 24 × 8 + 70 = 192 + 70 = 262
Arithmetic sum of 142 and 262 = (142 + 262)/2 =
202

70. Consider the following multiplication problem :


(PQ) × 3 = RQQ, where P, Q and R are different
digits and R > 0, R  0. What is the value of
(P+R) ÷ Q?
(a) 1
68. In an objective type test of 90 questions, 5 marks (b) 2
are allotted for every correct answer and 2 marks (c) 5
are deducted for every wrong answer. After
(d) Cannot be determined due to insufficient data
attempting all the 90 questions, a student got a
total of 387 marks. What is the number of incorrect Sol. (b)
responses? PQ × 3 = RQQ
(a) 9 (b) 13 Or (10P + Q) × 3 = 100R + 10Q + Q
(c) 27 (d) 43
Or 30P + 3Q = 100R + 11Q
Sol. (a)
Or 30P = 100R + 8Q
Total number of questions are 90, and 5 marks are
given for every correct answer. So, maximum marks The last digit of 30P will be 0, as well as that of
possible = 90 × 5 = 450 100R.
Now, 2 marks are deducted for every wrong answer. So, the last digit of 8Q must also be 0. So, the value
Essentially, it means that every incorrect answer of Q must be 5.
will decrease the maximum score by 5 + 2 = 7 marks. Hence, 30P = 100R + 8Q = 100R + 40

CSAT by Manjul Kumar Tiwari Sir


226 CSAT : 2021

Or 3P = 10R + 4 what is good or bad, and each one holds himself


If R = 1, then P = 14/3 (not an integer) responsible to the state”, refers to a state, which is
civil, and not necessarily a “ruling authority”
If R = 2, then P = 24/3 = 8
per se.
If R = 3, then P = 34/3 (not an integer, and in double
Option (c) is incorrect: The passage does strongly
digits)
hint towards the fact that man is inherently selfish
So, P = 8, Q = 5, and R = 2 per se, but it does not refer to him being immoral.
That is, 85 × 3 = 255 Option (d) is incorrect: The passage mentions that
So, (P + R)/Q = (8 + 2)/5 = 10/5 = 2 “every man who is in natural state consults only
his advantage and determines what is good or bad
Directions for the following 4 (four) items : Read the
according to his fancy and insofar as he has regard
following four passages and answer the items that follow. Your
for his advantage alone and holds himself
answers to these items should be based on the passages only.
responsible to no one save himself by any law”.
Passage -1 However, it does not connect this trait to survival.
S/he connects it to the concept of sin and a civil
Nothing can exist in a natural state which can be called
state.
good or bad by common assent, since every man who is
in natural state consults only his own advantage, and Passage-2
determines what is good or bad according to his own In the immediate future, we will see the increasing
fancy and insofar as he has regard for his own advantage commodification of many new technologies – artificial
alone, and holds himself responsible to no one save intelligence and robotics, 3D manufacturing, custom
himself by any law; and therefore sin cannot be conceived made biological and pharmaceutical products, lethal
in a natural state, but only a civil state, which is decreed autonomous weapons and driverless cars. This will pose
by common consent what is good or bad, and each one conundrums. The moral question of how a driverless car
holds himself responsible to the state. will decide between hitting jaywalker and swerving and
damaging the car has often been debated. The answer is
71. Which one of the following statements best reflects
both simple- save the human life- and complex. At which
the central idea of the passage given above?
angle should the car swerve- just enough to save the
(a) The conceptions of what is right or wrong
jaywalker or more than enough? If the driverless car is in
exist due to the formation of a state.
Dublin, who would take the decision ? The Irish
(b) Unless a ruling authority decides as to what
Government, or the car’s original code writer in
is right or wrong, no man would be morally
California, or a software programmer in Hyderabad to
right.
whom maintenance is outsourced ? If different national
(c) Man is inherently immoral and selfish in a
jurisdictions have different fine print on prioritizing a
natural state.
human life, how will it affect insurance and investment
(d) The idea of what is right or wrong is necessary
decisions, including transnational ones?
for the survival of human species.
Sol. (a) 72. Which of the following statements best reflect
Option (a) is correct: That the conceptions of what the rational, plausible, and practical implications
is right or wrong exist due to the formation of a that can be derived from the passage given above?
state. is justified from the lines, “sin cannot be 1. Too much globalization is not in the best
conceived in a natural state, but only a civil state, interests of any country.
which is decreed by common consent what is good 2. Modern technologies are increasingly
or bad, and each one holds himself responsible to blurring the economic borders.
the state.” 3. Innovation and capital have impinged on the
Option (b) is incorrect. The author, in the lines, “sin domain of the state.
cannot be conceived in a natural state, but only a 4. Public policy of every country should focus
civil state, which is decreed by common consent on developing its own supply chains.

CSAT by Manjul Kumar Tiwari Sir


Previous Year Solved Papers 227
5. Geopolitics will have to reconcile to many to lend more and boost economic activity. But bad debt
ambiguities and uncertainties. resolution and recapitalization are only a part of the
solution as they, by themselves, can do very little to rein
Select the correct answer using the code given
in reckless lending that has pushed the Indian banking
below:
system to its current sorry state. Unless there are systemic
(a) 1, 4 and 5 only
reforms that address the problem of unsustainable
(b) 1, 2, 3 and 4 only
lending, future credit cycles will continue to stress the
(c) 2, 3 and 5 only
banking system.
(d) 1, 2, 3, 4 and 5
Sol. (c) 73. Which one of the following statements best reflects
Statement 1 is incorrect: The assertion made in this the most logical, rational and practical suggestion
statement goes beyond the scope of this passage. implied by the passage given above?
(a) Lending by the banks should be closely
The lines, “If the driverless car is in Dublin, who
monitored and regulated by the central
would make the decision? The Irish Government,
Government.
or the car’s original code writer in California, or a
(b) Interest rates should be kept low so as to
software programmer in Hyderabad to whom induce banks to lend more, promote credit
maintenance is outsourced? If different national growth and thereby boost economic activity.
jurisdictions have different fine print on prioritizing (c) Merger of many banks into a few large banks
human life, how will it affect insurance and alone in the long-term solution to make them
investment decisions, including transnational viable and prevent their bad performance.
ones? “, are reflected in statements (2) and (3). (d) Indian banking system requires structural
Statement 4 is incorrect: The author makes no such reforms as a long-term solution for bad loans
reference in the passage. The author only poses a problem.
conundrum without providing any solutions for it Sol. (d)
at all. Option (a) is incorrect: The author refers to the need
Statement 5 is correct: The entire passages refers to for systemic reforms for addressing the problem of
and discusses these ambiguities and uncertainties, unsustainable lending. Systemic reforms go much
such as, “If different national jurisdictions have beyond the measure of simple monitoring or
different fine print on prioritizing a human life, how regulation.
will it affect insurance and investment decisions, Option (b) is incorrect: The author states that
including transnational ones? “moral questions “Unless there are systemic reforms that address the
like “how a driverless car will decide between problem of unsustainable lending, future credit
hitting a jaywalker and swerving and damaging cycles will continue to stress the banking system.”
the car”, at which angle should the car swerve- just Now, changing interest rates is a short-term measure
enough to save the jaywalker or more than enough? and not a systemic improvement. Also, the author
Etc. mentions recapitalization of public sector banks
Hence Option (c) is correct. specifically, and not interest rate lowering, as a
measure for providing capital to banks to lend. So,
Passage-3 this measure goes beyond the scope of the passage.
The resolution of bankruptcy cases of Indian banks under Option (c) is incorrect: It is not mentioned or referred
the Insolvency and Bankruptcy Code should help bring to anywhere in the passage, and hence goes beyond
non-performing assets (NPA) situation under some its scope.
control. Despite the slow pace of resolution by the Option (d) is correct: The author clearly states the
National Company Law Tribunal, the Code can be need for the same in the lines, “Unless there are
helpful in cleaning up bank books in future credit cycles. systemic reforms that address the problem of
The recapitalization of public sector banks too can help unsustainable lending, future credit cycles will
increase the capital cushion of banks and induce them continue to stress the banking system.”

CSAT by Manjul Kumar Tiwari Sir


228 CSAT : 2021

Passage-4 75. Consider the following Table :


In India, the objective of macroeconomics policy is to
enhance the economic welfare of the people, and any
one wing of such macro policy, monetary or fiscal, cannot
independently work without active support of another.

74. Which one of the following statements best reflects


Who is the fastest run scorer in the Test Match ?
the corollary to the passage given above?
(a) A (b) B
(a) The central bank cannot work independently
(c) C (d) D
of the Government.
(b) Government should regulate financial Sol. (b)
markets and institutions closely. Fastest run scorer means the batsman that has the
(c) Market economy is not compatible with the best runs scored : balls faced ratio.
socialist policies of the Government.
(d) Financial sector reforms are required for
enhancing the economic welfare of the people.
Sol. (a) For batsman A:

Option (a) is correct: The passage states (read Runs scored : Balls faced = 75/175 = 0.43
concludes) that “anyone wing of such macro policy, For batsman B:
monetary or fiscal, cannot independently work Runs scored : Balls faced = 55/97 = 0.57
without the active support of another.” Now, the For batsman C:
Central Bank deals with the monetary policy, while
Runs scored : Balls faced = 35/125 = 0.28
the government looks after the fiscal policy of the
country. Therefore, the statement in option (a) flows For batsman D:
as a natural corollary to the conclusion provided in Runs scored : Balls faced = 25/105 = 0.24
the passage. The best ratio is that of batsman B.
Option (b) is incorrect: Though the statement might
76. Half of the villagers of a certain village have their
be right in its own sphere, but it is beyond the scope
own houses. One-fifth of the villagers cultivate
of this passage.
paddy. One-third of the villagers are literate. Four-
Option (c) is incorrect: Not only is the statement in fifth of the villagers are under 25 years of age.
this option beyond the scope of the passage, but it Which one of the following statements is certainly
is also incorrect in its own right and a very extreme correct?
one. India, which follows a mixed economy, is a (a) All the villagers who have their own houses
classic counterexample to the assertion made in this are literate.
option. (b) Some villagers under 25 years of age are
Option (d) is incorrect: To quote a standard literate.
definition, “The financial sector is a section of the (c) Only half of the villagers who cultivate paddy
economy made up of firms and institutions that are literate.
provide financial services to commercial and retail (d) No villager under 25 years of age has his own
customers. This sector comprises a broad range of house.
industries including banks, investment companies,
Sol. (b)
insurance companies, and real estate firms.” The
50% of the villagers have their own house.
passage does not discuss financial sector reforms.
Rather, it deals with the macroeconomic framework 20% of the villagers cultivate paddy.
of monetary and fiscal policies, of which, the 33.33% of the villagers are literate.
financial sector is just one (relatively) small aspect. 80% of the villagers are under 25 years of age.

CSAT by Manjul Kumar Tiwari Sir


Previous Year Solved Papers 229
Now, as 80% of the villagers are under 25 years of 10th October 2023 will be Monday + 1, i.e. Tuesday.
age, and 33.33% of the villagers are literate, so at 10th October 2024 will be Tuesday + 2, i.e. Thursday.
least some of the villagers under 25 years of age We added 2, as 2024 is a leap year.
must be literate.
10th October 2025 will be Thursday + 1, i.e. Friday.
77. Consider two Statements and a Question :
10th October 2026 will be Friday + 1, i.e. Saturday.
Statement-1 : The last day of the month is a
10th October 2027 will be Saturday + 1, i.e. Sunday.
Wednesday.
79. Consider two Statements and four Conclusions
Statement-2 : The third Saturday of the month was
given below. You have to take the Statements to be
the seventeenth day.
true even if they seem to be at variance from the
Question : What day is the fourteenth of the given commonly known facts. Read all Conclusions and
month? then decide which of the given Conclusion(s)
Which one of the following is correct in respect of logically follows/follow from the Statements,
the Statements and the Question ? disregarding the commonly known facts.
(a) Statement-1 alone is sufficient to the answer Statement-1 : Some greens are blues.
of the Question
Statements-2: Some blues are blacks.
(b) Statement-2 alone is sufficient to answer the
Conclusion-1 : Some greens are blacks.
Question
Conclusion-2 : No green is black.
(c) Both Statement-1 and Statement-2 are
required to answer the Question Conclusion-3 : All greens are blacks.

(d) Neither Statement-1 alone nor Statement-2 Conclusion-4 : All blacks are greens.
alone is sufficient to answer the Question Which one of the following is correct?
Sol. (b) (a) Conclusion-1 and Conclusion-2 only
Considering statement 1: (b) Conclusion-2 and Conclusion-3 only
The last day of the month is a Wednesday. However, (c) Conclusion-3 and Conclusion-4 only
we do not know the number of days in the month,
(d) Neither Conclusion 1 nor 2 nor 3 nor 4
so we cannot find the day on the 14th of that month.
Hence, statement 1 alone is not sufficient to answer Sol. (d)
the question. The statements can be represented in the form of
Considering statement 2: Venn diagrams, as follows:

17th of that month is the third Saturday. So, 14th of


that month must be a Wednesday.
Hence, statement 2 alone is sufficient to answer the
question.
78. Which day is 10th October, 2027 ?
(a) Sunday (b) Monday Conclusion 1: Some greens are blacks - Not
(c) Tuesday (d) Saturday necessary. So, this conclusion is incorrect.
Sol. (a) Conclusion 2: No green is black - Not necessary. So,
10th October 2021 is a Sunday. this conclusion is incorrect.
So, 10th October 2022 will be Sunday + 1, i.e. Conclusion 3: All greens are blacks - Not necessary.
Monday. So, this conclusion is incorrect.

CSAT by Manjul Kumar Tiwari Sir


230 CSAT : 2021

Conclusion 4: All blacks are greens - Not necessary. Sol. (c)


So, this conclusion is incorrect. The given sequence is 2, 7, 22, 67, 202, X, 1822 The
So, none of the conclusions is correct. pattern being followed here is given below:

80. What is the value of ‘X’ in the sequence 2, 7, 22, 67,


202, X, 1822?
(a) 603 (b) 605
(c) 607 (d) 608



CSAT by Manjul Kumar Tiwari Sir


Previous Year Solved Papers 231

Previous Year
CSAT : 2022 Solved Papers

Directions for the following 3 (three) items: Statement 2 is correct: Again, the author directly
Read the following two passage and answer the items states, “the increasingly evident unsustainability
that follow the passages. Your answers to these items of production and consumption patterns. Current
should be based on the passage only. production models rely heavily on fossil fuels. We
Passage-1 now know that this is unsustainable because the
The main threat to maintaining progress to human resources are finite.
development comes from the increasingly evident Statement 3 is incorrect: The passage states that
unsustainability of production and consumption “Some developed countries have begun to alleviate
patterns. Current production models rely heavily on fossil the worst effects by expanding recycling”. This
fuels. We now know that this is unsustainable because means that some countries are trying to reduce the
the resources are finite. The close link between economic effects of unsustainable production via recycling
growth and greenhouse gas emissions needs to be severed and other methods. Therefore, recycling would
for human development to become truly sustainable. reduce the unsustainability in production patterns
Some developed countries have begun to alleviate the rather than aggravate it.
worst effects by expanding recycling and investing in
2. Consider the following statements :
public transport and infrastructure. But most developing
countries are hampered by the high costs and low Developed countries can support developing
availability of clean energy sources. Developed countries countries’ transition to sustainable human
need to support developing countries’ transition of development by
sustainable human development. 1. making clean energy sources available at low
cost
1. Unsustainability in production pattern is due to 2. providing loans for improving their public
which of the following? transport at nominal interest rates
1. Heavy dependence on fossil fuels 3. encouraging them to change their production
2. Limited availability of resources and consumption patterns
3. Expansion of recycling
Select the correct answer using the code given Which of the statements given above is/are
below. correct?
(a) 1 and 2 only (b) 2 only (a) 1 only (b) 1 and 2 only
(c) 1 and 3 only (d) 1, 2 and 3 (c) 2 and 3 only (d) 1, 2 and 3
Sol. (a) Sol. (b)
Statement 1 is correct: The passage directly Statement 1 is correct: The author mentions high
mentions, “the increasingly evident costs and low availability of clean energy sources.
unsustainability of production and consumption Immediately after that, he states that “Developed
patterns. Current production models rely heavily countries need to support developing countries’
on fossil fuels.” Therefore, heavy dependence on transition to sustainable human development”.
fossil fuels is a reason, as per the author, for the Therefore, the first statement is correct and it directly
unsustainability of production patterns. follows the assertion made.

CSAT by Manjul Kumar Tiwari Sir


232 CSAT : 2022

Statement 2: Whether this statement is correct or Select the correct answer using the code given
incorrect depends upon whether we opt for a narrow below.
or broader interpretation of the passage. The (a) 1 and 2 only (b) 2 and 3 only
passage mentions, “But most developing countries (c) 1 and 3 only (d) 1, 2 and 3
are hampered by the high costs and low availability Sol. (c)
of clean energy sources.” Therefore, it seems that
Statement 1 is correct: "Pockets of desert-like
the author is only focussing on issues related to
landscape are now appearing in other parts of the
high costs and low availability of clean energy
country including the Sutlej-Ganga Plains and
sources as hindrances for developed countries in
Deccan Plateau. Where only a few decades back
transitioning towards sustainable human
there used to be lush green forests with perennial
development.
streams and springs, there is only brown earth, bare
However, (taking a broader view) the question does
of vegetation, without any water in the streams and
mention “can”- and providing loans for improving
springs except in the rainy season.” Supports the
their public transport at nominal interest rates is
assertion of the author that deforestation and
certainly one way where developed countries can
denudation will ultimately lead to the depletion of
help developing countries especially to alleviate the
soil resources.
worst effects of unsustainable development.
Statement 2 is incorrect: This statement goes
Statement 3: The option although mention in the
beyond the scope of the passage. The passage
passage is not discussed with respect to developing
countries. Their by strictly bounding ourself to the doesnot mention anything related to the common
passage statement-3 can be considered incorrect. man.

Hence, option (b) is the answer. Statement 3 is correct: This is supported by the lines
"the harshness of the climatic conditions” and
Passage-2 “Where only a few decades back there used to be
Unless the forces and tendencies which are responsible lush green forests with perennial streams and
for destroying the country’s environment are checked in springs, there is only brown earth, bare of
the near future and afforestation of denuded areas is vegetation, without any water in the streams and
taken up on a massive scale, the harshness of the climatic springs except in the rainy season”.
conditions and soil erosion by wind and water will
4. What is the value of X in the sequence
increase to such an extent that agriculture, which is the
mainstay of our people, will gradually become 20, 10, 10, 15, 30, 75, X?
impossible. The desert countries of the world and our (a) 105 (b) 120
own desert area in Rajasthan are a grim reminder of the (c) 150 (d) 225
consequences of large-scale deforestation. Pockets of Sol. (d)
desert-like landscape are now appearing in other parts
The given series is:
of the country including the Sutlej-Ganga Plans and the
20, 10, 10, 15, 30, 75, X?
Deccan Plateau. Where only a few decades back there
used to be lush green forests with perennial streams and The terms are decreasing in the initial half, and then
springs, there is only brown earth, bare of vegetation, they start increasing. The speed at which they
without any water in the streams and springs except in increase at the latter half suggest that multiplication
the rainy season. may be involved.
The pattern is as follows: 20 × 0.5 = 10
3. According to passage given above, deforestation
10 × 1 = 10
and denudation will ultimately lead to which of
10 × 1.5 = 15
the following?
1. Depletion of soil resource 15 × 2 = 30
2. Shortage of land for the common man 30 × 2.5 = 75
3. Lack of water for cultivation 75 × 3 = 225

CSAT by Manjul Kumar Tiwari Sir


Previous Year Solved Papers 233
5. An identity Card has the number ABCDEFG, not On observing the options, we can see that C must
necessarily in that order, where each letter be 1 and D must be 2. So, C + D + E = 1 + 2 + 5 = 8.
represents a distinct digit (1, 2, 4, 5, 7, 8, 9 only).
6. Two friends X and Y start running and they run
The number is divisible by 9. After deleting the
together for 50 m in the same direction and reach a
first digit from the right, the resulting number is
point. X turns right and runs 60 m, while Y turns
divisible by 6. After deleting two digits from the
left and runs 40 m. Then X turns left and runs 50 m
right of original number, the resulting number is
and stops, while Y turns right and runs 50 m and
divisible by 5. After deleting three digits from the
then stop. How far are the two friends from each
right of original number, the resulting number is
other now?
divisible by 4. After deleting four digits from the
(a) 100 m (b) 90 m
right of original number, the resulting number is
(c) 60m (d) 50 m
divisible by 3. After deleting five digits from the
right of original number, the resulting number is Sol. (a)
divisible by 2. Which of the following is a possible The path taken by them has been represented below:
value for the sum of the middle three digits of the
number? Y - 50m X - 50m

(a) 8 (b) 9
Y - 40m X - 60m E
(c) 11 (d) 12
Sol. (a) 50m
W N

The number has 7 digits, and has been denoted by: S

ABCDEFG It’s pretty clear that they are 40 + 60 = 100 m apart


These letters can be replaced by 1, 2, 4, 5, 7, 8, 9, not at the end of their run.
necessarily in the same order. We have to find the
7. Which date of June 2099 among the following is
possible value of C + D + E
Sunday?
The original number (ABCDEFG) is divisible by 9.
(a) 4 (b) 5
It has to be as 1 + 2 + 4 + 5 + 7 + 8 + 9 = 36, which is
(c) 6 (d) 7
divisible by 9. This information is utterly useless.
Sol. (d)
After deleting 1 digit from the right, the resulting
Considering 5th June 2022 as the reference date
number (ABCDEF) is divisible by 6. It means that,
which was a Sunday.
F = 2, 4 or 8 (i.e. an even number).
Difference between both the years = 2099 – 2022 =
Also, if even after removing G, the remaining number
77 years
is divisible by 3, then it means G = 9.
Number of leap years in between = 77 /4 = 19 (we
After deleting 3 digits from the right, the resulting
will ignore the remainder 1) Hence, till 2099 we
number (ABCD) is divisible by 4. It means that, D
will have 58 normal years and 19 leap years.
= 2, 4 or 8 (i.e. an even number).
Number of odd days = (58 × 1) + (19 × 2) = 58 + 38 =
After deleting 5 digits from the right, the resulting
96 96/7 gives 5 as the remainder.
number (AB) is divisible by 2. It means that, B = 2, 4
or 8 (i.e. an even number). So, 5th June 2099 will be Sunday + 5 = Friday And,
so 7th June 2099 will be a Sunday.
So, F, D and B are even numbers (2, 4 or 8). And, A,
C, E, and G are odd numbers (1, 5, 7 or 9). 8. A bill for 1,840 is paid in the denominations of
After deleting 2 digits from the right, the resulting 50, 20 and 10 notes. 50 notes in all are used.
number (ABCDE) is divisible by 5. It means that, Consider the following statements:
E = 5. 1. 25 notes of 50 are used and the remaining
So, we just have to find C + D + E = C + D + 5, which are in the denominations of 20 and 10.
must be an even number as C is odd (1, or 7), and D 2. 35 notes of 20 are used and the remaining
is even (2, 4, or 8). are in the denominations of 50 and 10.

CSAT by Manjul Kumar Tiwari Sir


234 CSAT : 2022

3. 20 notes of 10 are used and the remaining 10. The digits 1 to 9 are arranged in three rows in such
are in the denominations of 50 and 20. a way that each row contains three digits, and the
Which of the above statements are not correct? number formed in the first row; and the number
(a) 1 and 2 only (b) 2 and 3 only formed in the third row is thrice the number formed
(c) 1 and 3 only (d) 1, 2 and 3 in the first row. Repetition of digits is not allowed.
Sol. (d) If only three of the four digits 2, 3, 7 and 9 are
allowed to use in the first row, how many such
Total amount = Rs. 1840
combinations are possible to be arranged in the
Let the number of Rs. 50, Rs. 20, and Rs. 10 notes be three rows?
a, b and c respectively. Now, let’s check the (a) 4 (b) 3
statements. (c) 2 (d) 1
Statement 1: 25 notes of Rs. 50 were used. So, Sol. (c)
remaining amount = 1840 – (25 × 50) = 1840 – 1250 We can only use three of the four digits – 2, 3, 7, and
= Rs. 590. 9, in the first row.
Even if all the remaining 25 notes are of Rs. 20 The first digit in the first row cannot be 7 or 9, as
denomination, we will only get Rs. 500. So, otherwise thrice the number will not be a three-digit
Statement 1 is definitely incorrect. number.
Statement 2: 35 notes of Rs. 20 were used. So, So, the first digit in the first row can either be 2, or 3.
remaining amount = 1840 – (35 × 20) = 1840 – 700 The possible cases are: 237, 273, 239, 293, 279, 297,
= Rs. 1140. 327, 372, 329, 392, 379, or 397.
Even if all the remaining 15 notes are of Rs. 50 On eliminating the numbers whose 3x is not a three-
denomination, we will only get Rs. 750. So, digit number, we are left with: 237, 273, 239, 293,
Statement 2 is definitely incorrect. 279, 297, 327, and 329.

Statement 3: 20 notes of Rs. 10 were used. So, We will check these numbers:
remaining amount = 1840 – (20 × 10) = 1840 – 200 = 237 × 2 =474 (digit repetition, and so eliminated)
Rs. 1640. 273 × 2 = 546; 273 × 3 = 819
Even if all the remaining 30 notes are of Rs. 50 239 × 2 = 478; 239 × 3 = 717 (digit repetition, and so
denomination, we will only get Rs. 1500. So, eliminated)
Statement 3 is definitely incorrect.
293 × 2 = 586; 293 × 3 = 879 (digit repetition, and so
9. Which number amongst 240, 321, 418 and 812 is the eliminated) 279 × 2 = 558 (digit repetition, and so
smallest? eliminated)
(a) 2 40 (b) 3 21 297 × 2 = 594 (digit repetition, and so eliminated)
(c) 4 18 (d) 8 12 327 × 2 = 654; 327 × 3 = 981
Sol. (b) 329× 2 = 658; 329× 3 = 987 (digit repetition, and so
eliminated) So, only two cases are possible.
The given numbers are: 240, 321, 418, and 812.
We can also write them as: 240, 321, 236, and 236. (as Directions for the following 4 (four) items:
4 = 22, and 8 = 23) Read the following two passages and answer the items
that follow the passages. Your answer to these items
So, we basically need to find the smallest one from
should be based on passage only.
among 236, and 321. As we cannot have two correct
answers, it must be 321. Passage-1
We can rewrite 236 and 321 as: 212 and 37 In simple matters like shoe-making, we think only a
special trained person will serve out purpose, but in
4096 > 2187
politics, we presume that everyone who knows how to
Hence, 321 is the smallest number. get votes knows how to administer a State. When we

CSAT by Manjul Kumar Tiwari Sir


Previous Year Solved Papers 235
are ill, we call for a trained physician, whose degree is poor’ and the debatable methodology used to count the
a guarantee of specific preparation and technical poor, but also because of a more fundamental
competence-we do not ask for the handsomest assumption underlying it. It exclusively relies on the
physician, or the most eloquent one : well then, the notion of poverty as insufficient income or insufficient
whole State is ill should we not look for the service and purchasing power. One can better categorize it by
guidance of the wisest and the best?” calling it income poverty. If poverty is ultimately about
deprivations affecting human well-being, then income
11. Which one of the following statements best reflects
poverty is only one aspect of it. Poverty of a life, in our
the message of the author of the passage?
view, lies not merely in the impoverished state in which
(a) We assume that in a democracy, any politician
the person actually lives, but also in the lack of real
is qualified to administer a State.
opportunity given by social constraints as well as
(b) Politicians should be selected from those
personal circumstances - to choose other types of living.
trained in administration.
Even the relevance of low incomes, meagre possessions,
(c) We need to devise a method of barring
and other aspects of what are standardly seen as
incompetence from public office.
economic poverty relate ultimately to their role in
(d) As voters select their administrators, the curtailing capabilities, i.e., their role in severely
eligibility of politicians to administer a State restricting the choices people have to lead variable and
cannot be questioned. valued lives.
Sol. (c)
12. Why is the methodology adopted in India to count
Option (a) is incorrect: While this option is close
the ‘poor’ debatable?
and might seem correct, the author clearly states
(a) There is some confusion regarding what
that “but in politics, we presume that everyone who
should constitute the ‘poverty line’/
knows how to get votes knows how to administer a
State”. He clearly specifies that the public/ voters (b) There are wide diversities in the condition of
presume that the person elected “knows how to the rural and urban poor.
administer a state” but not necessarily an (c) There is no uniform global standard for
assumption of qualifications. measuring income poverty.
Option (b) is incorrect: In each of the lines, (s)he (d) It is based on the proposition of poverty as
makes a bid to focus on the skill sets of the meagre income or buying capacity.
candidates standing for elections while voting. Sol. (d)
Option (c) is correct: This is a valid message being Option (a) is incorrect: There is no “confusion”
conveyed by the author throughout the passage. regarding the methodology or classification- the
Option (d) is incorrect: The author is clearly calling author does not mention anything as a confusion.
into question the eligibility of politicians and is He only states that the current methods of
urging the voters to vote for those who are estimating poverty are narrow, i.e. inadequate. He
specialised and trained in the sphere of tries to convey that poverty goes beyond income
administration. This is evident from the lines, “but and purchasing power- it is about restricted choices
in politics, we presume that everyone who knows and lack of real opportunity.
how to get votes knows how to administer a State.”, Option (b) is incorrect: The assertion made in this
“well then, when the whole State is ill should we option goes beyond the scope of the passage. In fact,
not look for the service and guidance of the wisest there is no talk of the rural or urban poor in the
and the best?” passage.
Passage-2 Option (c) is incorrect: This option also goes beyond
The poverty line is quite unsatisfactory when it comes the scope of the passage. No line in the passage
to grasping the extent of poverty in India. It is not only leads to the assumption that “There is no uniform
because of its extremely narrow definition of ‘who is global standard for measuring income poverty.”

CSAT by Manjul Kumar Tiwari Sir


236 CSAT : 2022

Option (d) is correct: This is the main message being Sol. (a)
stressed by the author, who explains through the Option (a) is correct: This option is the best answer.
various lines of the passage that measuring poverty Although the phrase, “all deprivations” might seem
through the lens of income or purchasing power extreme, the author does make a case for
(buying capacity) is only a narrow way of looking deprivations stemming from lack of choices (real
at poverty. opportunities), rather than solely income.
13. Why is income poverty only one measure of Option (b) is incorrect: The option goes beyond the
counting the ‘poor’? scope of the passage as there is no mention of rural
(a) It talks of only one kind of deprivation versus urban poor in the passage.
ignoring all others. Option (c) is incorrect: The option also goes beyond
(b) Other deprivations in a human life have the scope of the passage. There is no mention of
nothing to do with lack of purchasing power. diverse personal circumstances and missed
(c) Income poverty is not a permanent condition, opportunities therein. The author refers to the lack
it changes from time to time. of opportunities rather than missed ones.
(d) Income poverty restricts human choices only Option (d) is incorrect: This option states,
at a point of time. ‘restricting human choices permanently’. This
Sol. (a) cannot be assumed. (In fact, the very purpose of
Option (a) is correct: This option follows from the discussing poverty and poverty estimations is the
lines, “income poverty. If poverty is ultimately about hope of reversing the condition of lack of choices
deprivations affecting human well-being, then and opportunities). So, the use of the phrase
income poverty is only one aspect of it.” ‘permanently’ is incorrect here. No deprivation,
material or non-material, restricts choice
Option (b) is incorrect: This option is rather extreme
permanently.
and goes beyond the scope of the passage. Nothing
in the passage talks about the factors affecting the 15. X and Y run a 3 km race along a circular course of
purchasing power of a person. length 300 m. Their speeds are in the ratio 3:2. If
Option (c) is incorrect: This option is beyond the they start together in the same direction, how many
scope of the passage. There is no mention in the times would the first one pass the other (the start-
passage of income being a temporary or permanent off is not counted as passing)?
condition. (a) 2 (b) 3
(c) 4 (d) 5
Option (d) is incorrect as the passage does not lay
out a time consideration (i.e. when does income Sol. (b)
poverty affect human choices by restricting them- The faster runner will cross the slower one
whether only at a point of time or always, till such when he covers an extra 300 m. Let their speeds be
poverty exists) anywhere while suggesting how 3 m/sec and 2 m/sec.
income poverty affects or restricts human choices. So, their relative speed = 3 – 2 = 1 m/sec
So, the time taken by the faster runner to cross
14. What does the author mean by ‘poverty of a life’?
the slower one = Distance/Relative Speed =
(a) All deprivations in a human life which stem
300/1 = 300 seconds
not only from lack of income but lack of real
It basically means that the faster runner will cross
opportunities
the slower one every 300 seconds, or 5 minutes.
(b) Impoverished state of poor people in rural and
urban areas Now, the time taken for the faster racer to complete
(c) Missed opportunities in diverse personal the entire race = Total Distance/Speed = 3000/3
circumstances = 1000 seconds.
(d) Material as well as non-material deprivations So, the faster racer will cross the slower one 3 times
in a human life which restrict human choices during the entire race – after 300 seconds, 600
permanently seconds, and 900 seconds.

CSAT by Manjul Kumar Tiwari Sir


Previous Year Solved Papers 237
16. If the order of the letters in the English alphabet is 18. How many 3-digit natural numbers (without
reversed and each letter represents the letter whose repetition of digits) are there such that each digit
position it occupies, then which one of the is odd and the number is divisible by 5?
following represents ‘LUCKNOW’? (a) 8 (b) 12
(a) OGXPMLD (b) OGXQMLE (c) 16 (d) 24
(c) OFXPMLE (d) OFXPMLD Sol. (b)
Sol. (d) We need to find three-digit numbers in which:
We basically need to find the opposite letter of the * All digits are different, and all digits are odd. So,
letters in the given word. Opposite Letter Position = the three digits must be from amongst 1, 3, 5, 7, and
27 – Letter Position 9.
So, in case of LUCKNOW, we get: 27 – 12 = 15 = O * The number is divisible by 5, i.e. the units digit
27 – 21 = 6 = F is 5.
27 – 3 = 24 = X The number of ways we can fill the first two digits
from amongst 4 distinct digits = 4 × 3 = 12.
27 – 11 = 16 = P
27 – 14 = 13 = M 19. Consider the Question and two Statements given
27 – 15 = 12 = L below:

27 – 23 = 4 = D Question: Is x an integer?

The word that we get is OFXPMLD. Statement-1: x/3 is not an integer.

17. In a tournament of Chess having 150 entrants, a Statement-2: 3x is an integer.


player is eliminated whenever he loses a match. It Which one of the following is correct in respect of
is given that no match results in a tie/draw. How the Question and the Statements?
many matches are played in the entire tournament?
(a) Statement-1 alone is sufficient to answer the
(a) 151 (b) 150
Question
(c) 149 (d) 148 (b) Statement-2 alone is sufficient to answer the
Sol. (c) Question
The tournament starts with 150 players. (c) Both Statement-1 and Statement-2 are
After first round (in which 75 matches are held): 75 sufficient to answer the Question
players are eliminated, and 75 remain. After second (d) Both Statement-1 and Statement-2 are not
round (in which 37 matches are held): 37 players sufficient to answer the Question
are eliminated, and 38 remain. After third round Sol. (d)
(in which 19 matches are held): 19 players are Let’s consider the two statements one by one.
eliminated, and 19 remain. Statement 1: x/3 is not an integer.
After fourth round (in which 9 matches are held): 9 x may or may not be an integer. For example, if x = 2,
players are eliminated, and 10 remain. After fifth even then x/3 will not be an integer. Statement 2: 3x
round (in which 5 matches are held): 5 players are is an integer.
eliminated, and 5 remain. x may or may not be an integer. For example, if x =
After sixth round (in which 2 matches are held): 2 2/3, even then 3x will be an integer. On combining
players are eliminated, and 3 remain. After seventh the two statements, 3x is an integer, but x/3 is not.
round (in which 1 match is held): 1 player is x may or may not be an integer. For example, if x = 2,
eliminated, and 2 remain. even then x/3 will not be an integer. But 3x = 6 will
After eighth round (in which 1 match is held): 1 be an integer.
player is eliminated, and 1 remain. So, total number If x = 2/3, even then 3x will be an integer. But x/3 =
of matches = 75 + 37 + 19 + 9 + 5 + 2 + 1 + 1 = 149 2/9 will not be an integer.

CSAT by Manjul Kumar Tiwari Sir


238 CSAT : 2022

20. The increase in the price of a certain item was 25%. 21. Based on the above passage, the following
Then the price was decreased by 20% and then assumptions have been made:
again increased by 10%. What is the resultant 1. Development of agricultural technology is
increase in the price? confined to developed countries.
2. Agricultural technology is not adapted in
(a) 5% (b) 10%
developing countries.
(c) 12.5% (d) 15%
Which of the above assumptions is/are valid?
Sol. (b) (a) 1 only (b) 2 only
Let the initial price be Rs. 100. (c) Both 1 and 2 (d) Neither 1 nor 2
Sol. (d)
After 25% rise, the new price = 100 + 25% of 100
Assumption 1 is invalid: The passage mentions,
= Rs. 125
“Application of technology…Americas”. This does
After 20% fall, the new price = 125 – 20% of 125 not mean that technological development in
= Rs. 100 agriculture is confined to the West. Focus on the
After 10% rise, the new price = 100 + 10% of 100 use of the word “much” here. From this, we can
= Rs. 110 So, resultant percentage increase in price infer that the West holds a major share of
= 10% technological development in agriculture, but it
does not imply “no technological development (of
Directions for the following 3 (three) items: agriculture) in the developed countries”.
Read the following passages and answer the items that Assumption 2 is invalid: The author only makes a
follow the passage. Your answers to these items should caution about blindly copying and deploying the
be based on the passages only. technologies of the West with respect to their
suitability in different climatic regions of the
Passage-1
Tropics. He makes a case, instead, for investing in
In some places in the world, the productivity of staples more achievable things such as optimal fertilizer
such as rice and wheat has reached a plateau. Neither application and better infrastructure. However, this
new strains nor fancy agrochemicals are raising the does not imply that agricultural technology is not
yields. Nor is there much unfarmed land left that is adapted in developing countries at all. Also, this
suitable to be brought under the plough. If global assumption does not state agricultural technology
temperature continues to rise, some places will become developed in the western world- It refers to
unsuitable for farming. Application of technology can agricultural technology in general.
help overcome these problems. Agricultural technology 22. Based on the above passage, the following
is changing fast. Much of this change is brought about assumptions have been made:
by affluent farmers in the West/Americas. Techniques 1. Poor countries need to bring about change in
developed in the West are being adapted in some places their existing farming techniques.
to make tropical crops more productive. Technology is 2. Developed countries have better
of little use if it is not adapted. In the developing infrastructure and they waste less food.
world, that applies as much to existing farming Which of the above assumptions is/are valid?
techniques as it does to the latest advances in genetic (a) 1 only (b) 2 only
modification. Extending to the smallholders and (c) Both 1 and 2 (d) Neither 1 nor 2
subsistence farmers of Africa and Asia the best of Sol. (a)
today’s agricultural practices, in such simple matters Assumption 1 is correct: The passage specifies/
as how much fertilizers to apply and when, would talks about poor countries.
lead to a greatly increased availability of food for
Assumption 2 is incorrect: This assumption goes
humanity. So would things like better roads and storage
beyond the information provided in the passage.
facilities, to allow for the carriage of surpluses to markets
There is no mention of the infrastructure status of
and reduce wastage. the developed countries or the food wastage there.

CSAT by Manjul Kumar Tiwari Sir


Previous Year Solved Papers 239
Though the last lines of the passage do suggest that Which one of the following is correct in respect of
better roads and storage facilities will reduce the Question and the statements?
wastage, but it has been stated in a very general (a) Statement-1 alone is sufficient to answer the
way (with no comparison between developed Question
countries and developing countries whatsoever).
(b) Statement-2 alone is sufficient to answer the
The only comparison being made in the passage
Question
between developed and developing countries is
(c) Both Statement-1 and Statement-2 are
regarding agricultural technology.
sufficient to answer the Question
23. Based on the above passage, the following (d) Both Statement-1 and Statement-2 are not
assumptions have been made: sufficient to answer the Question
1. Growing enough food for future generations Sol. (d)
will be a challenge..
Let’s consider the two statements one by one.
2. Corporate farming is a viable option for food
security in poor countries.. Statement 1:
Which of the above assumptions is/are valid? X is the brother of Y, and Y is the brother of Z.
(a) 1 only (b) 2 only But we do not know whether Z is a male or a female.
(c) Both 1 and 2 (d) Neither 1 nor 2 So, we cannot say whether Z is the brother or sister
Sol. (a) of X.
Assumption 1 is correct: The author is talking about Statement 2:
growing enough food for future generations X, Y, and Z are siblings. It tells us nothing much.
Assumption 2 is incorrect: This assumption goes Here, also gender of Z is not known. So, Z can be
beyond the information provided in the passage. either brother or sister of X.
There is no mention or indication of corporate Even if we combine the two statements, we cannot
farming in the passage. answer the given questions.
24. The letters A, B, C, D and E are arranged in such a 26. On one side of a 1.01 km long road, 101 plants are
way that there are exactly two letters between A planted at equal distance from each other. What is
and E. How many such arrangements are possible? the total distance between 5 consecutive plants?
(a) 12 (b) 18
(a) 40 m (b) 40.4 m
(c) 24 (d) 36
(c) 50 m (d) 50.5 m
Sol. (c)
Sol. (b)
There are exactly 2 letters between A and E. So, the
Length of the road = 1.01 km = 1010 m
fifth letter must be either beside A or E. So, the
following four arrangements are possible: 101 plants are planted at equal distance from each
A__E_ other. So, there will be 100 gaps between those
_A__EE__A_ plants.

_E__A Length of each gap = 1010/100 = 10.1 m


The three blank spaces can be filled by three distinct Now, there must be 4 gaps between 5 consecutive
letters in 3 × 2 × 1 = 6 ways So, total possible plants. So, required distance = 4 × 10.1 = 40.4 m
arrangements = 6 × 4 = 24
27. A, B and C are three places such that there are
25. Consider the Question and two Statements given three different roads from A to B, four different
below: roads from B to C and three different roads from A
Question: Is Z brother of X. to C. In how many different ways can one travel
Statement-1 : X is the brother of Y and Y is a brother from A to C using these roads?
of Z. (a) 10 (b) 13
Statement-2 : X, Y and Z are siblings. (c) 15 (d) 36

CSAT by Manjul Kumar Tiwari Sir


240 CSAT : 2022

Sol. (c) If we have an arithmetic series, 1, 2, 3 …, then:


Number of ways to go from A to C = 3 Sum of first n terms = (n/2) (a + l), where ‘n’ is the
number of terms in the series, ‘a’ the first term, and
But we also know that, Number of ways to go from
‘l’ the last term.
A to B = 3, and Number of ways to go from B to
C = 4. So, sum of first 14 terms = (14/2) (1 + 14) = 7 × 15 =
105
So, Number of ways to go from A to C, via B = 3 × 4
= 12 So, total number of ways to go from A to C = 3 So, the term having 14 letters will have the 100th
+ 12 = 15 letter of the series.
It will start at the 92nd letter and end at 105th letter.
28. A has some coins. He gives half of the coins and 2
Hence, the 100th letter must be I.
more to B. B gives half of the coins and 2 more to C.
C gives half of the coins and 2 more to D. The 30. Three persons A, B and C are standing in a queue
number of coins D has now, is the smallest two- not necessarily in the same order. There are 4
digit number. How many coins does A have in the persons between A and B, and 7 persons between
bringing? B and C. If there are 11 persons between B and C
(a) 76 (b) 68 and 13 behind A, what could be the minimum
number of persons in the queue?
(c) 60 (d) 52
(a) 22 (b)28
Sol. (d)
(c) 32 (d)38
The number of coins with D = 10 Let’s work Sol. (a)
backwards now.
There are 4 persons between A and B, and 7 persons
D = (C/2) + 2 between B and C. So, there are four cases possible,
Or C/2 = 10 – 2 = 8 Or C = 16 as depicted below

Similarly,
C = (B/2) + 2
Or B/2 = 16 – 2 = 14 Or B = 28
And finally, B = (A/2) + 2
Or A/2 = 28 – 2 = 26 Or A = 52
So, A initially had 52 coins.
29. In the series AABABCABCDABCDE..., which letter
appears at the 100th place?
(a) G (b) H
(c) I (d) J Now, there are 11 persons ahead of C and 13 behind
A. As there are more than 11 persons ahead of C in
Sol. (c)
first case depicted above, we can eliminate it. We
The given series is: AABABCABCDABCDE… can proceed with the remaining three cases.
The pattern being followed here is: A, AB, ABC, To minimize the number of people in the queue, we
ABCD, ABCDE, … should just focus on the last/fourth case. The
number of people in the queue in the second case
It’s kind of an arithmetic series wherein the first
will be 28, in the third case 38, and in the fourth
term has 1 letter, second term has 2 letters, and so
case 22.
on. We have to estimate the length of the letter-series
near the 100th letter. So, the final arrangement will be as follows:

CSAT by Manjul Kumar Tiwari Sir


Previous Year Solved Papers 241
proposed natural selection) is amply supported by
scientific data. Indeed, to date no single zoological,
botanical, geological, palaeontological, genetic or
physical evidence has refuted either of the central two
main Darwinian ideas. If religion is not taken into
consideration, Darwinian laws are acceptable just like
the laws proposed by Copernicus, Galileo, Newton and
Einstein—sets of natural laws that explain natural
phenomena in the universe.

31. According to the passage, natural selection cannot


anticipate future environments on the earth as
1. species not fully prepared to face the
environmental changes that await them will
face extinction
2. all the existing species would get extinct as
their genomes will not withstand biological
Minimum possible number of people in the queue mishaps
= 22. 3. inability of the genome to withstand
environmental changes would result in
Directions for the following 4 (four) items:
extinction
Read the following two passages and answer the items
4. extinction of species is a common feature
that follow the passage. Your answers to these items
(a) 1, 2 and 3 (b) 2, 3 and 4
should be based on the passages only.
(c) 1, 3 and 4 (d) 1, 2 and 4
Passage-1 Sol. (c)
Natural selection cannot anticipate future environments Statement 1 is correct: According to the passage,
on the earth. Therefore, the set of existing organisms natural selection conveys that those species which
can never be fully prepared for environmental are not adapted to the environment will get extinct.
catastrophes that await life. An outcome of this is the The phrase, “Therefore, the set of existing organism
extinction of those species which cannot overcome can never be fully prepared for environmental
environmental adversity. This failure to survive, in catastrophe that await life. An outcome of this is
modern terms, can be attributed to the genomes which the extinction of those species which can not
are unable to withstand geological vagaries or overcome environmental adversity”.
biological mishaps (infections, diseases and so on). In
Statement 2 is incorrect: The phrase “All the
biological evolution on the earth, extinction of species
existing species” in this option is incorrect. The
has been a major feature. The earth may presently have
passage clearly conveys that only those species that
up to ten million species, yet more than 90% of species
are not able to adapt to changes in the environment
that have ever lived on the earth are now extinct. Once
will get extinct. This is inferred from the lines “the
again, the creationist doctrines fail to satisfactorily
extinction of those species which cannot overcome
address why a divine creator will firstly bother to create
environmental adversity. This failure to survive, in
millions of species and then allow them to perish. The
modern terms, can be attributed to the genomes
Darwinian explanation for extinct life is once again
which are unable to withstand geological vagaries
simple, elegant and at once convicing—organisms go
or biological mishaps (infections, diseases and so
extinct as a function of environmental or biological
on).” Hence this statement is incorrect.
assaults for which their inheritance deems them ill-
equipped. Therefore, the so-called Darwinian theory of Statement 3 is correct: As explained above, “This
evolution is not a theory at all. Evolution happens— failure to survive, in modern terms, can be attributed
this is a fact. The mechanism of evolution (Darwin to the genomes which are unable to withstand

CSAT by Manjul Kumar Tiwari Sir


242 CSAT : 2022

geological vagaries or biological mishaps elaborate evolution based on environmental and


(infections, diseases and so on)”. biological mishaps. and the existence for organism
Statement 4 is correct: The passage says that “In is attributed to adaptation to these changes rather
the biological evolution on the earth, extinction of than to creator. Hence this option is incorrect.
the species has been a major feature”. Hence we 33. With reference to the passage, the following
can consider this to be a common feature. assumptions have been made:

32. The passage suggests that Darwinian theory of 1. Only species that have the ability to overcome
evaluation is not a theory at all because environmental catastrophes will survive and
perpetuate.
(a) it does not satisfy the creationist doctrine
(b) extinction is a function of environment and 2. More than 90% of the species on the earth are
biological assaults in the danger of getting extinct due to drastic
(c) there are no evidences to refute it changes in the environment.
(d) existence of organisms is attributed to a 3. Darwin’s theory explains all the natural
creator phenomena.
Sol. (c)
(a) 1 only (b) 1 and 2 only
Option (a) is incorrect: As the passage states, the
creationist doctrine is not able to explain evolution. (c) 3 only (d) 1, 2 and 3
Hence there is no question of “undermining” its Sol. (a)
validity at all. Therefore, this option is incorrect.
Statement 1 is correct: as the passage states that
Option (b) is incorrect: As per the author, extinction the extinction of a species is primarily driven by
is indeed a function of environment and biological non adaptation to environmental catastrophe. It is
assaults as stated the in the phrase “The Darwinian reflected in the phrase “An outcome of this is the
explanation for extinct life is once again simple, extinction of those species which cannot overcome
elegant and at once convincing- organisms go environmental adversity. This failure to survive, in
extinct as a function of environmental or biological modern terms, can be attributed to the genomes
assaults for which their inheritance deems them which are unable to withstand geological vagaries
ill-equipped. Therefore, the so-called Darwinian or biological mishaps infections, diseases and so
theory of evolution is not a theory at all”. However, on)”.
this is not the reason/ does not explain why
Statement 2 incorrect: It is an incorrect inference as
Darwinian theory is not a theory. the passage does not predict the extinction
Option (c) is correct: The passage clearly states, probability in the future. It only provides the
“Therefore, the so-called Darwinian theory of extinction data for the past in the phrase, “The earth
evolution is not a theory at all. Evolution happens- may presently have up to ten million species, yet
this is a fact. The mechanism of evolution (Darwin more than 90% of species that have ever lived on
proposed natural selection) is amply supported by the earth are now extinct.”
scientific data. Indeed, to date no single zoological,
Statement 3 incorrect: It is incorrect that Darwin’s
botanical, geological, paleontological, genetic or
theory explains all natural phenomena. It only
physical evidence has refuted either of the central explains the natural phenomena of evolution. In
two main Darwinian ideas.” So, there is no evidence the passage, the author mentions the names of
to refute it, which enhances the credibility of the Copernicus, Galileo, Newton and Einstein for
Darwinian theory and establishes it as a fact. It is explaining natural laws that explain various
more of a fact or law like other natural laws. Hence phenomena in the universe- “Darwinian laws are
this option is correct. acceptable just like the laws proposed by
Option (d) is incorrect: The passage clearly Copernicus, Galileo, Newton and Einstein-sets of
establishes that the creationist doctrines fail to natural laws that explain natural phenomena in
satisfactorily address evolution. The passage the universe.”

CSAT by Manjul Kumar Tiwari Sir


Previous Year Solved Papers 243
Passage-2 example, both, dietary patterns and energy
With steady economic growth, higher literacy and generation, require water. Hence, the passage in its
increasing skill levels, the number of Indian middle- crux conveys that water management practices in
class families has gone up exponentially. Direct results India need to change quite a lot (dramatically) in
of the affluence have been changes in dietary patterns the coming years to sustain both aspects.
and energy consumption levels. People have moved to
35. How many seconds in total are there in x weeks, x
a higher protein-based diet like milk products, fish and
days, x hours, x minutes and x seconds?
meat, all of which need significantly more water to
(a) 11580x (b) 11581x
produce than cereal-based diets. Increasing use of
(c) 694860x (d) 694861x
electronic and electric machines/gadgets and motor
Sol. (d)
vehicles needs more and more energy and generation
of energy needs water. x weeks = 7x days = (7 × 24) x hours = 168x hours =
34. Which one of the following statements best reflects (168 × 60) x minutes = 10080x minutes = (10080 ×
the crux of the passage? 60)x seconds = 604800x seconds
(a) People should be persuaded to continue with x days = 24x hours = 1440x minutes = 86400x
the mainly Indian traditional cereal-based seconds x hours = 60x minutes = 3600x seconds
diets. x minutes = 60x seconds
(b) India needs to focus on developing So, total time in seconds = 604800x + 86400x +
agricultural productivity and capacity for 3600x + 60x + x = 694861x seconds
more energy generation in the coming years.
(c) Modern technological developments result in 36. P, Q, R, S, T and U are six members of a family. R is
the change of cultural and social behaviour the spouse of Q. U is the mother of T and S is the
of the people. daughter of U. P’s daughter is T and R’s son is P.
There are two couples in the family. Which one of
(d) Water management practices in India need to
the following is correct?
change dramatically in the coming years.
(a) Q is the grandfather of T
Sol. (d)
(b) Q is the grandmother of T
Option (a) is incorrect: The passage does not make (c) R is the mother of P
any suggestions for the way forward that whether (d) T is the granddaughter of Q
the shift in dietary patterns is, say, good for health
Sol. (d)
or whether people should shift back to cereal-based
diets merely due to the fact that it needs more water The family tree, as per the information provided in
for production. the question, has been depicted below

Option (b) is incorrect: Again, the passage only talks


about the issue of more water utilised in protein-
based diet production as compared to cereal-based
ones. It does not make any suggestions for the way
forward.
Option (c) is incorrect: The passage talks about
economic development and its impact on dietary
patterns. However, it does not discuss the social
behaviour of people at all. Hence this option is We can see that T is the granddaughter of Q.
incorrect. 37. Consider the Question and two Statement given
Option (d) is correct: Though, the use of the word, below in respect of three cities p, Q and R in a
“dramatically” makes this option look extreme, it State:
is the best option from the given set of options. The Question: How far is city P from city Q?
author does depict the increased water Statement-1 : City Q is 18 km from city R.
consumption in modern-day to be an issue- For Statement-2 : City P is 43 km from city R.

CSAT by Manjul Kumar Tiwari Sir


244 CSAT : 2022

Which one of the following is correct in respect of 39. Three Statements followed by three Conclusions
the Question and the Statements? are given below. You have to take the Statements
(a) Statement-1 alone is sufficient to answer the to be true even if they seem to be at variance from
Question the commonly known facts. Read all the
(b) Statement -2 alone is sufficient to answer the Conclusions and then decide which the given
Question Conclusions logically follows/follow from the
(c) Both Statement-1 and Statement-2 are Statements, disregarding the commonly known
sufficient to answer the Question facts:
(d) Both Statement-1 and Statement-2 are not Statement-1 : Some doctors are teachers.
sufficient to answer the Question Statement-1 : All teachers are engineers.
Sol. (d) Statement-3 : All engineers are scientists.
As we do not know the respective positions of P Conclusion-I: Some scientists are doctors.
and Q, we cannot find the distance between them, Conclusion-II : All engineers are doctors.
even after using the information in both the Conclusion-III : Some engineers are doctors.
statements. Which one of the following is correct ?
(a) Only Conclusion-I
38. Two Statements followed by four Conclusions are (b) Only Conclusion-II
given below. You have to take the Statements to be (c) Both Conclusion-I and Conclusion - III
true even if they seem to be at variance from the
(d) Both Conclusion - I and Conclusion - II
commonly known facts. Read all the Conclusions
Sol. (c)
and then decide which of the given Conclusions
logically follows/ follow from the Statements, We can draw the following possible Venn diagrams
disregarding the commonly known facts: based on the given two statements:
Statement-1 : All pens are books
Statement-2 : No chair is a pen.
Conclusion-I : All chairs are books.
Conclusion-II : Some chairs are pens.
Conclusion- III : All books are chairs.
Conclusion- IV : No chair is a book.
Which one of the following is correct?
(a) Only Conclusion-I
(b) Only Conclusion-II
(c) Both Conclusion-III and Conclusion- IV We can see that conclusions I and III follow.
(d) None of the Conclusions follows 40. Eight students A, B C, D, E, F, G and H around a
Sol. (d) circular table, equidistant from each other , facing
We can draw the following possible Venn diagrams the centre of the table, not necessarily in the same
based on the given two statements order. B and D sit neither adjacent to C nor opposite
to C. A sits in between E and D, and F sits in
between B and H. Which one of the following is
definitely correct?
(a) B sits in between A and G
(b) C sits opposite to G
(c) E sits opposite to F
(d) None of the above
Sol. (d)
The various possible circular arrangements, as per
the information provided in the passage, have been
We can see that none of the conclusion follows. depicted below

CSAT by Manjul Kumar Tiwari Sir


Previous Year Solved Papers 245
incorrect to infer that the author is suggesting that
man should not work hard. It only argues for a
balance in the work-life.
Option (b) is correct: The passage talks about
today’s workers being engaged in an excessive
amount of labour, which limits his spiritual and
intellectual development. Hence we can conclude
that the idea of the passage is that the great evil of
our age is overstrained.
Option (c) is incorrect: The passage does not talk
about the lack of the ability of man to think well per
se. It rather focusses on the less time left after
excessive (hours of) labour to ponder over or involve
We can see that none of (a), (b), or (c) is definitely
in spiritual aspects.
correct.
Option (d) is incorrect: The passage does not
Directions for the following 4 (four) items: suggest that man is incapable of caring for his
Read the following two passages and answer the items spiritual welfare. It just suggests that in order to go
that follow the passages. Your answers to these items towards spirituality or anything serious, people
should be based on the passages only. need a proper amount of spare time after work. But
people are incapable of devoting such time as a
Passage-1 result of being overstrained with work. So, they look
For two or three generations past, ever-increasing for purely entertaining pursuits in the little leisure
numbers of individuals have been living as workers time that they get.
merely, not as human beings. An excessive amount of 42. Man does not seek self- improvement because he
labour is rule today man’s spiritual element cannot (a) is not intellectually capable
thrive. He finds it very difficult to spend his little leisure (b) has no time to do so
in serious activities. He does not want to think; or he (c) is distracted by materialism
cannot even if he wants to. He seeks not self- (d) loves amusement and is mentally
improvement, but entertainment which would enable
Sol. (b)
him to be mentally idle and to forget his usual activities.
Option (a) is incorrect: The passage does not
Therefore, the so-called culture of our age is dependant
question the intellectual capability of man. It only
more on cinema than on theatre, more on newspapers,
talks about overstraining and the very less leisure
magazines and crime stories than on serious literature.
time at his disposal. Hence this statement is
41. The passage is based on the idea that incorrect.
(a) man should not work hard Option (b) is correct: A phrase from the passage
(b) the great evil of our age is overstain states that “He finds it very difficult to spend his
(c) man cannot think well little leisure in serious activities.” This statement
(d) man cannot care for his spiritual reflects the unavailability of time, due to which man
Sol. (b) does not seek self-improvement.
Option (a) is incorrect: The passage suggests the Option (c) is incorrect: The passage suggests that
less time left for leisure after work to be an issue as man does not seek self-improvement due to the lack
no time is left for spiritual growth- “An excessive of ability to do so as a result of lack of time. He is so
amount of labour is rule today in every circle of pressurised by excessive labour that he wants to be
society, with the result that man’s spiritual element mentally idle and forget his usual stress. The
cannot thrive. He finds it very difficult to spend his passage does not discuss man’s interest in
little leisure in serious activities.” It would be materialism.

CSAT by Manjul Kumar Tiwari Sir


246 CSAT : 2022

Option (d) is incorrect: This option shows man to age people. Hence the ratio of very old and very
intrinsically love being unproductive- as a lover of young will decrease for a while as long as the
amusement and being mentally idle. But the author demographic dividend remains. This is reflected in
of the passage shows man to be so as a result of the lines, “The demographic dividend is basically
overstrain, rather than being intrinsically like that. a swelling in the working age population, which
conversely means that the relative ratio of very
Passage-3
young and very old will, for a while, be on the
The demographic dividend, which has begun in India decline.”
and is expected to last another few decades, is a great Statement 3 incorrect: The passage does not
window of opportunity. The demographic dividend is discuss the relationship between demographic
basically a swelling in the working are population, dividend and population growth. Directly, it can
which conversely means that the relative ratio of very not be said that the population growth rate will
young and very old will, for a while, be on the decline. quickly stabilise on the operation of the demographic
From the experience of Ireland and China, We know dividend.
that this can be a source of energy and an engine of
economic growth. The demographic dividend tends to 44. With reference to the passage, which of the
raise a nation’s savings rate since in any nation, it is following inferences can be drawn?
the working age population that is the main saver. 1. Demographic dividend is an essential
And since the savings rate is an important driver of condition for a country to rapidly increase its
growth, this should help elevate our growth rate. economic growth rate.
However, the benefits of demographic dividend depend 2. Promotion of higher education is an essential
on the quality of the working age population And this condition for a country for its rapid economic
implies bringing back the importance of education, growth.
acquisition of skills and human capital. Select the correct answer using the code given
43. Which of the following would invariably happen below.
in a country, when the demographic dividend has (a) 1 only (b) 2 only
begun to operate? (c) Both 1 and 2 (d) Neither 1 nor 2
1. The number of illiterate people will decrease. Sol. (d)
2. The ratio of very old and very young will Statement 1 incorrect: As per the passage,
decrease for a while. demographic dividend provides a (great)
3. Population growth rate will quickly stabilize. opportunity for economic growth, as is exemplified
by the lines “The demographic dividend, which has
Select the correct answer using the code given
begun in India and is expected to last another few
below.
decades, is a great window of opportunity.”
(a) 1 and 2 only (b) 2 only
However, this does not imply that is an “essential
(c) 1 and 3 only (d) 1, 2 and 3
condition” for economic growth. That means that
Sol. (b)
economic growth might be possible even if a country
Statement 1 incorrect: The number of illiterate is not in its demographic dividend phase.
people will decrease depending upon the emphasis
Statement 2 incorrect: The passage only talks about
on the same given by the government and other
the importance of education (acquisition of skills
stakeholders. The operation of demographic
and human capital). It does specify higher
dividend has no relation with it. However, it is the
education as such- For example, vocational
other way round where literacy will increase the
education also helps in the acquisition of skills.
quality of demographic dividend will increase.
Hence, based on the information provided in the
Statement 2 correct: In the case of a demographic passage, we cannot infer that higher education is
dividend, the number of young people will increase an essential condition for a country for its rapid
as compared to the dependents- children and old economic growth.

CSAT by Manjul Kumar Tiwari Sir


Previous Year Solved Papers 247
45. Five friends P, Q, X, Y and Z purchased some So, his total travelling time = 90 – 25 = 65 minutes
notebooks. The relevant information is given below: The return route was 1.25 times the initial route. So,
1. Z purchased 8 notebooks more than X did. time taken must have increased by 25% too. So, if
2. P and Q together purchased 21 notebooks. initial time was 100 units, now it must be 125 units.
3. Q purchased 5 notebooks less than P did.
But it is also given that while returning he drove
4. X and Y together purchased 28 notebooks.
twice as fast. So, time taken must have been
5. P purchased 5 notebooks more than X did.
halved. So, time taken while returning back = 125/
If each notebook is priced Rs. 40, then what is the 2 = 62.5 units
total cost of all the notebooks? So, 100 + 62.5 = 65 minutes
(a) Rs. 2600 (b) Rs. 2400
Or 162.5 units = 65 minutes
(c) Rs. 2360 (d) Rs. 2320
So, 100 units = (65/162.5) × 100 = 40 minutes
Sol. (a)
As per the information provided in the question: Z So, the man took 40 minutes to reach to the village.
= X + 8 ….(i) So, the actual time at that moment = 14:30 + 40
P + Q = 21 ….(ii) minutes = 15:10 hours It’s pretty evident that the
village clock is 15:15 – 15:10 = 5 minutes fast
Q = P – 5….(iii) X + Y = 28 ….(iv)
P=X+5 ….(v) 47. A person X wants to distribute some pens among
We need to find the value of P + Q + X + Y + Z Using six children A, B, C, D, E and F. Suppose A gets
ii and iii, we get: twice the number of pens received by B, three times
that of C, four times that of D, five times that of E
P + Q = 21
and six times that of F. What is the minimum
Or P + (P - 5) = 21 Or P = 13
number of pens X should buy so that the number
So, Q = P – 5 = 13 – 5 = 8 of pens each one gets is an even number
Using v, we get: P = X + 5 (a) 147 (b) 150
Or X = P – 5 = 13 – 5 = 8 (c) 294 (d) 300
Using i, we get: Z = X + 8 Sol. (c)

Or Z = 8 + 8 = 16 Let the number of pens with A be the LCM of 2, 3, 4,


5, and 6 = 60 Then the number of pens with B = 60/
Using iv, we get: X + Y = 28
2 = 30
Or Y = 28 – 8 = 20
The number of pens with C = 60/3 = 20
So, P + Q + X + Y + Z = 13 + 8 + 8 + 20 + 16 = 65
The number of pens with D = 60/4 = 15 (an odd
So, total cost of all the notebooks = 65 × 40 = Rs.
number) The number of pens with E = 60/5 = 12
2600
The number of pens with F = 60/6 = 10
46. A man started from home at 14:30 hours and drove To ensure that all get an even number of pens, we
to village, arriving there when the village clock need to double the number of pens bought by A, i.e.
indicated 15:15 hours. After staying for 25 60 × 2 = 120
minutes, he drove back by a different route of length
So, total number of pens bought by X = 120 + 60 +
1.25 times the first route at a rate twice as fast
40 + 30 + 24 + 20 = 294
reaching home at 16:00 hours. As compared to the
clock at home the village clock is 48. Six persons A, B, C, D, E and F are sitting
(a) 10 minutes slow (b) 5 minutes slow equidistant from each other around a circular table
(c) 10 minutes fast (d) 5 minutes fast (facing the centre of the table).
Sol. (d) Consider the Question and two Statements given
Total time taken by the man to come back home = 16 below:
– 14.5 = 1.5 hours = 90 minutes Out of which he Question: Who is sitting on the immediate
stayed in the village for 25 minutes. left of A?

CSAT by Manjul Kumar Tiwari Sir


248 CSAT : 2022

Statement-1: B is sitting opposite to C and D is (d) Both Statement-1 and Statement-2 are not
sitting opposite to E. sufficient to answer the Question
Statement-2: F is sitting on the immediate left of B. Sol. (c)
It’s pretty evident that neither of the two statements
Which one of the following is correct in respect of
is sufficient alone. So, now let’s combine the two
the Question and the Statements?
statements.
(a) Statement-1 alone is sufficient to answer the
Let the present ages of Manisha and her mother be
Question
x and y respectively. As per statement 1, y = x + 24
(b) Statement-2 alone is sufficient to answer the
Question As per statement 2, (x + 5)/(y + 5) = 3/5 Or 5x + 25
(c) Both Statement-1 and Statement-2 are = 3y + 15
sufficient to answer the Question Or 3y – 5x = 10
(d) Both Statement-1 and Statement-2 are not Or 3(x + 24) – 5x = 10 Or 3x + 72 – 5x = 10 Or 2x = 62
sufficient to answer the Question Or x = 31
Sol. (d) So, Manisha is 31 years old at present. Both
It’s pretty evident that neither of the two statements statements together are sufficient to answer the
is sufficient alone. question.
Even on combining the two, we get two possible
50. Six lectures A, B, C, D, E and F, each of one hour
cases
duration, are scheduled between 8:00 a.m. and
2:00 p.m.
Consider the Question and two Statements given
below:
Question: Which lecture is in the third period?
Statement-1: Lecture F is preceded by A and
followed by C.

On observing the above two cases, we can see that Statement-2: There is no lecture after lecture B.
the person who is sitting of the immediate left of A Which one of the following is correct in respect of
cannot be determined. the Question and the Statements?
49. Consider the Question and two Statements given (a) Statement-1 alone is sufficient to answer the
below: Question
Question: What is the age of Manisha? (b) Statement-2 alone is sufficient to answer the
Question
Statement-1: Manisha is 24 years younger than her
mother. (c) Both Statement-1 and Statement-2 are
sufficient to answer the Question
Statement-2: 5 years later, the ages of Manisha and
(d) Both Statement-1 and Statement-2 are not
her mother will be in the ratio 3 : 5.
sufficient to answer the Question
Which one of the following is correct in respect of Sol. (d)
the Question and the Statements? It’s pretty evident that neither of the two statements
(a) Statement-1 alone is sufficient to answer the is sufficient alone. Even on combining the two, we
Question get multiple possible cases:
(b) Statement-2 alone is sufficient to answer the
AFC_ _ B
Question
(c) Both Statement-1 and Statement-2 are _AFC_B
sufficient to answer the Question _ _ AFCB

CSAT by Manjul Kumar Tiwari Sir


Previous Year Solved Papers 249
There may be even more cases as it’s not necessary Moreover, the author has cautioned that too much
that A, F and C are consequent lectures. So, even by of leisure can be tedious, and has recommended
using both the statements we cannot determine the that education is a necessity to best use leisure time.
answer. Option (c) is incorrect. The line “If a leisured
Directions for the following 3 (three) items: population has to be happy, it must be an educated
population…” restricts its domain to leisured
Read the following two passages and answer the
population. For a leisured population to be happy,
items that follow the passages. Your answer to these
it must be educated. If it is not educated, the leisure
items should be based on the passages only.
time might be detrimental to the population. This
Passage-1 analogy cannot be extrapolated to the non-leisure
In an economic organization, allowing mankind to population also. Therefore, it would be incorrect to
benefit by the productivity of machines should lead to say that an increase in the educated population
a very good life of leisure, and such leisure is apt to be leads to an increase in happiness among people.
tedious except those who have intelligent activities and Option (d) is incorrect, as already discussed above,
interests. If a leisured population is to be happy, it at the beginning of the explanation.
must be an educated population, and must be educated
So, from the above discussion, despite its extreme
with a view to enjoyment well as to the direct usefulness
nature, option (a) is the most appropriate amongst
of technical knowledge.
all the provided options.
51 Which of the following statements best reflects the
underlying tone of the passage? Passage-2
(a) Only an educated population can best make If presents bring less thrill now that we are grown up,
use of the benefits of economic progress. perhaps it is because we have too much already; or
(b) All economic development should be aimed perhaps it is because we have lost the fullness of the
at the creation of leisure. joy of giving, and with it the fullness of the joy of
(c) An increase in the educated population of a receiving. Children’s fears are poignant, their miseries
country leads to an increase in the happiness are acute, but they do not look too forward nor too far
of its people. backward. Their joys are clear and complete, because
(d) Use of machines should be encouraged in they have not yet learnt always to add ‘but’ to every
order to create a large leisured population. proposition. Perhaps we are too cautious, too anxious,
too sceptical. Perhaps some of our cares would shrink
Sol. (a)
if we thought less about them and entered with more
Given that there is a leisured population due to
single-minded enjoyment into the happiness that come
mechanization of economy, the author discusses
our way.
how to best channelize this leisured population.
This makes option (d) incorrect. Let’s consider the 52. With reference to the passage, which one of the
other options. following statements is correct?
Option (a) is correct: The passage clearly outlines (a) It is not possible for adults to feel thrilled by
the advantages of an educated population. It is presents.
happy; it is aware of the intelligent activities and (b) There can be more than one reason why adults
interests to make best use of the leisure time; it enjoys feel less thrilled by presents.
the technical knowledge etc. The use of ONLY (c) The author does not know why adults feel
makes it an extreme statement. Let’s put this answer less thrilled by presents.
option on hold and consider other options. (d) Adults have less capacity to feel the joy of
loving or being loved.
Option (b) is incorrect. The passage only touches
upon how to best leverage the leisure time due to Sol. (b)
mechanization in the economy. It would be an Option (a) incorrect: This option is an extreme
exaggeration to infer that ALL economic exaggeration of what the author conveys in the
development should lead to creation of leisure. passage. The author only speaks about a tendency

CSAT by Manjul Kumar Tiwari Sir


250 CSAT : 2022

for adults to be less thrilled by happy events. The 54. Let A, B and C represent distinct non-zero digits.
author nowhere makes any assertion or inference Suppose x is the sum of all possible 3-digit
that adults cannot be thrilled by presents at all. numbers formed by A, B and C without repetition.
Option (b) correct: The author states that adults Consider the following statements:
feel less thrilled due several reasons. For example, 1. The 4-digit least value of x is 1332.
(based on the passage) looking too forward or too 2. The 3-digit greatest value of x is 888.
far backward, being too cautious, too anxious, too
Which of the above statements is/are correct?
skeptical etc.
(a) 1 only (b) 2 only
Option (c) incorrect: The author, has properly (c) Both 1 and 2 (d) Neither 1 nor 2
reasoned why adults dont feel happy with presents,
Sol. (a)
Hence this option is incorrect.
The three-digit numbers have been represented by
Option (d) incorrect: The author does not talk about
ABC, wherein A, B, and C are non-zero digits. Using
the “capacity” of the adult to feel loved or joy. The
3 distinct digits we can make 3 × 2 × 1 = 6 three-
author only focuses only on the reasons why an
digit numbers.
adult should, but does not, feel thrilled.
So, x will be the sum of these 6 three-digit numbers.
53. The author of the passage is against We need to find the two values of x closest to 1000,
(a) worrying too much about the past and future one just below it (which will be the greatest 3-digit
(b) being in the habit of thinking about presents value of x), and the other just above it (which will
(c) not being thrilled by new things be the lowest 4-digit value of x).
(d) giving and receiving joy only partially Now, we have to do a bit of hit and try, so that the
Sol. (a) value of x reaches close to 1000. Let the three digits
Option (a) is correct: Worrying too much about the be the minimum possible ones, i.e. 1, 2, and 3.
past and future is the result of excessive thinking So, we get x = 123 + 132 + 213 + 231 + 312 + 321 =
which is reflected in the passage in the lines, 1332
“perhaps we are too cautious, too anxious, too This is the least possible value of x. So, statement 1
sceptical”. Hence option (a) is correct. is correct, but statement 2 is incorrect.
Option (b) is incorrect: The author is arguing for
thinking about the present and living it to the fullest. 55. There is a numeric lock which has a 3-digit PIN.
The author argues that adults, like children, must The PIN contains digits 1 to 7. There is no
not look forward or far backward. The focus should repetition of digits. The digits in the PIN from left
be on living with joy and completeness. Hence this to right are in decreasing order. Any two digits in
option is incorrect. the PIN differ by at least 2. How many maximum
attempts does one need to find out the PIN with
Option (c) is incorrect: The author reasons why
certainty?
adults feel less thrilled by presents, i.e. “because
(a) 6 (b) 8
we have too much already, or perhaps it is because
(c) 10 (d) 12
we have lost the fullness of the joy of giving and
Sol. (c)
with it the fullness of the joy of receiving. Children’s
fears are poignant, their miseries are acute, but they The PIN contains three digits out of – 1, 2, 3, 4, 5, 6,
do not look too forward nor too far backward. Their and 7.
joys are clear and complete, because they have not
Now, there is no repetition of digits, digits are in
yet learnt always to add ‘but’ to every proposition.
decreasing order from left to right, and any two
Perhaps we are too cautious, too anxious, too
digits in the PIN differ by at least 2.
sceptical.”
Option (d) is incorrect: The author is against giving The maximum attempts will be equal to all the
and receiving joy only partially, but it is only one of possible combinations of the PIN. Let us consider
the objections. Hence this option is incorrect. the various possible cases:

CSAT by Manjul Kumar Tiwari Sir


Previous Year Solved Papers 251
Case I: The rightmost digit is 1 Similarly, we consider BF as the diameter and one
The possible combinations are: 531, 631, 731, 641, side of the right-angled triangle, we can draw 6 right-
741, 751 (i.e. 6 possible combinations) angled triangles.
We can do the same when we consider CG and DH
Case II: The rightmost digit is 2
as the diameter and one side of the right-angled
The possible combinations are: 642, 742, 752 (i.e.
triangle.
3 possible combinations)
Hence, the total number of right-angled triangles
Case III: The rightmost digit is 3 that can be drawn = 6 + 6 + 6 + 6 = 24
The possible combinations are: 753 (i.e. 1 possible
combination) The rightmost digit cannot be more 57. 24 men and 12 women can do a piece of work in 30
than 3. days. In how many days can 12 men and 24 women
do the same piece of work?
So, the total number of possible combinations of
(a) 30 days
the PIN = 6 + 3 + 1 = 10
(b) More than 30 days
56. There are eight equidistant points on a circle. How (c) Less than 30 days or more than 30 days
many right-angled triangles can be drawn using (d) Data is inadequate to draw any conclusion
these points as vertices and taking the diameter as Sol. (d)
one side of the triangle? Since the comparative efficiencies of man and
(a) 24 (b) 16 women are not known, we cannot determine the
(c) 12 (d) 8 time taken by 12 men and 24 women to complete
Sol. (a) the given work.

In the following figure, we have drawn eight Hence, the data is inadequate to draw any
equidistant points on a circle - A, B, C, D, E, F, G, conclusion.
and H.
58. What is the remainder when

91 × 92 × 93 × 94 × 95 × 96 × 97 × 98 × 99 is divided
by 1261?
(a) 3 (b) 2
(c) 1 (d) 0
Sol. (d)
Given expression = 91 × 92 × 93 × 94 × 95 × 96 × 97
× 98 × 99 = X (let)

When we consider AE as the diameter and one side 1261 = 1 × 13 × 97


of the right-angled triangle, we can draw 6 right- So, its factors are 13 and 97.
angled triangles.
Since in expression X, multiples of 13 and 97 are
there, 1261 will completely divide the expression
X. Hence, the remainder = 0

59. Consider the following statements is respect of a


rectangular sheet of length 20 cm and breadth 8
cm:
1. It is possible to cut the sheet exactly into
4 square sheets.
2. It is possible to cut the sheet into 10 triangular
sheets of equal area.

CSAT by Manjul Kumar Tiwari Sir


252 CSAT : 2022

Which of the above statements is/are correct? Or x/z = 0.65/0.60, which is greater than 1.
(a) 1 only (b) 2 only Hence, x > z …..(ii)
(c) Both 1 and 2 (d) Neither 1 nor 2
From inequalities (i) and (ii), we get:
Sol. (c)
z<x<y
The rectangle is of dimensions 20 cm × 8 cm.
Directions for the following 3 (three) items:
Statement I: You may think that as the area of
rectangle is not a perfect square, it is not possible to Read the following two passages and answer the items
cut it into exactly 4 square sheets. But there’s a catch. that follow the passages. Your answer to these items
The statement never says that the 4 squares have to should be based on the passages only.
be equal in area. We can do so as follows:
Passage-1

The majority of people who fail to accumulate money


sufficient for their needs, are generally, easily influenced
by the opinions of others. They permit the newspapers
and the gossiping neighbours to do their thinking for
them. Opinions are the cheapest commodities on the
earth. Everyone has a flock of opinions ready to be
wished upon by anyone who will accept them. If your
are influenced by opinions when you reach decisions,
Statement II: You may think that we can cut the
you will not succeed in any undertaking.
rectangle into 8 triangles of equal area (as shown
below), but not in 10. 61. Which one of the following is implied by the
passage?
(a) Most of the people do not accumulate money
for their needs.
(b) Most of the people never fail to accumulate
money for their needs.
(c) There are people who fail to accumulate
money for their needs.
(d) There is no need to accumulate money.
Sol. (c)
Option (a) is incorrect as the passage is referring to
Hence, both statements 1 and 2 are correct those people who are not able to accumulate
sufficient money to meet their needs. It does not say
60. When 70% of a number x is added to another what proportion of people are able to able to or fail
number y, the sum becomes 165% of the value of y. to accumulate money. Thus, statement 1 that most
When 60% of the number x is added to another people do not accumulate money to meet their needs
number z, then the sum becomes 165% of the value is not implied by the passage.
of z. Which one of the following is correct?
Option (b) is incorrect Similarly, the implication
(a) z < x < y (b) x < y < z
given in statement 2 that most people do not fail to
(c) y < x < z (d) z < y < x
accumulate money is also incorrect.
Sol. (a)
Option (c) is correct as the passage talks about the
According to the question, 0.7 x + y = 1.65 y
majority of those people who are not able to
Or 0.7 x = 0.65 y accumulate sufficient money to meet their needs.
Or x/y = 0.65/0.70, which is less than 1. This would imply that there are groups of people
Hence, x < y ...(i) who are not able to accumulate money to meet their
Now, 0.6 x + z = 1.65 z Or 0.6 x = 0.65 z needs.

CSAT by Manjul Kumar Tiwari Sir


Previous Year Solved Papers 253
Option (d) is incorrect: The passage does refer to Passage-2
the necessity for accumulating money to meet one’s “The social order is a sacred right which is the basis
needs. And if one does not, then (s)he will be misled of all other rights. Nevertheless, this right does not
by a plethora of (incorrect) opinions and would come from nature, and must therefore be founded on
ultimately never succeed in life. This is reflected in conventions.”
the lines “The majority of people who fail to
63. With reference to the above passage, which of the
accumulate money sufficient for their needs, are
following statements is/are correct?
generally, easily influenced by the opinions of
others.” and “If you are influenced by opinions 1. Conventions are the sources of rights of man.
when you reach decisions, you will not succeed in 2. Rights of man can be exercised only when
any undertaking.” there is a social order.
Select the correct answer using the code given
62. What is the main idea of the passage?
below.
(a) People should not be influenced by the
opinions of others. (a) 1 only (b) 2 only
(b) People should accumulate as much money as (c) Both 1 and 2 (d) Neither 1 nor 2
they can.
Sol. (c)
(c) People should neither give nor accept the
opinions. Statement (1) is correct. The passage clearly
(d) People will succeed in any undertaking if they mentions, “all other rights. Nevertheless, this right
do not accept any opinion at all. does not come from nature, and must therefore be
founded on conventions.” The purpose of the first
Sol. (a)
line, “The social order is a sacred right which is the
Option (a) is correct as the passage concludes by
basis of all other rights.” is to establish that one
saying that to succeed in any undertaking, people
cannot enjoy any right if there is no social order
should not be influenced by the opinion of others
present.
when they make decisions.
Option (b) is incorrect as the author is not Statement (2) correct. The first line of the passage
suggesting a need to “accumulate” money as such, categorically states that the right of social order is
beyond one’s needs. It only talks about “money the basis of all other rights. From this we can safely
sufficient for their needs” and not as much money infer that the rights of man can be exercised only
as possible. when there is social order. This is an extreme stand,
Option (c) is incorrect as although the passage but is correct as per the passage.
states that opinions are the cheapest commodity, it 64. Two candidates X and Y contested an election.
does not suggest that people should stop giving or 80% of voters cast their vote and there were no
accepting advice altogether. There may be genuine
invalid votes. There was no NOTA (None of the
and important advice, which might be important
above) option. X got 56% of the votes cast and won
and vital. Understanding the entire tone of the
by 1440 votes. What is the total number of voters
passage, the author seems to only makes a case
against the unwarranted advice, which would in the voters list?
harm rather than benefit people. So, it prescribes (a) 15000 (b) 12000
that people should not be influenced by others’
(c) 9600 (d) 5000
opinions easily while reaching decisions.
Option (d) is incorrect as the passage says that Sol. (a)
people will not succeed if they accept others’ Let total number of voters in the voter list be x.
opinions when reaching a decision. However, we
So, Number of casted votes = 0.8x
cannot infer that if one does not accept such
opinions, the person would succeed in their X got 56% of the casted votes. So, Y got 44% of the
enterprise, as this might be dependent on many casted votes. Thus, X won by (56 – 44)%, i.e. 12% of
other factors as well. the casted votes.

CSAT by Manjul Kumar Tiwari Sir


254 CSAT : 2022

According to the question, 12% of 0.8x = 1440 1. p × q can be an odd number.


Or x = (1440 × 100)/(0.8 × 12) = 15000 2. q/p can be a prime number.
Thus, the total number of voters in the voter list is 3. p + q can be a prime number.
15000. Which of the above statements are correct?
65. What is the smallest number greater than 1000 that (a) 1 and 2 only (b) 2 and 3 only
when divided by anyone of the numbers 6, 9, 12, (c) 1 and 3 only (d) 1, 2 and 3
15, 18 leaves a remainder of 3? Sol. (d)
(a) 1063 (b) 1073 p is a prime number. So, p can be 2, 3, 5, 7, 11, 13, .
(c) 1083 (d) 1183 q is a composite number. So, q can be 4, 6, 8, 9, 10,
Sol. (c) Statement 1: p × q can be an odd number, e.g.
(3 × 9 = 27). Thus, statement 1 is correct. Statement
LCM of 6, 9, 12, 15 and 18 = 180
2: q/p can be a prime number, e.g. (4/2 = 2). Thus,
Smallest number greater than 1000 which is a
statement 2 is correct.
multiple of 180 is 1080. So, Required number = 1080
Statement 3: p + q can be a prime number, e.g. (3 + 4
+ 3 = 1083
= 7). Thus, statement 3 is correct. Thus, all the
66. Let p be a two-digit number and q be the number statements 1, 2 and 3 are correct.
consisting of same digits written in reverse order.
68. Consider the following statements:
If p × q = 2430, then what is the difference between
1. Between 3:16 p.m. and 3:17 p.m., both hour
p and q?
hand and minute hand coincide.
(a) 45 (b) 27
2. Between 4:58 p.m. and 4:59 pm., both minute
(c) 18 (d) 9
hand and second hand coincide.
Sol. (d)
p is a two-digit number, and q is the number Which of the above statements is/are correct?
consisting of the same digits in the reverse order. (a) 1 only (b) 2 only
It’s given that, p × q = 2430 (c) Both 1 and 2 (d) Neither 1 nor 2

The last digit of the product is 0, which indicates Sol. (c)


that one two of the digits must be 5. Let the From statement 1:
remaining digit be x. At 3 o’clock, the minute hand is 15 minute spaces
So, p = x5 = x × 10 + 5 = 10x + 5 apart from hour hand. To be coincident, it must
gain 15 minute spaces.
Reverse number, q = 5x = 5 × 10 + x = 50 + x
We know that, 55 minutes are gained in 60
According to the question,
minutes.
(10x + 5) × (50 + x) = 2430
So, 15 minutes are gained in (60/55) × 15 = 180/
Or 500x + 10x2 + 250 + 5x = 2430 Or 10x2 + 505x – 11 minutes = 16.36 minutes
2180 = 0
Thus, hour hand and minute hand will coincide
Or 2x2 + 101x – 436 = 0 at 3:16:36, which is between 3:16 pm and 3:17 p.m.
Or x(2x + 109) – 4(2x + 109) = 0 Or (x – 4)(2x + 109) Hence, statement 1 is correct.
=0 From statement 2:
Or x = 4, or –109/2 (can be neglected) So, x = 4 At 4:58 p.m. the second hand is at 12. In the next
Therefore, p = x5 = 45 And q = 54 minute, the second will definitely cross the minute
Required difference = 54 – 45 = 9 hand. Thus, between 4:58 p.m. and 4:59 p.m. the
minute hand and second hand will definitely
67. Consider the following statements in respect of two coincide.
natural numbers p and q such that p is a prime Hence, statement 2 is correct.
number and q is a composite number: Thus, both statements 1 and 2 are correct.

CSAT by Manjul Kumar Tiwari Sir


Previous Year Solved Papers 255
69. There are two containers X and Y. X contains 100 Sol. (d)
ml of milk and Y contains 100 ml of water. 20 ml of A pie-chart corresponds to 3600.
milk from X is transferred to Y. After mixing well,
Items B, C, D and E correspond to 900, 500, 450 and
20 ml of the mixture in Y is transferred back to X. If
750 respectively on the pie chart.
m denotes the proportion of milk in X and n denotes
the proportion of water in Y, then which one of the Angle corresponding to A in the pie-chart = 3600 –
following is correct? (900 + 500 + 450 + 750) = 1000
(a) m = n So, Percentage of expenditure on item A = (100/
(b) m > n 360) × 100 = (250/9)%
(c) m < n
Directions for the following 3 (three) items:
(d) Cannot be determined due to insufficient data
Sol. (a) Read the following two passages and answer the items
that follows the passages. Your answers to these items
Container X contains 100 ml of milk and container
should be based on the passages only.
Y contains 100 ml of water.
If 20 ml of milk transferred from container X to Passage-1
container Y, then: To encourage research is one of the functions of a
Amount of milk left in container X = 100 – 20 = university. Contemporary universities have encouraged
80 ml research, not only in those cases where research is
Amount of solution in container Y becomes = 100 necessary, but on all sorts of entirely unprofitable
ml water + 20 ml milk = 120 ml Ratio of milk and subjects as well. Scientific research is probably never
water in container Y = 20 : 100 = 1 : 5 completely valueless. However silly and insignificant
it may seem, however mechanical and unintelligent the
Amount of milk in 20 ml solution of container Y =
labours of the researchers, three is always a chance
(1/6) × 20 = 3.33 ml Amount of water in 20 ml
that the results may be of value to the investigator of
solution = 20 – 3.33 = 16.67 ml
talent, who can use the facts collected for him by
If 20 ml of this solution is transferred from container
uninspired but industrious researchers as the basis of
Y to container X, then:
some fruitful generalization. But where research is not
Amount of solution in container X becomes = (80 + original, but consists in the more rearrangement of
3.33), i.e. 83.33 ml milk + 16.67 ml water existing materials, where its object is not scientific but
Amount of solution in container Y becomes = (100 literary or historical, then there is a risk of the whole
– 16.67), i.e. 83.33 ml water + (20 – 3.33), i.e. 16.67 business becoming merely futile.
ml milk
71. The author’s assumption about scientific research
As per the question, m denotes the proportion of
is that
milk in X, and n denotes the proportion of water in
(a) it is never very valuable
Y. So, m = 83.33 ml and n = 83.33 ml
(b) it is sometimes very valuable
Thus, m = n (c) it is never without some value
(d) it is always very valuable
70. A pie chart gives the expenditure on five different
items A, B, C, D and E in a household. If B, C, D Sol. (c)
and E correspond to 90°, 50°, 45° and 75° Option (a) is incorrect as the author clearly lays out
respectively, then what is the percentage of a situation where seemingly silly and insignificant
expenditure on item A? research, when used by an intelligent investigator
can be very valuable.
112 125
(a) (b) Option (b) is incorrect. The author has vividly given
9 6
stated that research is always (and not sometimes)
155 250 valuable. This is exemplified in the lines, “Scientific
(c) (d) research is probably never completely valueless.
9 9

CSAT by Manjul Kumar Tiwari Sir


256 CSAT : 2022

However silly and insignificant it may seem, result may be of significant value, but does not mean
however mechanical and unintelligent the labours that it is always of value to an intelligent
of the researchers, there is always a chance that the investigator. This is exemplified in the lines “But
results may be of value to the investigator of talent, where research is not original, but consists in the
who can use the facts collected for him by mere rearrangement of existing materials, where its
uninspired but industrious researchers as the basis objects is not scientific but literary or historical, then
of some fruitful generalization.” there is a risk of the whole business becoming merely
Option (c) is correct. As explained above, the author futile.”
clearly explains that research is always valuable. Option (c) is correct: From the lines “Scientific
This can be seen in the lines, “Scientific research is research is probably never completely valueless”
probably never completely valueless. However silly we can infer that any result can be valuable and the
and insignificant it may seem, however mechanical lines, “there is always a chance that the results may
and unintelligent the labours of the researchers, be of value to the investigator of talent, who can use
there is always a chance that the results may be of the facts collected for him by uninspired but
value to the investigator of talent, who can use the industrious researchers as the basis of some fruitful
facts collected for him by uninspired but generalization” show the possibility that any result
industrious researchers as the basis of some fruitful can be valuable.
generalization.” Option (d) is incorrect: The passage does not
Option (d) is incorrect as the author provides a suggest that it is a must that the research must be of
scenario where the research could be futile, i.e. when some value to the intelligent investigator! It suggests
the research is not original and a mere that a seemingly futile research effort would find
rearrangement of existing materials with its object utility in the eyes of those who can utilise and make
being literary and historical. This indicates that benefit of it and that research never goes waste.
there is a situation when the research is not Passage-2
valuable. Thus, the research is not always very How best can the problems of floods and droughts be
valuable. addressed so that the losses are minimal and the system
becomes resilient? In this context, one important point
72. According to the author
that needs to be noted is that India gets ‘too much’
(a) not many research results can be of value to
water (about 75% of annual precipitation) during 120
an intelligent investigator
days (June to September) and ‘too little’ for the remaining
(b) a research results is always valuable to an
245 days. This skewed water availability has to be
intelligent investigator
managed and regulated for its consumption throughout
(c) any research result can be of value to an
the year.
intelligent investigator
(d) a research result must always be of some value 73. Which one of the following best reflects the
to an intelligent investigator practical, rational and lasting solution?
Sol. (c) (a) Constructing huge concrete storage tanks and
canals across the country
Option (a) is incorrect as the author is not
discussing how many results are of value to an (b) Changing the cropping patterns and farming
intelligent investigator. In the lines “there is always practices
a chance that the results may be of value to the (c) Interlinking of rivers across the country
investigator of talent”, the author is suggesting a (d) Buffer stocking of water through dams and
high possibility of the result of research being recharging aquifers
valuable, but it does not indicate or imply how Sol. (d)
many of these results are valuable or invaluable.
Option (a) is incorrect as constructing huge concrete
Option (b) is incorrect: Continuing on the previous storage tanks and canals across the country is not
option’s explanation, one can also see that any the most rational and practical solution due to the

CSAT by Manjul Kumar Tiwari Sir


Previous Year Solved Papers 257
large capital expenditure associated with such 75. What is the value of X in the sequence 2, 12, 36, 80,
dams, their viability in seismically active zones, and 150, X?
other socio-environmental problems associated (a) 248 (b) 252
with them. (c) 258 (d) 262
Also, the passage only discusses the temporal Sol. (b)
aspect of the same- “‘too much’ water (about 75% The pattern has been represented below:
of annual precipitation) during 120 days (June to
September) and ‘too little’ for the remaining 245
days.”- and not the spatial aspect. Whereas the
solution in this option discusses a spatial
component as well. Therefore, it does not quite
follow the passage and goes beyond its scope.
Option (b) is incorrect Apart from the reason
discussed above, changing the cropping patterns
and farming practices would not lead to practical Hence, option (b) is the correct answer.
water management as excess water still can not be 76. One non-zero digit, one vowel and one consonant
utilized or regulated which would flow to the from English alphabet (in capital) are to be used in
oceans and can not be available for other times. forming passwords, such that each password has
Option (c) is incorrect as the interlinking of rivers to start with a vowel and end with a consonant.
is a controversial issue due to sustainability issues How many such passwords can be generated?
associated with it. For example, the displacement (a) 105 (b) 525
of people, the disturbance of the ecosystem of (c) 945 (d) 1050
riparian regions, and the loss of biodiversity and Sol. (c)
productive lands. One non-zero digit = 1, 2, 3, 4, 5, 6, 7, 8 or 9, i.e. 9
Option (d) is correct Buffer stocking of water through possible values.
dams and recharging aquifers would mean that the There are 5 vowels and 21 consonants in English
excess water is being stored in a practical and cost- alphabet.
effective manner. Also, no sustainability issues are The password starts with a vowel and ends with a
involved with it. Recharged aquifer would also consonant. So, the digit will come at the middle.
ensure that groundwater is available for a longer The number of such passwords = 5 × 9 × 21 = 945
duration and during the off rainy seasons as well. Hence, 945 such passwords can be generated.

77. There are 9 cups placed on a table arranged in


74. If 15 × 14 × 13 × ... × 3 × 2 × 1 = 3m × n, where m and
equal number of rows and columns out of which 6
n are positive integers, then what is the maximum
cups contain coffee and 3 cups contain tea. In how
value of m?
many ways can they be arranged so that each row
(a) 7 (b) 6 should contain at least one cup of coffee?
(c) 5 (d) 4 (a) 18 (b) 27
Sol. (b) (c) 54 (d) 81
Sol. (d)
15 × 14 × 13 × 12 × 11 × 10 × 9 × 8 × 7 × 6 × 5 × 4 ×
3 × 2 × 1 = 3m × n The cups will be arranged in a 3 × 3 matrix.
Numbers which are multiple of 3 = 15 × 12 × 9 ×
6 × 3 = (3 × 5) × (3 × 4) × (3 × 3) × (3 × 2) × 3
= 36 × (5 ×4 × 2)
Therefore, the maximum value of m is 6. Hence,
option (b) is the correct answer.

CSAT by Manjul Kumar Tiwari Sir


258 CSAT : 2022

Now, let’s consider all the possible cases.  3x = x × (x2 - 1)


Case 1: Each row has 2 cups of coffee and 1 cup of  3x = x3 – x
tea.  x3 – 4x = 0
 x (x2 – 4) = 0
C C T
 x = 0, 2 or -2
C C T
Therefore, there are only 3 such possibilities.
C C T
Hence, option (c) is the correct answer.
One tea cup can be placed in a row in 3 ways.
79. What is the number of numbers of the form 0.XY,
So, Possible arrangements wherein all rows have 2 where X and Y are distinct non-zero digits?
coffee cups= 3×3×3 = 27 ways (a) 72 (b) 81
Case 2: One row has 3 cups of coffee. (c) 90 (d) 100
In such a case, one of the rows will have 2 cups of Sol. (a)
coffee and the other will have 1 cup of coffee. Since, X and Y are distinct non-zero digits
Since, the required number of numbers of the form
C C C All 'C' can be arranged in 1 way
0.XY = 9 × 8 = 72
C C T 2 C can be arranged in 3 ways
Hence, option (a) is the correct answer.
C T T 1 C can be arranged in 3 ways
80. The average weight of A, B, C is 40 kg, the average
If the first row has 3 coffee cups, then the possible weight of B, D, E is 42 kg and the weight of F is
arrangements = 3 × 3 = 9 ways equal to that of B. What is the average weight of A,
The rows having 3, 2 and 1 coffee cups can be B, C, D, E and F?
arranged in 3! = 6 ways (a) 40.5 kg
So, Possible arrangements wherein one row has (b) 40.8 kg
3 coffee cups = 9 × 6 ways = 54 ways (c) 41 kg
(d) Cannot be determined as data is inadequate
Therefore, total possible arrangements = 27 + 54
= 81 ways Sol. (c)
According to the question, (A + B + C)/3 = 40
Hence, option (d) is the correct answer.
Or (A + B + C) = 120 ………(i)
78. The sum of three consecutive integers is equal to (B + D + E)/3 = 42
their product. How many such possibilities are Or (B + D + E) = 126 ..……(ii)
there? F=B .…….(iii)
(a) Only one
From equations (i) and (ii), we get:
(b) Only two
A + B + C + B + D + E = 120 + 126
(c) Only three
(d) No such possibility is there Or A + B + C + D + E + B = 246

Sol. (c) Or A + B + C + D + E + F = 246 (as F = B)


So, average weight of A + B + C + D + E + F
Let the 3 consecutive numbers be x - 1, x, and x + 1
According to the question, = 246/6 = 41

(x – 1) + x + (x + 1) = (x - 1) × x × (x + 1)


CSAT by Manjul Kumar Tiwari Sir

You might also like